Superpage
Lung

Authors: Ali Alzeer, M.D., Atreyee Basu, M.D., Jennifer M. Boland, M.D., Rachel Bowden, M.B.B.S., M.Sc., Luka Brčić, M.D., Ph.D., Andrey Bychkov, M.D., Ph.D., Matthew J. Cecchini, M.D., Ph.D., Kyriakos Chatzopoulos, M.D., Ph.D., Heather I-Hsuan Chen-Yost, M.D., Luca Cima, M.D., Brittany Cody, D.O., Junya Fukuoka, M.D., Ph.D., Andréanne Gagné, M.D., M.Sc., Carolyn Glass, M.D., Ph.D., Hongxing "Simon" Gui, M.D., Ph.D., Hirotsugu Hashimoto, M.D., Ph.D., Tao Huang, Ph.D., Aliya N. Husain, M.D., Deepali Jain, M.D., Philippe Joubert, M.D., Ph.D., Jonathan Keow, M.D., Ph.D., Andras Khoor, M,D., Ph.D., Kristine Konopka, M.D. , Ihab Lamzabi, M.D., Brandon T. Larsen, M.D., Ph.D., Lili Lee, M.D., Hui-Hua Li, M.D., Ph.D., Sherman Lin, Ana Mataić, M.D., Alex McGeough, M.D., Phillip McMullen, M.D., Ph.D., M. Ángeles Montero-Fernández, M.D., Ph.D., Jijgee Munkhdelger, M.D., Ph.D., Alexis Musick, B.S., Nya D. Nelson, M.D., Ph.D., Gheorghe-Emilian Olteanu, M.D., Ph.D., Pembe Oltulu, M.D., Erdener Özer, M.D., Ph.D., Sevilay Özmen, M.D., Nat Pernick, M.D., William Perry, M.D., M.P.H., Jennifer Pogoriler, M.D., Ph.D., Jian-Hua Qiao, M.D., Sakda Sathirareuangchai, M.D., Jefree J. Schulte, M.D., Sana Shah, M.B.B.S., Divya Sharma, M.D., Brian D. Stewart, M.D., Emily O. Symes, M.D., Anusmita Tripathy, M.D., Ateeqa Mujeeb Ullah, M.D., Caroline I.M. Underwood, M.D., Elliot Weisenberg, M.D., Roseann I. Wu, M.D., M.P.H., Y. Albert Yeh, M.D., Ph.D., Eunhee S. Yi, M.D., Akira Yoshikawa, M.D., Bahaaeldin Youssef, M.D., Paul J.L. Zhang, M.D., Fang Zhou, M.D.
Resident / Fellow Advisory Board: David B. Chapel, M.D.
Editorial Board Members: Andrey Bychkov, M.D., Ph.D., Matthew J. Cecchini, M.D., Ph.D., Carolyn Glass, M.D., Ph.D., Marc Pusztaszeri, M.D., Jefree J. Schulte, M.D., Debra L. Zynger, M.D.
Deputy Editors-in-Chief: Andrey Bychkov, M.D., Ph.D., Patricia Tsang, M.D., M.B.A., Debra L. Zynger, M.D.
Editor-in-Chief: Debra L. Zynger, M.D.

Copyright: 2003-2024, PathologyOutlines.com, Inc.

Pulm / Heart related: Jobs, Fellowships, Conferences, Cases, CME, Board Review

Related chapters: Heart, Mediastinum, Pleura & peritoneum

Editorial Board oversight: Jefree J. Schulte, M.D. (last reviewed April 2022), Andrey Bychkov, M.D., Ph.D. (last reviewed January 2021)
Page views in 2023: 119
Page views in 2024 to date: 54

Acute fibrinous and organizing pneumonia
Definition / general
Essential features
  • It is a newly proposed subacute interstitial pneumonia similar to organizing pneumonia or organizing diffuse alveolar damage
  • Histologically, remarkable fibrin deposition (or "fibrin balls") along with plugs of organizing pneumonia in air spaces are characteristic
Terminology
  • Also called acute fibrinous organizing pneumonia
Epidemiology
Sites
  • Usually bilateral or sometimes unilateral lobes of the lung
Pathophysiology
  • In the initial study, acute fibrinous and organizing pneumonia (AFOP) was described as a possible variant of diffuse alveolar damage because of its similar aggressive behavior and mortality rate (Arch Pathol Lab Med 2002;126:1064)
  • However, recent studies and case reports have found that the clinical course and prognosis of AFOP is better and closer to that of organizing pneumonia (J Clin Pathol 2015;68:441, Chin Med J (Engl) 2015;128:2701)
  • Nowadays, AFOP is considered a histological variant of organizing pneumonia or a different type of lung disease similar to organizing pneumonia, which sometimes follows an aggressive course
  • Some idiopathic AFOP may be due to infection of undiagnosed bacteria
Etiology
Clinical features
  • Most patients present with mild to moderate subacute respiratory failure (J Clin Pathol 2015;68:441, Chin Med J (Engl) 2015;128:2701)
    • Fever, fatigue and malaise
    • Cough
    • Dyspnea
    • Sputum or sometimes hemoptysis
    • Duration of symptoms before diagnosis is 1 - 4 weeks
  • Some patients may follow fulminant course, need mechanical ventilation and die of the disease, similar to diffuse alveolar damage (Arch Pathol Lab Med 2002;126:1064, Medicine (Baltimore) 2016;95:e4073)
  • Abnormal chest auscultation
    • End inspiratory fine crackles in affected lobes
  • Mild to moderate restrictive or obstructive pattern in pulmonary function tests (Chin Med J (Engl) 2015;128:2701)
    • Decreased total lung capacity (TLC)
    • Decreased forced vital capacity (FVC)
    • Decreased diffusing capacity of the lung for carbon monoxide (DLCO)
Diagnosis
  • Based on clinical features, radiology and histology
    • No unique clinical or radiological findings have been identified to date
    • Open chest lung biopsy is recommended
      • Transbronchial lung biopsy or computed tomography guided needle lung biopsy may be diagnostic if clinical and radiological features are suggestive enough
    • Acute fibrinous and organizing pneumonia can be a background pattern with other disease present
      • If the specimen is too small and the main lesion is not included, acute fibrinous and organizing pneumonia can be underdiagnosed (Int J Clin Exp Pathol 2014;7:4493)
Laboratory
  • Increased C reactive protein
  • Increased serum surfactant proteins A and D
  • Increased serum ferritin may predict prognosis
  • Occasional positive sputum bacterial culture
  • Negative serum antibodies of connective tissue diseases and hypersensitivity pneumonitis
Radiology description
  • Simple chest radiography
    • Bilateral or unilateral ground glass opacity and consolidation
  • High resolution computed tomography (Radiographics 2013;33:1951):
    • Variable images, similar to organizing pneumonia
      • Typically, patchy mixture of ground glass opacity and consolidation
      • Size varies from a few centimeters to a whole lobe
    • Rapidly progressive variant may show bilateral diffuse opacity, similar to diffuse alveolar damage
Radiology images

Images hosted on other servers:
Chest radiograph Chest radiograph Chest radiograph

Chest radiograph

Acute fibrinous organizing pneumonia in left upper lobe Acute fibrinous organizing pneumonia in left upper lobe Acute fibrinous organizing pneumonia in left upper lobe

Acute fibrinous organizing pneumonia in left upper lobe


Multifocal opacities

Multifocal opacities

Before and after corticosteroid therapy Before and after corticosteroid therapy

Before and after corticosteroid therapy

Before disease onset and before / after corticoid therapy

Before disease onset and before / after corticoid therapy

Prognostic factors
Case reports
Treatment
  • In general, corticosteroid pulse therapy with / without cyclophosphamide improves the symptoms and prognosis (Arch Pathol Lab Med 2002;126:1064, J Clin Pathol 2015;68:441, Chin Med J (Engl) 2015;128:2701)
  • Treatment for underlying cause is also important for secondary acute fibrinous and organizing pneumonia
    • Antibiotics are effective for acute fibrinous and organizing pneumonia induced by bacterial infection
    • It is questionable if antibiotics can be a general therapeutic choice or not
  • Mechanical ventilation may be necessary for aggressive type
Gross description
  • Multiple patchy consolidated lesions
  • Ill defined, soft to firm gray areas
  • Mild increase in weight
  • Alveoli are filled with reddish fibrinous exudates
Microscopic (histologic) description
  • Major findings
    • Dominant findings of intra-alveolar fibrin, so called "fibrin ball"
      • Involves more than 20% of the alveolar spaces in the lesion
      • Neutrophils are usually scanty or absent
    • Organizing pneumonia: fibroblastic plugs in alveolar sacs and ducts with loose collagen matrix
    • Diffuse and patchy distribution
  • Minor findings
    • Mild to moderate interstitial changes
      • Lymphoplasmacytic infiltrate
      • Alveolar septal expansion with myxoid connective tissue
      • Limited within areas of fibrinous lesion
    • Type 2 pneumocyte hyperplasia
  • Pertinent negative findings; need to rule out secondary causes and other lung disease if present
    • Hyaline membranes
    • Eosinophilic inflammation
    • Extensive bronchopneumonia or abscess
    • Granulomatous inflammation
    • Vasculitis including capillaritis
    • Areas of necrosis
    • Marked dense fibrosis or honeycombing
  • See J Clin Pathol 2015;68:441
Microscopic (histologic) images

Contributed by Akira Yoshikawa, M.D. and Yale Rosen, M.D.
Low power

Low power

Medium power

Medium power

High power High power

High power

Elastica van Gieson staining Elastica van Gieson staining

Elastica van Gieson staining


Fibrin balls Fibrin balls Fibrin balls Fibrin balls Fibrin balls Fibrin balls

Fibrin balls


Fibrin balls

Fibrin balls

Acute fibrinous and organizing pneumonia Acute fibrinous and organizing pneumonia Acute fibrinous and organizing pneumonia Acute fibrinous and organizing pneumonia

Acute fibrinous and organizing pneumonia



Images hosted on other servers:
Fibrin balls in air spaces Fibrin balls in air spaces Fibrin balls in air spaces Fibrin balls in air spaces Fibrin balls in air spaces

Fibrin balls in air spaces


Fibrin balls in air spaces Fibrin balls in air spaces

Fibrin balls in air spaces

AFOP

AFOP

Positive stains
Negative stains
Differential diagnosis
Board review style question #1
Which two of the following findings are against the histological diagnosis of acute fibrinous and organizing pneumonia?

  1. Eosinophilic infiltration
  2. Hyaline membranes
  3. Lymphocytic infiltration
  4. Organizing pneumonia
  5. Type 2 pneumocyte hyperplasia
Board review style answer #1
A and B. Eosinophilic infiltration suggests eosinophilic pneumonia. Hyaline membranes suggest diffuse alveolar damage.

Comment Here

Reference: Acute fibrinous and organizing pneumonia

Acute interstitial pneumonia
Definition / general
  • In 1935, Hamman and Rich first reported autopsy cases of initially healthy individuals who developed a rapidly progressive and fatal type of interstitial lung disease, which differed from other interstitial pneumonia clinically and pathologically (Trans Am Clin Climatol Assoc 1935;51:154)
  • Katzenstein et al. coined the term "acute interstitial pneumonia (AIP)" (Am J Surg Pathol 1986;10:256)
  • In the multidisciplinary classification of idiopathic interstitial pneumonias by American Thoracic Society / European Respiratory Society, acute interstitial pneumonia is categorized as "acute / subacute interstitial pneumonia" (Am J Respir Crit Care Med 2013;188:733)
Essential features
  • Rare and aggressive type of idiopathic interstitial pneumonia with diffuse alveolar damage (DAD), characterized by diffuse inflammation with hyaline membrane and fibroblastic proliferation
  • Acute interstitial pneumonia shares common features with acute respiratory distress syndrome (ARDS) clinically and morphologically
Terminology
  • Also called Hamman-Rich syndrome and idiopathic diffuse alveolar damage
ICD coding
Epidemiology
  • Extremely rare (no conclusive epidemiological data available)
  • Mean age 50 years but can occur at any age (7 - 83 years) (Eur Respir J 2000;15:412)
  • No sex predilection
Sites
  • Bilateral lung, usually in all five lobes of the lung
Pathophysiology
  • Both endothelial and epithelial injury result in decreased integrity of the alveolar capillary membrane
  • Imbalance of proinflammatory and anti-inflammatory mediators
  • Neutrophils increase in alveoli and interstitium and release metabolites leading to lung injury
  • Alveolar epithelial cells may go through epithelial - mesenchymal transition to become myofibroblasts, resulting in interstitial organization and fibrosis (BMC Pulm Med 2014;14:67)
Etiology
  • No definite cause; no risk factors have been identified
Clinical features
  • Influenza-like illness, followed by progressive shortness of breath (Am J Surg Pathol 1986;10:256)
  • Vast majority of patients are previously healthy and lack history of lung disease
  • Many clinical characteristics of acute interstitial pneumonia are similar to acute respiratory distress syndrome (Chest 2003;124:554)
    • Acute interstitial pneumonia can progress to respiratory failure as profound as severe acute respiratory distress syndrome (PaO2/FIO2 ≤ 100 mm Hg) and almost all patients need mechanical ventilation and hospital care
    • Respiratory failure usually appears 1 - 3 weeks from the onset, later than acute respiratory distress syndrome (16.8 days vs. 2.2 days)
    • Multiple organ failure is less common in acute interstitial pneumonia
Diagnosis
Laboratory
  • Hypoxia
  • Increased serum ferritin, D dimer and C reactive protein
  • KL-6 may increase slightly
Radiology description
  • Heterogeneous bilateral ground glass opacity due to pulmonary edema
  • Chest radiograph
    • Ground glass opacity
    • Consolidation with air bronchogram
  • Chest CT
Radiology images

Images hosted on other servers:
Chest radiograph

Chest radiograph

Exudative phase on CT Exudative phase on CT

Exudative phase on CT

Organizing phase on CT Organizing phase on CT

Organizing phase on CT


Comparison on CT and histology Comparison on CT and histology

Comparison on CT and histology

Prognostic factors
  • Most patients die within 2 months unless appropriate treatment is provided (Eur Respir J 2000;15:412)
  • High dose steroid therapy drastically improves the prognosis with long term survival of more than 80% (Chest 2006;129:753, Chest 2003;124:554)
  • Survivors may suffer recurrences or develop chronic lung injury
Case reports
Treatment
  • Oxygen therapy for respiratory failure
    • Mechanical ventilation with positive end expiratory pressure
  • High dose steroid pulse (Chest 2006;129:753)
  • Direct hemoperfusion using polymyxin B immobilized fiber column was recently found to effectively improve the prognosis of acute interstitial pneumonia patients (Ther Adv Respir Dis 2017;11:261)
Gross description
  • Dark blue lungs with hemorrhagic dots on pleural surface
  • Heavy and firm due to edema and fibrosis
  • Dilatation of alveolar ducts
Gross images

Contributed by Yale Rosen, M.D.
Gross images of diffuse alveolar damage Gross images of diffuse alveolar damage

Gross images of diffuse alveolar damage

Microscopic (histologic) description
  • Acute interstitial pneumonia shows diffuse alveolar damage, which is almost completely identical to acute respiratory distress syndrome / diffuse alveolar damage morphologically (Eur Respir J 2000;15:412)
  • Proliferative / organizing (subacute) phase of diffuse alveolar damage is most common in acute interstitial pneumonia but also exudative (acute) phase and fibrotic (chronic) phase can be seen
  • Exudative phase
    • Hyaline membranes in alveolar duct or sacs; scattered or not apparent, unlike in acute respiratory distress syndrome
    • Interstitial and intra-alveolar edema
    • Collapsed alveoli
    • Denudation and necrosis of type I pneumocytes
    • Hemorrhage, usually mild
  • Proliferative / organizing phase (Am J Surg Pathol 1986;10:256, Eur Respir J 2003;21:187)
    • Organizing pneumonia with / without remnants of hyaline membrane
    • Interstitial and intra-alveolar proliferation of fibroblasts / myofibroblasts
    • Lymphocytic infiltration; usually more prominent than in acute respiratory distress syndrome
    • Proliferation of type II pneumocytes with occasional cellular atypia
    • Endothelial injury and fibrinous thromboembolism in arterioles / arteries
  • Fibrosis phase
    • Diffuse collagenous fibrosis
    • Microscopic honeycomb-like change
    • Traction bronchiolectasis
    • Squamous metaplasia
    • Organized thrombus
    • Thickening of pleura with dilatation of lymphatic / blood vessels
Microscopic (histologic) images

Contributed by Akira Yoshikawa, M.D. and Yale Rosen, M.D.
Low power

Low power

Architectural destruction Architectural destruction

Architectural destruction

Fibroblastic proliferation Fibroblastic proliferation

Fibroblastic proliferation


Type II pneumocyte hyperplasia

Type II pneumocyte hyperplasia

Lymphocytic infiltration Lymphocytic infiltration

Lymphocytic infiltration

Epithelial denudation

Epithelial denudation

Dense fibrosis with smooth muscle hyperplasia

Dense fibrosis with smooth muscle hyperplasia


Organizing phase - proliferation of fibroblasts and type II pneumocytes Organizing phase - proliferation of fibroblasts and type II pneumocytes Organizing phase - proliferation of fibroblasts and type II pneumocytes Organizing phase - proliferation of fibroblasts and type II pneumocytes Organizing phase - proliferation of fibroblasts and type II pneumocytes

Organizing phase - proliferation of fibroblasts and type II pneumocytes



Images hosted on other servers:
Organizing phase

Organizing phase

Organizing phase - fibroblastic proliferation Organizing phase - fibroblastic proliferation

Organizing phase - fibroblastic proliferation

Exudative phase and organizing phase Exudative phase and organizing phase

Exudative phase and organizing phase

Cytology description
Cytology images

Images hosted on other servers:
Atypical epithelial cells in BAL fluid

Atypical epithelial cells in BAL fluid

Positive stains
  • Elastica van Gieson (fiber staining) is helpful to evaluate architectural destruction of alveoli
Electron microscopy description
  • Proliferation of type II pneumocyte with cytoplasmic projection into alveolar septa, abnormally large lamellar bodies or denudation from basement membrane (Am J Surg Pathol 1986;10:256)
Differential diagnosis
Board review style question #1
Which of the following findings is not required for the diagnosis of acute interstitial pneumonia?

  1. Absence of exposure to causative factors of respiratory failure
  2. Absence of prior history of lung disease
  3. Bilateral shadows on chest radiograph
  4. Diffuse alveolar damage on histology
  5. PaO2/FIO2 ≤ 300 mm Hg
Board review style answer #1
E. PaO2/FIO2 ≤ 300 mm Hg is one of the diagnostic criteria of acute respiratory distress syndrome. Acute interstitial pneumonia can also cause the similar severe respiratory failure, however it is not a diagnostic requirement for acute interstitial pneumonia.

Comment Here

Reference: Acute interstitial pneumonia

Acute pneumonia
Definition / general
  • Broadly defined as acute inflammation of the lung parenchyma
  • Clinically characterized by fever, purulent sputum, leukocytosis and decline in oxygenation
  • Caused by bacteria and other microorganisms (virus and fungus)
Essential features
  • Classically divided by gross morphology into lobar and bronchopneumonia
  • Intra-alveolar fibrinopurulent exudate with neutrophils
  • Streptococcus pneumoniae is the most common bacteria causing community acquired pneumonia
  • Gram negative bacilli and Staphylococcus aureus are important causes of hospital acquired pneumonia and ventilator associated pneumonia
Terminology
  • Classification principles:
    • Pathogens: bacterial, viral, fungal
    • Clinical setting: community acquired pneumonia (CAP), hospital acquired pneumonia (HAP), ventilator associated pneumonia (VAP)
    • Extent:
      • Lobar pneumonia: involvement of the entire lung lobe
      • Bronchopneumonia: patchy involvement of the lung parenchyma, originating from the airway
  • Community acquired pneumonia (CAP):
    • Lung infection that is acquired from the normal environment
  • Hospital acquired pneumonia (HAP) (Clin Infect Dis 2016;63:e61):
    • Pneumonia not incubating at the time of hospital admission and occurring > 48 hours after admission
  • Ventilator associated pneumonia (VAP):
    • Pneumonia occurring > 48 hours after endotracheal intubation
  • Healthcare associated pneumonia (HCAP):
    • American Thoracic Society (ATS) / Infectious Diseases Society of America (IDSA) recommend abandoning this term in the most recent HAP / VAP guideline
ICD coding
  • ICD-10:
    • J13 - pneumonia due to Streptococcus pneumoniae
    • J14 - pneumonia due to Hemophilus influenzae
    • J15.0 - pneumonia due to Klebsiella pneumoniae
    • J15.1 - pneumonia due to Pseudomonas
    • J15.2 - pneumonia due to Staphylococcus
    • J15.3 - pneumonia due to streptococcus, group B
    • J15.4 - pneumonia due to other streptococci
    • J15.5 - pneumonia due to Escherichia coli
    • J15.6 - pneumonia due to other aerobic gram negative bacteria
    • J15.8 - other bacterial pneumonia
    • J15.9 - bacterial pneumonia, unspecified
  • ICD-11:
    • CA40.0 - bacterial pneumonia
    • CA40.01 - pneumonia due to Escherichia coli
    • CA40.02 - pneumonia due to Hemophilus influenza
    • CA40.03 - pneumonia due to Klebsiella pneumoniae
    • CA40.05 - pneumonia due to Pseudomonas aeruginosa
    • CA40.06 - pneumonia due to Staphylococcus
    • CA40.07 - pneumonia due to Streptococcus pneumoniae
    • CA40.08 - pneumonia due to beta hemolytic Streptococcus
    • CA40.0Y - pneumonia due to other specified bacteria
    • CA40.0Z - bacterial pneumonia, unspecified
Epidemiology
  • Leading cause of adult hospital admissions in the U.S. (Healthcare Cost and Utilization Project (HCUP) Statistical Briefs: Most Frequent Conditions in U.S. Hospitals, 2011):
  • For most cases of CAP (62%), a causative organism is not identified (N Engl J Med 2015;373:415)
    • Bacterial pneumonia accounts for 11% of CAP in the U.S.
    • Remainder: virus 23%, bacterial and viral pathogens 3%, fungus or mycobacterium 1%
  • Risk factors related to specific pathogens in CAP (Clin Infect Dis 2007;44:S27):
    • Alcohol use disorder: S. pneumoniae, oral anaerobes, Klebsiella pneumoniae, Acinetobacter species
    • Chronic obstructive pulmonary disease or smoking: Haemophilus influenzae, Pseudomonas aeruginosa, S. pneumoniae, Moraxella cararrhalis
    • Structural lung disease (e.g., bronchiectasis): P. aeruginosa, Burkholderia cepacia, S. aureus
    • Injection drug use: S. aureus, anaerobes, S. pneumoniae
    • Endobronchial obstruction: anaerobes, S. pneumoniae, H. influenzae, S. aureus
  • Risk factors for multidrug resistant pathogens in HAP / VAP (Clin Infect Dis 2016;63:e61):
    • Prior intravenous antibiotic use within 90 days
    • Septic shock at time of VAP
    • Acute respiratory distress syndrome preceding VAP
    • 5 or more days of hospitalization prior to the occurrence of VAP
    • Acute renal replacement therapy prior to VAP onset
Sites
  • Lungs, ranging from several acini to segments and lobes (up to total involvement)
  • Predilection
    • K. pneumoniae has a predilection for upper lobes (South Med J 1991;84:200)
    • Aspiration pneumonia tends to involve dependent areas of the lung; posterior segments of the upper lobes and apical segments of the lower lobes (Br J Radiol 2010;83:998)
Pathophysiology
  • Bacteria can reach the lungs in several ways (Semin Diagn Pathol 2017;34:498)
    • Airborne droplet spread
    • Microaspiration of pathogens that have colonized the oropharynx is a common mechanism in bronchopneumonia
    • Spread to the lungs via the pulmonary or systemic blood supply
  • Exudative spread throughout the lung via the pores of Kohn, potential channels between adjacent alveoli
  • Development of lobar pneumonia entails 4 stages (Kumar: Robbins & Cotran Pathologic Basis of Disease, 10th Edition, 2020)
    • Congestion: vascular engorgement, intra-alveolar fluid with few neutrophils, red cells and fibrin
    • Red hepatization: massive confluent exudation (neutrophils, red cells and fibrin), resulting in liver-like consistency
    • Gray hepatization: progressive disintegration of red cells, while fibrosuppurative exudate persists
    • Resolution: exudate broken down by enzymatic process, resulting in cellular debris, macrophages infiltrate and fibroblast proliferation
  • 5 - 15% of CAP are aspiration pneumonia (N Engl J Med 2001;344:665)
  • Pneumonia from hematogenous spread can be a result of:
    • Lemierre syndrome: infection and thrombosis of the internal jugular veins following throat and tonsillar infections by Fusobacterium necrophilum (N Engl J Med 2019;380:e16)
    • Infective endocarditis at the tricuspid and pulmonic valves
Etiology
  • Survey data in hospitalized patients from CAP in the U.S. (N Engl J Med 2015;373:415, Am J Respir Crit Care Med 2019;200:e45)
    • Most common: Streptococcus pneumoniae
    • Followed by Mycoplasma pneumoniae, Staphylococcus aureus, Legionella species and Enterobacteriaceae
  • Recent survey data in hospitalized patients from CAP in the U.S. (N Engl J Med 2015;373:415)
    • Most common bacteria: Streptococcus pneumoniae, followed by Mycoplasma pneumoniae, Staphylococcus aureus, Legionella species and Enterobacteriaceae
  • Causative organisms in HAP / CAP
    • Data from the U.S. Center for Disease Control and Prevention in 2009 - 2010: S. aureus (24.1%), P. aeruginosa (16.6%), Klebsiella species (10.1%), Enterobacter species (8.6%), Acinetobacter baumannii (6.6%) and E. coli (5.9%) (Infect Control Hosp Epidemiol 2013;34:1)
  • Streptococcus pneumoniae
    • Gram positive, diplococci, lancet shaped, facultative anaerobe
    • 100 known serotypes
    • Vaccine containing capsular polysaccharide available for common serotypes
  • Haemophilus influenza
    • Gram negative coccobacillus, can be encapsulated (typeable) or unencapsulated (nontypeable)
    • 6 serotypes based on capsular polysaccharide
    • Vaccine available for H. influenzae type b (Hib), the most virulent serotype
  • Staphylococcus aureus
    • Gram positive cocci
    • Important pathogen in HAP / VAP
    • Emerging cause of CAP over the past 2 decades (Semin Respir Crit Care Med 2020;41:470)
      • Panton-Valentine leukocidin (PVL): important virulence factor identified in community associated methicillin resistant S. aureus
      • Toxin causing lysis of leukocytes and necrosis of epithelial cells
    • Common coinfection with influenza virus
  • Klebsiella pneumoniae
    • Gram negative bacilli, facultative anaerobic, member of Enterobacteriaceae family
    • Thick, mucoid appearing sputum is characteristic
  • Pseudomonas aeruginosa
    • Gram negative bacilli, strictly aerobic
    • Important pathogen in cystic fibrosis patients and HAP / VAP
  • Moraxella catarrhalis
Diagrams / tables

Images hosted on other servers:

Lung immune system

Specimen handling

Organisms causing CAP

Hospitalization rate

Clinical features
  • Common signs and symptoms in CAP: dyspnea, cough, fever, chills and pleuritis (Lancet 2015;386:1097)
    • Elderly people: less evident symptoms (e.g., an altered state of consciousness, gastrointestinal discomfort and fever can be absent)
  • HAP / VAP presentation: increasing oxygen requirements, leukocytosis and secretions in the intensive care unit (Cleve Clin J Med 2020;87:633)
    • Suggestive scenarios: respiratory decline accompanied by fever and a productive cough, respiratory decline after a witnessed or suspected aspiration event in the hospital
Diagnosis
  • CAP (Am J Respir Crit Care Med 2019;200:e45):
    • Suggestive clinical features and compatible chest radiograph or other imaging technique
    • Microbiological data is not required for the diagnosis of pneumonia
      • Specific pathogens should be investigated when the testing result would significantly alter standard (empirical) management decisions
  • HAP / VAP (Clin Infect Dis 2016;63:e61):
    • Noninvasive sampling in suspected HAP: spontaneous expectoration, sputum induction, nasotracheal suctioning (uncooperative patient) and endotracheal aspiration (subsequent mechanical ventilation)
    • VAP should be diagnosed by noninvasive sampling (i.e., endotracheal aspiration) with semiquantitative cultures, rather than invasive sampling (i.e., bronchoscopy, blind bronchial sampling) with quantitative cultures and noninvasive sampling with quantitative cultures
  • Pneumonia in tissue specimens:
    • Occasionally diagnosed in surgical specimens as an accompanying disease
    • Much more commonly diagnosed at autopsy
    • Rarely diagnosed on biopsy tissue sampling
    • Purulent inflammation in respiratory cytology specimen (e.g., bronchoalveolar lavage [BAL], pleural effusion) is suggestive of bacterial pneumonia
Laboratory
  • Bacterial culture from respiratory specimens is required for proper treatment and epidemiological data (nosocomial infection):
    • Sputum
    • Invasive respiratory tract sample: bronchoalveolar lavage
    • Others: pleural fluid, lung tissue
  • Multiplex polymerase chain reaction (PCR) is commercially available for detection of common respiratory pathogens, including virus and bacteria (J Microbiol Immunol Infect 2019;52:920)
  • Rapid nasal swab for methicillin resistant Staphylococcus aureus (MRSA) PCR can be used to guide therapy (Antimicrob Agents Chemother 2014;58:859)
  • Tissue culture at autopsy (J Clin Microbiol 2014;52:1028):
    • Specimen should be obtained with sterile forceps and scalpel to avoid contamination and within 24 - 48 hours of death
    • Lung tissue specimens should be obtained with the organs being in situ and the organ surface should be sterilized
    • Even with the above precautions, cultures obtained during the autopsy procedure have limited value because of the high possibility of contamination and postmortem bacterial transmigration
      • Contamination is likely to occur from flora in upper respiratory tract migrating through bronchial secretion
      • Up to 50% tested positive for some organism(s), despite the lack of any further pathological evidence of infection
  • Blood culture:
  • Complete blood count: leukocytosis (white cells > 10,000/uL)
  • Urinary antigen for Streptococcus pneumoniae:
Radiology description
  • Common signs of bacterial pneumonia (AJR Am J Roentgenol 2014;202:479):
    • Consolidation:
      • Alveolar filling process that replaces air within the affected airspaces
      • Increasing in pulmonary attenuation and obscuring the margins of adjacent airways and vessels
    • Air bronchogram:
      • Visible air filled bronchi surrounded by dense, consolidated lung parenchyma
      • Normal lung: air filled bronchi are not visible because they are surrounded by aerated lung parenchyma
      • Differential diagnosis: nonobstructive atelectasis, aspiration and neoplasms
    • Silhouette sign:
      • Loss of a normal lung - soft tissue interface (loss of silhouette)
      • Commonly applied to the interface between the lungs and the heart, mediastinum, chest wall and diaphragm
      • Caused by any pathologic mechanism that replaces or displaces air within the lung parenchyma
    • Tree in bud opacity:
      • Visible small airways or terminal bronchioles filled with mucus, pus, fluid or cells, forming impactions that resemble a budding tree with branching nodular V and Y shaped opacities
    • Split pleura sign:
      • Visible thickened visceral and parietal pleura with fluid collection in between
      • Suggests the presence of empyema
  • Typical appearance of pneumonia on chest radiograph and CT scan (Diagn Interv Imaging 2012;93:431):
    • Lobar pneumonia: subpleural area of alveolar consolidation with blurred margins, which is restricted to the area next to the fissures, then progresses to a systematized segmental opacity affecting 1 or several contiguous segments or a lobe
    • Bronchopneumonia: centrilobular micronodules with blurred margin, areas of ground glass opacity or peribronchiolar consolidation with an acinar pattern and later progress to lobular, segmental or lobar consolidation
    • Accompanied by pleural effusions (20 - 60% of bacterial pneumonia)
  • Aspiration pneumonia (J Crit Care 2015;30:40):
    • New chest radiograph infiltrate in a dependent pulmonary segment
    • Bed bound patient: posterior segments of the upper lobes and the superior segments of the lower lobes
    • Ambulatory patient: lower lobes, especially the right lung
Radiology images

Images hosted on other servers:

Lobar pneumonia

Bronchopneumonia

Bronchopneumonia


Tree in bud opacity

Necrosis and cavitation

Pneumonia, hematogenous spread

Aspiration pneumonia

Prognostic factors
  • ATS / IDSA criteria for severe CAP patients who require admission to an intensive care unit (Am J Respir Crit Care Med 2019;200:e45):
    • Major criteria:
      • Septic shock with need for vasopressor
      • Respiratory failure requiring mechanical ventilation
    • Minor criteria - at least 3 of the following:
      • Altered mental status
      • Hypotension requiring fluid support
      • Temperature: < 36 °C (96.8 °F)
      • Respiratory rate: ≥ 30 breaths/minute
      • PaO2/FiO2 ratio: ≤ 250
      • Blood urea nitrogen: ≥ 20 mg/dL (blood urea 7 mmol/L)
      • Leukocyte count: < 4,000/uL
      • Platelet count: < 100,000/uL
      • Multilobar infiltrates
  • Pneumonia severity index (PSI): prognostic tool for the evaluation of immunocompetent patients with CAP (N Engl J Med 1997;336:243)
  • CURB-65 score is a prognostic tool based on 5 factors (Thorax 2003;58:377):
    • Confusion (disorientation to person, place or time)
    • Blood urea nitrogen: > 7 mmol/L (20 mg/dL)
    • Respiratory rate: ≥ 30/minute
    • Blood pressure: systolic < 90 mmHg or diastolic ≤ 60 mmHg
    • Age: ≥ 65 years
  • Risk factors associated with increased mortality rate in HAP / VAP (Clin Infect Dis 2016;63:e61):
    • Multidrug resistant pathogen
    • Bacteremia
    • Inadequate / inappropriate antibiotic therapy
Case reports
Treatment
  • Antibiotics for CAP (Am J Respir Crit Care Med 2019;200:e45):
    • Healthy outpatient adults: amoxicillin, doxycycline or macrolide (azithromycin, clarithromycin)
    • Outpatient adults with comorbidities:
      • Combination therapy (amoxicillin / clavulanate, cefpodoxime or cefuroxime and macrolide)
      • Monotherapy (levofloxacin, moxifloxacin or gemifloxacin)
    • Inpatient:
      • Combination therapy (ampicillin + sulbactam, cefotaxime, ceftriaxone or ceftaroline and macrolide)
      • Monotherapy (levofloxacin or moxifloxacin)
  • Antibiotics for HAP / VAP (Clin Infect Dis 2016;63:e61):
    • Piperacillin / tazobactam, cefepime, levofloxacin or imipenem + meropenem
      • Plus vancomycin of linezolid if methicillin resistant S. aureus is likely
    • VAP: consider antibiotics coverage for S. aureus, P. aeruginosa and other gram negative bacilli
  • Respiratory support
Gross description
  • Lobar pneumonia:
    • Characteristic for S. pneumoniae and K. pneumoniae
    • Uniform involvement of the whole lobe
    • Increased weight of the lung
    • 4 stages of inflammatory response (Kumar: Robbins & Cotran Pathologic Basis of Disease, 10th Edition, 2020):
      • Congestion: heavy, boggy, red lung
      • Red hepatization: red, firm and airless, with liver-like consistency
      • Gray hepatization: grayish brown color on cut
      • Resolution
    • Slimy mucoid appearance is characteristic for Klebsiella spp.
  • Bronchopneumonia:
    • Consolidation may be confined to 1 lobe or involve multiple lobes, poorly defined, gray-red to yellow in color
  • Cavitary lesion
    • Common in S. aureus, P. aeruginosa
Gross images

Images hosted on other servers:
Lobar pneumonia - various images Lobar pneumonia - various images

Lobar pneumonia

Bronchopneumonia - various images Bronchopneumonia - various images

Bronchopneumonia

Bronchopneumonia - various images Bronchopneumonia - various images

Bronchopneumonia


Lobar pneumonia - various images

Bronchopneumonia

Lobar pneumonia - various images

Streptococcus pneumoniae

Lobar pneumonia - various images

Staphylococcus aureus

Lobar pneumonia - various images

Streptococcus pyogenes

Lobar pneumonia - various images

Klebsiella pneumoniae

Lobar pneumonia - various images

Pseudomonas aeruginosa

Microscopic (histologic) description
  • Lobar pneumonia (Semin Diagn Pathol 2017;34:498):
    • Uniform inflammatory infiltrate, the changes are at the same stage throughout the entire lobe
    • Early stage: vascular engorgement, intra-alveolar fluid with few neutrophils and often bacterial colonies
    • Massive confluent exudate with intra-alveolar neutrophils, red cells and fibrin, correlates with red hepatization on gross exam
    • Progressive disintegration of red cells and the persistence of a fibrinosuppurative exudate, correlates with gray hepatization on gross exam
    • Resolution phase: exudates converted to fibromyxoid masses rich in macrophages and fibroblasts
    • Usually resolves with minimal fibrosis
    • Pleuritis can be seen
  • Bronchopneumonia:
    • Most common pattern of pulmonary infection
    • Different stages in the different areas
    • Patchy intra-alveolar fibrinopurulent exudate with neutrophils
  • Acute lung injury pattern:
  • Necrotizing pneumonia (Can Respir J 2014;21:239):
    • Characterized by necrotizing inflammation, leading to alveolar septa disruption and cavity formation
    • Common organisms: Staphylococcus aureus, Streptococcus pyogenes, S. pneumoniae (certain serotypes), Klebsiella, Acinetobacter, Pseudomonas and Burkhodoria
  • Aspiration pneumonia:
    • Foreign body giant cell reaction, characterized by multinucleated giant cells, granulomatous inflammation
    • Often necrotizing, abscess formation is common
    • Presence of food particles (e.g., lentils, vegetables, pill fragments)
Microscopic (histologic) images

Contributed by Sakda Sathirareuangchai, M.D. and Yuri Tachibana, M.D.

Intra-alveolar neutrophils

Acute bronchiolitis

Bacterial colonies

Diffuse alveolar damage

Granulomatous inflammation


Foreign body

Foreign body

Necrotizing inflammation

Alveolar exudate

Alveolar exudate

Virtual slides

Images hosted on other servers:

Acute
bronchopneumonia

Acute
bronchopneumonia
with DAD

Aspiration pneumonia

Cytology description
  • Bacterial pneumonia is rarely diagnosed by cytology alone
    • Sources of specimens: sputum, bronchoalveolar lavage, fine needle aspirate (FNA), pleural fluid
  • Acute purulent inflammation, with predominant neutrophils (Practical Pulmonary Pathology 2018;147)
  • Reactive pneumocytes can be mistaken for malignant cells
  • Bacteria can be seen with Diff-Quik preparation
    • Colonization must be differentiated from true infection
Cytology images

Images hosted on other servers:

S. pneumoniae (sputum, Gram stain)

P. aeruginosa (sputum, Gram stain)

BAL, purulent exudate

FNA, purulent exudate

FNA, gram negative bacilli

Positive stains
  • Gram stain:
    • 2 common methods in histology:
      • Brown and Brenn
      • Brown and Hopps
    • Gram positive bacteria stains blue, gram negative bacteria stains red
Negative stains
Videos

Bronchopneumonia

Lobar pneumonia

Sample pathology report
  • Lung, right middle lobe, transbronchial biopsy:
    • Acute bronchopneumonia (see comment)
    • Diffuse alveolar damage
    • Comment: Special stains for microorganisms (AFB and GMS) are negative.
  • Autopsy report
    • Final pathologic diagnosis
      • Acute bronchopneumonia involving right upper lobe
      • Diffuse alveolar damage
    • Microscopic description
      • Lungs: Sections of the lungs show intra-alveolar neutrophilic infiltrate with marked congestion. Alveolar septa are widened with mixed inflammatory cells. Reactive pneumocytes and hyaline membrane can be seen throughout the lung tissue.
Differential diagnosis
Board review style question #1

An 80 year old man presented with dyspnea, altered mental status and evidence of urinary tract infection. He was hospitalized in an intensive care unit for 2 weeks. The patient later developed fever, hypoxemia and new infiltrate on chest radiograph. An autopsy was performed and the lung showed the above histomorphology. What is the likely diagnosis?

  1. Bronchopneumonia
  2. Mycoplasma pneumonia
  3. Pulmonary tuberculosis
  4. Respiratory bronchiolitis
  5. Usual interstitial pneumonia
Board review style answer #1
A. Bronchopneumonia

Comment Here

Reference: Pneumonia
Board review style question #2
Which organism characteristically causes lobar pneumonia?

  1. Aspergillus fumigatus
  2. Escherichia coli
  3. Influenza virus
  4. Staphylococcus aureus
  5. Streptococcus pneumoniae
Board review style answer #2
E. Streptococcus pneumoniae

Comment Here

Reference: Pneumonia

Adenocarcinoma in situ
Definition / general
  • Preinvasive lung adenocarcinoma
  • Size: ≤ 30.0 mm
  • Pure lepidic growth pattern with no invasion (neoplastic cells along pre-existing alveolar structures)
  • Diagnosed only when all tumor has been sampled
  • Diagnosis cannot be made in small biopsy or cytology specimen
Essential features
  • Lepidic growth with no stromal, vascular or pleural invasion
  • Size: ≤ 30.0 mm
  • Diagnosed only after complete sampling of a resected lesion
Terminology
  • Bronchioloalveolar carcinoma (historical term, obsolete)
ICD coding
  • ICD-O:
    • 8250/2 - adenocarcinoma in situ, nonmucinous
    • 8253/2 - adenocarcinoma in situ, mucinous
Epidemiology
Sites
Pathophysiology
  • Multistep progression model (Ann Oncol 2015;26:156, Int J Mol Sci 2018;19:1259)
    • Adenocarcinoma in situ (AIS) is the step between atypical adenomatous hyperplasia (AAH) and minimally invasive carcinoma (MIA)
    • Incidence of EGFR mutations increases from atypical adenomatous hyperplasia → adenocarcinoma in situ → minimally invasive carcinoma
    • KRAS and BRAF mutations do not show same progression, suggesting other molecular alterations are also involved in tumor evolution
  • EPPK1, KMT2C, KMT2D, NOTCH3 and NF1 mutations may present as early events in the progression of lung adenocarcinoma (Am J Respir Crit Care Med 2020;201:697)
Etiology
  • Same as invasive lung adenocarcinoma
    • #1 risk factor is tobacco smoking
    • Possible risk factors in never smokers: secondhand smoke, radon, occupational exposures, air pollution
    • Most cases of lung cancer in never smokers are idiopathic, with different mutational profile than smoking related lung cancer
  • Reference: J Thorac Oncol 2012;7:1352
Clinical features
  • Often incidental
  • May occur alone or along with invasive adenocarcinoma as a separate focus
  • Slow growing
Diagnosis
  • May or may not be seen on CT or MRI, depending on size and association with scar
  • Radiologic features that suggest indolence (Ann Am Thorac Soc 2015;12:1193):
    • Small size
    • Longer volumetric doubling time (> 400 days)
    • Maximum standardized uptake value (SUV) < 1
  • Definitive diagnosis requires excision and histopathologic evaluation
Radiology description
Radiology images

Images hosted on other servers:

AIS as ground glass lesion

Prognostic factors
Case reports
Treatment
Gross description
Gross images

Contributed by Atreyee Basu, M.D. and Fang Zhou, M.D.

Firm, tan, ill defined lesion

Frozen section description
  • Lepidic pattern adenocarcinoma with mild to moderate cytologic atypia
  • Do not diagnose as adenocarcinoma in situ on frozen section; invasion may be present on deeper levels or in the remainder of the tumor that was not submitted for frozen section
  • If no invasion is seen on frozen, one may call it adenocarcinoma with the lepidic pattern on 1 representative section, pending evaluation of permanent sections (or the remainder of the tumor)
  • Reference: J Clin Pathol 2016;69:1076
Frozen section images

Contributed by Atreyee Basu, M.D. and Fang Zhou, M.D.

Neoplastic cells, lepidic pattern

Microscopic (histologic) description
  • Lepidic growth pattern only: back to back neoplastic cells growing along pre-existing alveolar structures only
  • No stromal, vascular or pleural invasion; no necrosis
  • Diagnosed only in resections after complete sampling
  • Cannot be diagnosed in small biopsies, cytology or frozen sections (differential diagnosis minimally invasive carcinoma and invasive adenocarcinoma)
  • Size: ≤ 30.0 mm
  • Nonmucinous adenocarcinoma in situ = mild to moderate cytologic atypia, consisting of various combinations of the following features: nuclear membrane irregularity, intranuclear pseudoinclusions, nuclear grooves, hyperchromasia, anisocytosis, small nucleoli, increased nuclear to cytoplasmic ratio, hobnailing
    • Not all features may be present
  • Mucinous type adenocarcinoma in situ = extremely rare; mucinous tumors are usually associated with invasion
    • Mucinous cells show minimal atypia with abundant intracellular mucin and basally oriented nuclei
  • References: J Thorac Oncol 2016;11:1204, Int J Mol Sci 2018;19:1259
Microscopic (histologic) images

Contributed by Atreyee Basu, M.D. and Fang Zhou, M.D.

Neoplastic cells, lepidic pattern

Neoplastic cells, nuclear atypia

Cytology description
  • Mild to moderate cytologic atypia
  • Nuclear membrane irregularity, nuclear grooves and intranuclear pseudoinclusions (resembles papillary thyroid carcinoma cytology)
  • Definitive diagnosis requires excision
Cytology images

Contributed by Atreyee Basu, M.D. and Fang Zhou, M.D.

Nuclear irregularities and anisocytosis

Nuclear pseudoinclusions and grooves

Nuclear atypia

TTF1 immunostain

Positive stains
Negative stains
Molecular / cytogenetics description
Videos

Adenocarcinoma in situ
by Dr. Thomas Colby

Sample pathology report
  • Lung, middle lobe, right, lobectomy:
    • Adenocarcinoma in situ (pTis) (see synoptic report)
    • Tumor size: 25.0 mm
    • Bronchial and vascular margins are negative for carcinoma

  • Lung, right upper lobe; biopsy:
    • Adenocarcinoma with lepidic pattern
  • Note: Adenocarcinoma in situ cannot be definitely diagnosed until the entire tumor is examined and invasive adenocarcinoma ruled out. Thus, on small biopsy specimens, potential cases of adenocarcinoma in situ are signed out as above.
Differential diagnosis


Table 1: Differential diagnosis of lepidic predominant lung tumors
Overall size ≤ 30.0 mm Overall size > 30.0 mm

Lepidic pattern only

Adenocarcinoma in situ
Adenocarcinoma, lepidic predominant
(stage as pT1a)
Invasion ≤ 5.0 mm Minimally invasive adenocarcinoma Adenocarcinoma, lepidic predominant
Invasion > 5.0 mm Adenocarcinoma, lepidic predominant Adenocarcinoma, lepidic predominant
Board review style question #1

What is the most commonly mutated gene associated with adenocarcinoma in situ?

  1. ALK
  2. BRCA
  3. EGFR
  4. PIK3CA
Board review style answer #1
Board review style question #2
On a resection specimen in which the tumor is entirely submitted, there is a 28.0 mm tumor with 2.0 mm of stromal invasion (acinar pattern). What is the diagnosis?

  1. Acinar predominant with lepidic adenocarcinoma
  2. Adenocarcinoma in situ
  3. Lepidic predominant with acinar adenocarcinoma
  4. Minimally invasive carcinoma
Board review style answer #2
D. Minimally invasive carcinoma

Comment Here

Reference: Adenocarcinoma in situ

Adenocarcinoma overview
Definition / general
  • Non-small cell lung carcinoma with glandular differentiation, mucin production or pneumocyte marker expression
Essential features
  • Most prevalent non-small cell lung carcinoma
  • 5 main histologic patterns (acinar, papillary, micropapillary, lepidic, solid); mucinous and nonmucinous subtypes
  • Positive for TTF1
Terminology
  • Terminology of lung adenocarcinoma was significantly revised in the 2015 WHO classification (J Thorac Oncol 2015;10:1243)
    • Discontinuation of the terms bronchioloalveolar carcinoma (BAC) and mixed subtype adenocarcinoma
    • Addition of adenocarcinoma in situ (AIS) as a preinvasive lesion to join atypical adenomatous hyperplasia
    • Addition of minimally invasive adenocarcinoma
    • Use of the term lepidic for a noninvasive component (previously classified as BAC) of an invasive adenocarcinoma
    • Introduction of the term invasive mucinous adenocarcinoma for adenocarcinomas formerly classified as mucinous BAC, excluding tumors that meet criteria for AIS or minimally invasive adenocarcinoma (MIA)
    • Discontinuation of the subtypes of clear cell and signet ring adenocarcinoma
    • Discontinuation of the term mucinous cystadenocarcinoma and inclusion of these under the category of colloid adenocarcinoma
ICD coding
  • ICD-O: 8046/3 - non-small cell carcinoma of bronchus or lung (international code)
  • ICD-10: C34.90 - malignant neoplasm of unspecified part of unspecified bronchus or lung
Epidemiology
Sites
Pathophysiology
Etiology
Diagrams / tables

Images hosted on other servers:
Histologic subtyping for surgeon

Histologic subtyping for surgeon

Histologic pattern and prognosis

Histologic pattern and prognosis

Treatment

Treatment

Grading of invasive nonmucinous adenocarcinomas

Grading of invasive nonmucinous adenocarcinomas

Clinical features
  • Cough (productive if mucinous adenocarcinoma), hemoptysis, dyspnea, weight loss, chest pain (Chest 2012;142:1338)
  • Paraneoplastic / endocrine syndromes are much less common than in small cell lung carcinoma
    • Hypertrophic pulmonary osteoarthropathy with clubbing of the fingers, symmetric polyarthritis, periostitis of the long bones (World J Clin Oncol 2014;5:197)
Diagnosis
  • Histological, based on morphology and staining pattern
Radiology description
Radiology images

Image hosted on other servers:
Mucinous adenocarcinoma

Mucinous adenocarcinoma

Adenocarcinoma on axial CT Adenocarcinoma on axial CT

Adenocarcinoma on axial CT

Prognostic factors
Case reports
Treatment
Gross description
Gross images

Contributed by Yale Rosen, M.D.
Peripheral adenocarcinoma

Peripheral adenocarcinoma



Images hosted on other servers:
Peripheral adenocarcinoma

Peripheral adenocarcinoma

Frozen section description
Frozen section images

Contributed by Caroline I.M. Underwood, M.D.
Mucinous lung adenocarcinoma

Mucinous lung adenocarcinoma

Microscopic (histologic) description
Microscopic (histologic) images

Contributed by Caroline I.M. Underwood, M.D., Andrey Bychkov, M.D., Ph.D., Fulvio Lonardo, M.D. and Negar Rassaei, M.D.
Adenocarcinoma in situ

Adenocarcinoma in situ

Acinar pattern

Acinar pattern

Lepidic pattern, architecture

Lepidic pattern, architecture

Lepidic pattern, cytologic features

Lepidic pattern, cytologic features

Micropapillary pattern

Micropapillary pattern


Papillary pattern Papillary pattern

Papillary pattern

Papillary pattern

Papillary pattern

Solid pattern, architecture

Solid pattern, architecture

Solid pattern, cytologic features

Solid pattern, cytologic features


Mucinous subtype, architecture

Mucinous subtype, architecture

Mucinous subtype, cytologic features

Mucinous subtype, cytologic features

Positive PDL1

Negative PDL1

Negative PDL1

ADC cocktail

Virtual slides

Images hosted on other servers:
Resection specimen

Resection specimen

Core needle biopsy

Core needle biopsy

Cytology description
  • 3D clusters of cohesive cells, foamy / vacuolated cytoplasm, fine chromatin, variable prominent nucleoli (J Thorac Oncol 2011;6:244)
  • Usually on pleural effusions or needle washes
Cytology images

Images hosted on other servers:
Large malignant cells

Large malignant cells

Positive stains
Electron microscopy description
Molecular / cytogenetics description
Molecular / cytogenetics images

Contributed by Ankur R. Sangoi, M.D. (Case #425)
<i>ALK</i> break apart FISH images (<i>ALK</i> gene in RED) <i>ALK</i> break apart FISH images (<i>ALK</i> gene in RED)

ALK break apart FISH images

Sample pathology report
  • Lung, left upper lobe, wedge resection:
    • Invasive adenocarcinoma, grade 2, acinar predominant with secondary solid growth pattern (see synoptic report)
Differential diagnosis
Board review style question #1

A 59 year old man presents with cough, hemoptysis and shortness of breath. A mass in his left lung was biopsied. Which of the following statements about this disease is true?

  1. Exposure to benzene is an important risk factor in the development of this disease
  2. Masses are most frequently found in central / hilar regions of both lungs
  3. The growth pattern indicated in the patient's biopsy above is a poor prognostic factor
  4. The most common site of metastasis is the liver
  5. This disease has a higher incidence in men than in women
Board review style answer #1
C. The growth pattern indicated in the patient's biopsy above is a poor prognostic factor. The H&E stain demonstrates a micropapillary pattern, which is a poor prognostic factor. Lung adenocarcinoma is associated with exposure to radon (not benzene) and is typically found in the peripheral regions or upper lobes of the lung. This cancer is also more common in women than in men and most frequently metastasizes to the brain, not the liver.

Comment Here

Reference: Adenocarcinoma overview
Board review style question #2
A 63 year old woman presented with mass in the upper lobe of her left lung with enlargement of the mediastinal lymph nodes. Surgical biopsy shows columnar tumor cells with abundant intracytoplasmic mucin in an acinar growth pattern. The malignant cells are most likely to be positive for which of the following mutations?

  1. ALK rearrangement
  2. BRAF
  3. EGFR
  4. HER2 amplification
  5. KRAS
Board review style answer #2
E. KRAS is the most common mutation associated with mucinous adenocarcinoma (76%). EGFR mutations are rare in the mucinous subtype. ALK, HER2 and BRAF are uncommon mutations overall.

Comment Here

Reference: Adenocarcinoma overview

Adenoid cystic carcinoma
Definition / general
  • #2 most common salivary gland-type carcinoma of lung
  • Usually in large bronchi, may involve the trachea
  • Prolonged course, but overall prognosis is poor
Essential features
  • See also Salivary glands - Adenoid cystic carcinoma
  • Primary pulmonary adenoid cystic carcinoma is rare, and metastasis from salivary glands must be excluded
  • Morphology is similar to adenoid cystic carcinomas in other sites, with cribriform, tubular and solid patterns
  • These tumors tend to arise in association with central airways and spread along neurovascular bundles
Terminology
  • Formally called bronchial adenoma, but now considered malignant
ICD coding
  • Use code specific for location of tumor
  • C34.90 Malignant neoplasm of unspecified part of unspecified bronchus or lung
Epidemiology
  • Primary pulmonary adenoid cystic carcinoma is very rare, < 0.2% of lung cancers
  • Typically adults
Sites
  • Usually central / endobronchial but may be peripheral
Pathophysiology
  • Slow growing but persistent, with recurrences over years, potentially with metastasis to lymph nodes and distant sites
Etiology
  • Unclear, probably arise from submucosal bronchial glands
Clinical features
  • Obstructive symptoms, i.e., pneumonia, dyspnea, cough, wheeze, hemoptysis
  • Peripheral lesions asymptomatic
Diagnosis
  • Exclude metastasis from salivary glands
Radiology images

Images hosted on other servers:
Adenoid cystic carcinoma<br>obstructing right upper<br>lobar bronchus

Adenoid cystic carcinoma
obstructing right upper
lobar bronchus

Prognostic factors
  • Variable by tumor stage
Case reports
Treatment
Clinical images

Images hosted on other servers:
Various images

Various images

Gross description
  • Large, centrally located, polypoid, intrabronchial mass
  • May grow along bronchi (subepithelial) causing thickened bronchial wall
  • Circumscribed, soft, yellowish white
Gross images

Images hosted on other servers:
75 year old woman with 1 cm lung lesion

75 year old woman with 1 cm lung lesion

Microscopic (histologic) description
  • Propensity for tracking along nerves and cartilaginous plates → bronchial margins more often positive than in other lung cancers
  • Infiltrative growth, cribriform / cylindromatous (islands and nests, with luminal matrix), tubular (gland-like spaces) or solid (insular, with scant matrix) - usually a mix of patterns are seen
  • Defining features are pseudocysts (rounded extracellular space containing basal lamina), intercellular spaces, basal lamina and true glandular lumens (Hum Pathol 1982;13:916)
  • Monotonous, polygonal, basaloid cells
  • Absence of mitoses, nuclear pleomorphism, necrosis and hemorrhage in most cases; solid type may show more mitoses
Microscopic (histologic) images

Contributed by Yale Rosen, M.D. and @ThatGlassTho on Twitter
Adenoid cystic carcinoma Adenoid cystic carcinoma Adenoid cystic carcinoma Adenoid cystic carcinoma

Adenoid cystic carcinoma

Adenoid cystic carcinoma Adenoid cystic carcinoma

Adenoid cystic carcinoma



Images hosted on other servers:
Cribriform-like structures with mucus

Cribriform-like structures with mucus

Focal tubular structure

Focal tubular structure

Various images

Various images

14 year old girl with 11 cm lung mass

14 year old girl with 11 cm lung mass


75 year old woman with 1 cm lung lesion (H&E, TTF1) 75 year old woman with 1 cm lung lesion (H&E, TTF1) 75 year old woman with 1 cm lung lesion (H&E, TTF1)

75 year old woman with 1 cm lung lesion (H&E, TTF1)

Cytology description
  • Cylinders or spheres of myxochondroid matrix within epithelial groups
  • Diagnosis more difficult if matrix is scarce, as in solid type
  • Cellular uniformity, distinct nucleolus, granular cytoplasm, distinct cell border, organoid cluster, hyaline globule and hyaline basement membrane material (J Pathol Transl Med 2015;49:511)
Cytology images

Images hosted on other servers:
Cytomorphology of pulmonary ACC

Cytomorphology of pulmonary adenoid cystic carcinoma

Immunohistochemistry & special stains
  • Usually not necessary for diagnosis
  • Luminal cells: c-kit/CD117+, p63-, actin-
  • Basal (myoepithelial) cells: c-kit/CD117-, p63+, actin+
  • Tumor cells are usually positive for keratin and S100, and negative for neuroendocrine markers
  • Basement membrane material is positive for collagen type IV or laminin
Electron microscopy description
  • May show evidence of partial myoepithelial differentiation
Molecular / cytogenetics description
Differential diagnosis

Adenoma (pending)
[Pending]

Adenosquamous
Definition / general
  • Malignant neoplasm showing components of both squamous cell carcinoma and adenocarcinoma (each comprising at least 10% of the tumor)
Essential features
  • WHO classification as non-small cell lung carcinoma with at least 10% components of both adenocarcinoma and squamous cell carcinoma by histomorphology that may be separate or intermingled
  • Diagnosis made on resection specimens, although it may be suspected on biopsy or cytology
  • May harbor targetable molecular alterations seen in adenocarcinomas
  • Rare incidence; 0.4 - 4% of all lung cancers
  • Clinical presentation frequently resembles other non-small cell carcinomas but it appears to have an adverse prognosis (Oncotarget 2017;9:8133)
  • Computed tomography (CT) scan shows central or peripheral location
  • Centrally located tumors commonly show postobstructive inflammatory changes
  • Peripheral tumors (90%) appear as ground glass opacities, often associated with scars
Terminology
  • By WHO classification, adenosquamous carcinoma contains at least 10% each of malignant squamous and glandular components
  • Differentiate from adenocarcinoma with squamoid features / squamous metaplasia as well as squamous cell carcinoma with pseudoglandular features
ICD coding
  • Use code specific for location of tumor
  • ICD-10: C34.90 - malignant neoplasm of unspecified part of unspecified bronchus or lung
Epidemiology
Sites
  • Tends to arise peripherally but may be central
Pathophysiology
  • More aggressive than pure carcinomas when matched for stage
  • Molecular mechanisms are not fully understood
  • Environmental factors are possible contributing factors
  • Chronic inflammation may influence pathogenesis
  • Aggressive behavior: more aggressive than adenocarcinoma or squamous cell carcinoma, leading to poorer prognosis
  • Metastatic potential: high risk of spreading to lymph nodes and distant organs
Etiology
  • Association with smoking, similar to other lung cancers
Diagrams / tables
N/A
Clinical features
Diagnosis
  • Diagnosis made on resection specimens, though can be suggested in biopsy or cytology as follows
    • Non-small cell carcinoma (see note)
    • Note: glandular and squamous components are present; the findings could represent adenosquamous carcinoma
Laboratory
N/A
Radiology description
Radiology images
N/A
Prognostic factors
  • Resectable adenosquamous carcinoma patients had higher pleural invasion incidence, poorer differentiation and worse survival compared with squamous cell carcinoma and adenocarcinoma patients (Jpn J Clin Oncol 2022;52:1456)
  • EGFR mutated adenosquamous carcinomas appear to have better prognosis (Pathol Int 2013;63:77)
  • Adenosquamous carcinomas without EGFR mutations show a moderate response to immune checkpoint inhibitor (ICI) based immunotherapy (Front Immunol 2022:13:944812)
  • High CXCR4 expression predicts a poor prognosis in resected lung adenosquamous carcinoma (J Cancer 2020;11:810)
  • Adenocarcinomatous predominant subtype of adenosquamous carcinoma of the lung (ASC) is associated with a better prognosis than the squamous cell carcinoma predominant subtype (J Cancer 2020;11:810)
Case reports
Treatment
Clinical images
N/A
Gross description
N/A
Gross images
N/A
Frozen section description
N/A
Frozen section images
N/A
Microscopic (histologic) description
  • Malignant cells with squamous or glandular differentiation with variable degree of admixture
  • Squamous component may show intercellular bridges or keratin pearls
  • Glandular components may show lumens with mucin; may have papillary, lepidic, acinar or tubular patterns
  • Metastasis in lymph node tend to be same histologic type as the major component but not necessarily (Cancer 1991;67:649)
Microscopic (histologic) images

Contributed by Roseann Wu, M.D., M.P.H. and Yale Rosen, M.D.
Glandular and squamous features Glandular and squamous features

Glandular and squamous features

Adenosquamous carcinoma Adenosquamous carcinoma Adenosquamous carcinoma

Adenosquamous carcinoma

Virtual slides

Images hosted on other servers:
Rosai collection: adenosquamous carcinoma in lung Rosai collection: adenosquamous carcinoma in lung Rosai collection: adenosquamous carcinoma in lung

Adenosquamous carcinoma in lung

Leeds University collection: 63 year old female, right lobectomy Leeds University collection: 63 year old female, right lobectomy

63 year old woman, right lobectomy

Cytology description
  • Variable depending on squamous or glandular components and degree of differentiation
Cytology images

Contributed by Roseann Wu, M.D., M.P.H.
Glandular and squamous features Glandular and squamous features Glandular and squamous features

Glandular and squamous features



Images hosted on other servers:
Adenosquamous carcinoma Adenosquamous carcinoma Adenosquamous carcinoma Adenosquamous carcinoma Adenosquamous carcinoma

Adenosquamous carcinoma

Immunofluorescence description
N/A
Immunofluorescence images
N/A
Positive stains
  • TTF1 in adenocarcinoma component and p63 and CK5/6 in squamous component
  • p40 (sometimes focally expressed in a subset of adenocarcinoma)
  • CK7
Negative stains
Electron microscopy description
  • Features of both squamous cell carcinoma and adenocarcinoma (ADC) are present in adenosquamous carcinoma
  • Adenocarcinoma features include intracellular and intercellular lumina and short, stubby microvilli
  • Squamous cell carcinoma features include well formed desmosomes with tonofilaments and intercellular bridges
  • A proportion of primitive cells (without ultrastructural features of glandular or squamous differentiation) may be present
Electron microscopy images
N/A
Molecular / cytogenetics description
  • EGFR and KRAS mutations, both characteristic of adenocarcinoma, have been reported in adenosquamous carcinoma
  • EGFR amplification is frequent in adenosquamous carcinoma
Molecular / cytogenetics images

Images hosted on other servers:
With <i>KRAS</i> and <i>EGFR</i> mutations

With KRAS and EGFR mutations

Videos

Metastatic adenosquamous carcinoma of the lung to wrist with squamous histology

Sample pathology report
  • Lung, right lower lobe, biopsy:
    • Compatible with adenosquamous carcinoma (see comment)
    • Comment: The tumor displays both glandular and squamous differentiation, characteristic of adenosquamous carcinoma. This is a rare and aggressive subtype of non-small cell lung carcinoma. The patient's clinical presentation and radiological findings support the pathological diagnosis. Further molecular testing and staging studies may be necessary for appropriate treatment planning. Immunohistochemical staining shows positivity for thyroid transcription factor 1 (TTF1) in the adenocarcinoma component, confirming its pulmonary origin. Positivity for p40 and p63 is observed in the squamous component, confirming its squamous differentiation.
Differential diagnosis
  • Adenocarcinoma with squamoid features or foci of benign squamous metaplasia:
    • Malignant features are present only in areas of glandular differentiation
    • Lacks malignant features in the foci of squamoid differentiation
  • High grade mucoepidermoid carcinoma:
    • Central airways
    • Typical morphology shows varying proportions of epidermoid cells, intermediate cells and mucocytes
    • Overt keratinization is rare
    • Mucous cells embedded in epidermoid cell nests or lining cystic spaces
    • Intermediate cells found within epidermoid cell nests or forming separate nests
  • Squamous cell carcinoma with pseudoglandular features or entrapped epithelium:
    • Pseudoglandular features of squamous cell carcinoma are typically positive for p40, p63 and CK5/6, while negative for TTF1 and napsin A
  • Collision tumor:
    • In a collision tumor, the 2 tumor components should be clearly separated, with distinct histological features for each
    • In adenosquamous carcinoma, the glandular and squamous components are typically intermingled within the same tumor mass
Board review style question #1
Which of the following is true about pulmonary adenosquamous carcinoma?

  1. Always shows both components in metastases
  2. Has a better prognosis than pure adenocarcinoma or squamous cell carcinoma
  3. May harbor EGFR or KRAS mutations
  4. Requires immunohistochemistry for diagnosis
  5. Shows a benign squamous component
Board review style answer #1
C. May harbor EGFR or KRAS mutations. It is important to recognize pulmonary adenosquamous carcinoma, as these tumors may harbor targetable mutations seen in adenocarcinomas, such as EGFR or KRAS. Answer B is incorrect because pulmonary adenosquamous carcinoma has a worse prognosis than either adenocarcinoma or squamous carcinoma. Answer A is always incorrect because only 1 of the 2 components may be seen in metastatic sites. Answer D is incorrect because the diagnosis is made with at least 10% of each component by histology, not immunohistochemistry. Answer E is incorrect because both the adenocarcinoma and squamous cell carcinoma components are malignant.

Comment Here

Reference: Adenosquamous carcinoma
Board review style question #2
Which of the following statements regarding lung adenosquamous carcinoma is accurate?

  1. Always benign and does not exhibit metastatic potential
  2. Characterized by a glandular and squamous differentiation
  3. Commonly presents as a purely neuroendocrine tumor
  4. Exclusively composed of squamous cell carcinoma elements
  5. Primarily affects the bronchial cartilage and not the epithelial cells
Board review style answer #2
B. Characterized by a glandular and squamous differentiation. Lung adenosquamous carcinoma is a subtype of non-small cell lung cancer (NSCLC) that contains both glandular (adenocarcinoma) and squamous (squamous cell carcinoma) differentiation. This histological pattern differentiates it from other types of lung cancer. Answers A, C, D and E are incorrect statements.

Comment Here

Reference: Adenosquamous carcinoma
Board review style question #3
Which of the following clinical features is commonly associated with lung adenosquamous carcinoma, setting it apart from other lung malignancies?

  1. Limited association with smoking history
  2. Overexpression of the ALK gene fusion
  3. Paraneoplastic production of PTH related peptide
  4. Peripheral, cavitary lesions on chest Xray
  5. Strong female predominance among affected patients
Board review style answer #3
D. Peripheral, cavitary lesions on chest Xray. Lung adenosquamous carcinoma often presents with peripheral cavitary lesions on chest Xray, which can be a distinguishing feature. This radiological appearance may be indicative of necrotic or cavitated areas within the tumor. Answers A, B, C and E are not commonly associated with lung adenosquamous carcinoma.

Comment Here

Reference: Adenosquamous carcinoma

Adenosquamous
Definition / general
  • Substantial amounts of malignant squamous and glandular differentiation (at least 10% of each component within tumor)
  • 90% peripheral, often associated with scars, with clinical / imaging features similar to adenocarcinoma
  • Poorer prognosis than either component alone
Essential features
  • WHO classification as non-small cell lung carcinoma with at least 10% components of both adenocarcinoma and squamous cell carcinoma by histomorphology
  • Diagnosis made on resection specimens, although it may be suspected on biopsy or cytology
  • May harbor targetable molecular alterations seen in adenocarcinomas
Terminology
  • By WHO classification, adenosquamous carcinoma contains at least 10% each of malignant squamous and glandular components
  • Differentiate from adenocarcinoma with squamoid features / squamous metaplasia as well as squamous cell carcinoma with pseudoglandular features
ICD coding
  • Use code specific for location of tumor
  • C34.90 Malignant neoplasm of unspecified part of unspecified bronchus or lung
Epidemiology
Sites
  • Tends to arise peripherally but may be central
Pathophysiology
  • More aggressive than "pure" carcinomas when matched for stage
Etiology
  • Association with smoking, similar to other lung cancers
Clinical features
  • Similar to lung adenocarcinoma
Diagnosis
  • Diagnosis made on resection specimens, though can be suggested in biopsy or cytology as follows:
    • Non-small cell carcinoma, see note
    • Note: Glandular and squamous components are present; the findings could represent adenosquamous carcinoma
Radiology description
Prognostic factors
  • Worse than "pure" adenocarcinoma or squamous cell carcinoma; associated with larger size, visceral pleural invasion, lymphatic invasion, ipsilateral second nodules (Ann Thorac Surg 2013;95:1189)
  • EGFR mutated adenosquamous carcinomas appear to have better prognosis (Pathol Int 2013;63:77)
Case reports
Treatment
Gross description
  • Similar to adenocarcinoma but may show more aggressive features, i.e. larger size, pleural invasion
Microscopic (histologic) description
  • Malignant cells with squamous or glandular differentiation with variable degree of admixture
  • Squamous component may show keratin, intercellular bridges, squamous pearls
  • Glandular components may show lumens with mucin; may have papillary, lepidic, acinar or tubular patterns
  • Metastasis in lymph node tend to be same histologic type as the major component (Cancer 1991;67:649) but not necessarily
Microscopic (histologic) images

Contributed by Roseann Wu, M.D., M.P.H. and Yale Rosen, M.D.
Adenosquamous carcinoma Adenosquamous carcinoma Adenosquamous carcinoma

Adenosquamous carcinoma



Images hosted on other servers:
Transbronchial lung biopsy with adenocarcinoma and squamous cell carcinoma Transbronchial lung biopsy with adenocarcinoma and squamous cell carcinoma

Transbronchial lung biopsy with adenocarcinoma and squamous cell carcinoma

Adenocarcinoma forming mucous membranes and parts of the squamous epithelial arcinoma Adenocarcinoma forming mucous membranes and parts of the squamous epithelial carcinoma

Adenocarcinoma forming mucous membranes and parts of the squamous epithelial carcinoma

Virtual slides

Images hosted on other servers:
Rosai collection: adenosquamous carcinoma in lung Rosai collection: adenosquamous carcinoma in lung Rosai collection: adenosquamous carcinoma in lung

Rosai collection: adenosquamous carcinoma in lung

Leeds University collection: 63 year old female, right lobectomy Leeds University collection: 63 year old female, right lobectomy

Leeds University collection: 63 year old female, right lobectomy

Cytology description
  • Variable depending on squamous or glandular components and degree of differentiation
Cytology images

Images hosted on other servers:
Adenosquamous carcinoma Adenosquamous carcinoma Adenosquamous carcinoma Adenosquamous carcinoma Adenosquamous carcinoma

Adenosquamous carcinoma

Positive stains
Negative stains
  • Neuroendocrine markers
Molecular / cytogenetics description
Molecular / cytogenetics images

Images hosted on other servers:
With <i>KRAS</i> and <i>EGFR</i> mutations

With KRAS and EGFR mutations

Videos


Differential diagnosis
Board review style question #1
Which of the following is true about pulmonary adenosquamous carcinoma?

  1. Has a better prognosis than pure adenocarcinoma or squamous cell carcinoma
  2. Always shows both components in metastases
  3. Requires immunohistochemistry for diagnosis
  4. May harbor EGFR or Kras mutations
  5. Shows a benign squamous component
Board review style answer #1
D. May harbor EGFR or Kras mutations. It is important to recognize pulmonary adenosquamous carcinoma, as these tumors may harbor targetable mutations seen in adenocarcinomas, such as EGFR or Kras. Pulmonary adenosquamous carcinoma has a worse prognosis than either adenocarcinoma or squamous carcinoma. Only one of the two components may be seen in metastatic sites. The diagnosis is made with at least 10% of each component by histology, not immunohistochemistry. Both the adenocarcinoma and squamous cell carcinoma components are malignant.

Comment Here

Reference: Adenosquamous carcinoma

Adenovirus
Definition / general
  • Lung disease caused by infection with adenovirus, a nonenveloped, lytic, double stranded DNA virus
Epidemiology
  • At least 51 serotypes of adenovirus have been associated with clinical syndromes; not well understood why specific sterotypes are associated with specific syndromes, but differences in mode of transmission and virus tropism are likely to play a role
  • Seven species are known (groups A - G); respiratory disease is caused by groups C, E and some members of group B
  • Infection is ubiquitous; most individuals have had several infections during childhood
Sites
  • In addition to lung, also occurs in the upper respiratory tract (tonsils, adenoids), liver, GI tract, CNS, heart, kidneys, urinary bladder, ear, eyes
Clinical features
  • Half of infections are subclinical
  • Represent 5% - 10% of febrile illness in infants and young children
  • Transmission is through respiratory droplets, fomites, or fecal-oral
  • In children, causes 5% of upper respiratory tract infections and 10% of cases of pneumonia; adult infection is less common
  • Can persist as a latent infection for years after primary disease; tonsils and adenoids are likely reservoirs
  • Outbreaks among military recruits have caused epidemics of serious disease at military training centers
  • Certain subgroups are associated with severe pneumonia, especially young children
  • Virulent strains have led to fatalities in previously healthy young adults (Clin Infect Dis 2003;37:e142)
  • Serious disease has occurred in immunocompromised hosts, especially patients with solid organ and hematopoietic stem cell transplants, although infection may be associated only with asymptomatic shedding
  • Adenovirus infections are only rare causes of significant disease in AIDS patients
  • When symptomatic, upper respiratory tract infection causes fever, sore throat, cough, hoarseness, rhinorrhea
  • Disease may mimic tonsillitis with group A streptococcal infection, due to exudative pharyngitis and enlarged cervical lymph nodes
  • Otitis media may occur children under age 1 year
  • Upper respiratory tract infection may progress to involve lower respiratory tract with cough and shortness of breath; often with systemic symptoms of fever, headache, myalgias; abdominal pain may occur
  • May cause a pertussis-like syndrome
  • Chest Xray: bilateral, patchy, ground glass opacities consistent with viral pneumonia
  • In hematopoietic stem cell transplants, the GI tract is more commonly affected than lungs; cold agglutinins are present in 20% of patients with adenovirus pneumonia
  • In solid organ transplants, usually the transplanted organ affected is by adenovirus
  • In lung transplant patients, disease usually occurs in early post-transplant period and is associated with graft failure, bronchiolitis obliterans, and often death
  • Disease in transplant patients may be due to primary infection or reactivation
  • Diagnosis: rountine tissue culture, PCR, serology, or antigen specific assays; in tissue samples, use immunostains
Prognostic factors
  • In transplant patients, increasing viral load by PCR and greater immunosuppression increases the risk of serious disease
  • Other risk factors are young patient age, graft vs. host disease, unrelated stem cell donor, T cell depletion of graft, cord stem cell transplant, low T cell counts post transplant, total body irradiation, use of antilymphocyte antibodies
Treatment
  • Only supportive care is known to be effective
  • Some studies have shown benefit of ribavirin therapy, but this is not universal
  • Immunotherapy is being investigated
  • Appropriate hand hygiene can reduce transmission
  • In the United States, military recruits formerly received vaccinations against some strains of adenovirus; production of this vaccine stopped, however, efforts to resume production are ongoing
Microscopic (histologic) description
  • Epithelial cells contain smudged nuclei with brick-like, intranuclear inclusions; start as small, eosinophilic inclusions, then briefly appear as basophilic inclusions with a thin halo; then enlarge to obscure the nuclear membrane to take on the typical smudgy appearance
  • Two patterns of disease are commonly encountered that may occur concurrently: diffuse alveolar damage like pattern with interstitial edema, mononuclear cell infiltrates and exudative fluid accumulation in alveoli with hyaline membranes; second pattern is necrosis of bronchi, bronchioles, and alveoli with neutrophilic and histiocytic infiltrates, with interstitial fibrosis and obliterative bronchiolitis as potential sequelae
Microscopic (histologic) images

Contributed by Elliot Weisenberg, M.D.
Adenovirus inclusions in lung from an infant chimpanzee

Adenovirus inclusions
in lung from an infant
chimpanzee



Images hosted on other servers:
Interstitial inflammation (no inclusions identifiable)

Interstitial inflammation (no inclusions identifiable)

Various images

Various images

Positive stains
  • Adenovirus immunohistochemistry
Electron microscopy description
  • 60 - 90 nm, icosahedral particles in crystalline array
Differential diagnosis
  • The diagnosis can be made by rountine tissue culture, PCR, serology, or antigen specific assays
  • In tissue samples, immunohistochemistry can confirm the diagnosis

Alectinib
Definition / general
  • Alectinib is a therapeutic agent that inhibits the activity of anaplastic lymphoma kinase (ALK)
  • Developed by Hoffmann-La Roche, Inc. and Genentech, Inc.
Trade name
  • Alecensa®
Indications
  • To treat patients with advanced or recurrent ALK positive non small cell lung cancer (NSCLC) or who could not tolerate crizotinib (Xalkori®) - approved by U.S. Food and Drug Administration (FDA) in December 2015
  • First line treatment of patients with ALK positive metastatic NSCLC - approved by U.S. FDA in November 2017
Pathophysiology
  • In normal cells, ALK is activated by dimerization of extracellular ligands and autophosphorylation of intracellular tyrosine kinase, mostly involved in the neurodevelopment of CNS, gut and testes (Figure 1)
  • ALK has been found to fuse with other genes in NSCLC, more commonly with EML4 gene; the resulting EML4-ALK fusion gene occurred in 3 - 6% of lung adenocarcinoma
  • ALK is activated by the 5' fusion partner (e.g. EML4) that serves as a functional promotor and expresses a domain in the fusion protein for dimerization (Figure 1)
  • Dimerized ALK rearranged fusion proteins activate downstream signaling pathways including STAT3, mTOR, PI3K, Ras and MEK to enhance cell survival, angiogenesis and cell cycle progression (Figure 1)
  • Alectinib inhibits ALK tyrosine kinase by binding to its ATP binding site and blocking autophosphorylation of the EML4-ALK fusion protein, thereby inhibiting downstream STAT3, Ras and PI3K signaling pathways (Figure 2)
  • Reference: Semin Cancer Biol 2017;42:81
Diagrams / tables

Images hosted on other servers:
Rearranged ALK signaling in NSCLC

Rearranged ALK signaling in NSCLC

Alectinib binds to tyrosine kinase

Alectinib binds to tyrosine kinase

ALK break apart probe for rearranged ALK

ALK break apart probe for rearranged ALK

Mechanisms of EML4-ALK rearrangement

Mechanisms of EML4-ALK rearrangement

Alectinib interacts with ALK P loop

Alectinib interacts with ALK P loop

Clinical information
  • Hepatotoxicity: monitor liver function tests every 2 weeks for first 3 months, then monthly
  • Not recommended in patients with moderate to severe hepatic impairment
  • Withhold the drug if patients develop interstitial lung disease
  • Withhold the drug if patients develop renal impairment
  • Bradycardia: monitor heart rate and blood pressure regularly
  • Myalgia: monitor creatine phosphokinase (CPK) every 2 weeks in first month
  • Embryo fetal toxicity: advise females to use elective contraception
  • Pharmacokinetics (Drug Saf 2019;42:199)
    • Bioavailability: 37% (taken with a meal)
    • Protein binding: > 99%
    • Metabolism: CYP3A4 and other CYP enzymes and aldehyde dehydrogenase
    • Plasma half life: 32.5 hours
    • Excretion: mainly via feces (98%), minor fraction in urine (< 1%)
    • Steady states: reached within 7 days
  • Reference: FDA: Highlights of Prescribing Information [Accessed 8 October 2020]
Uses by pathologists
  • ALK immunohistochemistry: cytoplasmic staining for altered ALK protein expression (N Engl J Med 2010;363:1693, Mod Pathol 2013;26:1545)
    • Clone ALK1, Dako mouse monoclonal ALK1 antibody CD246, sensitivity 100%, specificity 99%
    • Clone 5A4, Novocastra mouse monoclonal antibody p80 ALK, sensitivity 100%, specificity 98%
    • Clone D5F3, Cell Signaling Technology rabbit monoclonal ALK XP, sensitivity 100%, specificity 99%
    • Clone 1A4, Zeta mouse monoclonal anti-ALK antibody
  • ALK FISH: gold standard for evaluating ALK rearrangement
    • ALK 2p23 dual color break apart FISH probe for detecting ALK gene rearrangement (Figure 3)
    • ALK / EML4 tricolor FISH probe for detecting EML4-ALK gene fusion
  • ALK RT-PCR and next generation sequencing
Side effects
  • Fatigue (> 20%)
  • Constipation (> 20%)
  • Anemia (20%)
  • Peripheral edema (17%)
  • Myalgia (16%)
  • Increased blood bilirubin (15%)
  • Increased ALT (15%)
  • Increased AST (14%)
  • Nausea (14%)
  • Diarrhea (12%)
  • Increased weight (10%)
  • Dizziness (8%)
  • Musculoskeletal pain (7%)
  • Vomiting (7%)
  • Photosensitivity reaction (5%)
  • Dysgeusia (3%)
  • Blurred vision (2%)
  • Visual impairment (1%)
  • Alopecia (1%)
  • Increased γ glutamyltransferase (1%)
  • References: N Engl J Med 2017;377:829, FDA: Highlights of Prescribing Information [Accessed 8 October 2020]
Drug administration
Molecular theory
  • ALK is located at chromosome 2p21 and EML4 at 2p23.1-23.2
  • EML4-ALK fusion gene can be caused by chromosomal inversion or a complicated chromosomal translocation (chromothripsis) (Figure 4) (J Thorac Oncol 2014;9:1638)
  • Alectinib weakly interacts with the P loop of rearranged ALK protein shown in the cocrystal structures (Figure 5) (Lung Cancer 2017;110:32)
Board review style question #1
Which of the following lung primary tumors could be treated with alectinib?

  1. Adenocarcinoma with ALK rearrangement
  2. Adenocarcinoma with c-MET rearrangement
  3. Adenocarcinoma with EGFR mutations
  4. Small cell carcinoma with MYC rearrangement
Board review style answer #1
A. Adenocarcinoma with ALK rearrangement

Comment Here

Reference: Alectinib

Alveolar proteinosis
Definition / general
  • Rare disease with accumulation of acellular surfactant presumedly due to impaired clearance, likely due to dysfunction of alveolar macrophages
  • Considered a response to alveolar injury, not a specific entity
  • Congenital (2% of cases), primary (idiopathic) or secondary forms
  • Congenital cases are usually due to mutation in surfactant protein or GM-CSF receptor genes
  • Secondary (most common), associated with silica dust, aluminum, fiberglass, chemicals, immunosuppression, leukemia / lymphoma, Nocardia, Mycobacteria and Aspergillosis
  • Usually adults (more common in smokers), also children
  • Sputum contains chunks of gelatinous material; no inflammation
  • Usually does NOT progress to chronic fibrosis
  • Xray: resembles pulmonary edema; diffuse pulmonary opacification
Treatment
  • Whole lung lavage (also diagnostic)
Microscopic (histologic) description
  • Alveoli contain amorphous, eosinophilic and PAS+ material in lumina consisting of type II pneumocytes, lamellar bodies and necrotic alveolar macrophages
  • Variable fibrosis
  • Mild / no lymphocytic infiltration
Electron microscopy description
  • Exudate contains lamellar bodies, lipid particles and cellular debris

Amiodarone induced pulmonary toxicity
Definition / general
  • Oral antiarrythmic drug, usually used long term
  • 5% - 10% of patients develop lung injury
  • Risk factors are: advanced age, higher dose therapy, lung surgery, oxygen therapy and underlying pulmonary disease; disease may occur without risk factors or at lower doses
  • Drug inhibits phospholipid degradation by lysosomes, resulting in phospholipid accumulation within cells, especially alveolar macrophages
Case reports
Treatment
  • Drug withdrawal or dose reduction, corticosteroid therapy
  • May have 50% mortality if diffuse alveolar damage is present
Microscopic (histologic) description
  • Intra-alveolar exudate of finely vacuolated foamy macrophages, also present within alveolar septa
  • May have diffuse alveolar damage, cryptogenic organizing pneumonia type changes or changes of nonspecific interstitial pneumonia or desquamative interstitial pneumonia
Cytology description
  • Finely vacuolated, foamy macrophages; variable neutrophils
Electron microscopy description
  • Membrane bound cytoplasmic lamellar inclusions due to drug accumulation in the lungs
  • Morphologic findings are not specific and must be interpreted in conjunction with the clinical history

Amyloidosis
Definition / general
  • Amyloidosis in the lung can present in three distinct manners: diffuse alveolar septal amyloidosis, nodular pulmonary amyloidosis and tracheobronchial amyloidosis
  • Different types are distinguished by the pattern of deposition within the lung tissue; however, they are all characterized by deposition of an eosinophilic amorphous material that stains bright red with Congo red and shows an apple green birefringence upon polarization
  • Ultrastructurally, it is composed of haphazardly arranged nonbranching fibrils that measure 8 to 10 nm in diameter
Essential features
  • Heterogenous group of entities caused by deposition of amyloid, an abnormal protein with a beta pleated structure composed of haphazardly arranged nonbranching fibrils measuring 8 to 10 nm in diameter
  • Light microscopy shows amorphous eosinophilic deposits that show apple green birefringence under polarized light
  • Main pulmonary forms are diffuse alveolar septal, nodular and tracheobronchial
  • Most commonly caused by AL amyloidosis: the deposition of light chains due to a monoclonal hematologic process
    • AL is associated with hematologic malignancies, commonly multiple myeloma
  • Subtyping should be performed with mass spectrometry
Terminology
  • Amyloidosis in general (not organ / lung specific)
    • Primary amyloidosis (now systemic AL amyloidosis)
    • Secondary amyloidosis (now systemic AA amyloidosis)
    • Age related or senile systemic amyloidosis (now systemic wild type ATTR amyloidosis)
    • Familial amyloid polyneuropathy (now systemic hereditary ATTR amyloidosis) (Amyloid 2014;21:221)
  • Pulmonary amyloidosis subtypes
    • Diffuse alveolar septal amyloidosis
      • Diffuse parenchymal amyloidosis
    • Nodular pulmonary amyloidosis
      • Nodular parenchymal amyloidosis, nodular amyloidoma
    • Tracheobronchial amyloidosis
ICD coding
  • ICD-10:
    • E85.9 - Amyloidosis, unspecified
    • E85.2 - Heredofamilial amyloidosis, unspecified
    • E85.8 - Other amyloidosis
    • E85.3 - Secondary systemic amyloidosis
    • E85.4 - Organ limited amyloidosis
    • E85.82 - Wild type transthyretin related (ATTR) amyloidosis
  • ICD-11:
    • 5D00 - Amyloidosis
    • 5D00.1 - AA amyloidosis
    • 5D00.2 - Hereditary amyloidosis
    • 5D00.20 - Hereditary ATTR amyloidosis
    • 5D00.21 - Nonneuropathic heredofamilial amyloidosis
    • 5D00.2Y - Other specified hereditary amyloidosis
    • 5D00.2Z - Hereditary amyloidosis, unspecified
    • 5D00.3 - Dialysis associated amyloidosis
    • 2A83.5 - Monoclonal immunoglobulin deposition disease
    • 2A83.50 - Heavy chain deposition disease
    • 2A83.51 - Light and heavy chain deposition disease
    • 2A83.52 - Light chain deposition disease
    • 5D00.Y - Other specified amyloidosis
    • 5D00.Z - Amyloidosis, unspecified
Epidemiology
Sites
Pathophysiology
  • ~ 95% composed of fibril proteins
    • 31 human fibril proteins have met the criteria from the International Society of Amyloidosis (ISA) Nomenclature Committee (Amyloid 2014;21:221)
    • Insoluble fibril proteins from improper folding of soluble precursors
      • When produced in excessive amounts:
        • Immunoglobulin light chains
        • Serum amyloid A
        • Wild type transthyretin
      • Abnormal amino acid sequence (i.e., transthyretin variants)
  • ~ 5% serum amyloid P component and other glycoproteins
  • Systemic AL amyloidosis
    • Monoclonal plasma cell proliferative disorder
    • Monoclonal immunoglobulin light chains deposit in tissues
    • Can follow monoclonal gammopathy of undetermined significance (MGUS), multiple myeloma and Waldenström macroglobulinemia (N Engl J Med 2002;346:564)
  • Systemic AA amyloidosis
    • Deposition of serum amyloid A (Apo), an acute phase reactant produced in the liver
    • Usually secondary to a chronic inflammatory condition
  • Systemic wild type ATTR amyloidosis
    • Deposition of unmutated transthyretin (prealbumin) in tissues that accumulates with age
  • Systemic hereditary ATTR amyloidosis
    • Mutations in the transthyretin gene
Etiology
Clinical features
Diagnosis
  • Diffuse alveolar septal amyloidosis
    • Usually incidental
    • Symptoms due to amyloid deposition in other organs
    • Pulmonary involvement usually made at autopsy
  • Nodular pulmonary amyloidosis
    • Chest imaging
  • Tracheobronchial amyloidosis
  • DNA sequencing may be needed to distinguish between wild type and hereditary ATTR amyloidosis when a family history is not apparent (J Am Coll Cardiol 2015;66:2451)
Laboratory
  • Diffuse alveolar septal amyloidosis
  • Tracheobronchial amyloidosis
    • Decreased airflow with proximal airway involvement on pulmonary function tests
    • Normal airflow if just distal involvement
Radiology description
  • Nodular pulmonary amyloidosis
    • One or more solid lung nodules:
      • May have spiculated borders and be confused with malignancy
      • May have calcifications
    • Cystic features have been described (Eur Respir J 2007;30:589)
  • Tracheobronchial amyloidosis
    • Extensive thickening of the trachea or bronchi with or without calcification
    • May show postobstructive changes (atelectasis, pneumonia or long term bronchiectasis) (J Thorac Imaging 2005;20:41)
Radiology images

Contributed by Tan-Lucien H. Mohammed, M.D.

Tracheobronchial amyloidosis

Airway thickening

Ground glass consolidation


Pulmonary nodular amyloidosis

Basilar gradient

Scattered nodules

Multiple nodules and cysts



Prognostic factors
  • Systemic AL amyloidosis
    • Depends on the organs affected
  • Systemic AA amyloidosis
    • High mortality due to end stage renal disease, infection, heart failure, bowel perforation or GI bleeding if left untreated
  • Systemic wild type ATTR amyloidosis
    • Survival is better than AL amyloidosis
Case reports
Treatment
  • Typing of the amyloid is essential because treatment depends on the form
    • Systemic AL amyloidosis
    • Systemic AA amyloidosis
    • Systemic wild type ATTR amyloidosis
      • Chemotherapy and stem cell transplant contraindicated
    • Systemic hereditary ATTR amyloidosis
  • Diffuse alveolar septal amyloidosis
    • Prognosis worse with primary amyloidosis compared to other types
      • Death usually due to cardiac involvement or less often pulmonary hypertension
  • Nodular pulmonary amyloidosis
    • Conservative excision usually curative
    • Long term prognosis excellent
  • Tracheobronchial amyloidosis
Clinical images

Images hosted on other servers:

Primary endobronchial amyloidosis

Gross description
Microscopic (histologic) description
  • General: amyloid appears as glassy amorphous eosinophilic material
  • Diffuse alveolar septal amyloidosis
    • Low magnification: well preserved pulmonary architecture
    • Higher magnification
      • Alveolar septae thickened amyloid
      • Vessel walls involved and may form small nodules
      • Amyloid may be seen in visceral pleura
      • May see scant plasma cells
  • Nodular pulmonary amyloidosis
    • Well circumscribed nodules
    • Homogenous, densely eosinophilic material with associated small aggregates of lymphocytes and plasma cells
    • May see foreign body type giant cells, calcification, ossification and cartilage (Arch Pathol Lab Med 2017;141:247)
  • Tracheobronchial amyloidosis
    • Deposits of homogeneous eosinophilic material around seromucinous glands and cartilage
    • Involvement of submucosal vessels
    • Plasma cells, foreign body type giant cells, calcification, ossification (Arch Pathol Lab Med 1986;110:212)
Microscopic (histologic) images

Contributed by Brian D. Stewart, M.D.

Nodular deposits

Vascular deposition

Congo red

Congo red, polarized



Contributed by Andras Khoor, M.D., Ph.D.

Ossification


Diffuse septal deposition

Periarteriolar accentuation

Tracheobronchial deposition

Subepithelial deposits

Cytology description
  • Limited value in routine smears
  • Cell block of pleural fluid may show amorphous waxy material with a rim of reactive mesothelial cells (Diagn Cytopathol 2018;46:522)
Immunofluorescence description
  • Thioflavin T is a fluorescent dye and is viewed under a fluorescent microscope
Positive stains
Negative stains
Electron microscopy description
Electron microscopy images

Images hosted on other servers:

Beta pleated sheet

Amyloid fibrils

Molecular / cytogenetics description
  • Mass spectrometry based proteomic analysis
    • Performed on formalin fixed, paraffin embedded tissue after microdissection
    • Considered the preferred method for amyloid subtyping (Blood 2009;114:4957)
Videos

Definition, historical aspects and properties

Pathogenesis and classification

Pathology, clinical features and prognosis

Sample pathology report
  • Lung, right lower lobe, transbronchial biopsy:
    • Nodular pulmonary amyloidosis (see comment)
    • Comment: Sections show bronchial wall and alveolated lung parenchyma with nodular deposits of eosinophilic amorphous material deposited mostly within the vascular walls. Congo red special stain shows apple green birefringence upon polarization, supporting the diagnosis. The specimen will be sent for mass spectrometry for amyloid typing and these results will be issued as an addendum.
Differential diagnosis
Board review style question #1


The process demonstrated in the two photomicrographs of a transbronchial biopsy is most commonly caused by which of the following?

  1. Uric acid crystal deposition
  2. Deposition of improperly folded fibril proteins
  3. Calcium pyrophosphate crystal deposition
  4. Increased collagen production
  5. Microscopic thromboemboli
Board review style answer #1
B. Deposition of improperly folded fibril proteins. This is amyloidosis.

Comment Here

Reference: Amyloidosis
Board review style question #2
In the diffuse alveolar septal manifestation of amyloidosis, which of the following may be seen?

  1. Destruction of the pulmonary architecture
  2. Emphysematous changes
  3. Diffuse alveolar damage
  4. Thickened vascular walls
  5. Acute inflammatory reaction
Board review style answer #2
D. Thickened vascular walls

Comment Here

Reference: Amyloidosis
Board review style question #3


Which of the following is the preferred method for subtyping amyloid, shown in the images above?

  1. Polymerase chain reaction
  2. Electron microscopy
  3. Mass spectrometry
  4. Xray crystallography
  5. Gas chromatography
Board review style answer #3
C. Mass spectrometry

Comment Here

Reference: Amyloidosis

Anatomy
Definition / general
  • Main function of lungs is gas exchange
  • Trachea divides into right and left mainstem bronchi
  • Right lung has 3 lobes (upper, middle and lower); left lung has 2 lobes (upper and lower) lobes; lingual is part of left upper lobe and is somewhat analogous to right middle lobe
  • Right bronchus is more vertical and in line with trachea than left; thus aspirated material tends to enter right lung
  • Each main bronchus divides into lobar bronchi, then into segmental bronchi
  • Lobar bronchi are usually called secondary bronchi and segmental bronchi are called tertiary bronchi, except in Japan, where they are called first order and second order, respectively
  • Bronchial walls contain cartilage and submucosal glands
  • Bronchioles are generally < 3 mm in diameter and lack cartilage and submucosal glands in their walls; their diameter varies
  • Bronchioles are generally divided into nonrespiratory bronchioles (all bronchioles proximal to respiratory bronchioles, including terminal bronchioles that are proximal to respiratory bronchioles) and respiratory bronchioles (airways with alveoli budding from their walls)
  • Alveoli are "dead ends" of airways; their walls are composed only of alveoli, where gas exchange takes place
  • Lung has double arterial supply - pulmonary and bronchial arteries that accompany airways; in general, diameter of airway is similar to that of accompanying pulmonary artery
  • Lungs are surrounded by visceral pleural membrane; inner chest cavity is lined by parietal pleural membrane; these membranes define pleural space, which normally has minimal volume
  • Regional lymph nodes: paratracheal, pre- and retrotracheal, aortic, subcarinal, periesophageal, inferior pulmonary ligament, hilar, peribronchial and intrapulmonary
Embryology
  • The lungs develop from the ventral wall of the foregut
  • The midline trachea develops two lateral outpouchings, the right and left lung buds
  • The lung buds extend into primitive thoracic mesenchyme that becomes bronchial cartilage and other connective tissue
  • The right lung bud develops three branches that become mainstem bronchi, the left lung bud develops two branches that become mainstem bronchi
  • The phases of lung development are embryonic (3 1/2 to 6 weeks), pseudoglandular (6 to 16 weeks), canalicular (16 to 28 weeks), saccular (28 to 36 weeks) and alveolar (36 weeks to term)
Diagrams / tables

Images hosted on other servers:
Various images Various images Various images

Various images

Gross images

Images hosted on other servers:
Various images Various images Various images

Various images

Microscopic (histologic) images
Contributed by Grigory Demyashkin, M.D.
Lung, glandular stage

Lung, glandular stage

Lung, glandular stage; top: spine

Lung, glandular stage; top: spine

Top: spine; lower left: trachea and esophagus; center: lung, glandular stage

Top: spine; lower left: trachea and esophagus; center: lung, glandular stage


ARDS / DAD
Definition / general
  • In 1994, the American European Consensus Conference (AECC) defined acute respiratory distress syndrome (ARDS) as the acute onset of hypoxemia with bilateral infiltrates on frontal chest radiograph, with no evidence of left atrial hypertension (Am J Respir Crit Care Med 1994;149:818)
  • In 2012, the ARDS Task Force revised the new definition (see Diagnosis) (JAMA 2012;307:2526)
  • Diffuse alveolar damage (DAD) is manifested by injury to alveolar lining and endothelial cells, pulmonary edema, hyaline membrane formation and later by proliferative changes involving alveolar and bronchiolar lining cells and interstitial cells (Am J Pathol 1976;85:209)
Essential features
  • Acute and rapidly progressive hypoxia with bilateral pulmonary edema due to alveolar injury caused by pulmonary or systemic insults
  • 40% die within 28 days from onset of ARDS, mainly due to infection / sepsis and multiple organ dysfunction syndrome
  • Although DAD is the typical morphology of ARDS, clinical syndrome of ARDS is not synonymous with the pathologic diagnosis of DAD
  • DAD pattern is often characterized by hyaline membranes in acute phase but shows a wide variety of findings that makes the diagnosis challenging
Terminology
  • Previously known as adult respiratory distress syndrome
  • Acute lung injury, an overlapping entity suggested in AECC definition, was removed from Berlin definition; the AECC definition of acute lung injury non-ARDS is compatible with Berlin definition of mild ARDS (JAMA 2012;307:2526)
ICD coding
  • ICD-10: J80 - acute respiratory distress syndrome
Epidemiology
  • Mean age: 60 years old; from children to elderly
  • M:F = 3:2
  • Incidence of 10.6 - 78.9 per 100,000 person years (N Engl J Med 2005;353:1685, Intensive Care Med 2009;35:1352)
  • Incidence of ARDS in intensive care unit (ICU) (JAMA 2016;315:788):
    • 10.4% of all ICU patients
    • 67.2% of patients with acute hypoxemic respiratory failure
    • ARDS is frequently undiagnosed (60.2% recognized by clinician) or late diagnosed (34.0% recognized at the time of criteria fulfillment)
Sites
  • Diffusely involves bilateral lobes of the lung without upper or lower lobe predominance
  • Distribution can be regional, depending on the degree and cause of the inflammation
Pathophysiology
  • Pathogenesis (N Engl J Med 2000;342:1334, Lancet 2016;388:2416)
    • Exudative (acute) phase: 1 - 7 days
      • Neutrophil mediated inflammation destroys the alveolar capillary barrier (alveolar epithelium and endothelium), increases its permeability and causes intra-alveolar hemorrhage and edema
      • Protein rich edema interacts with alveolar surfactants, resulting in decreased pulmonary compliance
      • Hyaline membranes develop on alveolar wall where epithelium is denudated and disrupted
    • Proliferative / organizing (subacute) phase: 1 - 3 weeks
      • Proliferation of type II pneumocytes and subsequent differentiation into type I pneumocytes
      • Proliferation of myofibroblasts
      • Drainage of alveolar edema by restored type II pneumocytes
    • Fibrotic (chronic) phase: after 3 weeks
      • Collagenous fibrosis in alveolar spaces and interstitium
      • Refractory rigidity of alveoli due to architectural remodeling
  • Pathophysiology of hypoxia
    • Collapse of alveoli
    • Increased right to left intrapulmonary shunt
    • Decreased pulmonary compliance
      • Surfactant malfunction
      • Fibrosis
    • Decreased diffusing capacity due to pulmonary edema
    • Increased ventilation perfusion mismatch
    • Increased pulmonary vascular resistance
  • Mechanism of ARDS to cause multiple organ dysfunction syndrome is not clear
Etiology

Direct lung injury Indirect lung injury
  • Common causes
    • Infectious pneumonia
    • Aspiration of gastric contents
  • Common causes
    • Sepsis
    • Severe trauma or burn, especially
        with shock and multiple transfusions
  • Less common causes
    • Pulmonary contusion
    • Fat emboli
    • Near drowning
    • Inhalational injury (particle, gas)
    • Reperfusion pulmonary edema after
        lung transplantation or pulmonary embolectomy
  • Less common causes
    • Cardiopulmonary bypass
    • Drug
    • Acute pancreatitis
    • Autoimmune disease
    • Transfusion related acute lung injury
Diagrams / tables

Images hosted on other servers:
Acute phase

Acute phase

Organizing phase

Organizing phase

Clinical features
  • Acute and progressive respiratory failure
    • Usually starts 12 - 48 hours after the initial insult
    • Shortness of breath; dyspnea on exertion followed by dyspnea at rest
    • Hypoxia; PaO2 / FiO2 ≤ 300 mm Hg
  • Often followed by sepsis and multiple organ dysfunction syndrome (JAMA 2016;315:788)
  • Respiratory dysfunction and physical disability may persist for months after remission of ARDS, with gradual improvement (N Engl J Med 2003;348:683)
Diagnosis
  • Diagnosis of ARDS is based on clinical manifestation and its severity is evaluated with ratio of arterial partial pressure of oxygen to fraction of inspired oxygen (PaO2 / FiO2)
  • Berlin definition (JAMA 2012;307:2526)
    • Timing
      • Within 1 week of a known clinical insult or new or worsening respiratory symptoms
    • Chest imaging
      • Assessed by chest Xray or CT
      • Bilateral opacities; not fully explained by effusions, lobar / lung collapse or nodules
    • Origin of edema
      • Respiratory failure not fully explained by cardiac failure or fluid overload
      • Need objective assessment (e.g. echocardiography) to exclude hydrostatic edema if no risk factor present
    • Oxygenation (with positive end expiratory pressure [PEEP] or continuous positive airway pressure [CPAP] ≥ 5 cm H2O)
      • Mild: 200 < PaO2 / FiO2 ≤ 300 mm Hg
      • Moderate: 100 < PaO2 / FIO2 ≤ 200 mm Hg
      • Severe: PaO2 / FiO2 ≤ 100 mm Hg
  • Not all ARDS show DAD pattern
    • ARDS can present with organizing DAD and acute fibrinous organizing pneumonia as well
    • DAD can be seen as a nonspecific manifestation of the agonal phase and shock
  • Although rarely performed, lung biopsy can be helpful for critical care and prognosis estimation in patients with ARDS
Laboratory
  • Arterial blood gas test
    • Hypoxemia
    • Ratio of pulse oximetric oxygen saturation to FIO2 (SpO2 / FiO2) may be helpful for instant follow up (Chest 2007;132:410)
      • SpO2 / FiO2 of 235 ≈ PaO2 / FiO2 of 200 (sensitivity: 85%, specificity: 85%)
      • SpO2 / FiO2 of 315 ≈ PaO2 / FiO2 of 300 (sensitivity: 91%, specificity: 56%)
  • Other blood tests
    • Increased C reactive protein
    • Increased procalcitonin in septic ARDS
    • Increased ferritin (Am J Respir Crit Care Med 1999;159:1506)
    • Increased KL-6
    • Brain natriuretic peptide (BNP) test may helpful to distinguish ARDS and cardiogenic pulmonary edema (Chest 2007;131:964)
      • BNP ≤ 200 pg/mL is suggestive for ARDS (sensitivity: 40%, specificity: 91%)
      • BNP ≥ 1,200 pg/mL is suggestive for cardiogenic pulmonary edema (sensitivity: 52%, specificity: 92%)
Radiology description
  • General
    • Heterogeneous bilateral shadows due to pulmonary edema
    • Rule out atelectasis, pleural effusion and mass
    • Takes 12 - 24 hours from onset to be apparent
  • Chest Xray
    • Exudative phase: ground glass opacity and consolidation with air bronchogram
    • Proliferative / organizing and fibrotic phases: reticular shadow and volume reduction
  • Chest CT (Radiology 1999;211:859)
    • Exudative phase
      • Patchy to diffuse ground glass opacity, with or without interlobular septal thickening
      • Dorsal consolidation due to infiltrate
    • Proliferative / organizing phase
      • Ground glass opacity with bronchiolectasis and bronchiectasis
      • Volume reduction
    • Fibrotic phase
      • Septal thickening and reticular shadow in ground glass opacity
      • Peripheral cystic and honeycomb-like lesions due to fibrosis
Radiology images

Images hosted on other servers:
Changes in ARDS

Changes in ARDS

Exudative phase Exudative phase

Exudative phase

Exudative phase Exudative phase

Exudative phase

Exudative phase

Exudative phase


Proliferative / organizing phase Proliferative / organizing phase

Proliferative / organizing phase

Fibrotic phase

Fibrotic phase

ARDS on CT

ARDS on CT

ARDS on CT

ARDS on CT

ARDS on CT

ARDS on CT

Prognostic factors
Case reports
Treatment
  • Removal of the original insult
  • Oxygen therapy for respiratory failure (Lancet 2016;388:2416)
    • Mechanical ventilation
      • PEEP or CPAP ≥ 5 cm H2O
      • With or without neuromuscular blockade, prone position and extracorporeal membrane oxygenation
  • Other supportive care
    • Fluid management
    • Pharmacotherapy
      • Glucocorticoid
      • Anticoagulant
  • Since no treatment drastically improves the respiratory failure of ARDS, respiratory and systemic supportive care is needed until the patient survives from ARDS
Gross description
  • Brown to gray consolidative lesion with indistinct borders, diffusely involving bilateral lobes
  • Elastic hard and greasy due to transudate within alveolar spaces and interstitial edema
  • Heavy and shrunken due to fibrosis and collapse of the tissue
  • Dots of hemorrhage on pleural surface
Gross images

Contributed by Yale Rosen, M.D.
Diffuse alveolar damage (DAD) Diffuse alveolar damage (DAD) Diffuse alveolar damage (DAD)

Diffuse alveolar damage (DAD)

Patchy lung involvement

Patchy lung involvement


Diffuse alveolar damage (DAD) Diffuse alveolar damage (DAD) Diffuse alveolar damage (DAD)

Diffuse alveolar damage (DAD)

Microscopic (histologic) description
  • Histopathology of DAD progresses from exudative (acute) phase through proliferative / organizing (subacute) phase to chronic fibrotic phase roughly corresponding to the period of ARDS (Am J Pathol 1976;85:209, Arch Pathol Lab Med 2010;134:719, Clin Chest Med 2000;21:435, N Engl J Med 2000;342:1334)
    • Phase of the disease is almost synchronous throughout the lung
    • Features of different phases may be combined in the transitional period or with repeated bouts of lung injury
  • Exudative (acute) phase
    • Alveolar change
      • Hyaline membranes on alveolar duct or sacs
      • Interstitial and intra-alveolar edema
      • Collapsed alveoli
    • Epithelial change
      • Denudation and necrosis of type I pneumocytes
    • Vascular change
      • Necrosis of endothelial cells
      • Neutrophil aggregation
      • Microthromboemboli
      • Hemorrhage
  • Proliferative / organizing (subacute) phase
    • Alveolar change
      • Remnants of hyaline membrane with or without organization
      • Interstitial and intra-alveolar proliferation of myofibroblasts
      • Lymphocytic infiltration
    • Epithelial change
      • Proliferation / hyperplasia of type II pneumocytes
    • Vascular change
      • Endothelial injury and thromboemboli in arterioles
  • Fibrosis phase
    • Alveolar change
      • Collagenous fibrosis
      • Microscopic honeycomb-like change
      • Traction bronchiolectasis
    • Epithelial change
      • Squamous metaplasia / hyperplasia
    • Vascular change
      • Remodeling of arteries
      • Intimal fibrosis
      • Thickening of media
  • Others
    • May have superimposed pneumonia
    • Fibrin deposition
    • DAD with prominent organizing pneumonia is also called organizing DAD
Microscopic (histologic) images

Contributed by Akira Yoshikawa, M.D. and Yale Rosen, M.D.
Hyaline membrane Hyaline membrane

Hyaline membrane

Hyaline membrane Hyaline membrane

Hyaline membrane

Hyaline membrane

Hyaline membrane

Hyaline membranes and fibrin

Hyaline membranes and fibrin



Masson body

Masson body

Masson body

Masson body

Masson body

Masson body

Organizing DAD

Organizing DAD

Collagenous fibrosis

Collagenous fibrosis



Fibroblastic proliferation Fibroblastic proliferation

Fibroblastic proliferation

Fibroblastic proliferation Fibroblastic proliferation

Fibroblastic proliferation

Interstitial organization

Interstitial organization

Inflammatory cells

Inflammatory cells

Virtual slides

Images hosted on other servers:

Exudative phase

Cytology description
  • Bronchoalveolar lavage (BAL) fluid
    • Increased neutrophils in ARDS
    • Can be helpful for the diagnosis of underlying disease
Positive stains
  • Fiber staining (e.g. elastica van Gieson) is helpful to evaluate fibrosis and to evaluate the destruction of the alveolar architecture
  • Giemsa, Grocott and Ziehl-Neelsen stains are helpful to identify pathogens
  • Immunohistochemistry (not of practical utility):
    • Cytokeratin highlights collapsed alveoli and lung architecture
Electron microscopy description
  • Changes in epithelium, endothelium and interstitium (N Engl J Med 2000;342:1334)
    • Exudative phase
      • Vacuolization in damaged endothelial cells
      • Replacement of epithelial cells by hyaline membrane on the basement membrane
    • Proliferative / organizing and fibrotic phases
      • Reepithelialization of type II pneumocytes with microvilli and lamellar bodies with surfactant
      • Flattening of cytoplasm and loss of lamellar bodies and microvilli of type II pneumocyte, indicating transformation to type I pneumocyte
      • Collagen deposition
Videos

Histology of DAD

Sample pathology report
  • Lung, autopsy:
    • Diffuse alveolar damage (see comment)
    • Comment: Histologic sections reveal diffuse alveolar damage in subacute and chronic phases involving bilateral lobes; the former is represented by fibrin deposition, organizing pneumonia and focal hyaline membranes and the latter by interstitial fibrosis, fibroblastic foci and the destruction of alveolar architecture. Acute bacterial pneumonia is also superimposed.
Differential diagnosis
  • Major entities:
  • Minor entities:
    • Organizing pneumonia:
      • Migratory sign on imaging
      • Good response to corticosteroid
    • Eosinophilic pneumonitis:
      • Current smoker or history of particle inhalation
      • Eosinophilia (> 25%) in bronchoalveolar lavage
      • Good response to corticosteroid
    • Hypersensitivity pneumonitis:
      • History of antigen exposure
      • Remission of symptoms after removal of the causative antigen
      • Lymphocytosis (> 30%) in bronchoalveolar lavage
      • Diffuse centrilobular nodular shadow with ground glass opacity, usually in upper lobes
    • Miliary tuberculosis:
      • Diffuse nodular shadow (≤ 3 mm) involving whole lobes
      • Mycobacterium tuberculosis in bronchoalveolar lavage or biopsy specimen
    • Lymphangitic carcinomatosis:
      • Known malignancy
      • Tumor cells in bronchoalveolar lavage or biopsy specimen
Board review style question #1

Which phase of diffuse alveolar damage (DAD) is most clearly demonstrated in this histological image?

  1. Active
  2. Exudative (acute)
  3. Fibrotic (chronic)
  4. Postinflammatory
  5. Proliferative / organizing (subacute)
Board review style answer #1
E. Proliferative / organizing (subacute). Masson body, polypoid proliferation of spindle shaped fibroblasts, is noted in the alveoli.

Comment Here

Reference: ARDS / DAD
Board review style question #2
Which of the following findings is most suggestive of acute respiratory distress syndrome (ARDS) / diffuse alveolar damage (DAD)?

  1. Bacterial pneumonia
  2. Diffuse collagenous fibrosis
  3. Hyaline membranes
  4. Organizing pneumonia
  5. Proliferation of atypical pneumocytes
Board review style answer #2
C. Hyaline membranes. Hyaline membranes morphologically represent the damage of pneumocytes and endothelium in DAD.

Comment Here

Reference: ARDS / DAD

Asbestosis
Definition / general
  • Similar to other pneumoconiosis
  • Initial injury is at bifurcations of small airways and ducts; macrophages ingest fibers, release chemotactic factors and fibrogenic mediators, causing interstitial fibrosis similar to other fibrosing lung diseases such as UIP
  • Begins around respiratory bronchioles and alveolar ducts, extends distally; eventually causes honeycomb lungs
  • Begins in lower lobes and subpleurally (in contrast to coal workers' pneumoconiosisP and silicosis), progresses to middle and upper lobes
  • Visceral pleura becomes fibrotic, may bind lung to chest wall; may have associated Caplan syndrome
  • Symptoms: usually begin after 10 years of exposure, initially shortness of breath with exertion and later at rest; may progress to heart failure
  • Pleural plaques: well circumscribed plaques of dense collagen, often with calcium; on parietal pleura and dome of diaphragm; do not contain asbestos bodies, but rare if no asbestos history; may induce pleural effusions, usually no symptoms
  • Asbestos fiber detection: H&E, Prussian blue, incineration and EM
Asbestos
  • Crystalline hydrated silicates that form fibers
  • Causes localized fibrous plaques, pleural effusions, parenchymal interstitial fibrosis (asbestosis), bronchogenic carcinoma, mesothelioma, laryngeal carcinoma and possibly colon carcinoma
  • Increased incidence of mesothelioma in families of asbestos workers
  • Exists in serpentine / chrysotile (curly, flexible) and amphibole (straight, stiff, brittle) forms; most asbestos in industry are serpentine, but amphiboles are more pathogenic; link with mesothelioma is almost always with amphibole form
  • Chrysotiles usually are caught in upper respiratory passages, removed by mucociliary elevator; they are soluble and leached from tissue if they reach alveoli
  • Amphiboles (straight, stiff) go deeper into lungs; fibers > 8 mm and thinner than 0.5 mm are more injurious
  • Both types are fibrogenic; act as tumor initiator and promoter; generate free radicals; toxic chemicals (tobacco smoke) may also be adsorbed to asbestos fibers
  • Asbestos may act by countering antioxidant effect of Vitamin C (ascorbic acid) (Hum Pathol 2003;34:737)
  • Relative risks compared to normal population: asbestos and bronchogenic carcinoma has RR of 5; with tobacco use, RR is 55
  • Asbestos and mesothelioma (pleural, pericardial, peritoneal) has RR of 1000; no increased risk with smoking
  • Incidence of mesothelioma expected to increase until 2020 - 2025 due to lag time between exposure and diagnosis
  • Note: asbestos related tumors have no special histologic features
Gross images

Images hosted on other servers:
Pleural plaque

Pleural plaque

Microscopic (histologic) description
  • Early: interstitial pneumonia with desquamative features, hyperplastic alveolar cells with intracytoplasmic Mallory's hyaline tissue
  • Later: diffuse interstitial fibrosis with honeycombing (silicosis is nodular), asbestos bodies (golden brown, fusiform or beaded rods with translucent center; asbestos fibers coated with iron-containing proteinaceous material); iron from phagocyte ferritin
  • Asbestos fibers may have oxalate crystal deposition (Hum Pathol 2003;34:737)
  • Ferruginous bodies: inorganic particulates coated with phagocyte ferritin
Microscopic (histologic) images

Contributed by Jijgee Munkhdelger, M.D., Ph.D. and Andrey Bychkov, M.D., Ph.D.
Missing Image

Asbestos bodies in alveolar spaces

Missing Image

Ferruginous body

Missing Image

Asbestos body

Missing Image

Dumbbell shaped asbestos body


Aspergillus
Definition / general
  • Infection of the lung by a hyaline mold in genus Aspergillus, with various manifestation based on host immune status
  • Word origin: aspergillum, a ritual liturgical implement used in Roman Catholic ceremonies to sprinkle holy water (Arch Pathol Lab Med 2008;132:606)
Essential features
  • Aspergillus histomorphology: acute angle, dichotomous branching, septate hyphae; however, fungal culture is required for definite identification
  • Noninvasive form in immunocompetent host: cavitary lesion with fungal ball and surrounding chronic inflammation
  • Invasive form in immunocompromised host: necrotizing pneumonia with angioinvasion and infarct
Terminology
  • Allergic bronchopulmonary aspergillosis (ABPA)
  • Chronic pulmonary aspergillosis (CPA)
    • Simple / single aspergilloma
    • Aspergillus nodule
    • Chronic cavitary pulmonary aspergillosis (CCPA)
    • Chronic fibrosing pulmonary aspergillosis (CFPA)
    • Subacute invasive aspergillosis (formerly known as chronic necrotizing pulmonary aspergillosis)
  • Invasive pulmonary aspergillosis (IPA)
ICD coding
  • ICD-10:
    • B44.0 - invasive pulmonary aspergillosis
    • B44.1 - other pulmonary aspergillosis
  • ICD-11:
    • 1F20.0Y - invasive aspergillosis of other specified site
    • 1F20.12 - chronic pulmonary aspergillosis
    • 1F20.14 - aspergillus bronchitis
    • 1F20.15 - obstructing aspergillus tracheobronchitis
    • CA82.4 - aspergillus induced allergic or hypersensitivity conditions
Epidemiology
  • Causes a wide spectrum of diseases in humans, depending on the underlying immune status of the host (Clin Microbiol Rev 2019;33:e00140)
  • One of the most common causes of infectious death in severely immunocompromised patients
    • Mortality rate = 50% in neutropenic patients, 90% in hematopoietic stem cell transplantation (HSCT) recipients (QJM 2007;100:317)
  • Colonization in asymptomatic patient is common
Sites
  • Lung
  • Other commonly affected organs: central nervous system, eye, paranasal sinus, middle ear, heart, bone and soft tissue, skin, gastrointestinal tract (Clin Infect Dis 2016;63:e1)
  • Disseminated infection
Pathophysiology
Etiology
  • Saprophytic fungi in genus Aspergillus, ubiquitous in environment, can be isolated from soil, household, hospital environment, food, etc.
  • 300 species, only 8 species are responsible for infection in humans (Arch Pathol Lab Med 2008;132:606)
    • Aspergillus fumigatus (most common), followed by A. niger
    • Other: A. nidulans, A. terreus, A. clavatus, A. flavus, A. niveus and A. ustus
Diagrams / tables

Images hosted on other servers:

Aspergillus life cycle

Syndromes associated with aspergillosis

Spectrum of aspergillosis

Mechanism

Clinical features
  • 3 broad categories of clinical manifestation in pulmonary aspergillosis (Respir Med 2018;141:121)
    • Allergic bronchopulmonary aspergillosis (ABPA)
    • Chronic pulmonary aspergillosis (CPA)
    • Invasive pulmonary aspergillosis (IPA)
  • Allergic bronchopulmonary aspergillosis (ABPA)
  • Chronic pulmonary aspergillosis (CPA)
    • A spectrum of diseases in immunocompetent patients with a pre-existing pulmonary condition, including tuberculous and nontuberculous mycobacterial infections (most common), ABPA, COPD, treated lung cancer, asthma, pneumonia and fibrocavitary sarcoidosis (Respir Med 2018;141:121)
    • Aspergilloma (Respir Med 2018;141:121)
      • Fungal ball consisting of Aspergillus hyphae, fibrin and other debris
      • Present in almost all forms of CPA
    • Simple / single aspergilloma
    • Chronic cavitary pulmonary aspergillosis (CCPA)
      • Most common form of CPA
      • Can progress to chronic fibrosing pulmonary aspergillosis (CFPA) if untreated
    • Aspergillus nodule
      • Unusual form of CPA, mimicking malignancy, tuberculoma or coccidioidomycosis
      • Frequent necrosis but no tissue invasion
    • Subacute invasive aspergillosis
      • Commonly grouped under CPA but diagnosed and treated similarly to IPA (Respir Med 2018;141:121)
      • Formerly known as chronic necrotizing pulmonary aspergillosis
      • Mildly immunocompromised or severely debilitated patients
      • Similar clinical and radiological features to CCPA but progresses rapidly (Eur Respir J 2016;47:45)
    • Most common symptom is cough (> 90%) (Medicine (Baltimore) 2017;96:e8315)
    • Other symptoms: shortness of breath, chest pain, hemoptysis, fever, weight loss and night sweats
  • Invasive pulmonary aspergillosis (IPA)
Diagnosis
  • Allergic bronchopulmonary aspergillosis (ABPA)
    • Lung biopsy not routinely performed
    • International Society for Human and Animal Mycology (ISHAM) diagnostic criteria (Clin Exp Allergy 2013;43:850)
    • Predisposing conditions:
      • Asthma
      • Cystic fibrosis
    • Obligatory criteria (both required):
      • Type I aspergillus skin test positive or elevated IgE levels against A. fumigatus
      • Elevated total IgE levels > 1,000 IU/mL (unless all other criteria are met, then total IgE levels can be < 1,000 IU/mL)
    • Other criteria (at least 2 out of 3):
      • Presence of IgG antibodies against A. fumigatus or precipitating antibodies
      • Presence of fleeting or fixed pulmonary opacities on chest radiograph consistent with ABPA
      • Eosinophils > 500/μL in steroid naive patient (may be a historical value)
  • Chronic cavitary pulmonary aspergillosis (CCPA)
    • Diagnostic criteria from Infectious Diseases Society of America (IDSA) guidelines (Clin Infect Dis 2016;63:e1)
      • 3 months of chronic pulmonary symptoms or chronic illness or progressive radiographic abnormalities, with cavitation, pleural thickening, pericavitary infiltrates and sometimes a fungal ball
      • Aspergillus IgG antibody elevated or other microbiological data
      • No or minimal immunocompromise, usually with 1 or more underlying pulmonary disorders
  • Invasive pulmonary aspergillosis (IPA)
Laboratory
  • Standard fungal culture and identification
    • Species differentiation by fruiting body (conidial heads) morphology
  • Galactomannan (GM)
    • Polysaccharide cell wall component of Aspergillus spp., which is released during growth (Clin Infect Dis 2006;42:1417)
    • Can be measured in serum and bronchoalveolar lavage (BAL) fluid, using double sandwich ELISA technique
    • Marker of invasive aspergillosis in certain subpopulation (e.g. hematologic malignancy and HSCT recipient) (Clin Infect Dis 2016;63:e1)
    • Not recommended for screening in solid organ transplant (SOT) recipients (Scand J Infect Dis 2012;44:600)
  • Serum 1,3-beta-D glucan
    • Major cell wall components in various fungi (e.g., Aspergillus spp., Candida spp. and Pneumocystis jiroveccii) (J Clin Microbiol 2012;50:7)
    • Protease zymogen based colorimetric assay, commercially available as Fungitell®, an FDA cleared in vitro diagnostic rapid screening test
    • Marker of invasive fungal infection, not specific for genus Aspergillus
Radiology description
  • Allergic bronchopulmonary aspergillosis (ABPA)
    • Chest Xray (Indian J Radiol Imaging 2011;21:242)
      • May be normal in early stage
      • Tram line shadows, band-like (toothpaste) shadows, finger in glove opacities; represents mucoid impaction in dilated bronchi with occlusion of the distal end
    • High resolution computed tomography (HRCT) (J Clin Diagn Res 2014;8:RC05)
      • Centrilobular nodules, tree in bud pattern, mosaic attenuation and mucus impaction
    • High attenuation mucus (HAM) (J Clin Diagn Res 2014;8:RC05)
      • Pathognomonic for ABPA
      • Mucus that appears denser than the skeletal muscles on HRCT (70 - 90 Hounsfield units)
  • Chronic cavitary pulmonary aspergillosis (CCPA)
    • New or expanding pre-existing cavities of variable wall thickness in the setting of chronic lung disease, with or without fungal ball, often with pleural thickening and marked parenchymal destruction or fibrosis (Eur Respir J 2016;47:45)
  • Fungal ball: upper lobe, solid, round or oval, intracavitary mass, partially surrounded by a crescent of air (air crescent or Monad sign) (Eur Respir J 2016;47:45)
  • Invasive pulmonary aspergillosis (IPA)
    • HRCT is the imaging of choice when IPA is suspected (Clin Microbiol Infect 2018;24:e1)
    • Classical HRCT findings: macronodules (> 1 cm), surrounded by a halo of ground glass attenuation (halo sign)
    • Other findings: pleural based wedge shaped areas of consolidation, alveolar consolidations, mass-like lesion, ground glass opacities and pleural effusion
    • Delayed findings: cavity or air crescent sign
Radiology images

Images hosted on other servers:

ABPA, chest Xray and CT

CCPA, chest CT

CCPA, air crescent sign


IPA, chest Xray

IPA, chest CT

IPA, halo sign

Prognostic factors
  • Prognosis of ABPA is not well characterized (Mayo Clin Proc 2001;76:930)
  • Poor prognostic factors in CPA (Eur Respir J 2017;49:1601062)
    • History of a nontuberculous mycobacterial infection or COPD
    • Low body mass index (BMI)
    • Low serum albumin
    • Old age
    • Elevated inflammatory markers
    • Lower activity
    • Bilateral aspergillomas
  • Risk factor for developing IPA (Eur Respir Rev 2011;20:156)
    • Prolonged neutropenia (< 500 cells/mm3 for > 10 days)
    • Solid organ transplantation (highest risk is with lung transplantation and HSCT)
    • Prolonged (> 3 weeks) and high dose corticosteroid therapy
    • Hematological malignancy (risk is higher with leukemia)
    • Chemotherapy
    • AIDS
    • Chronic granulomatous disease
  • Baseline factors that predict treatment outcome in patients with invasive aspergillosis (Mycoses 2019;62:651)
    • Kidney and liver failure
    • ICU admission
    • Uncontrolled underlying disease
    • Prolonged neutropenia
    • Imaging results associated with negative outcome
      • Multiple consolidations
      • Bilateral pulmonary lesions
      • Pleural effusion
Case reports
Treatment
  • Based on guidelines from the Infectious Diseases Society of America (IDSA), endorsed by the Pediatric Infectious Diseases Society (PIDS) (Clin Infect Dis 2016;63:e1)
  • Allergic bronchopulmonary aspergillosis (ABPA)
    • Itraconazole
    • Alternative: oral voriconazole, posaconazole
    • Corticosteroids for exacerbation
  • Aspergilloma
    • Observation
    • Surgical resection in symptomatic patient (e.g., hemoptysis)
    • Alternative: itraconazole, voriconazole
  • Chronic cavitary pulmonary aspergillosis (CCPA), invasive pulmonary aspergillosis (IPA), tracheobronchial aspergillosis; same treatment
    • Voriconazole
    • Alternative: amphotericin B, isavuconazole, caspofungin, micafungin, posaconazole, itraconazole
Gross description
  • Allergic bronchopulmonary aspergillosis (ABPA)
    • Bronchiectasis from mucus impaction
    • Airway occlusion by mucus
  • Chronic cavitary pulmonary aspergillosis (CCPA)
    • Corresponds with the imaging finding of cavitary lesion, with or without fungal ball
  • Invasive pulmonary aspergillosis (IPA) (Arch Pathol Lab Med 2008;132:606)
    • Targetoid necrosis: central thrombosed vessels secondary to angioinvasion
    • Confluent bronchopneumonia or dense lobar consolidation may be seen
    • Foci of infarcted lung yields an infected pulmonary sequestrum
Gross images

Images hosted on other servers:
Abscess

Abscess

Aspergilloma Aspergilloma

Aspergilloma

Invasive aspergillosis

Systemic invasive aspergillosis

Microscopic (histologic) description
  • Organism: acute angle (< 45°) or dichotomous branching, septate hyphae, 2.5 - 4.5 µm in diameter
    • Aspergillus has a genus specific fruiting body (Arch Pathol Lab Med 2008;132:606)
      • Develops from mycelia in areas of high oxygen tension (e.g. lung, sinus cavities)
      • Does not develop in tissue
      • Composed of a vesicle and either 1 or 2 layers of phialides that produce conidia
    • Since histomorphology alone is not accurate in identification, definite classification should be based on microbiologic culture
  • Allergic bronchopulmonary aspergillosis (ABPA) (Arch Pathol Lab Med 2005;129:924)
    • Mucoid impaction of bronchi, composed of mucus and inflammatory cells (predominantly eosinophils), AKA allergic mucin
    • Bronchocentric granulomatosis
      • Airway centric necrotizing granulomatous inflammation with destruction of the airway wall
      • Dense inflammatory infiltrate with prominent eosinophils
      • May be seen in other conditions: nonfungal infection, malignancy, autoimmune disease
    • Eosinophilic pneumonia
    • Chronic or exudative bronchiolitis
    • Fungal hyphae are rarely identified, usually fragmented
  • Chronic pulmonary aspergillosis (CPA)
    • Characteristic cavitary lesion with fungal ball (Arch Pathol Lab Med 2008;132:606)
      • Cavity wall: superficial ulceration with granulation tissue, granulomatous inflammation or metaplastic squamous epithelium
      • Wall must be carefully examined, to exclude subacute invasive aspergillosis (chronic necrotizing pulmonary aspergillosis) which shows invasion of lung parenchyma but no angioinvasion
    • A. niger fungus ball can be associated with chronic pulmonary oxalosis (Arch Pathol Lab Med 2008;132:606)
      • Oxalic acid is produced by A. niger
      • Prothrombotic effect by oxalate leads to extensive ischemic necrosis
      • Resection of the fungus ball is definitive treatment
    • Splendore-Hoeppli phenomenon: deposition of antigen antibody complexes and debris from host inflammatory cells, resulting in amorphous eosinophilic material coating the mycelia (J Oral Maxillofac Pathol 2018;22:161)
  • Invasive pulmonary aspergillosis (IPA)
Microscopic (histologic) images

Contributed by Sakda Sathirareuangchai, M.D., Venna Maheshwar, M.D., Kiran Alam, M.D., Anshu Jain and Claudia Mendez, M.D.
Chronic cavitary pulmonary aspergillosis Chronic cavitary pulmonary aspergillosis Chronic cavitary pulmonary aspergillosis

Chronic cavitary pulmonary aspergillosis

Allergic bronchopulmonary aspergillosis Allergic bronchopulmonary aspergillosis

Allergic bronchopulmonary aspergillosis

Invasive pulmonary aspergillosis

Invasive pulmonary aspergillosis


Invasive pulmonary aspergillosis

Invasive pulmonary aspergillosis

Aspergilloma, PAS stain Aspergilloma, PAS stain

Aspergilloma, PAS stain

Fungal hyphae, PAS stain Fungal hyphae, PAS stain Fungal hyphae, GMS stain

GMS stain

Virtual slides

Images hosted on other servers:

ABPA

Aspergilloma

Invasive aspergillosis

Invasive aspergillosis, GMS

Cytology description
Cytology images

Images hosted on other servers:

Lung FNA, granuloma

Lung FNA, Aspergillus

BAL cytology, Aspergillus

Bronchial washing, Aspergillus

Sputum, calcium oxalate crystal

Bronchial brushing, Aspergillus

Bronchial brushing, Aspergillus


Invasive aspergillus tracheobronchitis: abundant necrotic material with multiple mycelia, consisting of septate hyphae branching at 45 degrees

Lung TBNA, Aspergillus

Positive stains
Negative stains
Electron microscopy images

Images hosted on other servers:

Scanning EM, A. fumigatus

Videos

Pulmonary aspergillosis

Sample pathology report
  • Lung, left upper lobe, wedge resection:
  • Cavitary lesion with chronic inflammation
    • Fungal hyphae identified (see comment)
    • Comment: Fungal hyphae is septate dichotomous branching. GMS stain is positive. The differential diagnosis includes Aspergillus spp., Fusarium and spp.
Differential diagnosis
  • Mucormycosis:
    • Previously known as zygomycosis
    • Nonseptate, broad hyphae with right angle (90°) branching, well stained by standard H&E
    • Less common than aspergillosis
    • Affects only immunocompromised host, especially diabetics
  • Pseudallescheria boydii (anamorph Scedosporium apiospermum):
  • Fusarium spp.:
    • An opportunistic mold with the same histomorphology as Aspergillus
    • Culture is required for definite identification (J Clin Diagn Res 2017;11:ED04)
  • Pulmonary tuberculosis:
    • Also has necrotizing granulomatous inflammation
    • Can coexist with chronic pulmonary aspergillosis
    • AFB stain positive
  • Non small cell lung carcinoma:
    • Can present with cavitary and mass lesion
    • Reactive pneumocytes from the surrounding tissue in aspergillosis may mimic carcinoma
Board review style question #1

A 65 year old man presented with a mass lesion in his left lung. He underwent a wedge biopsy, which showed benign lung parenchyma with chronic inflammation and an organism shown in the image. What is the most likely causative organism?

  1. Actinomyces israelii
  2. Aspergillus spp.
  3. Candida albicans
  4. Cryptococcus neoformans
  5. Mucor spp.
Board review style answer #1
B. Aspergillus spp.

Comment Here

Reference: Aspergillus
Board review style question #2
A 50 year old man presented with an enlarging mass-like lesion in an existing lung cavity from prior pulmonary tuberculosis. His sputum AFB was negative. There was no other parenchymal lesion in the lung. Fine needle aspiration of the lesion showed acute angle branching septate hyphae with chronic inflammation. Fungal culture grew Aspergillus fumigatus. Which is the proper diagnosis in this patient?

  1. Allergic bronchopulmonary aspergillosis
  2. Aspergillus nodule
  3. Chronic cavitary pulmonary aspergillosis
  4. Invasive pulmonary aspergillosis
  5. Tracheobronchial aspergillosis
Board review style answer #2
C. Chronic cavitary pulmonary aspergillosis

Comment Here

Reference: Aspergillus
Board review style question #3
Which of the following statement is true regrading allergic bronchopulmonary aspergillosis (ABPA)?

  1. Isolation of Aspergillus fumigatus is not required for diagnosis
  2. Lung biopsy is essential for diagnosis
  3. Mucous plug is rarely found in the airway
  4. Patients are immunocompromised host
  5. Serologic workup is not necessary
Board review style answer #3
A. Isolation of Aspergillus fumigatus is not required for diagnosis

Comment Here

Reference: Aspergillus

Asthma
Definition / general
  • Defined by the National Asthma Education and Prevention Program as a "chronic inflammatory disorder of the airways" in which many cells and cellular elements play a role - in particular, mast cells, eosinophils, T lymphocytes, macrophages, neutrophils and epithelial cells
  • In susceptible individuals, causes episodes of wheezing, breathlessness, chest tightness and coughing, particularly at night or early morning
  • Episodes are usually associated with widespread but variable airflow obstruction that is often reversible, either spontaneously or with treatment
  • Inflammation also causes an associated increase in the existing bronchial hyperresponsiveness to a variety of stimuli
  • Very common, affects 14 - 15 million Americans
  • Causes 3,000 US deaths annually (American Academy of Allergy, Asthma & Immunology)
  • Has increased in Western hemisphere over past 40 years
Pathophysiology
  • Atopic or Extrinsic: initial sensitization affects T helper 2 cells, which release IL4 / 5, which promote IgE release by B cells, mast cells, and eosinophils
  • Re-exposure to allergen leads to mediator release from mucosal mast cells
  • Acute / intermediate response is bronchoconstriction, edema, mucus secretion and vasodilation with increased vascular permeability
  • Late phase reaction is due to influx of other inflammatory cells stimulated by chemokines released by mast cells, epithelial cells, T lymphocytes and other cytokines; includes release of major basic protein from eosinophils, which causes epithelial damage and airway constriction
  • Putative mediators are leukotrienes C4, D4, E4 and acetylcholine; minor mediators are histamine, prostaglandin D2; associated with serum eosinophilia, sputum eosinophils
Clinical features
  • Atopic or Extrinsic: Type I hypersensitivity, generally due to allergens; begins in childhood, triggered by environmental allergens (dander, dust, pollen, food), often positive family history; more common in African American children; evidence of allergen sensitization; skin test causes wheel and flare reaction (CMAJ 2009;181:E181)
  • Noneosinophilic ("neutrophilic") asthma: a subgroup of atopic asthma not associated with eosinophilia; IL8 recruiting neutrophils are an important mechanism; patients tend to be less responsive to corticosteroids (Thorax 2011;66:942)
  • Nonatopic or Intrinsic: nonimmune; due to aspirin ingestion, pneumonia, cold, stress, exercise; follows respiratory infection (rhinovirus, parainfluenza virus); usually not familial; no evidence of allergen sensitization; normal serum IgE, negative skin tests; viral induced inflammation may lower threshold of subepithelial vagal receptors to irritants
  • Occupational asthma: due to repeated exposure to fumes, dusts, gases, chemicals, often in minute quantities; varying mechanisms of disease depending upon the stimulus
  • Drug induced asthma: associated with several drugs, but most noteworthy is aspirin use; rare, aspirin related cases are associated with recurrent rhinitis, nasal polyps and urticaria; patients are sensitive to small doses of aspirin; may be due to direct effects of aspirin on cyclooxygenase pathway
  • Status asthmaticus: unremitting attacks due to exposure to previously sensitized antigen; may be fatal, usually in patients with a long history of asthma
Gross description
  • Overdistended lungs, small areas of atelectasis, thick mucus plugs in proximal bronchi containing whorls of shed epithelium
Gross images

Images hosted on other servers:
Mucus plugs Mucus plugs

Mucus plugs

Status asthmaticus Status asthmaticus

Status asthmaticus

Microscopic (histologic) description
  • Curschmann spirals, eosinophils, extracellular Charcot-Leyden crystals (crystalloids composed of galectin-10, an eosinophil lysophopholipase), increased mucosal goblet cells and submucosal glands, thickened basement membrane, bronchial smooth muscle hypertrophy, airway wall edema
Microscopic (histologic) images

Contributed by Bobbi Pritt, M.D., Neil Harris, M.D. and Stacy Beal, M.D.

Curschmann spirals



Images hosted on other servers:
Smooth muscle hypertrophy and inflammatory cells Smooth muscle hypertrophy and inflammatory cells

Smooth muscle hypertrophy and inflammatory cells

Eosinophils and Charcot-Leyden crystals

Eosinophils and Charcot-Leyden crystals

Curschmann spirals Curschmann spirals Curschmann spirals

Curschmann spirals

Differential diagnosis

Atelectasis
Definition / general
  • Incomplete expansion of lung or collapse of previously inflated lung, due to obstruction / resorption, compression or contraction
  • Obstruction: resorption of oxygen causes collapse and mediastinal shifts towards affected lung; due to mucus plugs, asthma, chronic bronchitis, bronchiectasis, postoperative changes, foreign bodies, but not usually due to tumors, which cause only subtotal obstruction
  • Compression: due to fluid (congestive heart failure), air (pneumothorax), tumor and blood; causes mediastinal shifts away from affected lung
  • Contraction: due to fibrosis (restrictive lung disease)
  • Patchy: due to loss of surfactant in acute respiratory distress syndrome
  • Rounded: Localized area of subpleural lung collapse with associated pleural fibrosis; patients usually asymptomatic with occupational exposure to asbestos; usually posterior portion of a lower lobe
Gross description
  • Rounded: lung shows ill defined atelectasis with deeply invaginated pleural folds, which may disappear during grossing
Microscopic (histologic) description
  • Rounded: pleural fibrosis and invagination (use elastic stains to highlight) and atelectasis

Atypical adenomatous hyperplasia
Definition / general
  • Atypical adenomatous hyperplasia (AAH) is a small, localized proliferation of atypical pneumocytes (usually ≤ 5 mm) that line intact alveolar spaces
  • AAH is recognized as a precursor lesion to invasive adenocarcinoma (Cancer Metastasis Rev 2010;29:15)
Essential features
  • Small, localized lesion (usually ≤ 5 mm)
  • Discrete from alveolar parenchyma with proliferation of atypical pneumocytes
  • Forms a discontinuous monolayer of cells
  • Surrounding parenchyma should be devoid of substantial inflammation or fibrosis
Terminology
  • Obsolete terms (not recommended): atypical alveolar epithelial hyperplasia, atypical bronchoalveolar cell hyperplasia, atypical alveolar cell hyperplasia, bronchoalveolar cell adenoma, bronchial adenoma
ICD coding
  • ICD-O: 8250/0 - atypical adenomatous hyperplasia
  • ICD-11: 2F0Z & XH5QL3 - benign neoplasms of unspecified respiratory and intrathoracic organs & atypical adenomatous hyperplasia
Epidemiology
Sites
Pathophysiology
  • Conceptualized as the earliest morphologically recognizable lesion in the proposed stepwise progression of lung adenocarcinoma (Int J Mol Sci 2018;19:1259)
Etiology
  • Unknown; some series suggest that AAH is not correlated with smoking (Thorax 2001;56:302)
Clinical features
  • AAH is usually undetectable by imaging techniques and is typically found incidentally in surgical pathology specimens
  • Larger lesions may be present on high resolution CT imaging (Eur Radiol 2001;11:811)
Diagnosis
  • Typically diagnosed in lung resection specimens (often incidentally)
  • Usually not diagnosed on small core needle or transbronchial biopsies since the lesion should be examined in its entirety
Laboratory
  • Not typically assessed in AAH
Radiology description
Radiology images

Images hosted on other servers:

Conventional TSCT

Prognostic factors
  • Patients are cured upon resection
Case reports
Treatment
  • CT surveillance; frequency and duration dependent on size of nodule (JAMA 2022;327:264)
  • Patients are cured with resection
Gross description
  • Gray, tan-yellow, ill defined area
  • Can be multifocal
  • Localization: peripheral lung parenchyma, near pleura
  • Size: measures ≤ 5 mm
Frozen section description
  • Diagnosis is not typically made on frozen section, given the subtle changes associated with this lesion
  • Some cases with subtle atypia on frozen section, may turn out to be classified as AAH based on permanent sections
Microscopic (histologic) description
  • Proliferation of atypical pneumocytes
    • Mild to moderate atypia with increased cell size and nuclear to cytoplasmic ratio; nuclei are hyperchromatic and can have intranuclear eosinophilic inclusions
    • Cells are present along intact alveolar spaces
    • Atypical cells form a discontinuous monolayer of cells
    • Mitotic activity is typically minimal
  • Lesion is composed of a mixture of club cells and type II pneumocytes
    • Club cells can be recognized as columnar cells with eosinophilic and cytoplasmic snouts
    • Type II pneumocytes are cuboidal with finely vacuolated to clear cytoplasm
  • Absence of significant parenchymal inflammation or fibrosis
Microscopic (histologic) images

Contributed by Sherman Lin and Matthew J. Cecchini, M.D., Ph.D.

Small proliferation
of larger
hyperchromatic
cells

Proliferation of atypical pneumocytes

Discontinuous monolayer of atypical cells

Positive stains
Molecular / cytogenetics description
  • Lesional cells have been shown to be clonal in nature
  • Degree of copy number change and mutations has been shown in some studies to be less than in adenocarcinoma in situ (Nat Commun 2019;10:2978)
  • AAH has been found to have driver mutations such as KRAS, EGFR and BRAF (J Thorac Oncol 2018;13:1776)
  • Given that these lesions are cured with resection, molecular testing is not typically indicated in most clinical settings
Sample pathology report
  • Right lung, middle lobe, wedge resection:
    • Atypical adenomatous hyperplasia (AAH) (see comment)
    • 3 mm in greatest dimension
    • Comment: Atypical adenomatous hyperplasia (AAH) is a preinvasive lesion in the adenocarcinoma spectrum and is regarded as a precursor to adenocarcinoma in situ. No invasive or in situ malignancy is present.
Differential diagnosis
  • Nonmucinous (adenocarcinoma in situ):
    • Adenocarcinoma in situ (AIS) is typically > 5 mm
    • More cellular cuboidal / columnar cells forming a continuous monolayer
    • AIS typically has a sharper (abrupt) cutoff with the background lung parenchyma
    • Distinguishing between AIS and AAH can be challenging in some cases since there is overlap in morphology; however, this is of limited clinical relevance in most cases as both lesions are cured by resection
  • Reactive pneumocyte hyperplasia:
    • In reactive hyperplasia, the atypical pneumocytes tend to be more diffuse and do not form well defined nodular areas
    • There is often a background of inflammation, fibrosis or features of acute lung injury seen admixed with the atypical cells
Board review style question #1

Which of the following histologic features seen in the image above, best supports the classification of this sample as atypical adenomatous hyperplasia (AAH) over adenocarcinoma in situ (AIS)?

  1. Absence of significant inflammation
  2. Architectural changes
  3. Discontinuous nature of the atypical cells
  4. Loss of polarity
  5. Mild to moderate atypia
Board review style answer #1
C. Discontinuous nature of the atypical cells. Discontinuous atypical cells are most often seen in small atypical pneumocyte proliferations that are best classified as AAH. Architectural changes are not typically seen in AAH or AIS. Absence of significant inflammation can be seen in AAH and AIS. Mild to moderate atypia and loss of polarity can be seen in both AAH and AIS.

Comment Here

Reference: Atypical adenomatous hyperplasia
Board review style question #2
How is atypical adenomatous hyperplasia typically identified / detected?

  1. After workup for hemoptysis
  2. As an incidental solid lesion on CT scan
  3. Based on PET scan
  4. Incidental finding in resected lung tissue
Board review style answer #2
D. Incidental finding in resected lung tissue. Atypical adenomatous hyperplasia is typically identified as an incidental finding in resected lung tissue. It is not easily appreciated based on imaging studies and patients are normally asymptomatic.

Comment Here

Reference: Atypical adenomatous hyperplasia

Atypical carcinoid tumor / neuroendocrine tumor, grade 2
Definition / general
Essential features
  • Atypical carcinoids are defined as neuroendocrine tumors with 2 - 10 mitoses per 2 mm² or foci of necrosis
    • Tumors with morphologic features of carcinoid and > 10 mitoses per 2 mm² have been reported and are the focus of active research
  • Histologic features include neuroendocrine differentiation, with neuroendocrine growth patterns, salt and pepper chromatin with an inconspicuous nucleolus and moderate to abundant cytoplasm
  • Differentiation from typical carcinoid is crucial as they have a poorer prognosis and are more likely to metastasize
Terminology
  • Not recommended: moderately differentiated lung neuroendocrine carcinoma, grade 2 neuroendocrine carcinomas
  • Well differentiated neuroendocrine tumors G1 to G3 nomenclature is not currently applied in pulmonary carcinoids (Mod Pathol 2018;31:1770)
ICD coding
  • ICD-O: 8249/3 - atypical carcinoid tumor
  • ICD-10: C7A.090 - malignant carcinoid tumor of the bronchus and lung
  • ICD-11: 2C25.4 & XH51K1 - carcinoid or other malignant neuroendocrine neoplasms of bronchus or lung and neuroendocrine tumor, grade 2
Epidemiology
Sites
Etiology
  • Unknown
  • Can arise in the context of diffuse idiopathic pulmonary neuroendocrine cell hyperplasia and tumorlets (Thorax 2007;62:248)
Clinical features
Diagnosis
  • Even if a diagnosis of carcinoid tumor can be made with confidence on a biopsy or cytology sample, the definitive diagnosis of atypical carcinoid can only be made on a surgical resection, unless necrosis or increased mitotic activity is seen (Ann Oncol 2015;26:1604)
Laboratory
Radiology description
Radiology images
Prognostic factors
Case reports
  • 25 year old woman with a postpneumonectomy-like syndrome due to a bronchial atypical carcinoid tumor (BMC Pulm Med 2019;19:44)
  • 30 year old woman with a subcutaneous metastasis of a lung atypical carcinoid tumor (Medicine (Baltimore) 2018;97:e9415)
  • 45 year old woman with an endobronchial atypical carcinoid tumor with postobstructive mycobacterial infection (BMC Pulm Med 2019;19:41)
  • 77 year old woman with a bronchial typical carcinoid lung tumor and diffuse idiopathic neuroendocrine cell hyperplasia in the distal lung (J Thorac Dis 2017;9:E774)
Treatment
Clinical images

Images hosted on other servers:

Bronchial occlusion

Endobronchial tumor

Tumor near trachea

Gross description
  • Can be similar to typical carcinoids, as they are well circumscribed and round / ovoid tumors but differ in certain points (Cancer 2008;113:5)
    • On average, atypical carcinoids are larger
    • Cut surface is white-gray or tan like typical carcinoids but can be less homogeneous with pink or yellow-brown or red areas
Gross images
Frozen section description
Frozen section images
Microscopic (histologic) description
  • Diagnostic criteria:
    • Neuroendocrine morphology with 2 - 10 mitoses per 2 mm² or presence of necrosis
      • Necrosis can be in large zones but is usually punctate
      • Mitotic rate should be counted in the area with the highest proliferation rate (hot spot)
      • If mitotic rate is near cutoffs, assessment should be made on three sets of 2 mm² and their mean should count as the final mitotic rate
    • Rare tumors with morphologic features of carcinoid and > 10 mitoses per 2 mm² have been reported (Virchows Arch 2017;471:713, Am J Surg Pathol 2017;41:263, Diagn Pathol 2019;14:104)
  • Neuroendocrine histologic patterns similar to typical carcinoids: organoid, trabecular, rosette formation, papillary, pseudoglandular, follicular
  • Tumor cells are as typical carcinoid: uniform with a polygonal shape, round to oval nuclei with salt and pepper chromatin and inconspicuous nucleoli, along with moderate to abundant eosinophilic cytoplasm
  • Stroma is fine and highly vascularized; hyalinization, cartilage or bone formation are possible
Microscopic (histologic) images

Contributed by Philippe Joubert, M.D., Ph.D.
Peripheral carcinoid

Peripheral carcinoid

Histologic pattern: rosettes

Histologic pattern: rosettes

Histologic pattern: organoid

Histologic pattern: organoid

Histologic pattern: papillary

Histologic pattern: papillary

Histologic pattern: solid

Histologic pattern: solid

Cytologic features: salt and pepper chromatin

Cytologic features: salt and pepper chromatin


Cytologic features: spindle cell features

Cytologic features: spindle cell features

Cytologic features: eosinophilic cytoplasm

Cytologic features: eosinophilic cytoplasm

 Increased mitotic rate Increased mitotic rate

Increased mitotic rate

Necrosis (larger focus) Necrosis (larger focus)

Necrosis (larger focus)


Necrosis (punctate)

Necrosis (punctate)

Lymphovascular invasion with punctate necrosis

Lymphovascular invasion with punctate necrosis

Stroma: ossification

Stroma: ossification

AE1/3

AE1 / AE3

CD56

CD56

Chromogranin

Chromogranin


TTF1

TTF1

Ki67

Ki67

Elevated proliferation rate

Elevated proliferation rate

Virtual slides

Images hosted on other servers:

Atypical lung tumor with focal necrosis

Cytology description
  • Cells and architecture similar to typical carcinoid tumors but can differ in several ways (Cibas: Cytology - Diagnostic Principles and Clinical Correlates, 4th edition, 2014)
    • Groups of cells tend to be looser, with more isolated cells; rosette structures might be seen
    • Population of tumor cells can be less uniform with slight pleomorphism
    • Focal necrosis can be seen
    • Mitoses can be seen but should be rare
    • Even though the diagnosis can be suggested, a thorough examination of the surgical resection remains necessary to classify the tumor
Cytology images
Positive stains
Negative stains
Molecular / cytogenetics description
  • Mutations in the chromatin remodeling genes, including MEN1 and SWI/SNF complex (Transl Lung Cancer Res 2017;6:513)
  • RB1 and TP53 are uncommon
  • Low number of chromosomal imbalances
Sample pathology report
  • Right lung, superior lobe, transbronchial biopsy:
    • Carcinoid tumor, not further classified (see comment)
    • Comment: The presence of focal necrosis or increased mitotic activity on this biopsy combined with the classic morphology of a carcinoid tumor favors a diagnosis of atypical carcinoid. However, the definitive diagnosis will be made on the resection specimen.
Differential diagnosis
Board review style question #1

    A patient undergoes a lobectomy for a well circumscribed nodule. On H&E slide, the tumor exhibits a well differentiated neuroendocrine morphology and you observe the histologic features presented in the image. Which of the following statements is true?

  1. A Ki67 proliferation rate of > 10% is diagnostic
  2. It is defined as a well differentiated neuroendocrine tumor with 2 - 10 mitoses per 2 mm² or foci of necrosis
  3. It is defined as a well differentiated neuroendocrine tumor with 2 - 10 mitoses in 1 high power field or foci of necrosis
  4. This diagnosis can be made with certainty on small samples (biopsies and cytology)
Board review style answer #1
B. It is defined as a well differentiated neuroendocrine tumor with 2 - 10 mitoses per 2 mm² or foci of necrosis. The picture shows a carcinoid lung tumor with a classical neuroendocrine morphology and 2 mitoses in 1 high power field. Even though the whole tumor is not presented here, the presence of 2 mitoses is sufficient for an atypical carcinoid diagnosis.

While Ki67 proliferation rate is frequently > 10% in atypical carcinoids, this feature is not part of the diagnosis (A). C is nearly exact but mitotic count is not made on 1 high power field. Finally, the diagnosis can be suggested on small samples but a thorough examination of a resection specimen is necessary to confirm an atypical carcinoid diagnosis (D).

Comment Here

Reference: Atypical carcinoid
Board review style question #2

    Regarding pulmonary atypical carcinoids, which of the following statements is true?

  1. Differentiating them from typical carcinoid is crucial as they have a poorer prognosis and are more likely to metastasize
  2. Neuroendocrine immunohistochemical markers (chromogranin, synaptophysin and CD56) are systematically positive
  3. They are less likely to be peripherally located than typical carcinoids
  4. They frequently harbor ALK-ELM4 fusions
Board review style answer #2
A. Differentiating them from typical carcinoid is crucial as they have a poorer prognosis and are more likely to metastasize. Neuroendocrine immunohistochemical markers are not always positive and can be completely negative in a small subset of atypical carcinoids (B). Regarding their location, atypical carcinoids are more frequently peripherally located and typical carcinoids are more frequently central (C). ALK-EML4 fusions are not found in atypical carcinoids (D).

Comment Here

Reference: Atypical carcinoid

Benign metastasizing leiomyoma
Definition / general
  • Rare, benign appearing smooth muscle tumor in lung
  • Generally thought to represent hematogenous spread from a uterine leiomyoma, given evidence of clonal derivation / similarities in cytogenetic abnormalities (Mod Pathol 2006;19:130, Am J Surg Pathol 2007;31:737)
Essential features
  • Benign smooth muscle tumor in lung
  • Arises from hematogenous spread of uterine leiomyoma, usually years following surgical manipulation
  • Hormonally responsive
  • Multifocal; metastasis from leiomyosarcoma must be ruled out
Terminology
  • Metastasizing leiomyoma
  • First described by Steiner in 1939 as metastasizing fibroleiomyoma of the uterus
  • Older term was multiple fibroleiomyomatous hamartomas, when thought to originate in lung
ICD coding
  • ICD-O: 8898/1 - metastasizing leiomyoma
  • ICD-11: XH1EX8 - metastasizing leiomyoma
Epidemiology
Sites
Pathophysiology
  • Exact pathophysiology is unclear
  • Most accepted hypothesis in the literature is that lesions are clonally derived from uterine leiomyomas (Mod Pathol 2006;19:130, Am J Surg Pathol 2007;31:737)
  • Hormonally dependent; there is evidence of disease progression in pregnancy and regression after menopause
Etiology
  • Hematogenous spread (metastasis) from benign uterine leiomyoma, often after surgical manipulation (myomectomy or hysterectomy)
Clinical features
  • Asymptomatic; incidentally discovered on imaging
  • 30% of patients present with symptoms including chest pain, back pain, cough, dyspnea, hemoptysis, pneumothorax and respiratory failure
  • Lesions develop months to years after myomectomy; mean duration between hysterectomy and appearance of lung lesions is 23 years (Clin Chest Med 2016;37:589, Ann Thorac Surg 2016;101:253)
Diagnosis
Radiology description
Radiology images

Images hosted on other servers:

CT of benign metastasizing leiomyoma in lung

Prognostic factors
Case reports
Treatment
  • No specific guidelines for management
  • Watchful management for asymptomatic patients
  • Surgical resection of symptomatic lesions or lesions with interval growth
  • Medical hormonal suppression (J Clin Endocrinol Metab 2004;89:3183)
  • Oophorectomy for premenopausal women
Gross description
  • Tan-white, well circumscribed, rounded, rubbery to firm, with a whorled cut surface
Gross images

Contributed by Roseann Wu, M.D., M.P.H.

Homogenous whorled cut surface

Frozen section description
  • Bland spindle cell proliferation, arranged in fascicles
Microscopic (histologic) description
  • Well circumscribed, solitary or multiple nodules of smooth muscle, similar to uterine counterparts (BMC Pulm Med 2020;20:292, Case Rep Oncol Med 2014;2014:842801)
  • Whorled to intersecting fascicles of ovoid to elongated spindled cells with abundant eosinophilic cytoplasm, oval to cigar or boxcar shaped nuclei, inconspicuous nucleoli
  • Large, irregular, thick walled vessels
  • May contain cysts or entrapped epithelium
  • No atypia, no necrosis, no vascular invasion, no mitotic figures
Microscopic (histologic) images

Contributed by Roseann Wu, M.D., M.P.H.

Well circumscribed lesion

Bland spindle cells

Cytology description
  • Generally acellular or paucicellular on fine needle aspiration
  • Nonspecific cytologic findings: clusters of uniform, bland, cohesive spindle cells with no mitoses, no necrosis and no significant atypia
Cytology images

Images hosted on other servers:
FNA, large clusters of spindle shaped cells FNA, large clusters of spindle shaped cells

FNA, large clusters of spindle shaped cells

Electron microscopy description
  • Spindle cells show basal lamina material, pinocytotic vesicles, intracytoplasmic microfilaments with focal densities
Molecular / cytogenetics description
Molecular / cytogenetics images

Images hosted on other servers:

PCR amplification of lung and uterine tumors

Telomere length by FISH

Videos

Benign metastasizing leiomyoma

Sample pathology report
  • Lung, left upper lobe, mass, wedge resection:
    • Low grade spindle cell neoplasm, consistent with benign metastasizing leiomyoma (see comment)
    • Comment: Patient’s history of uterine leiomyoma is noted. Immunohistochemical studies were performed with adequate controls on block (A1). The tumor cells are positive for SMA, actin, desmin, ER and PR. The tumor cells are negative for pankeratin, TTF1, HMB45, STAT6 and chromogranin. The morphology and immunohistochemical profile supports the above diagnosis.
Differential diagnosis
Board review style question #1

A 45 year old woman presents with multiple incidentally discovered, well circumscribed, bilateral lung nodules on CT scan. She is asymptomatic and her history is only notable for a myomectomy, 10 years ago. A biopsy of one of the nodules is performed (see figure above). On immunohistochemical staining, the lesion is negative for STAT6 and the Ki67 proliferative index is low. What is the most likely diagnosis?

  1. Benign metastasizing leiomyoma
  2. Leiomyosarcoma
  3. Pulmonary hamartoma
  4. Solitary fibrous tumor
Board review style answer #1
A. Benign metastasizing leiomyoma

Comment Here

Reference: Benign metastasizing leiomyoma
Board review style question #2

A 45 year old woman presents with multiple incidentally discovered, well circumscribed, bilateral lung nodules on CT scan. She is asymptomatic and her history is only notable for a myomectomy 10 years ago. A biopsy of one of the nodules is performed (see figure above). On immunohistochemical staining, the lesion is negative for STAT6 and the Ki67 proliferative index is low. Based on this specific clinical presentation, what is the best choice for treatment?

  1. Chemotherapy
  2. Follow up imaging
  3. Hormonal therapy
  4. Surgical resection
Board review style answer #2
B. Follow up imaging

Comment Here

Reference: Benign metastasizing leiomyoma

Biopsies
Definition / general
  • Precise histologic classification of lung cancer is essential due to advances in tailored therapies for specific subtypes
  • Biopsy is the initial step for the classification of advanced or metastatic disease and for obtaining tissue for molecular / biomarker testing
  • Obtaining multiple biopsy specimens to allow for both IHC and molecular / biomarker testing is optimal
Essential features
  • Diagnosis in small biopsies utilizes morphologic features and judicial immunohistochemical / special stain evaluation in cases with ambiguous morphologies
  • Biopsy materials should be preserved as much as possible for molecular / biomarker testing
  • Diagnostic terminology can differ between small biopsies and resection specimens
Epidemiology
Sites
  • Biopsies may be obtained from either primary or metastatic sites
Diagrams / tables

Images hosted on other servers:

Terminology in small biopsy and cytology versus resection specimen

Small biopsy and cytologic preparation guidelines

Diagnosis
Terminology in small biopsy and cytology versus resection specimens for adenocarcinoma and squamous cell carcinoma (adapted from Table 3, J Thorac Oncol 2022;17:362)
Morphology / stains Terminology for small biopsies and cytology specimens Terminology for resection specimens
Morphologic squamous cell patterns clearly present Squamous cell carcinoma Squamous cell carcinoma
Morphologic adenocarcinoma patterns clearly present Adenocarcinoma (list patterns in the diagnosis) Adenocarcinoma predominant patterns:

Lepidic
Acinar
Papillary
Solid
Micropapillary
Adenocarcinoma with lepidic pattern (if pure, list the differential diagnosis on the right and add a comment that an invasive component cannot be excluded) Minimally invasive adenocarcinoma, adenocarcinoma in situ or an invasive adenocarcinoma with a lepidic component
Invasive mucinous adenocarcinoma (list the patterns; use the term mucinous adenocarcinoma with lepidic pattern if pure lepidic pattern and mention the differential diagnosis listed on the right) Invasive mucinous adenocarcinoma, minimally invasive adenocarcinoma or adenocarcinoma in situ, mucinous type
Adenocarcinoma with colloid features Colloid adenocarcinoma
Adenocarcinoma with fetal features Fetal adenocarcinoma
Adenocarcinoma with enteric features a Enteric adenocarcinoma
Morphologic squamous cell patterns not present but supported by stains (i.e., p40+) Non-small cell carcinoma, favor squamous cell carcinoma b Squamous cell carcinoma (nonkeratinizing pattern may be a component of the tumor)
Morphologic adenocarcinoma patterns not present but supported by special stains (i.e., TTF1+) Non-small cell carcinoma, favor adenocarcinoma b Adenocarcinoma (solid pattern may just be one component of the tumor)
No clear adenocarcinoma, squamous or neuroendocrine morphology or staining pattern Non-small cell carcinoma, NOS c Large cell carcinoma

  1. Metastatic carcinomas should be carefully excluded with clinical and appropriate but judicious immunohistochemical examination
  2. Categories do not always correspond to solid predominant adenocarcinoma or nonkeratinizing squamous cell carcinoma, respectively; poorly differentiated components in adenocarcinoma or squamous cell carcinoma may be sampled
  3. Non-small cell carcinoma, NOS pattern can be seen not only in large cell carcinomas but also when the solid, poorly differentiated component of adenocarcinomas or squamous cell carcinomas is sampled but does not express immunohistochemical markers or mucin


Terminology for small biopsies and cytology versus resection specimens for small cell carcinoma, large cell neuroendocrine carcinoma, adenosquamous carcinoma and pleomorphic carcinoma (adapted from Table 4, J Thorac Oncol 2022;17:362)
Terminology for small biopsies and cytology specimens Terminology for resection specimens
Small cell carcinoma Small cell carcinoma
Non-small cell carcinoma with neuroendocrine morphology and positive neuroendocrine markers, possible large cell neuroendocrine carcinoma Large cell neuroendocrine carcinoma
Morphologic squamous cell and adenocarcinoma patterns both present: non-small cell carcinoma, NOS

(Comment that adenocarcinoma and squamous components are present and that this could represent adenosquamous carcinoma)
Adenosquamous carcinoma (if both components ≥ 10%)
Morphologic squamous cell or adenocarcinoma patterns not present but immunohistochemical stains favor separate squamous and adenocarcinoma components: non-small cell carcinoma, NOS

(Specify the results of the immunohistochemical stains and the interpretation and comment that this could represent adenosquamous carcinoma but that diagnosis requires a resection specimen)
Adenocarcinoma, squamous cell carcinoma, adenosquamous carcinoma or large cell carcinoma with unclear immunohistochemical features
Non-small cell carcinoma with spindle cell or giant cell carcinoma

(Mention if adenocarcinoma or squamous carcinoma is present and comment that this could represent a pleomorphic carcinoma; however, that diagnosis requires a resection specimen)
Pleomorphic, spindle cell or giant cell carcinoma

Guidelines for good practice of small biopsies and cytologic preparations (adapted / reprinted from J Thorac Oncol 2022;17:362):
  1. For small biopsies and cytology, NSCC should be further classified into a more specific type, such as ADC or SQCC
  2. The term non-small cell lung carcinoma, NOS (NSCLC, NOS) should be used as little as possible
  3. When a diagnosis is made in a small biopsy or cytology specimen in conjunction with special studies, it should be clarified whether the diagnosis was established on the basis of light microscopy alone or if special stains were required
  4. The term nonsquamous cell carcinoma (non-SQCC) should not be used by pathologists
  5. Terminology in above tables should be used in routine diagnosis, future research and clinical trials, in order to ensure a uniform classification of disease cohorts in relation to tumor subtypes
  6. When paired cytology and biopsy specimens exist, they should be reviewed together to achieve the most specific and concordant diagnosis
  7. The terms AIS and minimally invasive ADC should not be used for diagnosis of small biopsies or cytology specimens (a noninvasive pattern is present in a small biopsy should be referred to as a lepidic growth pattern)
  8. The term large cell carcinoma should not be used for diagnosis in small biopsy or cytology specimens and should be restricted to resection specimens where the tumor is thoroughly sampled to exclude a differentiated component
  9. Tumors exhibiting sarcomatoid features (marked nuclear pleomorphism, malignant giant cells or spindle cell morphology) should be classified as specifically as possible (i.e., ADC; NSCC, favor ADC; SQCC; or NSCC favor SQCC) with an additional statement that giant or spindle cell features are present (note: if diagnosis specific features are not present, the term NSCC, NOS should be used, again with comment on the sarcomatoid features)
  10. Neuroendocrine immunohistochemical markers should be performed only in cases where there is suspected neuroendocrine morphology

(Abbreviations: ADC, adenocarcinoma; AIS, adenocarcinoma in situ; NOS, not otherwise specified; NSCC, non-small cell carcinoma; SQCC, squamous cell carcinoma)
Treatment
  • Identification of targetable mutations has led to overall decrease in NSCLC mortality disproportionate to the decrease in incidence (N Engl J Med 2020;383:640)
  • PDL1 evaluation recommended for all nonneuroendocrine carcinomas (J Thorac Oncol 2022;17:362)
Microscopic (histologic) description
  • Adenocarcinoma: gland formation, mucin production and variety of architectural patterns
  • Squamous cell carcinoma: intracellular bridges, keratinization
  • Adenosquamous carcinoma: mixed adenocarcinomatous and squamous differentiation or solid proliferation without obvious differentiation but with immunophenotypic evidence of both squamous and glandular differentiation; can only be diagnosed on resection specimens
  • Non-small cell carcinoma: carcinoma without obvious glandular or squamous differentiation further characterized by immunostaining
  • Neuroendocrine tumors: salt and pepper chromatin, organoid / trabecular / nested growth
  • Small cell carcinoma: fine chromatin, nuclear molding, marked mitotic activity, necrosis
Microscopic (histologic) images

Contributed by Phillip McMullen, M.D., Ph.D.
Lung mass Lung mass

Lung mass

TTF1

TTF1

p40

p40

Positive stains
Molecular / cytogenetics description
  • CAP / IASLC / AMP guidelines for molecular testing of lung cancer (Arch Pathol Lab Med 2018;142:321):
    • EGFR, ALK and ROS1 are “must test” genes
    • BRAF, MET, ERBB2 (HER2) and RET are recommended for patients suitable for clinical trials
    • KRAS sequencing may be helpful to exclude KRAS mutant patients from extended panel testing
  • While these recommendations are primarily regarding pulmonary adenocarcinoma, physicians may utilize molecular testing in nonadenocarcinomatous tumors when there are clinical features that may indicate the presence of an oncogenic driver
Sample pathology report
  • Right lung, upper lobe, transbronchial biopsies:
    • Non-small cell carcinoma, favor adenocarcinoma (see comment)
    • Comment: The tumor cells lack morphologic evidence of squamous or glandular differentiation. Immunohistochemical studies performed with appropriate controls reveal the tumor cells are positive for TTF1 and negative for p40. The results support the rendered diagnosis and favor adenocarcinoma of lung origin.
Board review style question #1
Transbronchial biopsy of a PET avid lung mass reveals a solid proliferation of malignant epithelial cells lacking obvious keratinization and glandular morphology. The tumor cells show variable p40 reactivity and are negative for both TTF1 and mucicarmine. What is the best designation for this tumor?

  1. Non-small cell carcinoma, favor adenocarcinoma
  2. Non-small cell carcinoma, favor squamous cell carcinoma
  3. Non-small cell carcinoma, not otherwise specified
  4. Squamous cell carcinoma
  5. Squamous cell carcinoma, nonkeratinizing
Board review style answer #1
B. Non-small cell carcinoma, favor squamous cell carcinoma. A poorly differentiated malignancy without morphologic evidence of keratinization / squamous differentiation or glandular / adenocarcinomatous differentiation requires judicious use of immunostains for further classification. A basic panel of TTF1 and p40 is sufficient to classify most lung tumors. Addition of mucicarmine is useful in occasional cases of TTF1 negative adenocarcinomas. Such undifferentiated tumors, in small biopsies, are primarily classified as non-small cell carcinoma with further classifications being favored based on the results of ancillary studies. Given the variable positivity for p40 and negative TTF1 and mucicarmine results, the best classification would be non-small cell carcinoma, favor squamous cell carcinoma.

Comment Here

Reference: Lung - Biopsies
Board review style question #2
Transbronchial biopsy of a PET avid lung mass reveals a solid proliferation of malignant epithelioid cells lacking obvious keratinization and glandular morphology. The tumor cells show no reactivity with TTF1, p40 or mucicarmine. The clinical radiologic features support a primary lung malignancy. What is the best designation for this tumor?

  1. Adenocarcinoma, solid pattern
  2. Non-small cell carcinoma, favor adenocarcinoma
  3. Non-small cell carcinoma, favor large cell carcinoma
  4. Non-small cell carcinoma, not otherwise specified
  5. Squamous cell carcinoma, nonkeratinizing
Board review style answer #2
D. Non-small cell carcinoma, not otherwise specified. A poorly differentiated malignancy without morphologic evidence of keratinization / squamous differentiation or glandular / adenocarcinomatous differentiation requires judicious use of immunostains for further classification. A basic panel of TTF1 and p40 is sufficient to classify most lung tumors. Addition of mucicarmine may be useful for occasional cases of TTF1 negative adenocarcinomas. Such undifferentiated tumors, in small biopsies, are primarily classified as non-small cell carcinoma with further classifications being favored based on the results of ancillary studies. In this case, there is no reactivity with ancillary studies to allow further designation. The absence of reactivity with ancillary studies could reflect a diagnosis of large cell carcinoma but could also represent sampling of an undifferentiated portion of an otherwise poorly differentiated squamous cell carcinoma or adenocarcinoma. The diagnosis of large cell carcinoma requires a resection specimen and thorough sampling to ensure that a differentiated component is not missed.

Comment Here

Reference: Lung - Biopsies
Board review style question #3


A 75 year old man with a 100 pack year smoking history presents with hemoptysis and is found to have a hilar mass and lymphadenopathy. Biopsy of one of the lymph nodes reveals the proliferation seen in the above images. Mitotic activity is approximately 35 mitoses per 2 mm2 and necrosis is present. Immunophenotyping is attempted; however, the block was inadvertently exhausted and stains are not available. What is the best diagnosis that can be rendered?

  1. Atypical carcinoid tumor
  2. Carcinoid tumor, not otherwise specified
  3. Epithelial neoplasm, not further classifiable
  4. No diagnosis can be rendered, rebiopsy is needed
  5. Small cell carcinoma
Board review style answer #3
E. Small cell carcinoma. Small cell carcinoma remains a morphologic diagnosis and does not require immunophenotyping for confirmation. The clinical history and description of the malignant cells are consistent with a diagnosis of small cell carcinoma. Immunophenotyping, including the use of Ki67, is most useful in cases with more ambiguous morphologic features or in crushed biopsies where morphologic features are obscured.

Comment Here

Reference: Lung - Biopsies

Bronchial atresia
Definition / general
  • Portion of bronchial tree with normal branching pattern, but without any demonstrable connection to the central bronchial tree
  • Congenital abnormality due to focal interruption of lobar, segmental or subsegmental bronchus with associated peripheral mucus impaction (bronchocele, mucocele) and hyperinflation of obstructed lung segment; most commonly affects apicoposterior segmental bronchus of left upper lobe, then segmental bronchi of right upper, middle and lower lobes
  • Cause unknown, but appears to occur before birth
  • Lobar bronchial atresia can mimic mainstem bronchial atresia on mid trimester ultrasound (Ultrasound Obstet Gynecol 2011;37:110)
Case reports

Bronchiectasis
Definition / general
  • Permanent dilation of bronchi and bronchioles caused by the destruction of mucosal and elastic tissues; often caused by or associated with chronic necrotizing infection of bronchi and bronchioles
Essential features
  • Gross pathology shows markedly distended peripheral bronchi, often with mucoid impaction and possible mucosal ulcerations
  • Histology shows a range of findings in the acute setting that include luminal dilation, mucostasis, acute inflammation and inflammatory debris
    • Usual respiratory type epithelium may be ulcerated and replaced by granulation tissue or squamous metaplasia
    • Chronic inflammation may be variably dense and include prominent lymphoid follicles and granulomas
    • Chronic findings include loss of smooth muscle, cartilage and elastic fibers with replacement fibrosis
  • Can mimic squamous cell carcinoma on frozen section; on histology, need to distinguish from other lung diseases, such as chronic obstructive pulmonary disease (COPD), emphysema, cystic fibrosis and asthma
ICD coding
  • ICD-10:
    • J47.9 - bronchiectasis, uncomplicated
    • J47.0 - bronchiectasis with acute bronchitis or lower respiratory infection
    • J47.1 - bronchiectasis with (acute) exacerbation
    • Q33.4 - congenital bronchiectasis
  • ICD-11:
    • CA24 - bronchiectasis
    • 1B10.0 - respiratory tuberculosis, confirmed
Epidemiology
  • About 350,000 to 500,000 adults in the United States have bronchiectasis (Chron Respir Dis 2017:14:377)
  • Increased risk of noncystic fibrosis bronchiectasis with extremes of ages (< 5 years, adults > 75 years)
  • Associated with underserved populations
  • Increased incidence of childhood bronchiectasis in Maori, Pacific Islanders of New Zealand, Australian aboriginal and Alaskan native children (Expert Rev Respir Med 2017;11:517)
Sites
  • Bronchi walls in lungs
Pathophysiology
  • Obstruction (i.e., due to tumor, foreign body, inspissated mucus) (Int J Chron Obstruct Pulmon Dis 2009;4:411)
    • Causes resorption of air distal to the obstruction
    • Causes atelectasis and accumulation of intraluminal secretions
  • Nonobstructive bronchiectasis (i.e., due to pneumonia and atelectasis) (Int J Chron Obstruct Pulmon Dis 2009;4:411)
    • Increased negative intrapleural pressure creates a force on bronchial walls, leading to dilation
  • Infectious bronchiectasis (N Engl J Med 2002;346:1383)
    • Enhanced cellular and mediator responses lead to destruction of tissue
    • Biopsies have shown infiltration by neutrophils and T lymphocytes
    • Chemoattractants have been detected (IL8, TNFα, prostanoids)
Etiology
  • Idiopathic (Paediatr Respir Rev 2011;12:104)
  • Congenital
    • Cystic fibrosis: most common cause in children
    • Primary ciliary dyskinesia and other mucociliary disorders
    • Immune deficiencies (hypogammaglobulinemia, MHC1 deficiency, TAP1 deficiency, IFNγ receptor deficiency)
    • Alpha-1 antitrypsin deficiency
    • Williams-Campbell syndrome
  • Infection
    • Mycobacterial infection: most common cause globally
    • Viral infection
    • Allergic bronchopulmonary aspergillosis
    • Postinfection sequelae (pneumonia, measles, whooping cough): most common, known noncystic fibrosis cause in adults
  • Autoimmune
    • Rheumatoid arthritis
  • Other causes
    • Chronic obstructive pulmonary disease / smoking
    • Aspiration
    • Extremes of age
    • Immune dysfunction: HIV
    • Foreign body / obstruction (i.e., tumor)
    • Malnutrition
Diagrams / tables

Images hosted on other servers:

Morphological types

Different etiologies of bronchiectasis

Clinical features
  • Cough and chronic sputum production (Thorax 2019;74:1)
    • Sputum is typically mucoid to mucopurulent, thick
    • Can be bloody due to erosive airway damage
  • Dyspnea and wheezing
  • Pleuritic chest pain
Diagnosis
  • Physical exam (Thorax 2019;74:1)
    • Abnormal breath sounds: crackles, wheezing, rhonchi
    • Digital clubbing
  • CT scan to differentiate between bronchiectasis and COPD (can overlap)
Laboratory
  • CBC with differential
  • Depends on cause of bronchiectasis
    • If infectious: sputum culture positive for infectious etiology
    • If congenital: positive sweat chloride test, positive for genetic mutations, etc.
  • Immunoglobulin quantitation
Radiology description
  • Xray (Radiopaedia: Bronchiectasis [Accessed 25 August 2022]):
    • Tram track opacities
    • Air fluid levels
    • Increase in bronchovascular markings
  • CT:
    • Tram track sign: thickened nontapering bronchial walls
    • Signet ring sign: dilated bronchus with pulmonary artery branch seen in cross section looks like a signet ring; bronchus and artery should be the same size
    • String of pearls sign
    • Bunch of grapes sign: closely apposed dilated bronchi appear to look like a bunch of grapes
    • Bronchus visualized within 1 cm of pleural surface
    • Lack of tapering
    • Increased bronchoarterial ratio
    • Bronchial wall thickening
    • Mucoid impaction, air trapping
Radiology images

Images hosted on other servers:

CT, coronal cross section

CT, axial cross section

Prognostic factors
  • Bronchiectasis severity index
    • Endorsed by the Thoracic Society of Australia and New Zealand
    • Uses clinical, radiological and microbiological features to predict morbidity and mortality in noncystic fibrosis bronchiectasis
    • Point system on a scale from 0 - 9
      • 0 - 4: mild bronchiectasis
      • 5 - 8: moderate bronchiectasis
      • 9+: severe bronchiectasis
    • Calculator: University of Dundee: Bronchiectasis Prediction Tools [Accessed 25 August 2022]
  • FACED score (Open Respir Med J 2015;9:46)
    • Uses a 0 - 7 point scale to determine the severity of bronchiectasis in noncystic fibrosis patients
    • FACED stands for different metrics
      • F: forced expiratory volume in 1 second (FEV1) (≥ 50% = 0 points, < 50% = 2 points)
      • A: age (< 70 years = 0 points, ≥ 70 years = 2 points)
      • C: chronic colonization (no Pseudomonas = 0 points, presence of Pseudomonas = 1 point)
      • E: extension (1 - 2 lobes affected = 0 points, > 2 lobes affected= 1 point)
      • D: dyspnea; modified Medical Research Council scale (mMRC) (0 - 2 = 0 points, 3 - 4 = 1 point)
Case reports
Treatment
  • Nonpharmacologic therapies (UpToDate: Bronchiectasis in Adults [Accessed 25 August 2022]):
    • Avoid lung irritant
    • Physiotherapy to clear airways
    • Mucolytics
    • Airway hydration
    • Pulmonary rehabilitation for severe cases
  • Antibiotics to prevent exacerbations:
    • Macrolides: especially for Pseudomonas aeruginosa
  • Other pharmacologic treatments:
    • Bronchodilators
    • NSAIDs
    • Glucocorticoids
Gross description
  • Markedly distended peripheral bronchi, usually in lower lobes; can be traced up to pleura (Zander: Pulmonary Pathology, 2nd Edition, 2017)
    • Variety of shapes present: saccular, cystic, cylindrical
  • Ulcerations of mucosal surfaces
  • Irregularly thickened bronchial walls
  • Honeycomb appearance on cut surface
  • Can have thick mucoid or mucopurulent secretions present in the bronchi
Gross images

Contributed by Kristine Konopka, M.D.

Dilated airways with mucoid deposition

Dilated airways

Frozen section description
  • Can be challenging on frozen section to distinguish from carcinoma (Int J Clin Exp Pathol 2015;8:7961)
    • Bronchial epithelial shedding
    • Epithelial squamous metaplasia: can be confused for carcinoma in situ
    • Bronchial mural destruction
Frozen section images

Images hosted on other servers:

Squamous metaplasia in bronchiectasis

Microscopic (histologic) description
  • Inflammatory cells along airways (Zander: Pulmonary Pathology, 2nd Edition, 2017)
    • Prominent chronic inflammation with lymphoid follicles and germinal centers
    • Varying degrees of neutrophilic inflammation
    • Can have eosinophils depending on the cause or if the patient has superimposed diseases
  • Epithelial ulceration
  • Over time
    • Loss of smooth muscle, elastic fibers, cartilage
    • Replacement fibrosis
  • Coexisting findings
    • Organizing pneumonia
    • Pulmonary hypertension
Microscopic (histologic) images

Contributed by Heather I-Hsuan Chen-Yost, M.D.

Lung explanted for cystic fibrosis

Diffuse inflammation and mucus plugging

Mixed acute and chronic inflammation

Lung explanted for idiopathic pulmonary fibrosis

Features of organizing pneumonia

Virtual slides

Images hosted on other servers:

Bronchiectasis with severe obstructive pneumonia

Mycetoma (aspergilloma) with background bronchiectasis

Cystic fibrosis (bronchiectasis with severe and chronic features)

Endobronchial hamartoma with postobstructive bronchiectasis

Cytology description
  • Seen in bronchial brushings and washings (Diagn Cytopathol 2008;36:13)
    • Mixed inflammation
    • Cellular debris
    • Mucus
    • Reactive bronchial cells
    • Squamous metaplasia
Videos

Histopathology: lung bronchiectasis

Sample pathology report
  • Bilateral lungs, explant:
    • Severe acute and chronic bronchiolitis with patchy fibrosis, consistent with bronchiectasis. Acute bronchopneumonia involving the left lower lobe.

  • Bilateral lungs, explant:
    • Bronchiectasis with severe and necrotizing acute and chronic bronchitis and bronchiolitis consistent with cystic fibrosis.
Differential diagnosis
  • Many things can mimic or be superimposed with bronchiectasis but key findings can differentiate other diseases from it
    • Bronchitis:
      • Increased goblet cells, thickened bronchial walls
    • Asthma:
      • More eosinophils, goblet cell hyperplasia and metaplasia, Charcot-Leyden crystals, smooth muscle hypertrophy
    • Aspiration pneumonia:
      • More neutrophils, abscesses, vegetable matter
    • Tuberculosis:
      • Necrotizing granulomas
    • Emphysema:
      • Gross findings with bullae
      • Alveolar wall destruction versus bronchiole destruction
    • Lymphoid malignancies:
      • Monotonous lymphoid populations with abnormal germinal centers
Board review style question #1


This histologic finding is most associated with which of the following congenital disease processes?

  1. Carney complex
  2. Congenital cystic airway malformation
  3. Cystic fibrosis
  4. Loeffler syndrome
  5. Pulmonary lymphangiectasis
Board review style answer #1
C. Cystic fibrosis. The histologic finding seen here is bronchiectasis. While the other answers can be associated with bronchiectasis, the most common cause in this question would be cystic fibrosis. Loeffler syndrome is not a congenital disease (associated with Ascaris infection).

Comment Here

Reference: Bronchiectasis
Board review style question #2

This explanted lung is most likely associated with which of the following diseases?

  1. Asthma
  2. Bronchiectasis
  3. Emphysema
  4. Interstitial fibrosis
  5. Tuberculosis
Board review style answer #2
B. Bronchiectasis. The gross image of the explanted lung comes from a patient who had a lung transplant for cystic fibrosis disease. The bronchi walls are thickened, ulcerated, dilated and there is thick mucin present in the lumen. The dilation extends to the pleura. Emphysema would have more bullae. Asthma would also have mucin but also hyperinflation. Tuberculosis would most likely show granulomas, areas of caseating necrosis or miliary seeding. Interstitial fibrosis often shows honeycombing, prominent cobblestoning of the surface and may or may not have concurrent bronchiectasis.

Comment Here

Reference: Bronchiectasis

Bronchocentric granulomatosis
Definition / general
  • Granulomatous disease of lungs in which almost all granulomas are centered in bronchi or bronchioles causing their destruction
  • Immunologic reaction related to chronic eosinophilic pneumonia and allergic bronchopulmonary aspergillosis
  • A histologic pattern of disease, not a specific diagnosis
  • Usually adults, often with asthma history and limited to lungs, may be asymptomatic
  • Usually solitary lesions that appear on chest Xray as atelectasis or consolidation, not nodules
  • Generally favorable prognosis
Gross description
  • Viscous material in involved bronchi
Microscopic (histologic) description
  • Large and medium bronchi infiltrated by neutrophils, eosinophils and necrotic debris surrounded by foreign body giant cells
  • Fragmented elastic tissue (with elastic stain)
  • Also bronchiolitis obliterans
  • No fibrinoid necrosis of vessels
Differential diagnosis

Bronchopulmonary dysplasia
Definition / general
  • Complication of mechanical ventilation and oxygen therapy for treatment of neonatal respiratory distress syndrome
  • Infants almost always < 1200 gram birth weight and < 30 weeks gestation
  • Definition: requirement for supplemental oxygen for longer than 28 days of age, and a postmenstrual age of at least 36 weeks (eMedicine)
Etiology
  • Due to decreased alveolar septation and capillary derangement, secondary to arrested septation of lungs, due to therapy in the saccular stage of lung development
  • Free radicals generated by oxygen therapy and inflammatory cytokines play a role in pathogenesis
Clinical features
  • Incidence has fallen with use of surfactant therapy in premature infants for neonatal respiratory distress syndrome (Semin Perinatol 2006;30:171)
  • Respiratory distress continues for months
  • Patients have limited pulmonary reserve, develop repeated infections, often have pulmonary hypertension and develop cor pulmonale
Microscopic (histologic) description
  • Bronchiolar and interstitial fibrosis, compensatory emphysema of less damaged acini
  • Inadequate alveolar development causes fewer but larger alveoli
Microscopic (histologic) images

Images hosted on other servers:
Severe fibrosis

Severe fibrosis

Large alveoli

Large alveoli

Additional references

Carcinoid tumorlet
Definition / general
  • Tumor of neuroendocrine differentiation, defined by size < 5 mm in diameter, mitotic count < 2 mitoses/2 mm² and absence of necrosis
Essential features
  • Can be found incidentally in lung resections but can arise in the context of diffuse idiopathic pulmonary neuroendocrine cell hyperplasia (DIPNECH)
  • Defined as a proliferation of neuroendocrine cells < 5 mm in diameter that extend through the bronchial basement membrane with < 2 mitoses/2 mm² and no necrosis
  • Progression to typical carcinoid tumors is possible
ICD coding
  • ICD-O: 8040/0 - tumorlet, benign
  • ICD-10: D3A.8 - other benign neuroendocrine tumors
  • ICD-11: 2F00.Y - other specified benign neoplasm of middle ear or respiratory system
Epidemiology
Sites
  • Tumorlets are located in the same region as inflammatory or fibrous lung disease (AJR Am J Roentgenol 2004;183:293)
  • When arising in the context of DIPNECH, they are typically located in the terminal bronchioles
Pathophysiology
Etiology
  • Unknown
Clinical features
Diagnosis
  • See DIPNECH for details on diagnosis for tumorlets arising in this setting
Radiology description
  • CT scan: can present as a bronchial wall thickening that can appear nodular or show multiple peribronchiolar spherical to ovoid solid or ground glass nodules (Clin Radiol 2015;70:317)
    • Images are similar to the ones for DIPNECH when arising in this context
    • When tumorlets arise in a context of chronic pulmonary lung disease, they can be subtle and masked by the underlying process (Med Sci Monit 2020;26:e926014)
Radiology images

Contributed by Andréanne Gagné, M.D., M.Sc. and Philippe Joubert, M.D., Ph.D.
CT scan: nodules

CT scan: nodules



Images hosted on other servers:

Peribronchiolar nodules

Multiple nodules

Nodules and air trapping (DIPNECH)

Prognostic factors
  • Most tumorlets are benign incidental findings for which surgical resection is curative and prognosis is excellent (Chest 2007;131:1635)
  • See DIPNECH for details on prognosis of tumorlets arising in this setting
Case reports
Treatment
  • See DIPNECH for details on tumorlets arising in this setting
Gross description
  • Difficult to identify but when visible, they are seen as small gray-white nodules < 5 mm in diameter, intimately associated with bronchioles
Frozen section description
Microscopic (histologic) description
  • Poorly defined nodule of neuroendocrine cells that cross the mucosal basal membrane in a fibrotic stroma:
    • Size < 5 mm with < 2 mitoses/2 mm² and absence of necrosis
    • Usually found in association with an airway
    • Composed of neuroendocrine cells that are oval to round or spindle shaped; cells have round to oval nuclei with salt and pepper chromatin and a moderate amount of eosinophilic cytoplasm
  • Complete features of DIPNECH or an underlying lung disease can be seen
Microscopic (histologic) images

Contributed by Andréanne Gagné, M.D., M.Sc. and Philippe Joubert, M.D., Ph.D.
Missing Image

Peripheral nodule

Missing Image

Poorly delineated nodule

Missing Image

Tumorlet

Missing Image

Tumorlet arising in DIPNECH

Missing Image

Small tumorlet


Missing Image

Clear cell features

Missing Image

Spindle cell features

Missing Image

Salt and pepper chromatin

Missing Image

CD56

Cytology description
Positive stains
Negative stains
Molecular / cytogenetics description
  • To date, molecular alterations of tumorlets are poorly described
Sample pathology report
  • See DIPNECH for an example of a report when tumorlets arise in this setting
Differential diagnosis
Board review style question #1

Regarding the lung nodule (tumor size < 5 mm without necrosis and mitoses) presented in the image, which of the following is true?

  1. EMA and CD56 stains are positive
  2. Many of them have EGFR mutations
  3. Their prognosis is poor
  4. They can be seen in the context of diffuse idiopathic pulmonary neuroendocrine cell hyperplasia (DIPNECH)
Board review style answer #1
D. They can be seen in the context of DIPNECH. A lung tumorlet is depicted in the image. While CD56 is positive in those tumors, EMA is negative and can help to differentiate with minute pulmonary meningothelial-like nodules (A). To date, molecular alterations of tumorlets are poorly described (B). Whether they occur in association with DIPNECH or with an underlying lung process, tumorlets have a good prognosis (C).

Comment Here

Reference: Carcinoid tumorlet
Board review style question #2
Which of the following is true about lung tumorlets?

  1. More than 2 mitoses/2 mm² can be seen
  2. Rare tumorlets of more than 1 cm have been reported
  3. They are confined to bronchial mucosa, without crossing the bronchial basal membrane
  4. They are frequently found in association with an airway
Board review style answer #2
D. They are frequently found in association with an airway. Tumorlets, by definition, have a size < 5 mm and < 2 mitoses/2 mm² with absence of necrosis (A and B). Statement C refers to diffuse idiopathic pulmonary neuroendocrine cell hyperplasia.

Comment Here

Reference: Carcinoid tumorlet

Carcinosarcoma
Definition / general
  • Subtype of sarcomatoid carcinoma, consisting of a mixture of nonsmall cell lung cancer (typically squamous cell carcinoma or adenocarcinoma) and sarcomatous heterologous elements
  • Monoclonal tumor with divergent lines of differentiation, leading to mixture of carcinomatous and sarcomatous elements (Am J Surg Pathol 2002;26:510)
Essential features
  • Biphasic tumor consisting of a nonsmall cell carcinoma with heterologous sarcomatoid differentiation
  • Rare tumor with poor prognosis
  • Recommend molecular testing according to associated histologic components; i.e. tumors with adenocarcinoma component should be tested for EGFR and ALK (J Thorac Oncol 2015;10:1243)
Terminology
  • Use specific term "carcinosarcoma" rather than general term "sarcomatoid carcinoma" whenever possible to avoid confusion (J Thorac Oncol 2015;10:1243)
  • "Biphasic sarcomatoid carcinoma" has been proposed as replacement term
ICD coding
  • Use code specific for location of tumor
  • C34.90 malignant neoplasm of unspecified part of unspecified bronchus or lung
Epidemiology
  • Rare, 0.1% of all lung cancers
  • M > F, most with smoking history, average age 60
Sites
  • Large airways and peripheral lung
Pathophysiology
  • Primary lung carcinoma undergoes sarcomatoid change
Etiology
  • Epithelial malignancy with divergent differentiation
Clinical features
  • Dependent on location: endobronchial lesions have pneumonia, cough, shortness of breath, hemoptysis; peripheral lesions may have no symptoms
Diagnosis
  • Difficult to diagnose on small biopsy / cytology but possible
Radiology images

Images hosted on other servers:
CT before / after targeted therapy CT before / after targeted therapy CT before / after targeted therapy

CT before / after targeted therapy

Large mass on CT scan

Large mass on CT scan

Mass lesion located left hilar localization

Mass lesion located left hilar localization

Prognostic factors
  • Poor prognosis overall; increased tumor size appears related to reduced survival (Am J Surg Pathol 1999;23:1514)
  • Better prognosis for smaller or endobronchial lesions
Case reports
Treatment
Gross description
  • May show central necrosis and hemorrhage if larger tumor
  • Irregular borders, tan to yellow
Gross images

Images hosted on other servers:
Bronchial carcinosarcoma

Bronchial carcinosarcoma

Microscopic (histologic) description
  • Mixture of carcinomatous and sarcomatous elements
  • Carcinoma component usually squamous but may be glandular or neuroendocrine
  • Heterologous sarcomatoid component, such as rhabdomyosarcoma, osteosarcoma, chondrosarcoma
  • Metastases may show carcinomatous or sarcomatous component or both
Microscopic (histologic) images
Scroll to see all images


Contributed by Yale Rosen, M.D. and Jian-Hua Qiao, M.D.
Giant cell carcinoma Giant cell carcinoma Giant cell carcinoma Giant cell carcinoma Giant cell carcinoma

Giant cell carcinoma

Core biopsy

Core biopsy


AE1 / AE3

AE1 / AE3

Cam 5.2

CAM 5.2



Images hosted on other servers:
Various images Various images Various images Various images

Various images

Initially presenting as invasive aspergillosis

Initially presenting as invasive aspergillosis

Blastomatoid variant

Blastomatoid variant


Lung parenchyma showing infiltration

Lung parenchyma showing infiltration

Lipoblastic elements with spindle cell component

Lipoblastic elements with spindle cell component

Adenocarcinoma with acinar growth pattern

Adenocarcinoma with acinar growth pattern

Malignant poorly formed epithelial glands

Malignant poorly formed epithelial glands

S100

S100

CK7 and TTF1

CK7 and TTF1


Carcinoma with atypical glands

Carcinoma with atypical glands

H&E

H&E

Virtual slides

Images hosted on other servers:
Rosai collection Rosai collection Rosai collection

Rosai collection

Cytology description
  • May only show epithelial component, i.e. squamous cell carcinoma
Cytology images

Contributed by Jian-Hua Qiao, M.D.
Highly pleomorphic giant cells Highly pleomorphic giant cells Highly pleomorphic giant cells Highly pleomorphic giant cells Highly pleomorphic giant cells Highly pleomorphic giant cells

Highly pleomorphic giant cells


Highly pleomorphic giant cells Highly pleomorphic giant cells Highly pleomorphic giant cells

Highly pleomorphic giant cells

Positive stains
  • Dependent on specific components of tumor
Negative stains
  • Dependent on specific components of tumor
Molecular / cytogenetics description
Differential diagnosis
Additional references
Board review style question #1
The most common epithelial component of a primary pulmonary carcinosarcoma is:

  1. Adenocarcinoma
  2. Adenosquamous carcinoma
  3. Large cell carcinoma
  4. Small cell carcinoma
  5. Squamous cell carcinoma
Board review style answer #1
E. Squamous cell carcinoma

Comment Here

Reference: Carcinosarcoma

Chronic bronchitis
Definition / general
  • Diagnosis: persistent cough with sputum for 3 months in 2 consecutive years without other apparent explanation (eMedicine)
  • Chronic disease of large airways, variable inflammation
Clinical features
  • Simple chronic bronchitis: cough but no physiologic evidence of airway obstruction
  • Chronic asthmatic bronchitis: hyperreactive airways with intermittent bronchospasm and wheezing
  • Obstructive bronchitis: often have associated emphysema, small airway disease
  • Causes: 4× - 10× more common in smokers, chronic irritation and infections may contribute
  • Other causes and contributors are air pollution including passive cigarette smoke, marijuana smoke and occupational dust exposure
  • Tobacco interferes with ciliary action, directly damages airway epithelium and inhibits ability of white blood cells to clear bacteria; infections maintain but do not initiate chronic bronchitis
  • Often diagnosed at time of acute respiratory illness (Prim Care Respir J 2010;19:371)
  • More infections, purulent sputum, hypercapnia, hypoxia than emphysema; clinically called "blue bloaters"
  • May cause secondary pulmonary vascular hypertension, cor pulmonale, congestive heart failure, death due to respiratory acidosis and coma, congestive heart failure and pneumothorax
  • Reid index: ratio of thickness of mucus gland layer to thickness of wall between epithelium and cartilage; normal is 0.4, increased in chronic bronchitis
Gross description
  • Boggy mucosa with excessive mucinous secretions, pus, prominence of bronchial mucosal pits overlying the orifices of bronchial mucous glands
Microscopic (histologic) description
  • Early: hypersecretion of mucus in large airways with hypertrophy of submucosal glands in tracheobronchial tree
  • Later: increase in goblet cells in small airways contributes to excessive mucus production and airway obstruction
  • Increased percentage of bronchial wall is occupied by submucosal mucous glands, as measured by Reid index; this directly correlates with sputum production, variable dysplasia, squamous metaplasia, bronchiolitis obliterans
  • Chronic inflammatory infiltrates range from absent to prominent
Microscopic (histologic) images

Images hosted on other servers:
Mucosal glandular hyperplasia

Mucosal glandular hyperplasia

Chronic inflammatory infiltrate

Chronic inflammatory infiltrate


CMV
Definition / general
  • Lung injury caused by infection by CMV, a β herpes virus
  • Usually immunocompromised patients
  • Often associated with pneumocystis and other infections
  • Xray: 2 - 4 cm peripheral nodules, miliary pattern, diffuse interstitial process
Epidemiology
  • 70% of Americans in large cities have latent infection
  • In immunocompetent individuals, primary infection is generally asymptomatic, but mononucleosis-like illness may occur; rarely, self limiting CMV pneumonia may occur during mononucleosis-like illness (eMedicine)
  • In organ transplant recipients, infection may occur when the donor organ is latently infected with CMV
  • When infection occurs in immunosuppressed patients, the risk of organ damage is higher the greater the immunosuppression
  • In AIDS patients, CMV disease generally occurs when the CD4+ cell count < 50/mm3
  • CMV is the most common life threatening infection in hematopoietic stem cell transplant recipients and is the most common opportunistic viral infection in AIDS
  • Prophylactic therapy and preemptive therapy, where CMV DNA levels are monitored by PCR and antiviral therapy against CMV is given prior to end organ damage, has greatly reduced the incidence of CMV induced disease in vulnerable patients
  • Highly active antiretrovirus therapy has reduced the incidence of end organ damage by 80% in HIV positive patients
Sites
  • In addition to lung, CMV causes severe disease in retina, brain, kidneys, GI
Etiology
  • Disease may occur with primary infection or secondarily when latent disease is reactivated
  • Virus can be spread transplacentally; neonatal disease can occur through exposure to cervical or vaginal secretions or from breast milk when active infection is present
  • In children and adults, infection is spread through saliva, respiratory droplets or sexual activity
  • Fecal oral transmission has also been documented
  • CMV causes down regulation of expression of MHC class I and II molecules, which helps it elude host immune responses
Clinical features
  • In immunocompetent patients, cough with interstitial infiltrates on chest xray
  • Immunosuppressed patients present with cough, fever, fatigue, shortness of breath, malaise, night sweats, muscle and joint pain
  • Note: positive culture, serology or PCR may not represent evidence of illness
  • Diagnosis is based on identification of viral inclusions in biopsy material
  • Acute CMV infection leads to transient but severe immunosupression
  • Prevention is key as CMV pneumonia has high mortality rate even with aggressive antiviral therapy
Treatment
  • Ganciclovir, valganciclovir, foscarnet, cidofovir, maribavir
Gross description
  • Lung is firm (similar to interstitial lung disease) with hemorrhagic nodules that may become confluent
Microscopic (histologic) description
  • Mononuclear infiltrates, usually mild edema and pneumocyte hyperplasia
  • Infected cells are generally large with a prominent basophilic nuclear inclusion, often with a clear halo, and smaller basophilic cytoplasmic inclusions are also present
  • Hemorrhagic necrosis may be present
  • In lung, usually infects endothelial and epithelial cells and alveolar histiocytes
Microscopic (histologic) images

Contributed by Dr. Michael P. Orejudos and Dr. Rosemarie Rodriguez

Cytology description
  • Diagnostic inclusions may be seen in brochiolalveolar lavage fluid, but diagnostic yield is very low
Positive stains
  • Immunohistochemical staining for CMV is confirmatory
  • PAS and GMS stains highlight the inclusions
Electron microscopy description
  • Particles are 150 - 200 nm with round core and double membrane
Differential diagnosis

Coal workers' pneumoconiosis
Definition / general
  • Incidence declining due to dust reduction measures

Simple coal workers' pneumoconiosis (CWP):
  • Patients have coal macules (1 - 2 mm collections of carbon laden macrophages) and coal nodules (coal macules and fibrosis) scattered throughout lung, more in upper lobe and upper lower lobe, near respiratory bronchioles
  • Usually minimal symptoms but 10% develop progressive massive fibrosis

Progressive massive fibrosis:
  • Also called complicated coal workers' pneumoconiosis
  • Intensely blackened scars > 2 cm, multiple and containing dense collagen and pigment
  • Center of lesion may be necrotic due to ischemia
  • Associated with pulmonary hypertension and cor pulmonale
  • May progress even if dust exposure ceases
  • Due to any pneumoconiosis, although most common in CWP and silicosis
  • Associated with increased incidence of clinical tuberculosis, chronic bronchitis and emphysema and independent of smoking
  • In isolation, not appear to increase the risk of lung cancer

Caplan syndrome:
  • Rheumatoid arthritis and pneumoconiosis cause rapidly developing nodular pulmonary lesions showing central necrosis surrounded by collagen, fibroblasts and macrophages (similar to rheumatoid nodules)
  • Associated with exposure to coal, asbestosis and silica dust
Radiology images

Contributed by @Andrew_Fltv on Twitter
Coal workers' pneumoconiosis

Coal workers' pneumoconiosis

Gross images

Contributed by @Andrew_Fltv on Twitter
Coal workers' pneumoconiosis Coal workers' pneumoconiosis Coal workers' pneumoconiosis

Coal workers' pneumoconiosis

Microscopic (histologic) images

Contributed by @Andrew_Fltv on Twitter
Coal workers' pneumoconiosis Coal workers' pneumoconiosis Coal workers' pneumoconiosis Coal workers' pneumoconiosis

Coal workers' pneumoconiosis


Coal workers' pneumoconiosis Coal workers' pneumoconiosis Coal workers' pneumoconiosis Coal workers' pneumoconiosis

Coal workers' pneumoconiosis



Images hosted on other servers:
Coal workers' pneumoconiosis

Coal workers' pneumoconiosis


Coccidioides
Definition / general
  • Lung infection caused by dimorphic fungi, Coccidioides spp.
Essential features
  • Highly prevalent in the endemic areas (southwest U.S., Central and South America)
  • Lower respiratory tract symptoms resembling bacterial pneumonia; mass lesion with cavity formation in chronic infection
  • Necrotizing granulomatous inflammation with characteristic spherules
Terminology
  • Pulmonary coccidioidomycosis
  • San Joaquin Valley fever, valley fever
ICD coding
  • ICD-10: B38.2 - pulmonary coccidioidomycosis, unspecified
  • ICD-11: 1F25.0 - pulmonary coccidioidomycosis
Epidemiology
Sites
Pathophysiology
  • Acquired via inhalation of aerosolized arthroconidia (3 - 5 microns) from the soil
  • A specialized structure called a spherule forms in the lung
  • Endospores are released from the ruptured spherule and develop into new spherules
  • Disseminate via hematogenous route
Etiology
  • Dimorphic fungi in the genus Coccidioides, which contain 2 species
    • Coccidioides immitis (central and southern California, San Joaquin Valley)
    • Coccidioides posadasii (isolated outside California)
  • No difference in clinical course and microscopic morphology between the 2 species
Diagrams / tables

Images hosted on other servers:
Endemic areas

Endemic areas

Endemic areas

Coccidioides spp. life cycle

Clinical features
  • Most infected individuals are asymptomatic or minimally symptomatic
  • Symptoms include cough, fever, dyspnea, pleuritic chest pain
  • Can be disseminated in 1 - 5% of cases, especially immunocompromised patients (Medicine (Baltimore) 2004;83:149)
  • Desert rheumatism refers to the immunologic phenomena triad, including fever, arthralgia and erythema nodosum
Diagnosis
  • Traveling history or residence in the endemic areas
  • Fungal organism identification by histology, culture or serology
Laboratory
  • Standard fungal culture: no specific plate morphology; lactophenol cotton blue stains shows septate hyphae with alternating barrel shaped arthroconidia
    • Biosafety level 3 pathogen: laboratory personnel should be aware of the diagnosis
  • Serologic testing for anticoccidioidal IgG, IgM
  • Increased serum (1→3)-β-D-glucan can be used to indicate invasive fungal disease, although not specific for coccidioidomycosis (J Clin Microbiol 2012;50:3060)
  • Complete blood count: eosinophilia found in 27% of cases (West J Med 1993;159:153)
Radiology description
  • Chest Xray and CT chest in acute infection: nonspecific, resembling acute bacterial pneumonia, including consolidation (75%), nodular opacities (20%), hilar adenopathy (20%), pleural effusion (15 - 20%); solitary pulmonary nodule may be seen (Radiographics 2014;34:912)
  • Disseminated infection: miliary nodules from hematogenous spread (Radiographics 2014;34:912)
  • Chronic cavitary lesion can be seen radiographically in 2% of cases, up to 11% by CT scan (Radiographics 2014;34:912)
  • Grape skin sign: classic finding in chronic pulmonary coccidioidomycosis (AJR Am J Roentgenol 2014;202:479)
    • Very thin walled cavitary lesion that develops in lung parenchyma previously affected by consolidation followed by central necrosis
Radiology description

Images hosted on other servers:
CT chest acute infection

CT chest acute infection

Case reports
Treatment
  • Oral azole antifungal
  • Amphotericin B
  • Surgical resection of the cavitary lesion if (Clin Infect Dis 2016;63:e112)
    • Cavities are persistently symptomatic despite antifungal treatment
    • Cavities have been present for more than 2 years and if symptoms recur whenever antifungal treatment is stopped
Gross description
  • Mass lesion can be seen in chronic infection, usually well demarcated, often with central cavitation
  • Fungus ball (mycetoma) was found in 28% of cases with cavity lesion (Hum Pathol 2014;45:153)
Microscopic (histologic) description
  • Necrotizing, often suppurative, granulomatous inflammation
  • Large (20 - 200 microns) thick walled spherules, with or without granular basophilic endospores (2 - 4 microns)
  • Eosinophilic infiltrate is common
  • Thin, septate hyphae with arthroconidia formation may be seen, more often in diabetes mellitus patients (Eur J Clin Microbiol Infect Dis 2008;27:813)
  • Cavitary lesion features (Hum Pathol 2014;45:153)
    • Palisading fibroblasts and fibrosis with chronic inflammation
    • Granuloma not seen in the cavity wall, while multinucleated giant cells occasionally present
    • Squamous metaplasia in the lining of the cavity wall
    • Hyphal forms identified in 62% of cases
    • Blood vessel lesion: arteritis with fibrinoid necrosis and rupture, thrombus and mural chronic inflammation
  • Surrounding lung parenchyma (Hum Pathol 2014;45:153)
    • Lymphoid hyperplasia
    • Chronic bronchiolitis
    • Organizing pneumonia
Microscopic (histologic) images

Contributed by Sakda Sathirareuangchai, M.D.
Spherules

Spherules

Spherule

Spherule

Spherules GMS

Spherules GMS

Cytology description
  • Bronchoalveolar lavage (BAL) cytology (Diagn Cytopathol 2016;44:195)
    • Various forms of spherules with associated acute inflammatory response
    • Spherules can be immature (not adsorb any stains), smaller, larger, empty or fractured
  • Granulomas were rare and mostly seen in lymph nodes and extra pulmonary sites (Diagn Cytopathol 2016;44:195)
  • Mycelial form in BAL is uncommon but has been reported (Diagn Cytopathol 2007;35:535)
Cytology images

Contributed by Sakda Sathirareuangchai, M.D.
Spherule cytologic smear

Spherule cytologic smear

Positive stains
Negative stains
Molecular / cytogenetics description
  • Real time PCR assay is available for various types of specimen (e.g., bronchoalveolar lavage, sputum, lung tissue, etc.)
  • In situ hybridization for C. immitis ribosomal RNA has been described (Diagn Mol Pathol 2010;19:99)
Videos

Dimorphic fungi: coccidioidomycosis by Glenn D. Roberts, Ph.D.

Sample pathology report
  • Left lung, core needle biopsy:
    • Necrotizing granulomatous inflammation (see comment)
    • Comment: Fungal organism identified on H&E and GMS stain, consistent with Coccidioides spp.
Differential diagnosis
  • Histoplasmosis:
    • Endemic in the central and eastern U.S. (Ohio and Mississippi River valleys)
    • Small (2 - 5 microns) budding yeast within the histiocytes
  • Blastomycosis:
    • Endemic in the eastern U.S. (Ohio, Mississippi River valleys, Great Lakes region)
    • Broad based budding yeast, smaller (8 - 15 microns) than Coccidioides sp.
  • Paracoccidioidomycosis:
    • Endemic in South America
    • Large (10 - 60 microns) spherical yeast with circumferential budding, resembling mariner’s wheel
  • Cryptococcosis:
    • No specific endemic area
    • Round, medium sized (4 - 7 microns), yeast with thick mucoid capsule
  • Tuberculosis:
    • Acid fast positive bacilli
Board review style question #1

A 40 year old man presents with chronic cough, low grade fever and mild dyspnea for several months. CT scan of the chest shows a 3 cm mass lesion in the right upper lobe of the lung. The H&E image of the core needle biopsy is shown. Which one of the followings is the most likely exposure history?

  1. Cave diving in Ohio
  2. Military training in Arizona
  3. Parakeet breeder
  4. Traveling to Africa
  5. Camping in Colorado
Board review style answer #1
B. Military training in Arizona. The patient has coccidiomycosis.

Comment Here

Reference: Coccidioides immitis
Board review style question #2
What is the pathognomonic microscopic feature of coccidioidomycosis in a histologic section?

  1. Broad based budding yeast
  2. Round yeast with large mucoid capsule
  3. Acute angle, branching septate hyphae
  4. Large spherule with numerous endospores
  5. Oval yeast with pseudohyphae budding
Board review style answer #2
D. Large spherule with numerous endospores

Comment Here

Reference: Coccidioides immitis

Colloid
Definition / general
  • Colloid adenocarcinoma is a type of invasive lung cancer characterized by extracellular mucin accumulation that distends alveolar spaces and destroys lung tissue
  • It is important to differentiate colloid adenocarcinoma from metastases of mucinous carcinoma originating in other organs and from invasive mucinous adenocarcinoma of the lung by identifying key characteristics, such as large pools of mucin destroying alveolar walls, which must be present in at least 50% of the tumor
Essential features
  • Invasive adenocarcinoma with extensive extracellular mucin pools distending alveolar spaces and destroying alveolar walls
  • Tumor cells are mucin filled, cuboidal to columnar, floating in mucin pools or lining fibrous walls of mucin filled spaces
  • Predominantly found in the peripheral lung parenchyma
  • Immunohistochemistry: positive for CK7 and CDX2 and CK20; weak or negative for TTF1, napsin A and EMA (MUC1)
  • Mostly indolent clinical course with a relatively favorable prognosis after complete surgical resection
Terminology
  • Recommended: colloid adenocarcinoma
  • Not recommended: mucinous cystadenocarcinoma; mucinous cystic tumor of borderline malignancy
ICD coding
  • ICD-O: 8480/3 - colloid adenocarcinoma
  • ICD-11: 2C25.Z & XH7GY6 - malignant neoplasms of bronchus or lung, unspecified & adenocarcinoma of lung, mucinous
Epidemiology
Sites
Pathophysiology
  • Pathogenesis of mucinous lung tumors, which may share similarities with mucinous / colloid gastrointestinal adenocarcinomas, could be attributed to gastrointestinal differentiation, as both types of tumors exhibit predominant extracellular accumulation of mucin
  • Colloid adenocarcinomas are generally negative for predictive biomarkers such as EGFR mutations and ALK rearrangements, while KRAS mutations are found in ~50% of these tumors
  • References: Virchows Arch 2015;467:675, Hum Pathol 2015;46:836
Etiology
Clinical features
  • In most cases, it is asymptomatic (J Surg Oncol 2016;114:211)
  • Incidental findings during routine radiographic evaluation or with no specific symptoms (i.e., cough, shortness of breath and unresolved pneumonia) (Hum Pathol 2015;46:836)
Diagnosis
Radiology description
  • On CT, colloid adenocarcinoma usually presents as an intrapulmonary mass with poor contrast enhancement (Intern Med 2018;57:3637)
  • On MRI on T1 weighted imaging (WI), colloid adenocarcinoma is characterized by low intensity and high intensity on T2WI, probably from the mucus component of the tumor (Intern Med 2018;57:3637)
  • On 18F fluorodeoxyglucose positron emission tomography (18F FDG PET), the majority of tumors show intense accumulation of 18F FDG (median standardized uptake value of 6.25, ranging from 3.0 to 8.6) (J Surg Oncol 2016;114:211)
Radiology images

Images hosted on other servers:
Radiological evaluation

Radiological evaluation

Radiology and bronchoscopy

Radiology and bronchoscopy

Prognostic factors
  • Estimated 5 year survival of 51% in patients who benefit from surgical resection (J Thorac Oncol 2011;6:1496)
  • Apparent better prognosis for tumors that are CDX2 and MUC2 positive compared with tumors that are CDX2 and MUC2 negative (Am J Surg Pathol 2004;28:442)
  • Clinical progression is generally slow; after comprehensive surgical removal, the prognosis is relatively favorable (Am J Surg Pathol 2004;28:442)
  • Poorer prognosis, involving recurrence and metastasis, is indicated by the existence of signet ring cells and a noncolloid component (Am J Surg Pathol 2004;28:442)
Case reports
Treatment
  • Complete surgical resection is the treatment of choice (Respirol Case Rep 2023;11:e01109)
  • KRAS mutations are the most common
  • Targetable alterations in EGFR and ALK are typically absent in colloid adenocarcinoma (i.e., EGFR mutations and ALK rearrangements) (Hum Pathol 2015;46:836)
Clinical images

Images hosted on other servers:
Intraoperative view of white mass with indistinct margins

Intraoperative view
of white mass
with indistinct
margins

Intraoperative view of giant cystic colloid adenocarcinoma

Intraoperative
view of giant
cystic colloid
adenocarcinoma

Gross description
  • Tumors appear as diverse, nonencapsulated, single, soft, jelly-like nodules with a mucoid appearance and protruding when cut; their sizes range from 5 to 100 mm (Mod Pathol 1992;5:634)
  • In rare instances, these tumors may display a notably cystic appearance (Hum Pathol 2015;46:836)
Gross images

Contributed by Matthew J. Cecchini, M.D., Ph.D.
Gross cut surface

Gross cut surface



Images hosted on other servers:
Gross cut surface

Gross cut surface

Cystic lesion with mucin pooling

Cystic lesion with mucin pooling

Microscopic (histologic) description
  • Abundant extracellular mucin
  • Distended and destroyed alveolar spaces
  • Invasive growth pattern
  • Mucin laden cuboidal / columnar cells
  • Possible signet ring morphology
  • Inconspicuous, well differentiated cells
  • Low mitotic count, no necrosis
  • Inflammatory infiltrate possible
  • Reference: Hum Pathol 2015;46:836
Microscopic (histologic) images

Contributed by Gheorghe-Emilian Olteanu, M.D., Ph.D., Ana Mataić, M.D. and Luka Brčić, M.D., Ph.D.
Abundant extracellular mucin

Abundant extracellular mucin

Mucin laden cells

Mucin laden cells

Hypercellularity

Hypercellularity

Inflammatory infiltrate

Inflammatory infiltrate

Nuclear and cytoplasm detail

Nuclear and cytoplasm detail


Siderophages and colloid adenocarcinoma cells

Siderophages and colloid adenocarcinoma cells

Extensive extracellular mucin

Extensive extracellular mucin

CK20 IHC stain

CK20 IHC stain

CDX2 IHC stain

CDX2 IHC stain

CK7 IHC stain

CK7 IHC stain

Virtual slides

Images hosted on other servers:
Well demarcated colloid adenocarcinoma with abundant mucin pools

Well demarcated
colloid adenocarcinoma
with abundant
mucin pools

Cytology description
  • Low cellularity
  • Tumor cells in tissue fragments
  • Single cells resembling histiocytes
  • Thick extracellular mucin pools
  • Reference: Cancer Cytopathol 2015;123:306
Cytology images

Images hosted on other servers:
Clusters of cells with basally oriented nuclei

Clusters of cells with basally oriented nuclei

Positive stains
Molecular / cytogenetics description
  • KRAS mutations are more common (mutations in codons 12 and 13 have been seen)
  • No EGFR mutations or ALK rearrangements
  • Reference: Hum Pathol 2015;46:836
Sample pathology report
  • Lung, lobectomy:
    • Colloid adenocarcinoma (see comment)
    • Comment: Abundant extracellular mucin pools with distended and destroyed alveolar spaces. Tumor cells are bland, mucin laden and cuboidal to columnar. Overall, the tumor has an invasive growth pattern.
Differential diagnosis
Board review style question #1
Which of the following uniquely represents invasive colloid adenocarcinoma of the lung, distinguishing it from other subtypes of lung adenocarcinoma?

  1. Absence of smoking as a risk factor
  2. Invasive lung cancer characterized by large pools of mucin destroying alveolar walls, with > 50% of the tumor demonstrating these characteristics
  3. Positive markers for CK7 and CDX2
  4. Presence of KRAS mutations
Board review style answer #1
B. Invasive lung cancer characterized by large pools of mucin destroying alveolar walls, with > 50% of the tumor demonstrating these characteristics. Invasive colloid adenocarcinoma of the lung is distinctly marked by large pools of mucin that destroy alveolar walls and this feature is present in > 50% of the tumor.

Answer D is incorrect as KRAS mutations are not unique to invasive colloid adenocarcinoma; they are one of the most commonly found mutations in all types of lung adenocarcinomas. Answer C is also incorrect since positive immunohistochemistry for CK7 and CDX2 is not exclusive to invasive colloid adenocarcinoma but is also found in mucinous lung adenocarcinoma and adenocarcinoma with enteric differentiation. Answer A is incorrect because smoking is a risk factor for all lung cancers, including colloid adenocarcinoma, and is not absent as a risk factor.

Comment Here

Reference: Colloid
Board review style question #2
Which of the following statements about invasive colloid adenocarcinoma is accurate?

  1. Negative for CK7 and TTF1
  2. Positive for CDX2 and SATB2
  3. Positive for CK7 and CDX2
  4. Typically presents EGFR mutations or ALK rearrangements
Board review style answer #2
C. Positive for CK7 and CDX2. Invasive colloid adenocarcinomas typically test positive for CK7 and CDX2. They usually display weak or negative expressions for TTF1 and are negative for SATB2 (it is important to note that the positivity for CDX2 is a common pitfall in colloid adenocarcinoma). Answer D is incorrect because invasive colloid adenocarcinomas more often have KRAS mutations and generally test negative for predictive biomarkers like EGFR mutations and ALK rearrangements. Answer B is incorrect because these tumors are typically negative for SATB2. Answer A is incorrect because invasive colloid adenocarcinomas are generally positive for CK7, not negative. While they can be weak or negative for TTF1, this statement does not represent the full picture.

Comment Here

Reference: Colloid
Board review style question #3

A 67 year old man presented with difficulty breathing, cough and expectoration. A chest CT reveals a mass in the lower right portion of the lung measuring 53 mm in diameter. A biopsy finds abundant mucin but no atypical cells. Following a lobectomy, immunohistochemistry of tumor cells indicates expression of CK7, CK20 and CDX2 but not TTF1 or SATB2. The patient has no history of previous tumors and no malignancy in the gastrointestinal tract is currently detected. Given the image and the provided information, what is your pathological diagnosis?

  1. Adenocarcinoma with enteric differentiation
  2. Invasive colloid adenocarcinoma of the lung
  3. Invasive mucinous adenocarcinoma of the lung
  4. Metastatic adenocarcinoma of the gastrointestinal tract
Board review style answer #3
B. Invasive colloid adenocarcinoma of the lung. The patient's immunohistochemistry results and morphological characteristics suggest invasive colloid adenocarcinoma of the lung. Although invasive mucinous adenocarcinomas and enteric type adenocarcinomas are possible considerations based on the immunohistochemistry, the morphology leans towards a diagnosis of colloid lung adenocarcinoma. Answer C is incorrect as the presence of abundant mucin, specific immunohistochemical markers and the lack of atypical cells makes invasive mucinous adenocarcinoma less likely. Answer A is incorrect as adenocarcinoma with enteric differentiation typically expresses SATB2, which is not observed in this case. Answer D is ruled out by the patient's negative history of previous tumors, the absence of current gastrointestinal malignancy and negative SATB2 expression.

Comment Here

Reference: Colloid

Congenital pulmonary airway malformation (CPAM)
Definition / general
  • Congenital pulmonary airway malformations encompass a spectrum of cystic and noncystic lung malformations that develop in utero
Essential features
  • Cysts are interconnected with each other and with alveolar spaces and typically lined by ciliated columnar epithelium; there is substantial overlap in cyst size between type 1 and 2 CPAMs
  • Type 1 and 3 CPAMs are associated with mosaic activating KRAS mutations and an increased risk of metastatic mucinous adenocarcinoma if incompletely resected (Mod Pathol 2022 Jul 6 [Epub ahead of print])
  • Type 2 CPAMs are associated with bronchial atresia and do not have KRAS mutations (Semin Pediatr Surg 2003;12:17, Mod Pathol 2022;35:1404)
  • Terms type 0 and type 4 CPAM are now obsolete (see Terminology below)
Terminology
ICD coding
  • ICD-10: Q33.9 - congenital malformation of lung, unspecified
Epidemiology
  • Congenital lung lesion
  • Type 2 CPAMs are the most common but are usually asymptomatic (Am J Surg Pathol 2019;43:47)
  • Clinical course is dependent upon the size of the lesion and timing of resection
  • Nearly all type 1 CPAMs and some type 3 CPAMs are caused by mosaic activating KRAS mutations; these are usually symptomatic (Mod Pathol 2022 Jul 6 [Epub ahead of print])
Sites
  • Most commonly involve a single lobe of the lung
  • Rarely, 1 lesion may involve multiple lobes or a patient may have 2 distinct lesions
Etiology
Clinical features
Diagnosis
  • Lesions are typically detected on prenatal ultrasound
  • Definitive diagnosis requires pathologic examination following resection
Radiology description
  • Ultrasound: multiple hypoechoic lung cysts (Neonatology 2016;110:101)
  • Fetal MRI: T2 hyperintense lesion with cystic spaces
Prognostic factors
  • Type 1 and type 3 CPAMs:
    • Incomplete resection in infancy or resection later in childhood is associated with an increased risk of developing metastatic mucinous adenocarcinoma (Am J Surg Pathol 2020;44:1118)
    • There are no reports of metastatic mucinous adenocarcinoma arising in CPAMs that are completely resected in infancy or early childhood
Case reports
Treatment
  • Complete surgical resection
  • Thoracoamniotic shunts may be placed to decompress type 1 CPAMs in utero until resection can be performed after birth
    • Maternal steroids may be given for symptomatic patients whose cysts are too small for shunt
  • Reference: Eur J Pediatr 2017;176:1559
Gross description
  • Type 1 (large cyst) CPAM
    • Grossly cystic lung lesion
    • Largest cyst size is highly variable and ranges from < 0.5 cm to > 7 cm in greatest dimension (Mod Pathol 2022 Jul 6 [Epub ahead of print])
    • Cysts are interspersed with relatively normal appearing lung parenchyma (alveoli are enlarged and simplified)
    • No mucocele
  • Type 2 (small cyst) CPAM
    • Variable gross appearance
    • In cystic specimens, cysts can measure up to 2.5 cm in greatest dimension and cysts are interspersed with normal appearing lung parenchyma
    • Some specimens may have no grossly identifiable lesion
    • May have a mucocele or grossly evident bronchial atresia
  • Type 3 CPAM
Gross images

Contributed by Nya D. Nelson, M.D., Ph.D. and Jennifer Pogoriler, M.D., Ph.D.
Type 1 CPAM

Type 1 CPAM

Type 2 CPAM

Type 2 CPAM

Type 3 CPAM

Type 3 CPAM


Microscopic (histologic) description
  • Type 1 (large cyst) CPAM
    • Readily identifiable cysts lined by ciliated cuboidal to stratified columnar epithelium with interspersed alveolar type spaces (Mod Pathol 2022 Jul 6 [Epub ahead of print])
    • Cysts connect with adjacent alveoli with frequent transitions from thick cyst wall to adjacent alveolar wall
    • Often has epithelial complexity - papillary projections, irregularly shaped small cystic spaces
    • May have solid appearing areas with features of both type 1 and type 3 CPAM
    • Mucinous cell clusters are seen in ~75% (when carefully searched for)
      • Presence of mucinous cell clusters is specific but not entirely sensitive for a KRAS mutation
      • Mucinous cell clusters in infants are often multifocal and have papillary or acinar architecture when they lepidically cover a small alveolus (Am J Surg Pathol 2020;44:1118)
      • Most pediatric pathologists do not diagnose mucinous adenocarcinoma in specimens from infants when only these small clusters are present
    • May have foci of cartilage within the large cyst walls or foamy appearing pneumocytes with vacuolated cytoplasm (Mod Pathol 2022 Jul 6 [Epub ahead of print])
    • Thoracoamniotic shunt placement may result in reactive changes, such as squamous metaplasia
  • Type 2 (small cyst) CPAM
    • Spectrum of histologic changes ranging from readily identifiable cysts to mildly malformed alveolar type spaces
    • Cysts may be focal with the majority of the malformation having only mild changes
    • Cysts are typically round and lined by ciliated columnar epithelium
    • Epithelial complexity is rare
    • Often with prominent evidence of mucostasis, including pools of mucin and foamy intra-alveolar macrophages
    • Transitions from cystic spaces into adjacent alveolar type spaces are rare
    • Cases that are clinically considered to be CPAMs but do not have cysts show milder parenchymal maldevelopment with enlarged and simplified alveoli (formerly congenital lobar emphysema)
    • Striated skeletal muscle may occasionally be seen in the septa between cysts
  • Type 3 CPAM
    • Consist predominantly of small irregularly shaped airway spaces lined by ciliated cuboidal to columnar epithelium (Mod Pathol 2022 Jul 6 [Epub ahead of print])
    • Surrounding septa often appear thickened, with prominent mesenchyme and cuboidal epithelium
    • Interspersed alveolar type spaces (with thin walls and flat pneumocytes) are rare / absent in most type 3 CPAMs
    • Mucinous cell clusters are seen in ~45%
      • Mucinous cell clusters are often multifocal and have papillary or acinar architecture (Am J Surg Pathol 2020;44:1118)
      • Most pediatric pathologists do not diagnose mucinous adenocarcinoma in specimens from infants when only these small clusters are present
Microscopic (histologic) images

Contributed by Nya D. Nelson, M.D., Ph.D. and Jennifer Pogoriler, M.D., Ph.D.
Type 1 CPAM low power

Type 1 CPAM

Type 1 CPAM complexity

Type 1 CPAM complexity

Mucinous cell clusters

Mucinous cell clusters

Small mucinous cell cluster

Small mucinous cell cluster

Type 2 CPAM low power Type 2 CPAM

Type 2 CPAM


Type 2 CPAM - mucostasis

Type 2 CPAM - mucostasis

Congenital lobar emphysema low power Congenital lobar emphysema

Congenital lobar emphysema

Type 3 CPAM low power Type 3 CPAM

Type 3 CPAM

Positive stains
  • PAS stain can be used to highlight mucinous cells in type 1 and type 3 CPAMs but is not necessary
  • MUC5 highlights mucinous cells but these can be found on H&E
Molecular / cytogenetics description
  • Nearly all classic type 1 CPAMs (> 98%) have an activating KRAS mutation in the mucinous and nonmucinous epithelium
    • KRAS p.G12D is the most common mutation reported in type 1 CPAMs
  • Type 3 CPAMs are associated with activating KRAS mutations or mutations in other genes involved in cell cycle regulation and growth
    • 50% have a KRAS mutation and p.G12V is the most common variant
    • 2 reported cases with mutations in other genes (FGFR2 and NEK9)
  • Presence of mucinous cell clusters is highly specific but not entirely sensitive for a KRAS mutation
  • Reference: Mod Pathol 2022 Jul 6 [Epub ahead of print]
Sample pathology report
  • Lung, right upper lobe, lobectomy:
    • Type 1 (large cyst) congenital pulmonary airway malformation, with atypical mucinous cells (see comment)
    • Comment: Sections throughout the lobe demonstrate numerous, variably enlarged and cystically dilated airspaces lined by ciliated respiratory epithelium. Foci of epithelial complexity, including scattered foci of mucinous cells (1 - 2 mm each) and branching papillae are seen. The mucinous cells are seen in association with the epithelium of the cysts and in the adjacent alveoli. Children and young adults with unresected or incompletely resected type 1 CPAMs have rarely developed metastatic mucinous adenocarcinoma. While the mucinous cells in this lesion are histologically indistinguishable from mucinous adenocarcinoma in situ, recurrence / metastasis has not been reported following complete resection by lobectomy in infancy. Type 1 CPAMs are associated with mosaic activating KRAS mutations throughout the epithelium and should be entirely resected (Mod Pathol 2022 Jul 6 [Epub ahead of print]).

  • Lung, right upper lobe, lobectomy:
    • Type 2 (small cyst) congenital pulmonary airway malformation (see comment)
    • Comment: The epithelium is flat and simple with prominent mucostasis, consistent with bronchial atresia sequence.
  • Differential diagnosis
    • Type I pleuropulmonary blastoma:
      • Gross: peripherally located multiseptated cyst
      • Back to back cysts without intervening alveoli
      • Septa may have primitive CD56+ round cells but may be regressed
      • Associated with DICER1 mutations
    • Intrapulmonary bronchogenic cyst:
      • Gross: single thick walled cyst
      • Single cyst wall does not connect to adjacent alveolar spaces
      • Usually cartilage or submucosal glands in wall
      • May have type 2 CPAM changes adjacent (due to obstruction during development)
    • Intrapulmonary sequestration:
      • Gross: systemic feeding vessel entering away from the hilum
      • Large cartilage containing airway paired with the feeding vessel (aberrant hilum)
      • Parenchymal maldevelopment is similar to type 2 CPAMs ranging from mild alveolar enlargement to type 2 CPAM-like change
      • Pulmonary hypertensive changes may be present in older patients
    Board review style question #1

    Routine prenatal imaging revealed a cystic lung lesion. The pregnancy was otherwise uncomplicated and the lesion was resected at 3 weeks of age following a term delivery. What is the most appropriate diagnosis?

    1. Type 1 (large cyst) CPAM
    2. Type 2 (small cyst) CPAM
    3. Type 3 CPAM
    4. Type I pleuropulmonary blastoma
    Board review style answer #1
    A. Type 1 (large cyst) CPAM. At least 1 cyst is relatively large. The thicker cyst walls frequently directly transition abruptly into alveolar type spaces. Some of the cyst epithelium has prominent papillary infoldings.

    Comment Here

    Reference: Congenital pulmonary airway malformation (CPAM)
    Board review style question #2

    A right upper lobe resection is performed in a 2 month old baby for an asymptomatic cystic lesion identified on routine prenatal ultrasound. It is most likely associated with which of the following?

    1. An aberrant systemic feeding vessel
    2. Bronchial atresia
    3. DICER1 syndrome
    4. Mosaic activating KRAS mutations
    Board review style answer #2
    B. Bronchial atresia. The cysts here are relatively small with a continuous wall (few transitions to alveolar type walls) and a flat epithelium without complexity. This is a type 2 CPAM, which is associated with bronchial atresia.

    Comment Here

    Reference: Congenital pulmonary airway malformation (CPAM)

    COVID-19
    Definition / general
    Essential features
    • COVID-19 is a viral infection caused by coronavirus SARS-CoV-2 that can progress to severe acute respiratory syndrome with pneumonia and acute respiratory distress syndrome
    • The disease spread rapidly and became a pandemic with > 100 million confirmed cases and over 2 million deaths worldwide by end of January 2021
    • Histologically, COVID-19 shows diffuse alveolar damage corresponding to the phase of the disease (acute to fibrotic), divided into 3 main injury patterns: epithelial, vascular and fibrotic
    • Definite diagnosis is based on detection of viral RNA by RT-PCR
    Terminology
    • COVID-19 is also called novel coronavirus pneumonia
    • Severe acute respiratory syndrome coronavirus 2 (SARS-CoV-2) is also called 2019 novel coronavirus (2019-nCoV)
    ICD coding
    • ICD-10 (emergency codes):
      • U07.1 - COVID-19 confirmed by laboratory testing
      • U07.2 - suspicious for COVID-19 with inconclusive laboratory testing
    • ICD-11:
      • RA01.0 - COVID-19 (definite)
      • RA01.1 - COVID-19 (suspected or probable)
    Epidemiology
    Sites
    • Upper respiratory tract in mild disease
    • Bilateral lobes of the lung in more severe disease
    Pathophysiology
    • Spike surface glycoprotein of the virus binds to the host via receptor binding domains of the angiotensin converting enzyme 2 (ACE2), which is most abundant in type II alveolar cells (J Virol 2020;94:e00127-20)
    • After a SARS-CoV-2 attaches to a target cell, the virion releases RNA into the cell, initiating replication of the virus which further disseminates to infect more cells (Cell 2020;181:271)
    • SARS-CoV-2 produces several virulence factors that promote shedding of new virions from host cells and inhibit immune response
    • Virus independent immunopathology in fatal COVID-19 (Am J Respir Crit Care Med 2021;203:192)
      • Organ injury and death in COVID-19 is immune mediated rather than pathogen mediated
      • Tissue inflammation and organ dysfunction in fatal COVID-19 do not correlate with the tissue and cellular distribution of SARS-CoV-2
    Etiology
    • Severe acute respiratory syndrome coronavirus 2 (SARS-CoV-2), a positive sense, single stranded RNA virus having close genetic similarity to bat coronaviruses (Nature 2020;579:270)
    Diagrams / tables

    Images hosted on other servers:

    SARS-CoV-2

    Pathogenesis of COVID-19

    Epidemic timeline of COVID-19


    Timeline of tissue changes

    Timeline of autopsy studies

    Autopsy PPE

    Clinical features
    • Average time from exposure to symptom onset is 5 days (Ann Intern Med 2020;172:577)
    • Asymptomatic infection rate is 46% (J Med Virol 2020;92:2543)
    • Rare in children, about ~2 - 5% of confirmed cases, with milder symptoms and very low hospitalization rate (< 7%) (JAMA 2020;324:782)
    • Common symptoms in hospitalized patients (JAMA 2020;324:782):
      • Fever (70 - 90%)
      • Dry cough (60 - 86%)
      • Shortness of breath (53 - 80%)
      • Fatigue (38%)
      • Myalgias (15 - 44%)
      • Nausea / vomiting or diarrhea (15 - 39%)
      • Headache, weakness (25%)
    • Patients can present with nonclassical symptoms (JAMA 2020;324:782):
      • Isolated gastrointestinal symptoms
      • Isolated anosmia or ageusia (3%)
    • COVID-19 can progress to severe acute respiratory syndrome and its major clinicopathological phenotypes include pneumonia and acute respiratory distress syndrome
    • Patients who required ICU supportive care presented with acute respiratory distress syndrome, acute cardiac injury, acute kidney injury and shock; up to 15% of them had fatal outcomes (Travel Med Infect Dis 2020;34:101623)
    • Common complications among hospitalized patients (JAMA 2020;324:782):
      • Pneumonia (75%)
      • Acute respiratory distress syndrome (15%)
      • Acute liver injury (19%)
      • Cardiac injury (7 - 17%): troponin elevation, acute heart failure, dysrhythmias, myocarditis (JAMA 2020;324:782)
      • Prothrombotic coagulopathy resulting in venous and arterial thromboembolic events (10 - 25%)
      • Acute kidney injury (9%)
      • Acute cerebrovascular disease (3%)
      • Shock (6%)
    • A rare multisystem inflammatory syndrome similar to Kawasaki disease has recently been described in children (2 per 100,000 persons aged < 21 years) (JAMA 2020;324:782)
    • Post acute COVID-19 syndrome: persistent symptoms, delayed or long term complications beyond 4 weeks
      • Persistent ≥ 1 symptom was reported in 32.6 - 87.4% of cases (fatigue, joint pain, chest pain, dyspnea, cough, loss of taste / smell, headache, diarrhea) (Nat Med 2021;27:601)
    Diagnosis
    Laboratory
    Radiology description
    • Ground glass opacities, crazy paving pattern and consolidation in bilateral lobes are common findings (Radiology 2020;295:715)
    • 15% of CT and 40% of chest radiograph findings are normal early in the disease (JAMA 2020;324:782)
    • Evolution of abnormalities occurs in the first 2 weeks after onset
    Radiology images

    Images hosted on other servers:

    Chest CT images

    Severe changes

    Prognostic factors
    Case reports
    Treatment
    Gross description
    Gross images

    Images hosted on other servers:

    Grossing protocol

    Thickened alveolar septa, thromboemboli

    Patchy consolidations

    Subsegmental pulmonary embolism

    Microscopic (histologic) description
    • Pulmonary changes are the most significant, although nonspecific (Lancet Respir Med 2020;8:420, J Thorac Oncol 2020;15:700, Mod Pathol 2020;33:1007)
      • Findings of diffuse alveolar damage (DAD) corresponding to the phase of disease:
        • Exudative phase: hyaline membrane formation, desquamation of pneumocytes, cellular or proteinaceous exudates, alveolar hemorrhage, fibrinoid necrosis of small vessels
        • Organizing phase: interstitial and intra-alveolar proliferation of fibroblasts, lymphocytic infiltration, type II pneumocyte hyperplasia, fibrin deposition
        • Fibrotic phase: dense collagenous fibrosis, architectural remodeling
      • Lung injury patterns (Mod Pathol 2020;33:2128, Eur J Clin Invest 2020;50:e13259):
        • Epithelial (85%): DAD with varying degrees of organization, denudation, hyperplasia of pneumocytes
        • Vascular (59%): diffuse intra-alveolar fibrin, microvascular damage, (micro) thrombi, acute fibrinous and organizing pneumonia
          • More vascular changes (hemangiomatosis-like changes, thromboemboli, pulmonary infarcts, perivascular inflammation) in fatal COVID-19 cases compared to DAD of other causes (Am J Respir Crit Care Med 2022;206:857)
        • Fibrotic (22%): fibrotic DAD, interstitial fibrosis
          • Prominent ongoing fibrosis in explanted lungs, accompanied by abundant macrophage infiltration, neoangiogenesis and persistent microvascular thrombosis (Am J Clin Pathol 2022;157:908)
      • Viral infection changes:
        • Multinucleated enlarged pneumocytes with large nuclei, amphophilic cytoplasm and prominent nucleoli in alveolar spaces
        • Intranuclear inclusions
      • Bacterial pneumonia may be superimposed
    • Extrapulmonary changes (Mod Pathol 2020;33:2128):
      • Cardiovascular: mild pericardial edema, some serosanguinous pericardial effusion, mild myocardial edema, low grade interstitial infiltration of mononuclear cells, endotheliitis
        • Widespread systemic vasculitis with associated thromboemboli is not as common as initially thought (Lancet 2020;396:320)
      • Hepatobiliary: hepatic congestion, mild steatosis, patchy hepatic necrosis, Kupffer cell hyperplasia, increased number of lymphocyte predominant inflammatory cells in the portal tracts and sinusoids, endotheliitis
      • Renal: varying degrees of acute tubular injury, lymphocytic tubule interstitial infiltration, fibrin or hyaline thrombi in blood vessel, glomerular capillary dilatation, lymphocytic endotheliitis (Kidney Int 2020;98:219)
      • Gastrointestinal: epithelial damage, prominent endotheliitis, ischemic enterocolitis
      • Spleen: reduced number of lymphocytes with necrosis, atrophy, congestion, hemorrhage, infarction
      • Bone marrow: histiocytic hyperplasia, hemophagocytosis (Mod Pathol 2020;33:2139)
      • Other: cutaneous, prostatic manifestations, inflammation and clots in placenta with funisitis
    Microscopic (histologic) images

    Contributed by Toru Igari, M.D., Jin Takahashi, M.D. and Shinyu Izumi, M.D.
    Bronchial change

    Bronchial change

    Squamous metaplasia

    Squamous metaplasia

    Acute phase

    Acute phase

    Hyaline membranes

    Hyaline membranes

    Organizing phase

    Organizing phase


    Organizing phase Organizing phase

    Organizing phase

    Organizing phase Organizing phase

    Organizing phase

    Fibroblastic proliferation

    Fibroblastic proliferation


    Lymphocytic infiltration

    Lymphocytic infiltration

    Pneumocyte hyperplasia Pneumocyte hyperplasia Pneumocyte hyperplasia

    Pneumocyte hyperplasia

    Epithelial denudation

    Epithelial denudation


    Epithelial denudation

    Epithelial denudation

    Perivascular lymphocytes

    Perivascular lymphocytes

    Clear cell change

    Clear cell change

    Multinucleation Multinucleation

    Multinucleation



    Images hosted on other servers:

    Lung injury patterns


    SARS-CoV-2 in lung

    Cytology description
    • Bronchoalveolar lavage (BAL):
      • Inflammatory background (lymphohistiocytic, histiocytic, neutrophil predominant), virus related changes (atypical nuclei, nuclear cytoplasmic inclusion, multinucleation), nonspecific reactive changes (bronchial, alveolar, reserve cell hyperplasia) (Acta Cytol 2022;66:532)
      • Abundant activated plasma cells, as per a single case report (J Thorac Oncol 2020;15:e65)
      • Alveolar macrophages may feature nuclear clearing or intranuclear cytopathic inclusions
      • High neutrophil proportions in mechanically ventilated critically ill patients (Clin Respir J 2022;16:329)
    Cytology images

    Images hosted on other servers:

    Inflammatory background

    Nuclear, cytoplasmic inclusion


    Multinucleated alveolar macrophage

    Multinucleated giant cell

    BAL ThinPrep

    Positive stains
    Electron microscopy description
    • Spherical particles sized 60 - 140 nm
    • Distinctive spikes on the surface (9 - 12 nm) giving virions the appearance of a solar corona, consistent with the Coronaviridae family
    • Inclusion bodies filled with virus particles in membrane bound vesicles in cytoplasm of the respiratory epithelium (N Engl J Med 2020;382:727)
    Electron microscopy images

    Images hosted on other servers:

    Scanning electron microscopy

    Transmission electron microscopy

    Virus in epithelial cells

    Molecular / cytogenetics description
    Videos

    Pathology of COVID-19: a pulmonary pathologist explains

    Sample management

    Autopsy


    SARS-CoV-2 detection in tissues

    Pulmonary pathology

    Cardiovascular pathology


    COVID-19 autopsy pathology findings

    Histology of acute respiratory distress syndrome

    Sample pathology report
    • Usually reported as part of autopsy findings:
      • Diffuse alveolar damage due to severe acute respiratory syndrome coronavirus 2 (SARS-CoV-2) infection
      • Diffuse alveolar damage with changes compatible with viral infection
    Differential diagnosis
    Additional references
    Board review style question #1

    Which of the following is the main histologic finding in the organizing phase of diffuse alveolar damage in COVID-19 infection?

    1. Alveolar hemorrhage
    2. Dense collagenous fibrosis
    3. Fibrinoid necrosis of small vessels
    4. Fibroblastic proliferation
    5. Hyaline membrane formation
    Board review style answer #1
    D. Fibroblastic proliferation

    Comment Here

    Reference: COVID-19
    Board review style question #2



    Which of the following is the target organ affected by SARS-CoV-2 and is responsible for the major manifestation of COVID-19?

    1. Brain
    2. Heart
    3. Kidney
    4. Liver
    5. Lung
    Board review style answer #2
    E. Lung

    Comment Here

    Reference: COVID-19
    Board review style question #3
    Which of the following findings suggests viral infection of the lung?

    1. Alveolar hemorrhage
    2. Fibrin deposition
    3. Neutrophilic aggregation
    4. Nuclear inclusions
    5. Type II pneumocyte hyperplasia
    Board review style answer #3
    D. Nuclear inclusions

    Comment Here

    Reference: COVID-19

    Cystic fibrosis
    Definition / general
    • Autosomal recessive disorder
    • 1 in 20 Caucasians in US are carriers; much lower incidence in African Americans, Asians, Hispanics
    • In US, approximately 1 in 2500 live births have disease
    • Over 1300 known mutations of cystic fibrosis transmembrane conductance regulator / CFTR, most common mutation (seen in 70% in UK) is #508 (Eur J Hum Genet 2002;10:583)
    Etiology
    • Mutations cause reduced chloride ion in secretions and thicker respiratory secretions
    • Primary defect is abnormal function, deficiency or absence of cystic fibrosis transmembrane conductance regulator / CFTR that regulates chloride channel in epithelial cells
    • Allelic variation correlates with some aspects of disease, but lung function, neonatal intestinal obstruction, diabetes and anthropometry display strong genetic control independent of CFTR, and candidate gene studies have revealed genetic modifiers underlying these traits (Ann NY Acad Sci 2010;1214:57)
    Associated Infections
    • Allergic bronchopulmonary aspergillosis: increased incidence
    • Burkholderia cepacia: unique to cystic fibrosis, seen in 20% of patients; causes rapid deterioration of pulmonary status and death; transmitted person to person, has marked social impact as those infected are excluded from social functions (camps) and ineligible for transplant; treat with chloramphenicol and trimethoprim-sulfamethoxazole
    • Hemophilus influenza: common
    • Mycobacteria (atypical): common
    • Pseudomonas aeruginosa: bacteria produces alginate, a capsular protein that mediates adherence; mucoid phenotype is unique to cystic fibrosis; bacteria is never eradicated from lung; treat with ceftazidime (BMC Med 2011 Apr 4;9:32)
    • Staphylococcus aureus: infection persists despite treatment
    • Stenotrophomonus maltophilia: aerobic Gram-negative rod, multidrug resistant, smells like onions; treat with trimethoprim-sulfamethoxazole, resistant to imipenim
    • Also anaerobes, although difficult to detect (New Microbiol 2010;33:185)
    • Note: many taxonomic changes have occured (Clin Microbiol Rev 2010;23:299)
    Diagrams / tables

    Images hosted on other servers:
    CFTR CFTR

    CFTR

    Clinical features
    • Disease usually manifests in young children, but also prenatally or in adolescents
    • Recurrent infections cause chronic lung disease, the most severe manifestation of the disease; recurrent upper respiratory infections occur, late sequela are pancreatic insufficiency, steatorrhea, malnutrition, cirrhosis
    • Mutations also cause defective cilia and infertility
    • Meconium ileus seen in 5% - 10% of patients; also intussusception
    • Heterozygotes (carriers) also have higher incidence of respiratory and pancreatic disease than general population
    Gross description
    • Emphysema, bronchiectasis, abscess, fibrosis
    Differential diagnosis
    • Kartegeners (defective cilia syndrome)

    Cytology
    Table of Contents
    Definition / general
    Definition / general
    • Sputum or bronchial brushings are 80 - 90% sensitive for lung carcinoma if examine 5+ sputum samples
    • Fine needle aspirate is superior to bronchoalveolar lavage for diagnosis of pathologic pathology (Mod Pathol 2002;15:1259)
    • False positives: due to misidentification of macrophages or pneumocytes - includes bronchiectasis, fungal infections, infarct, lipoid pneumonia, postradiation changes, viral pneumonia
    • False negatives: often due to sampling (Diagn Cytopathol 2012;40:556); pleural fluid cytology is often falsely negative for small cell carcinoma
    • Also useful for mutation analysis (Acta Cytol 2012;56:661)

    Desquamative interstitial pneumonitis
    Definition / general
    • In 1965, Liebow et al reported a series of cases with interstitial pneumonia characterized by massive proliferation and desquamation of epithelial cells and coined the term "desquamative interstitial pneumonia" (DIP) (Am J Med 1965;39:369)
    • In 1977, Tubbs et al determined, based on electron microscopy, that most cells within alveolar spaces were macrophages (Chest 1977;72:159)
    • In 2013, American Thoracic Society / European Respiratory Society (ATS / ERS) categorized DIP as a smoking related interstitial pneumonia, as well as respiratory bronchiolitis related interstitial lung disease (Am J Respir Crit Care Med 2002;165:277, Am J Respir Crit Care Med 2013;188:733)
    Essential features
    • A rare but characteristic type of smoking related lung disease
    • Histologically, massive accumulation of intra-alveolar macrophages is the key finding
    Terminology
    • Although “desquamated cells” are now recognized as intra-alveolar macrophages and ATS / ERS once considered to change the name of this entity to “alveolar macrophage pneumonia”, it is still officially called “desquamative interstitial pneumonia” (Am J Respir Crit Care Med 2002;165:277)
    ICD coding
    • Desquamative interstitial pneumonia: J84.117
    Epidemiology
    Sites
    • Bilateral (or sometimes unilateral) lobes of the lung
    Pathophysiology
    • Respiratory bronchiolitis, respiratory bronchiolitis associated interstitial lung disease (RB-ILD) and DIP are now considered on a spectrum and it can be difficult in some cases to separate these diseases clearly (Histopathology 2011;58:509)
      • Deaths are somewhat common in DIP (6 - 30%) but rare in respiratory bronchiolitis and RB-ILD
    • Studies with mouse models suggest a two hit development (Chest 2015;148:1307, Antiviral Res 2011;92:319)
      • Primary smoking exposure induces GM-CSF secretion from airway epithelial cells
      • Subsequent infection provokes macrophage activation and accumulation
    Etiology
    Clinical features
    • Slight to mild chronic respiratory failure
      • Dyspnea on exertion
      • Dry cough
      • Digital clubbing
    • Abnormal chest auscultation
      • End inspiratory fine crackles in bibasilar lung
    • Mild systemic symptoms may accompany
      • Fever
      • Fatigue
      • Weight loss
    • Normal or slightly abnormal pulmonary function tests (restrictive, obstructive or mixed pattern)
      • Decreased forced vital capacity (FVC)
      • Decreased diffusing capacity of the lung for carbon monoxide (DLCO)
    • See details in review (Eur Respir Rev 2013;22:117, Histopathology 2011;58:509, Chest 2017;3692:33197)
    Diagnosis
    • Based on clinical features, radiology and histopathology (Am J Respir Crit Care Med 2002;165:277)
      • Lung biopsy is required to establish a firm diagnosis since the clinical and radiological findings are often not specific
      • However, a biopsy may not be necessary if the clinical and radiological features are suggestive enough (Radiology 1999;211:555)
    Laboratory
    • Increased serum KL-6
    Radiology description
    • Simple chest radiograph (Clin Radiol 2003;58:259)
      • Variable and nonspecific, including ground glass opacity and reticulonodular shadow
    • High resolution CT (Clin Radiol 2003;58:259, Chest 2017;3692:33197)
      • Bilateral and subpleural ground glass opacity
      • Middle to lower zones are affected predominantly; however, the upper zone can be involved
      • Linear shadow, cystic spaces, emphysema, traction bronchiectasis and honeycombing can be accompanied
    Radiology images

    Images hosted on other servers:
    High resolution CT High resolution CT High resolution CT

    High resolution CT

    Prognostic factors
    Case reports
    Treatment
    Gross description
    • Diffuse bronchocentric multinodular change with lower lobe predominance
    • Firm and consolidated
    • Pale gray to white
    • Mild to moderate increase in weight of the lung
    • Honeycomb change can be seen but sparsely
    Microscopic (histologic) description
    • Diffuse and massive accumulation of intra-alveolar macrophages (smoker's macrophages)
      • Brown pigments, more finely granular than hemosiderin, are seen in the eosinophilic cytoplasm (smoker's pigments)
      • Giant cells can be also seen
    • Mild to moderate interstitial fibrosis or cellular change
      • Diffuse and homogeneous involvement (similar to NSIP) along with alveolar macrophages
      • Lymphoid follicles and eosinophils are often present
      • Marked dense fibrosis, architectural distortion, fibroblastic focus or presence of intact alveolar walls in the area of accumulated macrophages should suggest another diagnosis
    • See details (Histopathology 2011;58:509, Chest 2017;3692:33197, Chest 2015;148:1307, Eur Respir Rev 2013;22:117)
    Microscopic (histologic) images

    Contributed by Akira Yoshikawa, M.D.
    Low power

    Low power

    Middle power Middle power

    Middle power

    Smoker's macrophages Smoker's macrophages

    Smoker's macrophages


    Giant cells

    Giant cells

    Lymphoid follicles

    Lymphoid follicles

    Low power

    Low power

    Middle power Middle power

    Middle power


    Smoker's macrophages and interstitial fibrosis Smoker's macrophages and interstitial fibrosis Smoker's macrophages and interstitial fibrosis

    Smoker's macrophages and interstitial fibrosis



    Images hosted on other servers:
    Trichrome staining

    Trichrome staining

    H&E and IHC of CD68

    H&E and IHC of CD68

    Virtual slides

    Images hosted on other servers:
    Dr. Rosai's collection<br>- Desquamative interstitial<br>pneumonitis

    Dr. Rosai's collection
    - Desquamative interstitial
    pneumonitis

    Cytology description
    • Increased eosinophil or neutrophil ratio in bronchoalveolar lavage fluid
    Positive stains
    Electron microscopy images

    Images hosted on other servers:
    Numerous lysosomes in macrophages Numerous lysosomes in macrophages Numerous lysosomes in macrophages

    Numerous lysosomes in macrophages

    Videos

    Histopathology Lung - Desquamative interstitial pneumonia

    Differential diagnosis
    • Alveolar hemorrhage
      • Hemosiderin laden macrophages are seen
      • Hemosiderin is positive for Prussian blue but more coarse and larger
    • Eosinophilic pneumonia
      • Marked interstitial eosinophils and edema, organizing pneumonia, mucus plugs and fibrin deposition are seen
    • Hypersensitivity pneumonitis
      • Clinically, smoking history is less common
      • Patinets have a history of exposure to antigens such as spores of fungi, animal proteins and chemicals
      • Positive for serum antibody to causative antigens
      • Airway centered change, interstitial cellular infiltration and granuloma are seen
    • Nonspecific interstitial pneumonias
      • Accumulation of intra-alveolar macrophages may be seen but focal or mild (DIP-like reaction)
      • Clinically, patients with NSIP often associated with collagen tissue disease
    • Pneumoconiosis
      • Clinically, the patients usually have a history of occupational exposure
      • Macrophages, giant cells and causative particles (e.g. asbestos body, anthrosilicotic dust) are seen especially in interstitium
    • Pulmonary Langerhans cell histiocytosis
      • Bronchocentric nodules of Langerhans cells
      • Positive for CD1a, S100 and Langerin
    • Respiratory bronchiolitis
      • Most common lung disease in smokers
      • Clinical manifestation is slight or absent
      • Macrophage accumulation in bronchocentric predominance is seen on histology
      • Interstitial pneumonia, lymphoid follicles, giant cells and eosinophils are absent
    • Respiratory bronchiolitis-associated interstitial lung disease
      • Less common lung disease in smokers
      • Physical deterioration, including respiratory symptoms and abnormal pulmonary function, is significant enough to diagnose as interstitial lung disease
      • Almost identical to respiratory bronchiolitis on histology
    • Usual interstitial pneumonia
      • Marked dense fibrosis, architectural distortion and fibroblastic focus are seen
      • Clinically, UIP / IPF is more progressive and not responding to corticosteroid therapy
    Board review style question #1
      Which clinical and morphological finding is NOT suggestive for DIP?

    1. A history of passive smoking
    2. Bilateral ground glass opacity
    3. Intra-alveolar cells positive for M. tuberculosis
    4. Intra-alveolar cells positive for Prussian blue
    5. Intra-alveolar giant cells
    Board review style answer #1
    C. Accumulated intra-alveolar cells of DIP are called smoker’s macrophages, which harbor aspirated particles in the cytoplasm. DIP is not related to mycobacterial infection.

    Comment Here

    Reference: Desquamative interstitial pneumonitis

    Diffuse idiopathic pulmonary neuroendocrine hyperplasia
    Definition / general
    • Generalized proliferation of pulmonary neuroendocrine cells that can be limited to the respiratory mucosa or invade locally to form tumorlets and carcinoid tumors
    Essential features
    • Diffuse idiopathic pulmonary neuroendocrine hyperplasia (DIPNECH) is considered a preinvasive lesion that may progress into tumorlets or carcinoid tumors (usually typical carcinoid)
    • Histologic features comprise a generalized intramucosal proliferation of pulmonary neuroendocrine cells in monolayers or small groups that can penetrate through the bronchial basement membrane to form tumorlets
    • Diagnostic criteria are still debated; the latest WHO edition separates diffuse idiopathic pulmonary neuroendocrine hyperplasia into a pathological diagnosis based on histologic features and a clinical diagnosis
      • Some authors consider it a syndrome with a distinctive clinical presentation, typical radiological findings and a pathological demonstration
    • Must be differentiated from secondary DIPNECH, which is a localized neuroendocrine proliferation secondary to another chronic lung disease
    • Symptomatic patients present with insidious onset of a nonproductive chronic cough with dyspnea and wheezing that mimics asthma, chronic obstructive pulmonary disease or gastroesophageal reflux
    ICD coding
    • ICD-O: 8040/0 - tumorlet, benign
    • ICD-10: D3A.8 - other benign neuroendocrine tumors
    • ICD-11: 2F00.Y - other specified benign neoplasm of middle ear or respiratory system
    Epidemiology
    Sites
    • Proliferation of neuroendocrine cells arise from terminal bronchioles
    Pathophysiology
    • Not well described
    • Idiopathic by definition
      • Secondary DIPNECH is caused by other chronic lung diseases
    • Considered a preneoplastic proliferation as neuroendocrine cells can cross the basement membrane to form tumorlets and carcinoid tumors (Histopathology 2011;59:751)
    • Clinical symptoms are thought to be caused by the production of peptides by the neuroendocrine cells, which leads to a peribronchiolar fibrosis and an obstructive respiratory defect (Chest 2015;147:415)
    Etiology
    Clinical features
    Diagnosis
    Laboratory
    Radiology description
    • CT scan:
      • Bilateral mosaic perfusion with air trapping is the most important feature (Clin Radiol 2015;70:317, Clin Imaging 2015;39:243)
        • Better evaluated on expiratory CT scan
        • Secondary to a disturbed airflow caused by the proliferation of tumor cells within small airways and constrictive bronchiolitis
      • Multiple bronchiolocentric parenchymal nodules of a solid or ground glass appearance (Clin Radiol 2015;70:317)
        • Represents aggregates of neuroendocrine cells, tumorlets or carcinoid tumors
    Radiology images

    Contributed by Andréanne Gagné, M.D., M.Sc. and Philippe Joubert, M.D., Ph.D.
    CT scan: mosaic perfusion

    CT scan:
    mosaic perfusion

    CT scan: nodules

    CT scan: nodules

    Prognostic factors
    • Poorly understood
    • Prognosis is mostly linked to the progressive worsening of pulmonary function (Chest 2015;147:415)
      • Progression can be rapid (within 2 years) or over a long period
      • Presence of constrictive bronchiolitis on histology is not a prognostic factor
    Case reports
    Treatment
    Gross description
    • Not visible macroscopically
    Frozen section description
    Microscopic (histologic) description
    • Generalized intramucosal proliferation of pulmonary neuroendocrine cells forming monolayers or small groups that can protrude into bronchial lumen
    • Cells do not cross the mucosal basal lamina
    • Cells are round, oval or spindle shaped, have a moderate amount of eosinophilic cytoplasm and have round to oval nuclei with a salt and pepper chromatin
    • If neuroendocrine cells cross the mucosal basal lamina = tumorlet
      • Poorly defined nodules with infiltrative margins in a fibrotic stroma, usually found in relation to an airway
      • Size < 5 mm with < 2 mitoses/2 mm² and absence of necrosis
    • A minimum number of foci of proliferating neuroendocrine cells and tumorlets has been proposed but no consensus exists (Hum Pathol 2015;46:176):
      • Presence of ≥ 5 neuroendocrine cells distributed in a linear fashion or in clusters within the basement membrane in ≥ 3 bronchioles
      • Combined with an association to ≥ 3 tumorlets
    • Must be differentiated from secondary DIPNECH, which is a localized neuroendocrine proliferation associated to an underlying lung pathology
    • Can be associated with constrictive bronchiolitis (44% of patients) (Chest 2015;147:415)
    Microscopic (histologic) images

    Contributed by Andréanne Gagné, M.D., M.Sc. and Philippe Joubert, M.D., Ph.D.
    Small nodules

    Small nodules

    Linear proliferation

    Linear proliferation

    Nodule occluding a bronchiole

    Nodule occluding a bronchiole

    Discrete proliferation

    Discrete proliferation

    Tumorlet

    Tumorlet


    Tumorlet

    Tumorlet

    Salt and pepper chromatin

    Salt and pepper chromatin

    Chromogranin

    Chromogranin

    Synaptophysin

    Synaptophysin

    CD56

    CD56

    Virtual slides

    Images hosted on other servers:
    Missing Image

    Wedge resection with DIPNECH

    Positive stains
    Molecular / cytogenetics description
    Sample pathology report
    • Lung, left upper lobe, wedge resection:
      • Multiple foci of neuroendocrine proliferation (neuroendocrine hyperplasia and neuroendocrine tumorlet) (see comment)
      • Comment: The presence of foci of neuroendocrine proliferation raises the possibility of a diffuse idiopathic pulmonary neuroendocrine cells hyperplasia (DIPNECH). Correlation with clinical symptoms, pulmonary function tests and high resolution chest CT scan with an expiratory phase is recommended to confirm this diagnosis.
    Differential diagnosis
    Board review style question #1

    A 55 year old woman with a long history of chronic cough and asthma undergoes an upper right lobectomy for a lung nodule. The final diagnosis is a typical carcinoid tumor. The adjacent lung parenchyma presents numerous lesions diffusely throughout the lobectomy as depicted in the image. Concerning this diagnosis, which of the following is true?

    1. Diagnostic criteria are well defined and combine histological, radiological and clinical findings
    2. EGFR mutations are frequently found
    3. Constrictive bronchiolitis can be found
    4. Most patients are men
    Board review style answer #1
    C. Constrictive bronchiolitis can be found. The diagnosis here is diffuse idiopathic pulmonary neuroendocrine cell hyperplasia (DIPNECH). Diagnostic criteria for this entity are still debated (A). The WHO classification defines diffuse idiopathic pulmonary neuroendocrine cell hyperplasia with histologic features and some authors have proposed precise diagnostic criteria. However, other groups consider it a syndrome with a distinctive clinical presentation, typical radiological findings and a pathological demonstration. No specific molecular alterations have been described in diffuse idiopathic pulmonary neuroendocrine cell hyperplasia lesions (B). Mengoli et al. (Am J Surg Pathol 2018;42:646) tested a panel of targetable genes and have not found any specific molecular alterations. Concerning histologic findings, constrictive bronchiolitis can be found and is thought to be secondary to the production of peptides by the neuroendocrine cells (C). Finally, an overwhelming majority of patients with diffuse idiopathic pulmonary neuroendocrine cell hyperplasia are women (F:M, > 10:1) (D).

    Comment Here

    Reference: Diffuse idiopathic pulmonary neuroendocrine hyperplasia
    Board review style question #2
    Regarding the histologic features of diffuse idiopathic pulmonary neuroendocrine cell hyperplasia (DIPNECH), which of the following is true?

    1. There is a generalized intramucosal proliferation of pulmonary neuroendocrine cells
    2. There is a localized intramucosal proliferation of pulmonary neuroendocrine cells around bronchiectasis
    3. Tumorlets can have a size of > 5 mm
    4. A tumorlet is a nodule of neuroendocrine cells that is limited to the bronchial mucosa
    Board review style answer #2
    A. There is a generalized intramucosal proliferation of pulmonary neuroendocrine cells. A generalized intramucosal proliferation of pulmonary neuroendocrine cells (A) is found in diffuse idiopathic pulmonary neuroendocrine cell hyperplasia cases. It must be differentiated from localized pulmonary neuroendocrine cell proliferations that can be found associated to chronic lung diseases (B). Tumorlets are, by definition, < 5 mm (C) and cross the basal membrane of the mucosa (D).

    Comment Here

    Reference: Diffuse idiopathic pulmonary neuroendocrine hyperplasia

    Diffuse pulmonary lymphangiomatosis
    Definition / general
    • Diffuse proliferation of abnormal, complex lymphatic channels (lymphangiomas) that may involve lung or other organs (Lymphat Res Biol 2011;9:191)
    • Pulmonary involvement often accompanied by chylous pleural effusions
    • May be progressive and fatal from respiratory failure
    Terminology
    • Lymphangioma if proliferation is focal
    Epidemiology
    • Rare condition, typically presents in children and young adults
    • No sex predilection
    Sites
    • Localized to lung / chest or can also involve multiple organs, including bone, spleen, soft tissue, liver
    Pathophysiology / etiology
    • Progressive disease with recurrent effusions and respiratory failure secondary to infections and chylous accumulation
    • Congenital, possibly from abnormal lymphatic development
    Clinical features
    • Generally presents with dyspnea, wheezing, cough and may be confused with asthma
    • Occasionally with chyloptysis or hemoptysis
    Diagnosis
    • Clinical presentation and imaging findings may suggest diagnosis, particularly if bone involvement is noted
    • Most cases diagnosed with open lung biopsy
    • Transbronchial biopsy may confirm disease if vesicles seen on bronchoscopy (Clin Radiol 2005;60:921)
    Laboratory
    • Thoracocentesis may reveal chylous effusion (elevated triglycerides)
    Radiology images
    • Xray: nonspecific findings, including pleural effusion, diffuse pulmonary opacities, interstitial thickening
    • CT: bilateral ground glass opacities, smooth thickening of interlobular septa, bronchovascular bundles, pleura; may show infiltration of the mediastinal soft tissue by cystic fluid densities
    Radiology images

    Images hosted on other servers:
    Lower lung heterogeneous opacities

    Lower lung heterogeneous opacities

    Thickening with effusions

    Thickening with effusions

    Prognostic factors
    Case reports
    Treatment
    Clinical images

    Images hosted on other servers:
    Subpleural lymphatic vessels

    Subpleural lymphatic vessels

    Gross description
    • Subpleural and septal thickening
    Microscopic (histologic) description
    • Diffuse proliferation of complex, anastomotic lymphatic channels without significant dilatation, lined by benign appearing, flattened endothelial cells
    • Prominence of lymphatic channels in visceral pleura, interlobular septa, and bronchovascular bundles with surrounding bundles of spindle cells, interspersed collagen, vessels
    • Lymphatic channels may contain acellular, eosinophilic, proteinaceous material
    • May show hemorrhagic kaposiform component, with compressed vascular channels, plump spindle cells, hemosiderin
    Microscopic (histologic) images

    Images hosted on other servers:
    D2-40

    D2-40

    Native lung

    Native lung

    Multiple proliferating vascular spaces

    Multiple proliferating vascular spaces

    Positive stains
    Negative stains
    Differential diagnosis

    Drug induced pneumonitis
    Table of Contents
    Definition / general
    Definition / general
    • Hundred of drugs are associated with lung injury, including pulmonary edema, pulmonary hemorrhage, airway disease, pleural changes and vascular changes
    • Bleomycin and amiodarone (5% - 15% of patients) causes pneumonitis and fibrosis
    • Methotrexate, nitrofurantoin causes hypersensitivity pneumonitis

    Emphysema
    Definition / general
    • Abnormal enlargement of air spaces distal to the terminal bronchioles characterized by destruction of the alveolar septa with little or no fibrosis
    Essential features
    • Destruction of acinar structure and airspace enlargement
    • Affects multiple pulmonary functions and causes chronic respiratory symptoms
    Terminology
    • By the Global Initiative for Chronic Obstructive Lung Disease (GOLD), an international organization launched by the National Heart, Lung, and Blood Institute, National Institutes of Health, U.S. and the World Health Organization (WHO)
    • Chronic obstructive pulmonary disease (COPD) (Global Initiative for Chronic Obstructive Lung Disease: 2021 GOLD Reports [Accessed 20 December 2021])
      • Common progressive disease that is characterized by persistent respiratory symptoms and usually associated with the prolonged exposure to noxious particles or gases, especially smoking
      • Airflow limitation is caused by a combination of airways disease (chronic bronchitis) and lung parenchymal destruction (emphysema)
    • Major types of emphysema (Lancet 2004;364:709, Int J Chron Obstruct Pulmon Dis 2008;3:193):
      • Centriacinar, centrilobular or proximal acinar emphysema (mostly associated with smoking and COPD)
      • Panacinar or panlobular emphysema
      • Paraseptal or distal acinar emphysema
    • Minor types of emphysema:
      • Combined pulmonary fibrosis and emphysema (CPFE)
      • Interstitial emphysema
      • Bullous emphysema
      • Senile emphysema
      • Irregular emphysema
      • Congenital lobar emphysema
    ICD coding
    • ICD-10:
      • J43.1 - panlobular emphysema, panacinar emphysema
      • J43.2 - centrilobular emphysema
      • J43.8 - other emphysema
      • J43.9 - emphysema, unspecified
    • ICD-11:
      • CA21.1 - panlobular emphysema, panacinar emphysema
      • CA21.2 - centrilobular emphysema
      • CA21.Y - other specified emphysema
      • CA21.Z - emphysema, unspecified
    Sites
    • Centriacinar emphysema
      • Upper lobe predominant
      • Respiratory bronchioles and surrounding lung parenchyma
    • Panacinar emphysema
      • Lower lobe predominant
      • Entire acinus
    • Paraseptal emphysema
      • Upper lung
      • Distal part of acinus in subpleural area
    Pathophysiology
    • Destruction of lung parenchymal tissue due to chronic inflammation
    • Protease mediated destruction of elastin is an important feature (Thorax 2016;71:105)
    • Morphological progression (Int J Chron Obstruct Pulmon Dis 2016;11:2287, Clin Anat 2015;28:227)
      • Increase in size and number of small fenestrae in alveolar walls (pores of Kohn), which leads to loss of elastic recoil
      • Breakdown and merging of fibrovascular trabeculae (framework)
      • Remodeling of acini results in airspace enlargement
    Etiology
    Diagrams / tables

    Images hosted on other servers:

    Components of COPD

    Classification of emphysema

    High resolution CT diagram

    Clinical features
    • Dyspnea; chronic, progressive and usually irreversible
    • Chest inflation
    • Combined pulmonary fibrosis and emphysema (CPFE) (Chest 2012;141:222, Eur Respir J 2005;26:586)
      • Coexistence of interstitial fibrosis and emphysema of unknown causes
      • Patients with CPFE have different pulmonary function tests and outcomes compared to patients with pure emphysema or pure fibrosis
    • Interstitial emphysema (Am J Surg Pathol 2014;38:339)
      • Air gains access to the pulmonary interstitium to cause air leak and pneumothorax
      • Common in premature infants
      • Adults: commonly in usual interstitial pneumonia but can occur in any interstitial lung diseases
    • Bullous emphysema
      • Formation of multiple bullae > 1 cm with thin wall
      • Can cause bullae inflation and pneumothorax
    • Senile emphysema
      • Due to age related alteration of acini
    • Irregular emphysema
      • Occurs in relation to scars
    • Congenital lobar emphysema (Pediatr Clin North Am 1994;41:453)
      • Hyperinflation of one or more lobes due to malformation of bronchioles
      • Causes respiratory distress
      • Can be sporadic or caused by autosomal dominant inheritance
    Diagnosis
    • Primarily diagnosed by imaging studies with compatible clinical manifestation
    • Rarely diagnosed solely in biopsy specimens
      • More commonly diagnosed in autopsy specimens or as a background finding in resection specimens
    Laboratory
    • Screening for alpha-1 antitrypsin deficiency (AATD) is recommended by some experts in all patients with COPD (Chronic Obstr Pulm Dis 2016;3:668)
      • Alpha-1 antitrypsin (AAT) serum level and genotype
    Radiology description
    • Xray: overinflation of the lung but it may not be seen in early stages
    • Chest CT: emphysematous regions are represented by low attenuation areas (Eur Respir J 2016;48:216)
    Radiology images

    Images hosted on other servers:

    Centriacinar emphysema

    Centriacinar emphysema

    Panacinar emphysema

    Paraseptal (distal acinar) emphysema


    Interstitial emphysema

    CPFE

    AATD

    Prognostic factors
    Case reports
    Treatment
    Gross description
    • Hyperinflation or ballooning due to entrapment of the air from airway obstruction
      • Lungs may be overlapping the heart
      • Upper lobes are more involved
    • Centriacinar: sparse empty spaces with pigmentation (anthracosis) corresponding enlarged airspaces
    • Panacinar: airspaces are more or less evenly inflated throughout the secondary lobules
    • Paraseptal: inflated subpleural airspaces with thin walls
    • Bullous emphysema
      • Descriptive term regarding emphysematous lung with visible bullae upon gross examination
      • Bulla: an air filled space of 1 cm in diameter within the lung which has developed because of emphysematous destruction of the lung parenchyma (StatPearls: Bullous Emphysema [Accessed 5 January 2022])
    Gross images

    Images hosted on other servers:

    Centriacinar emphysema

    Centriacinar emphysema

    Panacinar emphysema


    Paraseptal (distal acinar) emphysema

    Centriacinar versus panacinar emphysema

    Cystic formation from alveolar wall destruction

    Lung bullae

    "Dirty holes" in subtle emphysema


    Combined emphysema and fibrosis

    Microscopic (histologic) description
    • General findings
      • Airspace enlargement; the size of airspace in the background parenchyma serves as a reference; although exact microscopic criteria have not been established
      • Fragmented alveolar walls
        • If the acinar arrangement is maintained, it is representing pores of Kohn
        • If not, it is representing acinar destruction
      • Mild fibrotic change can be seen
      • Any degrees of inflammation can accompany
      • Subtype is determined with histological landmarks, although exact classification is better appreciated at gross exam:
        • Bronchovascular bundle of terminal bronchiole and arteriole is in the center of acinus
        • Connective tissue septa (secondary lobule of Miller) are the periphery of acinus, which is often ambiguous in less inflamed lung
      • Note: similar appearance can be seen due to inadequate inflation or fixation of specimen
    • Combined pulmonary fibrosis and emphysema (CPFE)
      • Centrilobular emphysema in upper lobes
      • Usual interstitial pneumonia in lower lobes
    • Interstitial emphysema
      • Elongated or angulated spaces in fibrotic interstitium
      • Surrounded by dense fibrosis and lining of multiple giant cells but not epithelium
      • Most commonly around bronchovascular bundles
    Microscopic (histologic) images

    Contributed by Akira Yoshikawa, M.D.
    Normal lung architecture

    Normal lung architecture

    Peribronchiolar region

    Peribronchiolar region

    Centriacinar involvement Centriacinar involvement Centriacinar involvement

    Centriacinar involvement

    Panacinar emphysema

    Panacinar emphysema


    Paraseptal emphysema

    Paraseptal emphysema

    Virtual slides

    Images hosted on other servers:

    Severe emphysema

    Centriacinar emphysema

    Panacinar emphysema

    Paraseptal emphysema with fibrosis

    Positive stains
    • Elastic stains, such as elastic van Gieson and elastica Masson
    Electron microscopy images

    Images hosted on other servers:

    Remodeling of elastin and collagen

    Videos

    Emphysema histology

    Emphysema classification

    Sample pathology report
    • Lung, right upper lobe, wedge resection:
      • Squamous cell carcinoma (see comment)
      • Comment: Background lung parenchyma with emphysema, respiratory bronchiolitis and smoking related interstitial fibrosis.
    Differential diagnosis
    Board review style question #1

    What is the most common etiology associated with this alveolar abnormality?

    1. Autoimmune disease
    2. Cigarette smoking
    3. Congenital defect
    4. Infection
    5. Malignancy
    Board review style answer #1
    B. Cigarette smoking has a strong association with emphysema, particularly centriacinar

    Comment Here

    Reference: Emphysema
    Board review style question #2
    Which one of the following lung pathologies can be found in individual with alpha-1 antitrypsin deficiency (AATD)?

    1. Bronchiolitis
    2. Centriacinar emphysema
    3. Interstitial emphysema
    4. Panacinar emphysema
    5. Usual interstitial pneumonia
    Board review style answer #2
    D. Lung pathologies in AATD patient include panacinar emphysema and bronchiectasis

    Comment Here

    Reference: Emphysema

    Enteric
    Definition / general
    • Pulmonary enteric adenocarcinoma (PEAC) is an extremely rare subtype of non-small cell lung cancer that is characterized by pathological features similar to those of colorectal adenocarcinoma
    Essential features
    Terminology
    • Adenocarcinoma, enteric type
    • Primary lung enteric adenocarcinoma
    • Pulmonary intestinal type adenocarcinoma (not recommended)
    • Pulmonary adenocarcinomas showing enteric differentiation (not recommended)
    ICD coding
    • ICD-O: 8144/3 - adenocarcinoma, intestinal type
    • ICD-10: C34 - malignant neoplasm of bronchus and lung
    • ICD-11: 2C25.0 & XH0349 - adenocarcinoma of bronchus or lung & adenocarcinoma, intestinal type
    Epidemiology
    Sites
    Pathophysiology
    Etiology
    Clinical features
    Diagnosis
    • Tumor with enteric morphology and expression of intestinal markers (CDX2, CK20, HNF4a or MUC2) and coexpression of TTF1 or CK7
    • Considering the common clinical features with other adenocarcinomas, a combination of clinical signs, histopathology, IHC and molecular features is required for a definitive diagnosis
    • Colorectal adenocarcinoma should be ruled out by endoscopy and radiology, even if the pathological results correspond to primary enteric adenocarcinoma of the lung
    • Pathology and immunohistochemistry results are mostly used in the diagnosis of PEAC (World J Clin Cases 2021;9:9236)
      • The patient can finally be diagnosed with PEAC when primary PEAC consists mainly of tissue with > 50% intestinal differentiation, tumor cells are positive for at least 1 immunohistochemical marker associated with colorectal cancer (e.g., CK20, CDX2, MUC2, villin, etc.) and tumors of gastrointestinal origin are excluded
      • CK7 is important for pulmonary origin
    Laboratory
    Radiology description
    Radiology images

    Images hosted on other servers:
    Right upper lobe and mediastinal lymph node metastasis

    Right upper lobe
    and mediastinal
    lymph node
    metastasis

    Tumor in the lower lobe of the left lung

    Tumor in the lower lobe of the left lung

    Prognostic factors
    Case reports
    Treatment
    • Current treatment strategy is the same as for primary adenocarcinoma
    • A comprehensive treatment supported by surgical treatment, chemotherapy, radiotherapy or targeted therapy is selected according to clinical stage (Transl Oncol 2021;14:101123, World J Clin Cases 2021;9:9236)
    • Patients may benefit from anti-HER2 therapy and immune checkpoint inhibitors (Transl Oncol 2021;14:101123)
    • EGFR tyrosine kinase inhibitor in PEAC is unreasonable and inefficient (J Transl Med 2018;16:81)
    • Patients may be more likely to benefit from checkpoint blocking immunotherapy (J Transl Med 2018;16:81)
    • Although only limited data are available, alterations eligible for tyrosine kinase inhibitor therapy seem to be less common than in other non-small cell lung cancer subtypes (Mod Pathol 2019;32:855)
    Gross description
    Frozen section description
    • An adenocarcinoma with enteric morphology may be said to be present on frozen sections but without immunohistochemical study and radiological and clinical correlation, it cannot be determined whether the malignancy in the lung is primary or secondary
    Microscopic (histologic) description
    • Enteric pattern exhibits the features of colorectal adenocarcinoma, which has glandular, papillary or cribriform structures with luminal necrosis, tall columnar cells with pseudostratified and atypical nuclei, and eosinophilic cytoplasm (Oncol Lett 2017;13:4651, Int J Clin Exp Pathol 2014;7:1266)
    • Enteric pattern can be admixed with other patterns of adenocarcinoma
    • Mitotic figures are often found
    • Stroma is often desmoplastic and associated with prominent inflammatory cell infiltrates
    • Histologically, PEAC has features of intermediate differentiation and sometimes forms a cribriform pattern with tall columnar cells arranged in irregular acini or with extensive central necrosis (Oncotarget 2017;8:63442)
    Microscopic (histologic) images

    Contributed by Sevilay Özmen, M.D. and Pembe Oltulu, M.D.

    Moderately differentiated glands with necrotic debris

    Cribriform and acinar growth pattern

    Glands filled with necrotic debris

    Cribriform growth pattern

    Cribriform growth pattern

    Glands filled with necrotic debris

    Glands filled with necrotic debris


    CK7 positivity CK7 positivity

    CK7 positivity

    CK20 positivity CK20 positivity

    CK20 positivity

    CDX2 positivity

    CDX2 positivity

    TTF1 positivity

    TTF1 positivity

    Cytology description
    • Sheets or papillary clusters of high columnar cells
    • Intracytoplasmic mucin
    Cytology images

    Contributed by Sevilay Özmen, M.D.

    Small clumps of tumor cells

    Cluster of tumor cells

    Cluster of tumor cells

    Papillary cluster

    Papillary cluster

    Acinar cluster

    Acinar cluster

    Positive stains
    Molecular / cytogenetics description
    Sample pathology report
    • Lung, right upper lobe, resection:
      • Pulmonary enteric adenocarcinoma (see synoptic report)

    • Lung, biopsy:
      • Adenocarcinoma with enteric differentiation (see comment)
      • Comment: In the case where malignancy in the gastrointestinal tract was excluded, the findings are consistent with enteric differentiated adenocarcinoma of the lung.
    Differential diagnosis
    Board review style question #1
    What gene mutation is most commonly associated with enteric adenocarcinoma?

    1. BRCA
    2. EGFR
    3. KRAS
    4. NRAS
    5. PIK3CA
    Board review style answer #1
    C. KRAS. The KRAS gene in pulmonary enteric adenocarcinoma has a high mutation rate of ~40 - 50%.

    Comment Here

    Reference: Enteric adenocarcinoma
    Board review style question #2

    A 60 year old man presented with complaints of cough, chest tightness and shortness of breath. He was a former smoker with 20 pack years. Chest tomography showed a 6 cm mass in the periphery of the lung. A biopsy was performed. Tumor cells immunohistochemically express CK7, CK20, CDX2 and TTF1 and the patient does not show malignancy in the gastrointestinal tract. According to the image above, what is the pathological diagnosis?

    1. Adenocarcinoma with enteric differentiation
    2. Invasive mucinous adenocarcinoma of the lung
    3. Large cell carcinoma of the lung
    4. Small cell lung cancer
    5. Squamous cell lung cancer
    Board review style answer #2
    A. Adenocarcinoma with enteric differentiation

    Comment Here

    Reference: Enteric adenocarcinoma

    Eosinophilic granulomatosis with polyangiitis (EGPA)
    Definition / general
    • Also called Churg-Strauss syndrome
    • Very rare
    • Systemic vasculitis resembling polyarteritis nodosa or microscopic polyangiitis; associated with asthma, peripheral eosinophilia, pulmonary involvement and fever
    • Rarely presents without pulmonary disease as fever of unknown origin
    • < 50% have positive ANCA, often antimyeloperoxidase
    Treatment
    • Steroids: effective, but patients may relapse
    Microscopic (histologic) description
    • Lung and extrapulmonary sites (skin, heart, nervous system, GI) have prominent eosinophilic infiltrate, granulomatous reaction around necrotic foci with radially arranged histiocytes and pallisading giant cells near small arteries or arterioles, eosinophilic vasculitis
    • May have fibrin rich edema, lymphocytes, sarcoid-like granulomas, focal fibrosis and eosinophilic microabscesses
    Differential diagnosis

    Eosinophilic pneumonia
    Definition / general
    • Acute eosinophilic pneumonia:
      • Diagnosis of exclusion
      • Lung disease associated with eosinophils in alveolar and interstitial spaces, usually with peripheral eosinophilia but excluding Langerhans cell histiocytosis
      • Must exclude drug reactions (antibiotics, cytotoxic or anti-inflammatory drugs), immune disorders (Churg-Strauss syndrome, collagen vascular disease, asthma, hypereosinophilic syndrome, chronic eosinophilic leukemia NOS, myeloid and lymphoid neoplasms with eosinophilia and rheumatoid arthritis), infections (bacteria, Aspergillus, HIV, parasites - helminths, Dirofiliaria and filarial) or tobacco (flavored cigars, new onset of smoking (Chest 2007;131:1234, JAMA 2004;292:2997)
    • Chronic eosinophilic pneumonia:
      • Reaction to drugs, Aspergillus or other fungi, occurs with some malignancies and connective tissue diseases
      • Prolonged (months) febrile illness with cough, weight loss, generalized fatigue, drenching night sweats and peripheral eosinophilia
      • Associated with chronic asthma, usually in setting of allergic bronchopulmonary aspergillosis
      • Xray: patchy infiltrates in peripheral lungs with central sparing
    Clinical features
    • Symptoms: fever, weight loss and shortness of breath
    • Xray: peripheral infiltrate
    • Classified as simple, acute or chronic
    • Simple eosinophilic pneumonia (see Loeffler syndrome)
    • Acute eosinophilic pneumonia: onset in 1 - 4 days, accompanied by fever, cough, dyspnea and chest pain; unknown cause, prominent eosinophils in bronchoalveolar lavage fluid and diffuse alveolar damage at biopsy (Am J Respir Crit Care Med 2002;166:1235)
    Case reports
    • 6 year old boy post chemotherapy for neuroblastoma with bilateral pulmonary infiltrates (Case #105)
    Treatment
    • Steroids cause dramatic response / complete resolution to acute or chronic forms
    Gross description
    • Chronic eosinophilic pneumonia: consolidation, mucus plugs in distal bronchi or bronchioles
    Microscopic (histologic) description
    • Acute eosinophilic pneumonia:
      • Acute form has diffuse alveolar damage
      • Alveolar and interstitial infiltration by eosinophils, also plasma cells and histiocytes
      • May have Charcot-Leyden crystals
      • Variable angiitis, granulomatosis, fibrosis, mucus plugging and bronchiolitis with necrosis
    • Chronic eosinophilic pneumonia:
      • Patchy intraalveolar edema, interstitial inflammation with giant cells and eosinophils with scattered histiocytes and plasma cells
      • Mucus plugs composed of inflammatory cells and cellular debris
      • Charcot-Leyden crystals may be present
      • Often bronchiolitis obliterans
      • Blood vessel infiltration by inflammatory cells is common but no vascular necrosis
      • No diffuse alveolar damage
    Microscopic (histologic) images

    Case #105

    Acute eosinophilic pneumonia

    Differential diagnosis
    Additional references

    Epithelial myoepithelial carcinoma
    Definition / general
    • Low grade malignancy that arises from submucosal bronchial glands, mimics similar salivary gland tumor
    • Very rare in lung, only case reports and small series
    • Long interval to recurrence or metastasis
    Essential features
    • Rare, low grade, primary salivary gland-type carcinoma of the lung
    • Typically arises in bronchial tree from submucosal glands
    • Characterized by biphasic proliferation of duct-like epithelial cells with surrounding myoepithelial cells
    Terminology
    • Previous designations include adenomyoepithelioma, epithelial myoepithelial tumor, epithelial myoepithelial tumor of unknown malignant potential, pneumocytic adenomyoepithelioma (Hum Pathol 2009;40:366, Mod Pathol 2001;14:521)
    ICD coding
    • C33 Malignant neoplasm of trachea
    • C34.00 Malignant neoplasm of unspecified main bronchus
    • C34.01 Malignant neoplasm of right main bronchus
    • C34.02 Malignant neoplasm of left main bronchus
    • Code more peripheral lesions depending on specific lobe, laterality and extent
    Epidemiology
    Sites
    • Bronchial tree, usually endobronchial but a few cases peripheral / parenchymal
    Pathophysiology
    Etiology
    • Arises from submucosal bronchial glands, the lung counterpart of the intercalated duct of the salivary gland (Oncol Lett 2015;10:175)
    Clinical features
    • Cough, hemoptysis, dyspnea or obstructive symptoms if endobronchial location
    • Peripheral lesions may be asymptomatic
    Diagnosis
    • Challenging to diagnose with small biopsy specimens
    • Exclude metastasis from salivary gland
    Radiology images

    Images hosted on other servers:
    Chest Xray and<br>CT of peripheral nodule

    Chest Xray and
    CT of peripheral nodule

    Prognostic factors
    • Complete surgical resection generally curative
    • High mitotic rate, tumor necrosis and nuclear pleomorphism appear to be adverse prognostic factors (Am J Surg Pathol 2001;25:1508)
    Case reports
    Treatment
    Clinical images

    Images hosted on other servers:
    Bronchoscopy with lobulated<br>endobronchial mass

    Bronchoscopy with lobulated
    endobronchial mass

    Gross description
    • Intraluminal polypoid mass in bronchus; may invade parenchyma
    • Well circumscribed, unencapsulated, tan, firm
    • Variable size averaging a few centimeters
    Gross images

    Images hosted on other servers:
    Incidental endobronchial mass

    Incidental endobronchial mass

    Peripheral tumor

    Peripheral tumor

    Microscopic (histologic) description
    • Well circumscribed mass with pushing margin
    • Thin fibrous septa with variable hyalinization or sclerosis
    • Biphasic proliferation of inner epithelial and outer myoepithelial cells with formation of bilayered, duct-like structures
    • Appearance and proportion of two cell types may be variable
    • Epithelial cells flattened, cuboidal or columnar with somewhat clear to eosinophilic cytoplasm
    • Myoepithelial cells may be indistinct, flattened, spindled or clear
    • Lumens may contain pale to eosinophilic amorphous material or debris
    • Scant inflammatory infiltrate may be present
    • Mitoses, significant nuclear atypia, necrosis and hemorrhage typically absent
    Microscopic (histologic) images

    Contributed by Roseann Wu, M.D., M.P.H. and Case #357
    Epithelial myoepithelial carcinoma arising from airway Epithelial myoepithelial carcinoma arising from airway

    Epithelial myoepithelial carcinoma arising from airway

    Lung biopsy Lung biopsy Lung biopsy Lung biopsy

    Lung biopsy


    Lung biopsy

    Lung biopsy



    Images hosted on other servers:
    Tumor with tubular and<br>glandular structures

    Tumor with tubular and
    glandular structures

    Incidental endobronchial mass - H&E Incidental endobronchial mass - H&E

    Incidental endobronchial mass - H&E

    Incidental endobronchial mass - calponin

    Incidental endobronchial mass - calponin

    Incidental endobronchial mass - S100

    Incidental endobronchial mass - S100

    Incidental endobronchial mass - AE1 / AE3

    Incidental endobronchial mass - AE1 / AE3


    Incidental endobronchial mass - CK7

    Incidental endobronchial mass - CK7

    Incidental endobronchial mass - CK903

    Incidental endobronchial mass - CK903

    Incidental endobronchial mass - Ki67

    Incidental endobronchial mass - Ki67

    Positive stains
    Negative stains
    Molecular / cytogenetics description
    Differential diagnosis
    Additional references
    Board review style question #1
      Epithelial myoepithelial carcinoma primary to the lung is believed to arise from what cell type(s)?

    1. Club cells
    2. Ciliated columnar cells
    3. Submucosal bronchial glands
    4. Type 1 pneumocytes
    5. Type 2 pneumocytes
    Board review style answer #1
    C. Submucosal bronchial glands

    Comment Here

    Reference: Epithelial myoepithelial carcinoma

    Epithelioid hemangioendothelioma
    Definition / general
    • Formerly called intravascular bronchioloalveolar tumor (see also Soft tissue topic)
    • 80% women, usually young adults
    • Neoplastic, but usually not metastatic
    • Progressive growth, usually remains within thoracic cavity, may cause death from respiratory insufficiency
    • Other sites: liver, bone
    • 10% have peripheral eosinophilia
    • Poor prognosis if vascular spread, pleural involvement, severe symptoms
    Case reports
    Gross description
    • Multiple round, well demarcated nodules < 2 cm, often in lower lung, with a gray-white peripheral rim and chondroid appearance
    • Resembles metastasis, granulomatous disease or interstitial lung disease
    • May spread along pleura or pericardium and resemble mesothelioma
    Microscopic (histologic) description
    • Central hyalinized stroma, eosinophilic amorphous material or coagulative necrosis with variable calcification surrounded by thin rim of plump eosinophilic endothelial cells
    • Clusters fill alveoli, apparently through pores of Kohn, and occasionally bronchioles, arteries, veins
    • Nuclei are bland, round / oval, may have cytoplasmic vacuoles
    • No / minimal mitoses
    • Lung architecture preserved
    Positive stains
    Electron microscopy description
    • Endothelial features: well developed basal lamina, pinocytotic vesicles, occasional Weibel-Palade bodies
    Differential diagnosis
    • Metastatic tumor from liver or other sites: destroy lung architecture, high mitotic rate
    • Sclerosing hemangioma: destroys lung architecture, negative for vascular markers

    Fetal
    Definition / general
    • Rare tumor resembling fetal lung in pseudoglandular stage
    • Either high grade or low grade
    • May be a variant of pulmonary blastoma without malignant mesenchymal components
    • Low grade tumors associated with upregulation of Wnt signaling pathway (Mod Pathol 2002;15:617)
    • High grade variant is very aggressive
    Microscopic (histologic) description
    • Irregular tubular structures of columnar epithelial cells with clear cytoplasm and oval nuclei, optically clear nuclei rich in biotin
    • Resembles fetal lung in pseudoglandular stage
    • High grade: disorganized glands, large vesicular nuclei, prominent nucleoli, pronounced anisonucleosis, transition to conventional adenocarcinoma, broad areas of necrosis, desmoplastic stroma, p53+, AFP+, no morules (Am J Surg Pathol 1998;22:399)
    Microscopic (histologic) images

    Images hosted on other servers:
    Well differentiated tumor Well differentiated tumor Well differentiated tumor

    Well differentiated tumor

    Cytology description
    • Low grade tumors: subnuclear vacuoles with tigroid features, small, uniform, round nuclei, morules, neuroendocrine differentiation in glandular epithelial cells (Am J Clin Pathol 2010;134:894)
    Cytology images

    Images hosted on other servers:
    Various images

    Various images


    Germ cell (pending)
    [Pending]

    Giant cell interstitial pneumonia
    Definition / general
    • Rare; currently not considered a form of idiopathic interstitial pneumonia
    Microscopic (histologic) description
    • Intra-alveolar multinucleated giant cells and other inflammatory cells
    • Giant cells often phagocytose other histiocytes

    Goodpasture syndrome
    Definition / general
    • Autoimmune disease affecting men > women, usually ages 15 - 29 years, with simultaneous sometimes massive hemorrhagic interstitial pneumonitis and rapidly progressive (crescentic) glomerulonephritis
    • Sometimes preceded by chemical or drug exposure, viral infection or malignancy
    • Genetic predisposition, higher incidence with some HLA subtypes
    • Death common due to uremia
    • Due to IgG antibodies to basement membrane of alveoli and glomeruli
    Treatment
    • Plasma exchange (removes antibodies and chemical mediators)
    • Immunosuppressive therapy (prevents further antibody production)
    Microscopic (histologic) description
    • Lungs: heavy, focal necrosis of alveolar wall, fibrous thickening of septa with mild hyperplasia of alveolar lining cells, organization of blood in alveolar space and hemosiderin laden macrophages; linear deposits of immunoglobulin along basement membrane

    • Kidney: focal proliferative to crescentic glomerulonephritis with linear deposits of immunoglobulin and complement along basement membrane, similar to lung
    Differential diagnosis
    • Mitral stenosis, periarteritis nodosa, SLE and systemic vasculitis all cause secondary pulmonary alveolar bleeding and hemosiderosis
    • Immune complex glomerulonephritis: granular pattern of immunofluorescence
    • Idiopathic pulmonary hemosiderosis: no antibodies

    Granulomatosis with polyangiitis (GPA)
    Definition / general
    • Granulomatosis with polyangiitis (GPA) is a systemic autoimmune vasculitis syndrome commonly involving the lower respiratory tract, the upper respiratory tract and the kidney
    • Characterized by a necrotizing vasculitis and a systemic granulomatous inflammatory process which replaces the involved tissues
    Essential features
    • Classic GPA clinicopathological triad
      • Vascular, respiratory and renal involvement
      • Histopathological features
        • Necrotizing vasculitis of small to medium sized arteries and veins
        • Eosinophilic palisading granulomas
        • Glomerulonephritis
      • Association with antineutrophil cytoplasmic antibodies (ANCA) positivity
        • C-ANCA is more specific for GPA
    Terminology
    • Obsolete term (not recommended): Wegener granulomatosis (WG), name changed due to Wegener's Nazi association in the 1930s (Lancet 2006;367:1362)
    ICD coding
    • ICD-11: 4A44.A1 - granulomatosis with polyangiitis
    Epidemiology
    • Incidence
    • Risk factors
    Sites
    • Systemic vasculitis may manifest in a classic triad involving
      • Head and neck region (upper respiratory tract)
      • Lower respiratory tract
      • Kidney
    • Can also occur in a combination of 1 or 2 of the classic sites, along with additional sites
      • Skin, joints, middle ear, eye and nervous system
      • Solitary lung involvement can occur and is difficult to diagnose for the pathologist
    • Reference: Autoimmun Rev 2014;13:1121
    Pathophysiology
    • Exact pathophysiology is unclear and pathogenesis is likely multifactorial
    Etiology
    • Exact mechanism is unclear
    • Thought to be a result of exposure to infectious, environmental or drug induced triggers in patients with predisposing genetic background
    • Recent discoveries suggest excessive activation of neutrophils that form neutrophil extracellular traps (NETs) (Front Immunol 2019;10:2617)
      • Excessive NET formation is involved in ANCA mediated vascular injury and production of ANCA themselves
      • Viscous cycle of NET formation and ANCA production is thought to drive GPA pathogenesis
    Clinical features
    • Typically manifest in upper / lower respiratory tract and kidney; pulmonary symptoms / signs in the absence of upper respiratory symptoms / signs are unusual
    • Most common presenting symptoms / signs (local): rhinorrhea, purulent / bloody nasal discharge, oral or nasal ulcers, sinus pain, cough, hemoptysis and chest pain
    • Most common presenting symptoms / signs (systemic): polyarthralgias, myalgias, fever, malaise, weight loss
    • Reference: Arthritis Rheum 1990;33:1101
    Diagnosis
    • Clinical setting: corresponding sites of pathology, symptoms, radiological findings and patient demographics
    • Positive ELISA serum C-ANCA test or less commonly P-ANCA test
    • Biopsy or resection shows characteristic histologic findings consistent with GPA
    • Special stains and cultures exclude infections
    • Reference: Arthritis Rheum 1990;33:1101
    Laboratory
    • ELISA test for serum antineutrophil cytoplasmic antibodies (ANCA) serotypes
      • Cytoplasmic (C) [PR3]-ANCA (proteinase 3); highly specific for GPA
      • Perinuclear (P) [MPO]-ANCA (myeloperoxidase); less specific for GPA
    • C-ANCA found in 90% with active generalized disease and 60% with limited disease
    • Positive test in an appropriate clinical setting supports GPA diagnosis
    • Negative test does not exclude the diagnosis, especially with characteristic histopathology
    • ANCA positivity strongly related to relapse in GPA with renal involvement (J Am Soc Nephrol 2015;26:537)
    Radiology description
    • Upper respiratory tract involvement: sinonasal mucosal thickening with bony / cartilaginous erosion
    • Lower respiratory tract involvement: 4 patterns of involvement (Chest 1990;97:906, J Thorac Imaging 1988;3:33, Clin Radiol 1982;33:545)
      • Nodules with or without cavitation, often around bronchovascular bundles or in a subpleural distribution
      • Pulmonary hemorrhage, which can occur with nodules but in some cases the consolidation associated with the hemorrhage predominates
      • Reticulonodular pattern can be the first pattern and is often asymptomatic
      • Peripheral wedge-like consolidation
    • Renal involvement: typically a hypovascular mass with unclear margins; in some cases, opacity of the perirenal fat and para-aortic lymph nodes can be seen
    Radiology images

    Images hosted on other servers:

    Multiple nodules and ground glass opacities

    Large bilateral pulmonary cavities

    Prognostic factors
    • Can be fatal without treatment
    • Excellent prognosis with treatment; 5 year survival is > 80%
    • Increased incidence of infection with higher disease burden, long term exposure to glucocorticoids and kidney involvement
    • Reference: Nat Rev Dis Primers 2020;6:71
    Case reports
    Treatment
    • Usually treated with corticosteroids and cyclophosphamide with excellent prognosis
    • Recent addition of rituximab (monoclonal antibody) for poorly responding cases or relapses suggested (N Engl J Med 2010;363:221)
    • Methotrexate serves as effective maintenance therapy for mild / limited GPA (Am J Med 2003;114:463)
    Gross description
    • Multiple bilateral pulmonary nodules with frequent cavitation; rarely manifests as solitary lung lesion
    Gross images

    Contributed by Yale Rosen, M.D. and Philip Kane, M.D.
    Nodules with central cavitation

    Nodules with central cavitation

    Solid nodule

    Solid nodule

    Solid nodule with areas of cavitation

    Solid nodule with areas of cavitation

    Frozen section description
    • Necrotizing granulomatous inflammation, reactive changes in the background lung should not be interpreted as malignancy
    Microscopic (histologic) description
    • Necrotizing granulomatous inflammation
    • Deeply basophilic necrosis due to the presence of nuclear debris derived from necrosis and karyorrhexis; this type of blue necrosis is referred to as dirty necrosis
    • Necrosis is often described as suppurative given the large number of neutrophils that are often present
    • Necrosis is often described as geographic, which is best appreciated at low magnification and refers to the irregular contours of the necrosis that resemble the outlines of countries on a map (Am J Surg Pathol 1991;15:315)
    • Multinucleated giant cells in the granulomatous inflammation often have strikingly hyperchromatic nuclei
    • Medium to small arteries and veins are heavily inflamed in GPA, the inflammatory cells being neutrophils, histiocytes and lymphocytes
    • Necrotizing vasculitis is one of the diagnostic features and consists of necrosis of the vessel wall by an inflammatory infiltrate; this is best recognized when the vasculitis focally involves the vessel and the remainder of the vessel is intact (Hum Pathol 1988;19:1065)
    • Elastic stains can help identify blood vessels; however, this may present a pitfall since completely necrotic blood vessels within zones of necrosis can be present that do not qualify for true necrotizing vasculitis
    • GPA can also present as diffuse alveolar hemorrhage due to the destruction of capillaries in the alveolar septa
      • This produces an inflammatory infiltrate that is rich in neutrophils and karyorrhectic debris that produces a histologic picture similar to leukocytoclastic vasculitis of the skin
      • This histologic finding is typically classified as capillaritis (Semin Respir Crit Care Med 2004;25:475)
      • In these cases, the adjacent alveoli are filled with hemosiderin laden macrophages, red blood cells and neutrophils
    Microscopic (histologic) images

    Contributed by Matthew J. Cecchini, M.D., Ph.D.
    Blue (dirty) necrosis

    Blue (dirty) necrosis

    Geographic necrosis

    Geographic necrosis

    Blue (dirty) necrosis in core needle biopsy

    Blue (dirty) necrosis in core needle biopsy

    Granulomatous inflammation Granulomatous inflammation

    Granulomatous inflammation


    Vasculitis

    Vasculitis

    Focal vasculitis

    Focal vasculitis

    Small vessel vasculitis

    Small vessel vasculitis

    Hyperchromatic giant cells

    Hyperchromatic giant cells

    Capillaritis

    Capillaritis

    Virtual slides

    Images hosted on other servers:

    GPA with necrotic mass lesion

    Granulomatosis with polyangiitis

    Positive stains
    • Elastic stains can aid in the identification of areas of vasculitis
    Negative stains
    Videos

    Granulomatosis with polyangiitis (pathophysiology, symptoms, treatment)

    GPA symptoms, diagnosis, treatment


    GPA review

    GPA discussion at 5:01

    Sample pathology report
    • Lung, right, lower lobe, wedge biopsy:
      • Necrotizing granulomatous inflammation and vasculitis, most consistent with granulomatosis with polyangiitis (GPA) (see comment)
      • Comment: In this specimen, there is prominent granulomatous inflammation and vasculitis. Histochemical stains for fungal and acid fast organisms are negative. The features are most consistent with a diagnosis of granulomatosis with polyangiitis (GPA). Correlation with the clinical findings and serology is recommended for definitive classification.
    Differential diagnosis
    • Granulomatous infection:
      • Cultures or special stains are positive
    • Eosinophilic granulomatosis with polyangiitis:
      • P-ANCA positive
      • Eosinophil rich inflammation
    • Microscopic polyangiitis:
      • Absence of granulomatous inflammation
    • Acute lupus pneumonitis:
      • Nonspecific findings with alveolar wall damage, inflammation, edema and features of acute lung injury including hyaline membranes
    • Idiopathic pulmonary hemosiderosis:
      • Absence of vasculitis and granulomatous inflammation
    Board review style question #1
    Which of the following serology tests is the most specific for granulomatosis with polyangiitis?

    1. ANA
    2. C-ANCA
    3. CRP
    4. ESR
    5. P-ANCA
    Board review style answer #1
    B. C-ANCA. Cytoplasmic (C) [PR3]-ANCA (proteinase 3) is highly specific for GPA and found in 90% of cases with generalized disease. Answer A is incorrect because ANA detects antinuclear antibodies and is seen in systemic autoimmune disorders such as systemic lupus erythematosus. Answer C is incorrect because CRP stands for C reactive protein and is a test that measure general inflammation in the body and is not specific. Answer D is similarly incorrect because erythrocyte sedimentation rate (ESR) is a general nonspecific marker of inflammation. Answer E is incorrect because P-ANCA is more commonly seen in eosinophilic granulomatosis with polyangiitis.

    Comment Here

    Reference: Granulomatosis with polyangiitis (GPA)
    Board review style question #2

    What histologic finding is seen in this image with septal expansion by neutrophils and karyorrhectic debris?

    1. Acute bronchopneumonia
    2. Acute lung injury
    3. Capillaritis
    4. Granulomatous inflammation
    5. Organizing pneumonia
    Board review style answer #2
    C. Capillaritis. Neutrophils and karyorrhectic debris is best classified as capillaritis. Answers A, B, D and E are incorrect because they should not have neutrophils or karyorrhectic debris in the interstitium.

    Comment Here

    Reference: Granulomatosis with polyangiitis (GPA)

    Grossing & features to report
    Grossing
    • 1 section per cm of tumor diameter, representative of grossly distinct areas
    • Margins (bronchial, pulmonary artery / vein, visceral pleura, other surgical margins)
    • Tumor and adjacent lung
    • Tumor and bronchial wall
    • Tumor and pleura
    • Bronchial mucosa proximal to tumor
    Frozen section
    • May be difficult to distinguish:
    • Touch preparation cytology recommended to supplement frozen section findings (J Clin Pathol 2010;63:675)
    Features to report
  • Editor's note
  • Specimen type and site (involved bronchi or segments, laterality, intact or fragmented)
  • Procedure
  • Histologic type
  • Histologic grade (incorporating nuclear atypia, mitotic index)
  • Tumor size
  • Surgical resection margins (bronchi, peribronchial and perivascular soft tissue)
  • Involvement of bronchi, surfaces covering tumor (pleura, thoracic wall, diaphragm), adjacent structures
  • Angiolymphatic invasion
  • Presence of multiple tumors (intrapulmonary metastases)
  • Pleural dissemination
  • Lymph node involvement (site of lymph nodes, number obtained, number positive, size of largest metastasis, extracapsular invasion)
  • Presence of disease in uninvolved lung / bronchi
  • Presence of histologic treatment effect (if prior chemoradiation therapy)
  • Molecular studies, as appropriate
  • Features to report by organization:

  • Herpes simplex
    Definition / general
    • Herpes simplex virus (HSV), serotypes HSV-1 and HSV-2, cause acute infection followed by latent phase that may reactivate to infectious form
    • See also Skin - nontumor chapter
    • HSV-1 and HSV-2 are alpha group herpes viruses with similar genotypes but serologic differences; may both infect the lung
    • HSV-1 usually affects oral pharynx; HSV-2 usually causes genital infection
    • Virus spreads via sensory neurons where latent infection is established
    • Reactivation may be symptomatic or asymptomatic, virus spreads via nerves to mucus membranes or skin, where it replicates
    • Newborns (see below), patients with impaired mucosal defenses, burn patients, trauma patients, and severely immunocompromised patients are at greatest risk
    • Also associated with coexisting bacterial pneumonia (J Crit Care 2011;26:432.e1)
    • Also be caused by intubation of patient with active oral disease
    • Immunostains, PCR, culture or electron microscopy can confirm the diagnosis if necessary
    Gross description
    • Diffusely firm lungs with small yellow / red necrotic areas
    Gross images

    Contributed by Yale Rosen, M.D.
    Various images Various images

    Various images

    Microscopic (histologic) description
    • Interstitial pneumonia with necrosis of bronchial and alveolar epithelium and acute and chronic inflammatory infiltrate
    • Occasional intranuclear viral inclusions at edge of necrotic areas
    • Neutrophilic infiltrates may resemble bacterial bronchopneumonia, but Cowdry type A or B nuclear inclusions are generally diagnostic
    Microscopic (histologic) images

    Images hosted on other servers:
    Immunostain highlights<br>HSV+ cell in<br>broncheo-alveolar lavage

    Immunostain highlights
    HSV+ cell in
    broncheo-alveolar lavage


    Histology
    Definition / general
    • Lung: organ participating in and in charge of respiration, namely, gas exchange
    • Lung histology: basic microscopic structures of the unaffected lung (i.e. nonneoplastic, noninflammatory and nondegenerated lung tissue)
    Essential features
    • Bronchus has cartilage and bronchial glands, while bronchiole lacks them (Mills: Histology for Pathologists, 5th Edition, 2019)
    • Bronchi and bronchioles up to terminal bronchioles are pure conducting airways, while respiratory bronchioles and alveoli play a role in gas exchange
    • There are 2 different definitions of a lobule (secondary lobule), namely, Miller's definition and Reid's definition (Radiology 2006;239:322)
      • Miller: small lung units separated by interlobular septa and pulmonary veins, according to lung anatomy (Radiology 2006;239:322)
      • Reid: an approximately equal sized unit (about 10 mm in diameter) of areas containing 3 to 5 acini, regardless of the interlobular septum, on the basis of bronchography (Radiology 2006;239:322)
    • The term "primary pulmonary lobule," which indicates alveolar structures distal to respiratory bronchioles, is not commonly used; therefore, a lobule usually means a secondary lobule (Radiology 2006;239:322)
    • Pulmonary acinus is defined as the largest unit participating in gas exchange, although there are different definitions of acinus (Radiology 2006;239:322)
      • One definition of the acinus is a lung unit supplied by a single terminal bronchiole; the other is a lung unit supplied by a respiratory bronchiole
    Physiology
    • Ciliated cell: transports foreign bodies and mucous stream from the bronchus or bronchiole toward the proximal airway (Mills: Histology for Pathologists, 5th Edition, 2019)
    • Mucous cell (goblet cell): secretes sulfomucin
    • Basal cell: acts as a stem cell differentiating into other cells
    • Neuroendocrine cell (Kulchitsky cell): secretes hormones (e.g. catecholamine, serotonin and gastrin releasing peptide) (Lowe: Stevens & Lowe's Human Histology, 4th Edition, 2014)
    • Club cell (formerly Clara cell): plays a key role in lung remodeling, secretes surfactant and repairs airway epithelium
    • Type I alveolar epithelium: responsible for respiration; namely, taking oxygen from the outside into the body and transporting carbon dioxide in the opposite direction
    • Type II alveolar epithelium: secretes surfactant and plays a role as a precursor cell of type I alveolar epithelium (Lung Cancer 1994;10:S53)
    • Pulmonary artery: supplies hypooxgenic blood into the lung in order to exchange the gas between airway and blood
    • Bronchial artery: feeds bronchial wall, bronchiole wall and other connective tissue except alveolar septum with oxygen rich blood (Pawlina: Histology - A Text and Atlas, 8th Edition, 2018)
    • Pulmonary vein: transports oxygen rich blood from the lungs to the left atrium
    • Lymphatics: dual system, namely, flow along bronchovascular bundle from hilum and flow along interlobular septum from visceral pleura
    Gross description
    • Location: periphery of the pulmonary hilum
    • Lungs consist of the branched bronchus (airways) and vasculatures
    • Broncho (bronchiolo) vascular bundle: includes bronchus (bronchiole) and pulmonary artery (peripheral branch) located in the center of pulmonary lobe (or lobule)
    • Pulmonary vein: located in the interlobular septum (Katzenstein: Diagnostic Atlas of Non-Neoplastic Lung Disease, 1st Edition, 2016)
    • Peripheral lung: shows sponge-like appearance with fine wall reflecting the microscopic appearance of alveoli
    Gross images

    Contributed by Hirotsugu Hashimoto, M.D., Ph.D.

    Cut surface of the lung

    Microscopic (histologic) description
    • Bronchus:
    • Bronchiole:
      • Small airway, usually 1 mm in diameter or less, lined by single layer of ciliated epithelium
      • Lacks bronchial glands and cartilage
    • Ciliated cell:
    • Mucous cell (goblet cell):
      • Tall columnar epithelium with clear cytoplasm containing mucin, lacking cilia
    • Basal cell:
      • Laying on the basal membrane of bronchus or bronchiole with high nucleocytoplasmic ratio (Lung Cancer 1994;10:S53)
    • Neuroendocrine cell (Kulchitsky cell):
      • Laying scattered on the basal membrane with granular scant cytoplasm and relatively high nucleocytoplasmic ratio
      • Usually difficult to distinguish from basal cell only by H&E staining
    • Club cell (formerly Clara cell):
      • Toll epithelial cell, no cytoplasmic mucin, no cilia
      • Number of club cells increase as bronchiole heads toward periphery
    • Bronchial gland:
      • Mixed gland composed of both serous and mucinous epithelium
      • Its duct opens to bronchial lumen
    • Interlobular septum:
      • Fibrous tissue separating Miller's secondary lung lobule, which contains pulmonary vein and lymphatic vessel
      • Note that not every secondary lobule is separated by this septum
    • Bronchovascular bundle:
      • Fibrous tissue connecting from pulmonary hilum, which contains bronchus, pulmonary artery, bronchial artery and lymphatic vessel
    • Alveolar duct:
      • Terminal airway opening to alveolus, of which alveolar entrance rings constitute the walls
      • Alveolar entrance rings contain elastic fibers and smooth muscles
    • Alveolus:
      • Sac-like structure lined by alveolar epithelium
      • Alveolar septum contains pulmonary capillaries
      • Strictly speaking, alveolar structure is not sac shaped but is a polyhedral architecture
    • Type I alveolar epithelium (pneumocyte):
      • Thin flat epithelium covering about 90 - 95% of alveolar lumen in area, about 40% of all alveolar epithelia in number
    • Type II alveolar epithelium (pneumocyte):
      • Cuboid epithelium covering 5 - 10% of the alveolar lumen in area, about 60% of all alveolar epithelia in number
    • Pulmonary artery:
    • Pulmonary vein:
      • Vein composed of collagenous fiber, smooth muscle and elastic fiber in various rates
      • The central part contains a lot of elastic fiber but there is less in the periphery
      • Runs in the interlobular septum or periphery of Miller's secondary lung lobule
    • Bronchial artery:
      • Muscular artery supplied by thoracic aorta or costal artery
      • Size is far smaller than bronchus beside it
    • Lymphatic vessel:
      • Thin vasculature lined by endothelia
      • Runs with bronchovascular bundle or interlobular septum
    • Anthracotic pigment:
      • Black pigment, which usually is deposited in the bronchovascular (or bronchiolovascular) bundle and pleura
    Microscopic (histologic) images

    Contributed by Hirotsugu Hashimoto, M.D., Ph.D.

    Bronchovascular bundle

    Bronchial wall

    Miller's secondary lung lobule

    Miller's secondary lung lobule (elastica Masson)

    Peripheral airway (bronchiole to alveoli)

    Alveoli

    Positive stains
    Negative stains
    Electron microscopy description
    • Club cell (formerly Clara cell): elevated higher than ciliated cells nearby (Lowe: Stevens & Lowe's Human Histology, 4th Edition, 2014)
    • Type I alveolar epithelium: fewer organelles
    • Type II alveolar epithelium: lamellar body and electron dense vesicles in the cytoplasm; microvilli on the alveolar surface
    Electron microscopy images

    Images hosted on other servers:

    Type II alveolar epithelium (pneumocyte)

    Board review style question #1
    Which of the following is expressed in the alveolar epithelium?

    1. Cytokeratin 5/6
    2. Cytokeratin 7
    3. Cytokeratin 20
    4. p40
    Board review style answer #1
    B. Alveolar epithelium expresses cytokeratin 7 but does not express cytokeratin 20, high molecular weight cytokeratins (e.g. cytokeratin 5/6 and 34 beta E12) or p40.

    Comment Here

    Reference: Lung nontumor - Normal histology
    Board review style question #2


    Which of the following is located in the center of Miller's secondary lung lobule shown here in the H&E and elastic stain?

    1. Alveolar septum
    2. Alveoli
    3. Bronchiole
    4. Bronchus
    Board review style answer #2
    C. Bronchiole. A bronchiolovascular bundle, including terminal bronchiole, is located in the center of Miller's secondary lung lobule. A bronchiole lacks bronchial glands and cartilage.

    Comment Here

    Reference: Lung nontumor - Normal histology

    Histoplasma
    Clinical features
    • Deep fungal infection indigenous to Ohio and Missouri River valleys and the Caribbean basin (eMedicine)
    • See also Skin - nontumor topic
    • Fungi are thermally dimorphic; spore producing hyphae outside body, but yeast at body temperature
    • Infection due to inhaling dust or soil particles contaminated with bird or bat droppings
    • "Capsulatum" is incorrect - no capsule is present
    • In immunocompetent, generally causes a self limited or latent infection, but chronic pneumonia preferentially involving lung apices associated with systemic symptoms and cough can occur; localized lung lesions, with or without lymph node involvement, are common; also localized lesions, often calcified, in adrenal, lymph nodes, liver, spleen, meninges; resembles tuberculosis with epitheliod granulomas with caseous necrosis; necrotic foci may coalesce to produce large areas of necrosis; with drug therapy or endogenous control, fibrosis and calcification occur, causing "tree bark" lesion
    • In immunocompromised, disease is often virulent and widely disseminated
    • If necessary, microbiologic studies, serology, or molecular studies are confirmatory
    Pathophysiology
    • Infection is incompletely understood, but is usually controlled by helper T cells and heat shock protein that activate macrophages to kill yeast
    Gross description
    • Resembles tuberculosis, may see "tree bark" appearance (due to fibrosis and calcification) or coin lesion
    Microscopic (histologic) description
    • Diagnosis based on identifying small, budding, intracellular yeast in tissue, 2 - 5 microns
    • Yeasts usually visible with H&E, but may need special stains; GMS more sensitive than PAS, but microcalcifications may create false positives
    • No / minimal granulomas in immunosuppressed or neonates; yeasts fill histiocytes and are widely disseminated
    Microscopic (histologic) images

    Contributed by Jamie Shutter, M.D.
    Left images: H&E; right images: GMS Left images: H&E; right images: GMS

    H&E

    Left images: H&E; right images: GMS Left images: H&E; right images: GMS

    GMS



    Images hosted on other servers:
    GMS staining

    GMS staining

    Negative stains

    HIV
    Clinical features
    • Lung disease is most common cause of morbidity and mortality in AIDS patients
    • Diagnose infection or mimickers with bronchoalveolar lavage, transbronchial biopsy or open lung biopsy
    • Nonspecific features resemble DIP or lymphocytic interstitial pneumonia
    • Patients often have multiple infections
    • Lung biopsies in AIDS patients should routinely be stained for pneumocystis, fungi and mycobacteria
    • Characteristically cavitary lesions: Staphylococcus, fungi (Candida, Aspergillus, Cryptococcus [J Acquir Immune Defic Syndr 2010;54:269], Histoplasma, Blastomyces), Mycobacterium tuberculosis, Mycobacterium avium complex, other nontuberculous mycobacteria; Rhodococcus equi, Fusarium, Pseudallescheria
    • AIDS patients also have infections from CMV, Pneumocystis jeroveci (formerly Pneumocystis carinii), toxoplasma, microsporidia, herpesvirus types 1 and 2, adenovirus, Strep. pneumonia, H. influenza, Gram negative rods, Legionella pneumophilia, cryptosporidium, zygomycetes, herpes varicella zoster, Penicillium marneffei (patients from Southeast Asia) and bartonella
    • Rarely pulmonary alveolar proteinosis (J Med Case Reports 2011;5:46)
    • Noninfectious causes of pulmonary infiltrates: Kaposi sarcoma, drug reaction, primary lung cancer, non Hodgkin lymphoma
    • Multiple infections and infections with concurrent malignancy are common in AIDS patients
    Microscopic (histologic) images

    Images hosted on other servers:

    CMV and pulmonary alveolar proteinosis


    Hyalinizing clear cell carcinoma
    Definition / general
    • Low grade and indolent malignant neoplasm rarely seen in lung
    • Head and neck is the most common location of this neoplasm
    Essential features
    • Rare, malignant, low grade neoplasm that arises in the bronchial glands of the middle aged population but is more common in salivary glands of the oral cavity
    • Composed of trabeculae and solid areas separated by dense hyalinizing stroma
    • Keratin and p40 positive
    • Most important differential diagnosis is squamous cell carcinoma, which also stains positively with the same immunohistochemical markers but is much more aggressive
    • Diagnosis can be confirmed by demonstrating EWSR1::ATF1 fusion
    Terminology
    • Hyalinizing clear cell carcinoma of lung (HCCC)
    ICD coding
    • ICD-O: 8310/3 - clear cell adenocarcinoma, NOS
    Epidemiology
    Sites
    • These tumors are commonly located in the oral cavity, followed by uncommon locations, such as trachea, bronchi and nasopharynx (Int J Surg Pathol 2023;31:1187)
    Pathophysiology
    Etiology
    • Etiology of this neoplasm remains unclear
    Clinical features
    • Most cases are detected incidentally during regular physical examination and a few present with dyspnea, chest pain and hemoptysis (Front Oncol 2023;13:1175279)
    Diagnosis
    • These tumors are identified as masses incidentally on radiological imaging and on tumor cells with demonstration of EWSR1::ATF1 gene rearrangement being used as a confirmatory test (Am J Clin Pathol 2017;148:73)
    Laboratory
    • Cytogenetic or molecular study: Ewing sarcoma breakpoint region 1 (EWSR1) fluorescence in situ hybridization is a helpful ancillary test in diagnosing pulmonary hyalinizing clear cell carcinoma; most of these tumors demonstrate EWSR1::ATF1 fusion transcript (Am J Clin Pathol 2017;148:73)
    Radiology description
    Radiology images

    Images hosted on other servers:
    Lobulated mass in right lower lobe of lung

    Lobulated mass in right lower lobe of lung

    Solitary, noncalcified nodule in left lower lobe

    Solitary, noncalcified nodule in left lower lobe

    Preoperative CT of pulmonary HCCC

    Preoperative CT of pulmonary HCCC

    Prognostic factors
    • Excellent prognosis
    • Long term follow up may be helpful in cases having lymph node metastasis, perineural invasion and tumor necrosis, as there is limited data on the prognosis of these cases (Int J Surg Pathol 2023;31:1187)
    Case reports
    Treatment
    • Surgical resection is curative
    Clinical images

    Images hosted on other servers:
    Intrabronchial protrusion of mass

    Intrabronchial protrusion of mass

    Bronchoscopy of pulmonary HCCC and SCC

    Bronchoscopy of pulmonary HCCC and SCC

    Gross description
    • Solitary, solid, well circumscribed / lobulated and creamy white
    Gross images

    Images hosted on other servers:
    Well demarcated, creamy white intrabronchial mass

    Well demarcated,
    creamy white
    intrabronchial
    mass

    Solid, white and well demarcated lung mass

    Solid, white and well demarcated lung mass

    Microscopic (histologic) description
    • Neoplasm with well demarcated borders composed of tumor cells arranged in nests, trabeculae and occasional glands
    • Background stroma is composed of hyalinized collagen bands
    • Tumor cells have clear or eosinophilic cytoplasm, round to oval nuclei with inconspicuous nucleoli (Mod Pathol 2018;31:923)
    • Neural invasion is commonly seen in this neoplasm when compared with mucoepidermoid carcinoma (Mod Pathol 2018;31:923)
    Microscopic (histologic) images

    Contributed by Aliya N. Husain, M.D.
    Nest, trabeculae and glandular pattern

    Nest, trabeculae and glandular pattern

    Monomorphic tumor cells

    Monomorphic tumor cells

    Hyalinized stroma

    Hyalinized stroma

    Inconspicuous nucleoli

    Inconspicuous nucleoli


    Clear to eosinophilic cytoplasm

    Clear to eosinophilic cytoplasm

    Stroma is densely hyalinized

    Stroma is densely hyalinized

    p40 nuclear positivity

    p40 nuclear positivity

    Cytoplasmic positivity with AE1 / AE3 immunostain

    Cytoplasmic positivity with AE1 / AE3 immunostain

    Positive stains
    Negative stains
    Molecular / cytogenetics description
    • Fluorescence in situ hybridization (FISH) assay using EWSR1 gene break apart probe was performed in all cases as a specific diagnostic ancillary technique; all cases harbored the characteristic EWSR1 gene rearrangement
    Molecular / cytogenetics images

    Images hosted on other servers:
    Detection of <i>EWSR1::ATF1</i>

    Detection of EWSR1::ATF1

    Sample pathology report
    • Lung, left, CT guided biopsy:
      • Malignant neoplasm with clear cell features (see comment)
      • Comment: Immunohistochemical studies (done with appropriate controls) showed that the tumor cells are immunoreactive for AE1 / AE3, p63 and negative for PAX8, TTF1, S100, SOX10 and HMB45. The morphology and staining pattern are most consistent with hyalinizing clear cell carcinoma. Fluorescence in situ hybridization assay using EWSR1 gene break apart probe to demonstrate presence of EWSR1 gene rearrangement can be used to confirm the diagnosis.
    Differential diagnosis
    Board review style question #1

    Which one of the following cytogenetic abnormalities is commonly seen in this rare pulmonary neoplasm, which is positive for keratin AE1 / AE3 and p40?

    1. EWSR1::ATF1 fusion
    2. EWSR1::CREM fusion
    3. EWSR1::ETV1 fusion
    4. EWSR1::FEV fusion
    Board review style answer #1
    A. EWSR1::ATF1 fusion. 93% of hyalinizing clear cell carcinoma cases demonstrate EWSR1::ATF1 fusion transcript. Answer B is incorrect because EWSR1::CREM fusion is rarely seen (Am J Surg Pathol 2018;42:1182). Answers C and D are incorrect because EWSR1::ETV1 and EWSR1::FEV fusions have not been demonstrated in this neoplasm.

    Comment Here

    Reference: Hyalinizing clear cell carcinoma

    Hyalinizing granuloma
    Definition / general
    • Rare, nodular lung lesion
    • Usually multiple, bilateral and cause unknown
    • Inflammatory reaction of unknown stimulus
    • Patients often have a history of autoimmune disease, exposure to fungal or mycobacterial antigens
    • Associated with sclerosing mediastinitis, retroperitoneal fibrosis and lymphoma
    • May be progressive, but does not cause death
    Microscopic (histologic) description
    • Central keloid-like collagen, arranged in whorls, surrounded by foreign body giant cells simulating nodular amyloidosis
    • May have plasma cells and lymphocytes between collagenous bands
    • Usually no epithelioid granulomas, no necrosis
    Negative stains
    Differential diagnosis

    Hypersensitivity pneumonitis
    Definition / general
    • Also known as extrinsic allergic alveolitis
    • A complex health syndrome of varying intensity, clinical presentation and natural history
    • Due to immunologically induced inflammation of lung parenchyma in response to inhalation of a large variety of antigens (Am J Respir Crit Care Med 2005;171:792)
    Essential features
    • An interstitial pneumonia with acute to chronic respiratory failure caused by inhalation exposure to a variety of natural or chemical antigens
    • Histologically characterized by airway centered inflammation with fibrosis and poorly formed nonnecrotizing granulomas
    Terminology
    • Also called extrinsic allergic alveolitis
    ICD coding
    • J67.8
    Epidemiology
    • 4 - 15% of interstitial lung diseases (Eur Respir J Suppl 2001;32:114s)
    • Prevalence and incidence vary by climate, geographical condition, living environment and agricultural practice
    • UK incidence is 0.9 per 100,000 person per year (QJM 2007;100:233)
    • Higher prevalence and mortality rate in farmers and agricultural industries (Am J Ind Med 2006;49:997)
    • Mean age is 50 - 60 years old
    • No sexual predominance
    • Smoking is related to lower prevalence but worse prognosis (Intern Med 1995;34:966)
    Sites
    • Predominant in middle to upper lobes of the lung; usually bilateral
    Pathophysiology
    • Gene polymorphisms related to the acquired immune response may predispose to HP (Am J Respir Crit Care Med 2012;186:314):
      • Major histocompatibility complex class 2 (MHC class II)
      • Proteasome subunit beta type 8 (PSMB8)
      • Transporter associated with Antigen Processing 1 (TAP1)
    • Malfunction of regulatory T cells (Eur Respir J 2011;37:632)
    • Antigen exposure results in early formation of type III immune complexes, followed by type IV delayed hypersensitivity
    Etiology
    • HP is developed through inhalation and exposure to a causative antigen
    • Causative agents: spores of bacteria, fungi, mycobacteria, animal proteins and chemicals from hay, grain, sugar cane, bark, cheese, cork and animal feces (J Investig Allergol Clin Immunol 2015;25:237)
    • According to the causative antigen, HP may have different names:
      • Air conditioner lung: due to thermophilic bacteria
      • Byssinosis: in textile workers due to fibers from cotton, linen and hemp
        • Resembles asthma clinically
        • Disease mechanism may not be immune mediated, endotoxin from bacterial contamination of cotton may play a role
      • Farmer's lung: from moldy hay containing spores of thermophilic actinomycetes
      • Maple bark stripper’s lung: fungal spores
      • Pigeon breeder's lung: also called bird fancier's disease; proteins from serum, feathers excreta
      • Hot tub lung: nontuberculous Mycobacterium
    Clinical features
    • HP is classified into acute, subacute and chronic, however there is a lack of consensus for the criteria
    • Acute hypersensitivity pneumonitis
      • Influenza-like syndrome a few hours after exposure to an antigen: fever, dyspnea, cough, crackles may be detected on chest auscultation
      • Symptoms resolve several hours after antigen removal
      • Repeated acute episodes of farmer’s lung leads to centriacinar emphysema (Eur Radiol 2003;13:2212)
    • Subacute hypersensitivity pneumonitis
      • Slowly progressive respiratory failure over weeks to months
      • Fever, dyspnea, cough, fatigue, crackles may be detected on chest auscultation
      • Pulmonary function may be normal
      • Probably results from continuous low level exposure to the antigen
    • Chronic hypersensitivity pneumonitis
      • Slowly progressive and insidious respiratory failure; often without acute episodes
      • Dyspnea, cough, fatigue, weight loss, fine crackles on chest auscultation
      • Restrictive pattern on pulmonary function tests
        • Decreased total lung capacity (TLC)
        • Decreased forced vital capacity (FVC)
        • Decreased diffusing capacity of the lung for carbon monoxide (DLCO)
      • Often associated with bird antigen exposure
      • Acute exacerbation followed by respiratory deterioration within 1 - 2 months can occur; usually without further antigen exposure (Chest 2008;134:844, Chest 2008;134:1265)
    Diagnosis
    • Diagnosis is based on clinical, radiological (high resolution computed tomography, HRCT) and pathological examination
    • Surgical lung biopsy is often necessary to differentiate subacute and chronic hypersensitivity pneumonitis from other interstitial lung disease; however, it is rare for acute hypersensitivity pneumonitis to be biopsied
    • Although several diagnostic criteria have been proposed, none are widely accepted
    • A large cohort study by HP Study Group suggested clinical predictors for the diagnosis of HP (Am J Respir Crit Care Med 2003;168:952)
      • Exposure to a known offending antigen
      • Positive precipitating antibodies
      • Recurrent episodes of symptoms
      • Inspiratory crackles
      • Symptoms 4 - 8 hours after exposure
      • Weight loss
    • Bronchoalveolar lavage is supportive in the diagnosis but lacks standardization (Chest 2012;142:208, Am J Respir Crit Care Med 2012;186:314)
      • Increased total cell count
      • Increased lymphocyte percentage ≥ 30% for nonsmokers or exsmokers or ≥ 20% for current smokers; a normal lavage rules out the presence of active HP (Am Rev Respir Dis 1990;141:S169, Br J Ind Med 1986;43:401)
      • CD4 / CD8 ratiois usually decreased in HP but can be increased as high as in sarcoidosis; CD4 / CD8 ratio is now considered to vary by clinical conditions such as causative antigen and smoking status
    • Inhalation challenge is supportive but lacks standardization (Chest 2012;142:208, Am J Respir Crit Care Med 2012;186:314, Eur Respir J 2014;44:1658)
      • Antigen exposure at the workplace or home or direct inhalation of the specific antigen after a period of avoidance provokes symptoms of HP and decreases FVC in 8 - 12 hours
      • The patient should be monitored at least for 24 hours after the inhalation in case of severe attack of HP
    Laboratory
    • Serum IgG antibody to causative antigens may be increased; however, serum antibody could be positive in 31% of non HP subjects (Am J Respir Crit Care Med 2003;168:952)
      • Avian antigens: pigeon, parakeet, budgerigar, chicken
      • Fungus: trichosporon, aspergillus
      • Bacteria: actinomycete
      • Mycobacteria: Mycobacterium avium-intracellulare
      • Chemicals
    • Increased serum KL-6, often over 1000 IU (normal limit is < 500 IU)
    Radiology description
    Radiology images

    Images hosted on other servers:
    Chest X-ray and CT image of HP

    Chest X-ray and CT image of HP

    CT of HP

    CT of HP

    CT of acute HP due to isocyanate

    CT of acute HP due to isocyanate

    CT of subacute HP of a farmer

    CT of subacute HP of a farmer

    CT of chronic HP of a plastic industrial worker

    CT of chronic HP of a plastic industrial worker

    CT of HP due to nontuberculous mycobacterium

    CT of HP due to nontuberculous mycobacterium

    Prognostic factors
    Case reports
    Treatment
    • Avoidance of antigen is the key of HP management
    • Oral or systemic corticosteroids are considered for severe case or when the antigen is not removable; however, steroids do not change long term outcome and are not standardized (Chest 2013;144:1644)
      Gross description
      • Diffuse involvement with mild to moderate increase in lung weight
      • Bronchocentric fibrotic changes may be seen
        Microscopic (histologic) description
        • Common findings(J Clin Pathol 2013;66:888, Am J Respir Crit Care Med 2012;186:314, J Investig Allergol Clin Immunol 2015;25:237, Arch Pathol Lab Med 2008;132:199):
          • Airway centered (peribronchiolar) change
          • Interstitial cellular infiltration
          • Poorly formed nonnecrotizing granulomas or interstitial giant cells with cholesterol clefts
            • Well formed granulomas can be found but may raise differential diagnosis with sarcoidosis if the granulomas are numerous and predominant
        • Acute HP
          • Airway centered inflammation with little fibrosis
          • Neutrophilic infiltration with / without capillaritis
          • Intra-alveolar fibrin deposition
        • Subacute HP
          • Airway centered infiltration with fibrosis
          • Lymphocytic infiltration with granulomas or giant cells
        • Chronic HP
          • Predominantly airway centered inflammation with diffuse fibrotic change
          • Lymphocytic infiltration with granulomas or giant cells
          • Often overlaps with NSIP, UIP, organizing pneumonia and airway centered interstitial fibrosis
          • Bridging fibrosis (fibrotic band connecting bronchioles with each other and with lobular septa) and peribronchiolar metaplasia can be a diagnostic clue to differentiate HP from IPF (J Clin Pathol 2013;66:888, Histopathology 2012;61:1026)
        • Additional findings
          • Byssinosis bodies (hemosiderin coated strands of fiber within fibrous tissue) can be found in byssinosis
          • Foamy macrophages in alveolar spaces
          • Organizing pneumonia
        Microscopic (histologic) images

        Scroll to see all images:

        Contributed by Akira Yoshikawa, M.D.
        Case 1: 69 year old woman with history of exposure to bird antigen Case 1: 69 year old woman with history of exposure to bird antigen Case 1: 69 year old woman with history of exposure to bird antigen Case 1: 69 year old woman with history of exposure to bird antigen Case 1: 69 year old woman with history of exposure to bird antigen Case 1: 69 year old woman with history of exposure to bird antigen

        Case 1: 69 year old woman with history of exposure to bird antigen


        Case 2: 67 year old man with history of exposure to garden chemicals Case 2: 67 year old man with history of exposure to garden chemicals Case 2: 67 year old man with history of exposure to garden chemicals Case 2: 67 year old man with history of exposure to garden chemicals Case 2: 67 year old man with history of exposure to garden chemicals Case 2: 67 year old man with history of exposure to garden chemicals

        Case 2: 67 year old man with history of exposure to garden chemicals


        Case 2: 67 year old man with history of exposure to garden chemicals Case 2: 67 year old man with history of exposure to garden chemicals

        Case 2: 67 year old man with history of exposure to garden chemicals

        Case 3: 72 year old male woodcutter Case 3: 72 year old male woodcutter Case 3: 72 year old male woodcutter Case 3: 72 year old male woodcutter

        Case 3: 72 year old male woodcutter


        Case 3: 72 year old male woodcutter Case 3: 72 year old male woodcutter Case 3: 72 year old male woodcutter Case 3: 72 year old male woodcutter Case 3: 72 year old male woodcutter

        Case 3: 72 year old male woodcutter



        Images hosted on other servers:
        Airway centered lymphocytic infiltration with granuloma Airway centered lymphocytic infiltration with granuloma Airway centered lymphocytic infiltration with granuloma

        Airway centered lymphocytic infiltration with granuloma

        Airway centered lymphocytic infiltration with fibrosis

        Airway centered lymphocytic infiltration with fibrosis

        Poorly formed granulomas Poorly formed granulomas

        Poorly formed granulomas


        Poorly formed granulomas Poorly formed granulomas

        Poorly formed granulomas

        Peribronchiolar metaplasia

        Peribronchiolar metaplasia

        Intra-alveolar fibrindeposition

        Intra-alveolar fibrindeposition

        Giant cells

        Giant cells


        Comparison of<br>granulomatous lesions<br>of different diseases

        Comparison of
        granulomatous lesions
        of different diseases

        Cytology description
        • Bronchoalveolar lavage fluid shows lymphocytosis (see Diagnosis)
        Cytology images

        Images hosted on other servers:
        Bronchoalveolar lavage fluid

        Bronchoalveolar lavage fluid

        Positive stains
        Differential diagnosis
        Board review style question #1
        Which two clinical and morphological findings are suggestive for HP?

        1. Intra-alveolar fibrin deposition
        2. Monocytosis in bronchoalveolar lavage
        3. Necrotizing granuloma
        4. Organizing pneumonia
        5. Weight loss
        Board review style answer #1
        A. and E.

        Comments:
        1. Intra-alveolar fibrin deposition is suggestive for acute HP, also for acute fibrinous and organizing pneumonia
        2. Typical bronchoalveolar lavage of HP shows lymphocytosis
        3. Necrotizing granuloma is more suggestive for tuberculosis
        4. Organizing pneumonia can be seen in HP but it is not specific
        5. Weight loss is suggestive for HP according to the large cohort study (see Diagnosis)

        Comment Here

        Reference: Hypersensitivity pneumonitis

        Hypoplasia
        Definition / general
        • Incomplete development of lung; lung usually weighs less than normal with fewer alveoli than expected for gestational age
        • Bilateral hypoplastic disease is fatal
        • Causes: oligohydramnios (renal agenesis, fetal membrane rupture), decreased intrathoracic space (renal cystic disease, diaphragmatic hernia), reduced breathing (anencephaly, musculoskeletal disorders, brainstem hypodevelopment, Virchows Arch 2009;454:451)
        • References: eMedicine
        Gross images

        Images hosted on other servers:
        Severe pulmonary hypoplasia Severe pulmonary hypoplasia

        Severe pulmonary hypoplasia


        Idiopathic pulmonary hemosiderosis
        Definition / general
        • Rare, usually children or teenagers with hemoptysis, hypoxemia, dypsnea and iron deficiency anemia similar to Goodpasture’s disease, but no antibodies in serum or tissue, no renal disease
        • Clinical triad of hemoptysis, iron deficiency and diffuse pulmonary infiltrates
        • May be related to consanguinity or environmental factors
        • Fever, lymph node, liver and spleen enlargement may occur
        • Usually fatal after several years
        • Diagnosis of exclusion
        Microscopic (histologic) description
        • Hemosiderin laden macrophages in alveolar lumina, shedding and hyperplasia of alveolar epithelial cells and marked alveolar capillary congestion
        • Varying interstitial fibrosis, intra-alveolar hemorrhage
        • No necrosis, no vasculitis, no granulomas and no lymphoid follicles
        Differential diagnosis
        • Note: these disorders all cause secondary pulmonary alveolar bleeding and hemosiderosis
          • Mitral stenosis
          • Periarteritis nodosa
          • SLE (neutrophils in alveolar septal capillaries)
          • Systemic vasculitis

        Infantile lobar emphysema
        Definition / general
        • Also called congenital lobar emphysema
        • Infants or young children
        • Abnormal bronchial development resulting in air trapping, perhaps due to hypoplasia of bronchial cartilage; associated with other cardiopulmonary anomalies
        • Affects left or right upper lobe or right middle lobe
        • May cause severe compression of other pulmonary lobes
        • Not emphysema since no tissue destruction
        Case reports
        Microscopic (histologic) description
        • Massive distention of alveolar spaces, but no tissue destruction
        Microscopic (histologic) images

        Images hosted on other servers:
        Various images Various images

        Various images


        Infarct / pulmonary emboli
        Definition / general
        Essential features
        • Pulmonary embolism is the third most common acute cardiovascular condition worldwide, after myocardial infarction and stroke
        • True antemortem clots and emboli display alternating darker and lighter striations due to erythrocyte and fibrin / platelet rich layers (lines of Zahn)
        • Thromboembolism is then organized by migrating fibroblasts and myofibroblasts if thrombolysis does not occur within 1 to 2 days
        Terminology
        • Pulmonary infarct, venous thromboembolism (VTE), pulmonary embolism
        ICD coding
        • ICD-10: I26.99 - other pulmonary embolism without acute cor pulmonale
        Epidemiology
        • Third most common acute cardiovascular condition worldwide, trailing only myocardial infarction and stroke (Arterioscler Thromb Vasc Biol 2014;34:2363)
        • Responsible for ~100,000 fatalities in the United States annually (Arch Intern Med 2003;163:1711)
        • M > F (Arch Intern Med 2003;163:1711)
        • Increased incidence with age, particularly in women, to the point that it has an incidence of > 500 per 100,000 after the age of 75 (Am J Med 2013;126:832.e13)
        • Risk factors can be divided into genetic versus acquired
          • Genetic factors include hypercoagulable conditions, such as factor V Leiden mutation, protein C and S deficiency, prothrombin mutations, antiphospholipid syndrome, etc.
          • Acquired risk factors include but are not limited to pregnancy, hospitalization, obesity, oral contraceptive use, malignancy, long bone fracture, stroke, indwelling central venous lines, acute spinal cord injury and patients who have had joint arthroplasty or repair
        Sites
        • Pulmonary arteries or any of its branches
        Pathophysiology
        • Pathogenesis of pulmonary embolism is similar to that of thrombus formation (i.e., Virchow's triad)
        • Virchow's triad includes venous stasis, endothelial damage and hypercoagulability
        • Range of pathophysiologic reactions can occur when an embolus lodges in the lung:
          • Pulmonary infarction due to tiny thrombi that lodge distally in segmental and subsegmental arteries in ~10% of people (Chest 1991;100:598)
          • Ventilation perfusion mismatch (V/Q mismatch) in the lung is caused by insufficient perfusion, resulting in hypoxemia
          • Hypotension due to increased pulmonary vascular resistance (PVR) (Clin Chest Med 1984;5:403)
            • PVR increases due to the presence of thrombi and hypoxic vasoconstriction of lung capillaries
            • Increased PVR, in turn, obstructs right ventricular outflow, resulting in right ventricular dilatation and flattening or bending of the intraventricular septum
            • Both decreased right ventricular flow and right ventricular dilation decrease left ventricular preload, reducing cardiac output
        Etiology
        • Most emboli originate in the proximal veins of the lower extremities (iliac, femoral and popliteal)
        • At the time of presentation, more than half of patients with proximal deep vein thrombosis have concurrent pulmonary embolism (Am J Respir Crit Care Med 2001;164:1033)
        • Most thrombi form in lower extremity veins at regions of diminished flow, such as valve cusps or bifurcations
        Diagrams / tables

        Images hosted on other servers:

        Pathophysiology of hemodynamic instability

        Clinical features
        • Common symptoms at presentation include (Chest 1991;100:598)
          • Dyspnea at rest or with exertion
          • Pleuritic pain
          • Cough
          • Orthopnea
          • Wheezing
          • Hemoptysis
          • Calf or thigh pain or swelling
        • Common presenting symptoms found on examination include (Am J Med 2007;120:871)
          • Tachypnea
          • Tachycardia
          • Calf or thigh swelling, erythema, edema, tenderness, palpable cords
          • Rales
          • Accentuated pulmonic component of the second heart sound
          • Decreased breath sounds
          • Jugular venous distension
          • Fever, mimicking pneumonia
        Diagnosis
        • Pretest probability for pulmonary embolism should always be calculated using a validated pretest probability score or approximated clinically if pulmonary embolism is suspected (e.g., Wells score, modified Wells score or modified Geneva score)
        • Criteria for Wells and modified Wells score (JAMA 2006;295:172):
          • Clinical signs and symptoms of deep vein thrombosis = 3
          • Alternative diagnosis is less likely than pulmonary embolism = 3
          • Heart rate > 100 = 1.5
          • Immobilization for ≥ consecutive days or surgery in the previous 4 weeks = 1.5
          • Previously diagnosed with pulmonary embolism or deep vein thrombosis = 1.5
          • Hemoptysis = 1
          • Malignancy (on treatment, treatment in the last 6 months or palliative) = 1

        • Interpretation of Wells and modified Wells score:
          3 tier 2 tier
          • 0 - 1: low risk
          • 2 - 6: moderate risk
          • > 6: high risk
          • ≤ 4: unlikely
          • ≥ 4.5: likely

        • Computed tomographic pulmonary angiography (CTPA) or magnetic resonance pulmonary angiography (MRPA): a filling defect in any branch of the pulmonary artery (main, lobar, segmental, subsegmental) that becomes evident after contrast enhancement is diagnostic of pulmonary embolism
        • CTPA, MRPA: very sensitive and specific, fast and readily available imaging study to rule pulmonary embolism in or out; has essentially replaced conventional angiography
        • Chest Xray: wedge shaped infarct after 12 - 36 hours
        • Nuclear scan: macroaggregates of labeled albumin with perfusion lung scanning; angiography is the most definitive diagnostic test
        Laboratory
        • D dimer testing is an excellent validated screening test for thromboembolic disease; a negative result essentially rules out pulmonary embolism
        • Laboratory tests are not diagnostic but they can raise clinical suspicion for pulmonary embolism, confirm the presence of other illnesses and offer prognostic information if pulmonary embolism is diagnosed
        • These include a complete blood count and serum chemistries, arterial blood gases and D dimer
        • Electrocardiogram (ECG) abnormalities are common but nonspecific in those with suspected pulmonary embolism
          • The most prevalent findings are tachycardia and nonspecific ST segment and T wave changes (70%) (Eur Respir J 2005;25:843)
        Radiology description
        • CT scan findings (Insights Imaging 2019;10:114):
          • Polo mint sign: the appearance of the polo mint sign inside the pulmonary artery refers to a blood clot surrounded by contrast material
          • Railway sign: the railway track sign is used in chest radiography or CT to refer to the presence of contrast in the pulmonary artery surrounding a partial filling defect in the setting of acute pulmonary embolism
        • Radiograph findings (Radiopaedia: Pulmonary Embolism [Accessed 5 October 2022]):
          • Fleischner sign: enlarged pulmonary artery
          • Hampton hump: peripheral wedge of airspace opacity and implies lung infarction
          • Westermark sign: regional oligemia and the highest positive predictive value
          • Pleural effusion
          • Palla sign: enlarged right descending pulmonary artery
          • Chang sign: dilated right descending pulmonary artery with sudden cutoff
          • Knuckle sign: abrupt tapering or cutoff of a pulmonary artery secondary to a pulmonary embolus
        Radiology images

        Images hosted on other servers:

        Saddle PE on CTA

        Prognostic factors
        Case reports
        • 37 year old woman with no previous medical history presented with chest discomfort, acute coronary syndrome was suspected and was later determined to be a case of pulmonary embolism (BMC Pulm Med 2019;19:44)
        • 40 year old woman developed pulmonary embolism following a 1.5 meter stair fall with a craniothoracic impact point (Ann Med Surg (Lond) 2022;78:103868)
        • 51 year old woman with progressive pain in her left shoulder for 6 months developed an asymptomatic pulmonary embolism after shoulder arthroscopy (Orthop Surg 2021;13:1119)
        • 55 year old woman was admitted to the acute medical unit with fluctuations in consciousness and in a septic state (BMJ Case Rep 2014;2014:bcr2013201320)
        • 61 year old man presented with chest pain, difficulty breathing, hemoptysis and general discomfort and was found to have a massive pulmonary embolism along with S1Q3T3 ECG findings (BMC Cardiovasc Disord 2021;21:224)
        Treatment
        • Stabilize the patient until a clinical examination and conclusive diagnostic tests are performed
        • Start anticoagulation even before establishing the diagnosis of pulmonary embolism if the risk/benefit ratio between the suspicion of pulmonary embolism and the risk of bleeding appears favorable
        • Inferior vena cava (IVC) filters should be placed in most pulmonary embolism patients for whom anticoagulation is contraindicated or when the risk of bleeding is high
        • For patients who present with hemodynamic instability, thrombolytic therapy, embolectomy and catheter directed modalities like ultrasound assisted thrombolysis are potentially acceptable treatment options
        • Elastic graded compression stockings, frequent ambulation and IVC filters are adjunct therapies that can be used in addition to anticoagulation in patients with pulmonary embolism
        • References: Am J Cardiol 1959;4:611, Circulation 2005;112:e28
        Gross description
        • Early stages of pulmonary infarct are often red-blue and have a raised appearance
        • Fibrinous exudate covers the apposed pleural surface
        • Within 48 hours, the infarct grows paler and finally red-brown as hemosiderin is produced
        • Fibrous replacement begins at the margins as a gray-white peripheral zone and gradually transforms the infarct into a contracted scar
        • Saddle embolus arises when a large emboli lodges in the main pulmonary artery or one of its major branches or at the bifurcation
        • Smaller emboli can induce bleeding or infarction in more peripheral arteries
        • References: Kumar: Robbins & Cotran Pathologic Basis of Disease, 10th Edition, 2020, Husain: Thoracic Pathology - High Yield Pathology, 1st Edition, 2012
        Gross images

        Contributed by Aliya N. Husain, M.D. and @yro854 on Twitter
        Pulmonary thromboembolism Pulmonary thromboembolism

        Pulmonary thromboembolism

        Infarct / pulmonary emboli

        Infarct / pulmonary emboli



        Images hosted on other servers:
        Thromboemboli in pulmonary artery Thromboemboli in pulmonary artery Thromboemboli in pulmonary artery

        Thromboemboli in pulmonary artery

        Saddle embolus

        Saddle embolus


        Hemorrhagic infarct

        Hemorrhagic infarct

        Organizing infarct

        Organizing infarct

        Fibrous band

        Fibrous band

        Microscopic (histologic) description
        • Antemortem clots display erythrocyte and fibrin layering
        • Because of erythrocyte and fibrin / platelet rich layers, true ante mortem clots and emboli exhibit alternating darker and lighter striations (lines of Zahn)
        • Thromboembolus is then organized by migrating fibroblasts and myofibroblasts if thrombolysis does not take place within 1 to 2 days
        • Clot organization develops from the periphery to the center
        • These migratory cells combine with granulation tissue in large arteries and cause clot contraction to one side, resulting in an eccentric thickening of the affected vessel
        • Hemorrhagic area exhibits ischemic necrosis of the alveolar walls, bronchioles and arteries
        • Infarction results in the loss of alveolar nuclei and eventual fibrosis
        • Infected embolus causes a septic infarct with a significant neutrophilic response
        • Fat embolism / bone marrow embolus: the presence of fat cells with or without marrow elements in the pulmonary circulation
          • Most cases are due to long bone and pelvic fractures
          • Rounded clear cells can be observed in the pulmonary vessel
        • Pulmonary infarcts may present in several ways:
          • Typical textbook description is a wedge shaped subpleural zone of hemorrhage with coagulative necrosis
          • However, it is essential to remember that various configurations, such as geographical, spherical and mixed forms, can be seen at low magnification
        • All of them have coagulative necrosis and significant hemorrhage in the lung parenchyma
        • Geographic form is marked by coagulative necrosis in the subpleural region with a serpentine distribution and patches of healthy lung parenchyma frequently encircled by large areas of eosinophilic necrosis
        • Round configuration has a thick fibrous capsule that encases a nearly spherical zone of necrosis, while the mixed pattern has segments with both rounded and geographical arrangements
        • Granulation tissue covers the airspaces around the necrotic zone and is bordered by thicker interstitium
          • Foamy macrophages and histiocytes may also surround the coagulative necrosis
        • Squamous metaplasia is also commonly found in pulmonary infarcts and frequently demonstrates substantial reactive changes that lead to a differential diagnosis of cancer
        • It may also manifest with vascular inflammation, frequently leading to a differential diagnosis of vasculitis
        • References: Kumar: Robbins & Cotran Pathologic Basis of Disease, 10th Edition, 2020, Husain: Thoracic Pathology - High Yield Pathology, 1st Edition, 2012, Mod Pathol 2009;22:679
        Microscopic (histologic) images

        Contributed by Aliya N. Husain, M.D.
        Lines of Zahn

        Lines of Zahn

        Postmortem clot

        Postmortem clot

        Organizing thrombus

        Organizing thrombus

        Organized thromboembolism

        Organized
        thromboembolism


        Organized thromboembolism

        Organized
        thromboembolism

        Recanalized thromboembolism

        Recanalized
        thromboembolism

        Organized thrombus

        Bone marrow embolism

        Bone marrow embolism

        Positive stains
        Sample pathology report
        • Lung, left lower lobe, wedge biopsy:
          • Organized pulmonary artery thromboembolism with associated infarct (see comment)
          • Comment: An irregular intimal plaque with fibrosis is evident within the artery, along with recanalization of the vessel and the presence of small lumina, suggesting a previous clot. Areas of calcification are present within the fibrosis in the pulmonary artery. In addition, necrosis of alveolar walls with loss of nuclei in the neighboring alveoli represents an area of infarction.
        Differential diagnosis
        Board review style question #1

        A 78 year old man was brought to the emergency room with complaints of shortness of breath, chest discomfort and cough. He died as the medical team was attempting to revive him. The family requested a postmortem examination to determine the cause of death. On investigation, a clot was found lodged in the right main pulmonary artery. Which one of the following characteristics of the clot will help identify if it formed antemortem and contributed to his death?

        1. Appearance of currant jelly on chicken fat
        2. Gelatinous, soft and rubbery appearance on gross examination
        3. Presence of lines of Zahn
        4. Red blood cells and coagulated serum mixed in 2 separate layers
        Board review style answer #1
        C. The presence of lines of Zahn or lighter layers of platelets and fibrin alternating with darker layers of red blood cells in a clot indicates that it was formed before the patient died.

        Comment Here

        Reference: Infarct / pulmonary emboli
        Board review style question #2

        A 62 year old woman presents to the emergency room with a right femoral fracture. 3 hours later, she develops sudden shortness of breath and altered mental status. On physical examination, she is tachycardic and a red petechial rash can be observed on her chest. A Doppler ultrasound is performed and there are no signs of a deep venous thrombosis. Which of the following best describes what the pulmonary artery histology would demonstrate?

        1. Atypical cells with a high N:C ratio
        2. Foamy macrophages and fibrin deposits
        3. Presence of neutrophils in a clot
        4. Rounded clear cells with hematopoietic elements are observed in the pulmonary vessel
        Board review style answer #2
        D. Rounded clear cells with hematopoietic elements are observed in the pulmonary vessel. Bone marrow embolism is defined by the presence of fat cells and hematopoietic elements in the pulmonary circulation, as seen in the image. Hypoxemia, neurologic problems and petechial rash are the characteristic triad for patients experiencing fat embolism syndrome (FES); however, none of these characteristics are unique to FES.

        Comment Here

        Reference: Infarct / pulmonary emboli

        Inflammatory bowel disease
        Definition / general
        • Vast majority of patients with lung disease have history of IBD in GI tract, but lung disease may occur without GI disease
        • Lung involvement more common with ulcerative colitis than Crohn's disease
        • Acute or chronic bronchiolitis with nonnecrotizing granulomas, bronchopneumonia, interstitial pneumonia and organizing pneumonia
        • May be due to mesalamine (anti-inflammatory component of sulfasalazine, used to treat IBD)
        • May have CMV or atypical mycobacteria infections
        Differential diagnosis
        Additional references

        Inflammatory myofibroblastic tumor
        Definition / general
        • Category used to designate a variety of benign proliferative lesions forming a lung mass, including neoplastic and nonneoplastic entities
        • Neoplastic: renamed inflammatory myofibroblastic tumor (IMT), may show clonal cytogenetic abnormalities involving 2p23 that encodes ALK gene, typically occurs in children and young adults, well demarcated but non encapsulated, usually solitary mass that replaces underlying lung tissue
        • Nonneoplastic: older age, ill defined or irregular contour due to prominent organizing pneumonia component and fibrosis at edge (Arch Pathol Lab Med 2010;134:417)
        Essential features
        • Inflammatory pseudotumor is a heterogeneous group of benign lesions that may occur in the lung, with a variable mix of inflammatory cells and fibroblasts / myofibroblasts
        • Inflammatory myofibroblastic tumor is recognized as a neoplastic subset that can be associated with ALK gene rearrangement and is typically seen in the younger age group
        • The differential diagnosis is broad, and diagnosis typically requires surgical excision
        Terminology
        • Variable definitions in use of "inflammatory pseudotumor"; some use term to describe any circumscribed or irregular inflammatory nodule, mass or consolidation; others favor a more descriptive diagnosis
        • Inflammatory myofibroblastic tumor means neoplastic, fibrohistiocytic, associated with ALK rearrangement
        • Other names used in nonneoplastic lesions: plasma cell granuloma, hyalinizing granuloma, plasma cell pseudotumor, inflammatory myofibrohistiocytic proliferation
        Epidemiology
        • Incidence unclear since terminology / definition varies
        • M = F, broad age range but usually < 30 if IMT
        • 3% bilateral
        Sites
        • Typically parenchymal, but may involve pleura
        • Rarely intrabronchial
        Etiology
        • Unknown, but may be autoimmune (IgG4) related or due to an infectious process
        Clinical features
        • 50% with cough, shortness of breath, chest pain, hemoptysis
        Diagnosis
        • Surgical excision with histologic examination
        Radiology description
        • Xray: single, well defined, round or oval mass
        • CT: may show pleural retraction if lesion involves pleura, may show cavitation or calcification
        Prognostic factors
        • Usually remains stable or grows very slowly, may spontaneously resolve
        • Poor prognostic factors for IMT: metastases, necrosis > 15% of surface area examined, local recurrence, bizarre giant cells, > 3 mitotic figures / 50 HPF, advanced stage, high cellularity, poor circumscription
        Case reports
        Treatment
        • Complete excision
        • Rarely causes death due to local extension
        Clinical images

        Images hosted on other servers:
        Bronchoscopy with biopsy

        Bronchoscopy with biopsy

        Gross description
        • Variable size from < 1 cm to very large (> 30 cm)
        • Well circumscribed, non encapsulated, usually solitary, white to gray, firm, fleshy parenchymal nodule
        • May be yellow and friable or show hemorrhage, necrosis, calcification
        Microscopic (histologic) description
        • Variable patterns, e.g. fibrohistiocytic, plasma cell granuloma, largely sclerosed, compact spindle cells, hypocellular fibrous, myxoid / vascular, fibroxanthomatous
        • Generally fibroinflammatory with variable numbers of plasma cells, lymphocytes, histiocytes and myofibroblasts
        • May show mast cells, eosinophils, neutrophils, multinucleated cells, hemosiderin, calcification
        • May resemble nodular fasciitis, fibrous histiocytoma or fibromatosis
        • May show features of organizing pneumonia including lymphohistiocytic inflammation and fibrosis with preservation of alveolar architecture in early lesions; also bordering alveoli with foamy macrophages and hyperplastic pneumocytes
        • Lymphoplasmacytic variant with mostly plasma cells and lymphocytes, germinal centers, paucicellular collagen, endothelialitis, fibrinous pleuritis
        Microscopic (histologic) images

        Contributed by Roseann Wu, M.D., M.P.H.



        Images hosted on other servers:
        Various images

        Myofibroblasts with plump oval nuclei

        Various images

        Lymphoid hyperplasia and lymphocytic infiltration

        Various images

        Nodular lesions and infiltrating lymphocytes

        Variable appearance of ALK<br>rearranged IMTs

        Variable appearance of ALK rearranged IMTs

        Positive stains
        Negative stains
        Electron microscopy description
        • Elongated cytoplasmic processes with pinocytotic vesicles, subplasmalemmal plaques, thin filaments, abundant endoplasmic reticulum
        Molecular / cytogenetics description
        Childhood inflammatory myofibroblastic tumor
        Definition / general
        • Most common isolated lung lesion in children, usually asymptomatic
        • Mostly neoplastic with frequent ALK gene rearrangements (in inflammatory myofibroblastic tumor)
        • Benign, although rare cases of malignant behavior have been reported

        Essential features
        • Inflammatory myofibroblastic tumor (IMT) occurs more often in childhood and in young adults and shows morphology similar to the adult form
        • IMT in children is more often associated with ALK gene rearrangement than in adults

        Terminology
        • Renamed inflammatory myofibroblastic tumor (formerly inflammatory pseudotumor)

        Epidemiology
        • Most common lung tumor in children age 16 and younger

        Sites
        • Lung, mediastinum, trachea, bronchus, abdomen, orbit

        Radiology description
        Case reports
        Treatment
        • Excision or radiation therapy

        Gross description
        • Solitary, small peripheral nodules, yellow, firm, covered by intact pleura or polypoid bronchial mass

        Gross images

        Images hosted on other servers:
        Right lower lobe

        Right lower lobe



        Microscopic (histologic) description
        • Plasma cells (often abundant), lymphocytes, histiocytes and myofibroblasts
        • May have vascular proliferation, collagenous or hyalinized stroma, myxoid change, xanthoma cells, hemosiderin
        • May resemble nodular fasciitis, fibrous histiocytoma or fibromatosis

        Microscopic (histologic) images

        Images hosted on other servers:
        3 year old child

        Spindle cells arrayed in fascicles

        3 year old child

        Proliferation of regular myofibroblasts

        3 year old child

        ALK1 reactivity

        3 year old child

        AML reactivity



        Molecular / cytogenetics description
        • More often associated with ALK rearrangement than adult cases

        Influenza
        Epidemiology
        • Most common cause of viral pneumonia
        • RNA virus, nucleoprotein determines type of virus: A, B or C (Virol J 2009 Mar 13;6:30)
        • Envelope contains the viral hemagglutinin and neuraminidase that establish the viral subtype, e.g., H1N1 (Wikipedia)
        • Preexisting antibodies to specific hemagglutinins and neuraminidases prevent or amerliorate future infections
        • Type A: causes most severe disease; major cause of epidemics and pandemics; also infects pigs, horses, birds; one subtype predominates at any given time
        • Antigenic drift leading to epidemics is caused by small mutations in hemagglutinins and neuraminidases, which allow the virus to evade most host antibodies
        • Antigenic shift leads to pandemics, and is caused by recombination of hemagglutinin and neuraminidase RNA with animal influenza RNA, leading to absence of human immunity
        • Antigenic drift and shift do NOT occur with influenza types B and C, which usually cause mild, self limited illness in children
        • References: World Health Organization Influenza update
        Case reports
        • 43 year old man in India with with fever for 3 days and sudden onset of shortness of breath (Case of the Week #459)
        Clinical features
        • Cause mild acute lung injury to necrotizing pneumonia to BOOP-like changes
        • May cause bronchiolitis or diffuse alveolar damage
        • Viral cytopathic changes do NOT occur
        • Airways show mucosal hyperemia with infiltrate of lymphocytes, histiocytes, plasma cells
        • Swelling and increased mucus production impair bronchociliary elevator, and may lead to secondary bacterial infection; small airways may become occluded causing focal atelectasis; in severe disease, this is more widespread with distal airways plugged by fibrinopurulent debris
        • If process is prolonged, obliterative bronchiolitis with irreversible lung damage may occur
        • Histologic findings are nonspecific; the diagnosis can be confirmed with molecular testing, viral culture, DFA, IHC, serology, antigen detection assays
        Microscopic (histologic) images

        Contributed by Dr. K.V. Vinu Balraam
        Lung Lung Lung Lung

        Lung

        Pulmonary lymph node

        Pulmonary lymph node

        Kidney

        Kidney

        Board review style question #1
        Which histological feature is highly specific to influenza A/H1N1 infection in diffuse alveolar damage?

        1. Neutrophilic aggregation
        2. Organizing pneumonia
        3. Alveolar hemorrhage
        4. All of the above
        5. None of the above
        Board review style answer #1
        E. No histological finding specific to influenza infection has been identified. The findings in A, B and C are quite common in diffuse alveolar damage of any etiology.

        Comment Here

        Reference: Influenza
        Board review style question #2

        This image is from an autopsy of a 43 year old man with influenza A/H1N1 infection. Which histological feature is highly specific to this disease?

        1. Neutrophilic aggregation
        2. Organizing pneumonia
        3. Alveolar hemorrhage
        4. None of the above
        Board review style answer #2
        D. None of the above

        Comment Here

        Reference: Influenza

        Interstitial pneumonia with autoimmune features
        Definition / general
        • Interstitial pneumonia is one of the common clinical manifestation of connective tissue disease (CTD); this entity is called CTD associated interstitial lung disease (CTD-ILD)
        • Some patients initially diagnosed with idiopathic interstitial pneumonias (IIPs) eventually present with systemic autoimmune symptoms and develop a complete form of CTD; thus, it is often difficult to distinguish CTD-ILD from IIPs before the symptoms appear (Respiration 1995;62:248)
        • IIP cases with certain features suggesting an underlying autoimmune process but insufficient to fulfill the diagnostic requirements of any specific CTD were previously categorized as an idiopathic or unclassifiable interstitial pneumonia (Am J Respir Crit Care Med 2013;188:733)
        • In 2015, ATS / ERS working group proposed the term "interstitial pneumonia with autoimmune features (IPAF)" as a possible spectrum of CTD-ILD along with its provisional criteria (Eur Respir J 2015;46:976)
        Essential features
        • An interstitial lung disease with clinical, serologic or histologic features suggestive of CTD which does not conclusively meet current rheumatologic criteria
        • Histologically, nonspecific interstitial pneumonia (NSIP) and organizing pneumonia (OP) patterns are prevalent; lymphocytic inflammation is suggestive of IPAF
        Terminology
        • Before the proposal from ATS / ERS, studies used nomenclature of:
          • Lung dominant connective tissue disease (lung dominant CTD)
          • Undifferentiated connective tissue disease associated interstitial lung disease (UCTD-ILD)
          • Autoimmune featured interstitial lung disease (autoimmune featured ILD)
        Epidemiology
        • Mean age 50 - 60 years old
        • Female predominant
        • No correlation with smoking status
        Sites
        • Unilateral or bilateral lower lobes of the lungs
        Etiology
        Clinical features
        • Mild to moderate chronic respiratory failure
          • Shortness of breath
          • Dyspnea on exertion
          • Cough
          • Fatigue
          • Weight loss
        • Abnormal chest auscultation
          • End inspiratory fine crackles in lower lobes of the lungs
        • Restrictive pattern in pulmonary function tests:
          • Decreased forced vital capacity (FVC)
          • Decreased diffusing capacity of the lung for carbon monoxide (DLCO)
        • Extrathoracic autoimmune manifestation or serologic abnormality accompanythe respiratory symptoms (See Diagnosis)
        Diagnosis
        • The proposed criteria consists of a priori requirements and three domains: clinical, serologic and morphologic, as below (Eur Respir J 2015;46:976)
        • Findings of "multi compartment involvement" are detected by various modalities such as simple chest X ray, high resolution computed tomography (HRCT), surgical lung biopsy, pulmonary function tests and right heart catheterization
        1. Presence of an interstitial pneumonia (by HRCT or surgical lung biopsy)
        2. Exclusion of alternative etiologies and
        3. Does not meet criteria of a defined CTD and,
        4. At least one feature from at least two of these domains:
          1. Clinical domain
          2. Serologic domain
          3. Morphologic domain
        1. Clinical domain criteria
          1. Distal digital fissuring ("mechanic hands")
          2. Distal digital tip ulceration
          3. Inflammatory arthritis or polyarticular morning joint stiffness ≥ 60 min
          4. Palmar telangiectasia
          5. Raynaud's phenomenon
          6. Unexplained digital edema
          7. Unexplained fixed rash on the digital extensor surfaces ("Gottron’s sign")
        1. Serologic domain
          1. Antinuclear antibody (ANA) ≥ 1:320 titer; diffuse, speckled, homogeneous patterns or
            1. ANA nucleolar pattern (any titer) or
            2. ANA centromere pattern (any titer)
          2. Rheumatoid factor (RF) ≥ 2x upper limit of normal
          3. Anti-CCP
          4. Anti-dsDNA
          5. Anti-Ro (SS-A)
          6. Anti-La (SS-B)
          7. Anti-ribonucleoprotein
          8. Anti-Smith
          9. Anti-topoisomerase (Scl-70)
          10. Anti-tRNA synthetase (e.g. Jo-1, PL-7, PL-12; others are EJ, OJ, KS, Zo, tRS)
          11. Anti-PM-Scl
          12. Anti-MDA-5
        1. Morphologic domain
          1. Suggestive radiologic patterns on HRCT
            1. NSIP
            2. OP
            3. NSIP with OP overlap
            4. Lymphoid interstitial pneumonia (LIP)
          2. Histopathology pattern in morphologic domain
            1. NSIP
            2. OP
            3. NSIP with OP overlap
            4. LIP
            5. Diffuse lymphoplasmacytic infiltration with or without lymphoid follicles
          3. Multicompartment involvement
            1. Unexplained pleural effusion or thickening
            2. Unexplained pericardial effusion or thickening
            3. Unexplained intrinsic airways disease
            4. Unexplained pulmonary vasculopathy
        Laboratory
        • Not included in the IPAF criteria because of their low specificity for CTD: low titer ANA, low titer RF, erythrocyte sedimentation rate, C reactive protein, creatine phosphokinase (Eur Respir J 2015;46:976)
        Radiology description
        • NSIP pattern
          • Diffuse ground glass opacity
          • Reticular opacity
          • Traction bronchiectasis
          • Often difficult to make a definite diagnosis without histopathological examination
        • OP pattern
          • Patchy and migratory consolidation in a subpleural and peribronchial area
          • Band-like pattern
        • LIP pattern
          • Reticular opacity
          • Air space opacity
          • Often difficult to make a definite diagnosis without histopathological examination
        • UIP and other radiological patterns can also be seen
        Radiology images

        Images hosted on other servers:
        NSIP pattern on CT NSIP pattern on CT NSIP pattern on CT NSIP pattern on CT

        NSIP pattern on CT


        OP pattern on CT OP pattern on CT

        OP pattern on CT

        LIP pattern on CT

        LIP pattern on CT

        Prognostic factors
        Treatment
        Clinical images

        Images hosted on other servers:

        Clinical features in systemic sclerosis
        Digital ulceration Digital ulceration

        Digital ulceration

        Telangiectasia Telangiectasia

        Telangiectasia


        Raynaud's phenomenon

        Raynaud's phenomenon
        and telangiectasia



        Clinical features in dermatomyositis

        Mechanic hand

        Gottron's sign

        Gross description
        • Diffuse involvement with mild to moderate increase in weight
        • Shrunken lung due to fibrotic changes in lower lobes
        • Pleural thickening
        Microscopic (histologic) description
        • Histological NSIP pattern is the most common (about 50%) in IPAF cases, followed by NSIP with OP overlap pattern (Respir Med 2016;119:150, Eur Respir J 2016;47:1767)
        • NSIP pattern
          • Diffuse and uniform inflammation in alveolar walls
          • Cellular or fibrotic changes
        • OP pattern
          • Polyps of fibroblastic organization obstructing alveolar ducts
        • LIP pattern
          • Marked and extensive infiltration of lymphocytes in alveolar wall
        • Other findings suggestive of IPAF
          • Lymphoid aggregates with germinal centers
          • Diffuse lymphoplasmacytic infiltration
          • Additional supportive findings (Chest 2010;138:251)
            • Extensive pleuritis
            • Dense perivascular collagen
        • Other histological patterns, such as UIP pattern and respiratory bronchiolitis interstitial lung disease (RB-ILD) pattern, can be seen with lesser frequency
        • UIP pattern may be related to poorer prognosis than NSIP pattern (Chest 2015;147:165)
        Microscopic (histologic) images

        Contributed by Akira Yoshikawa, M.D.
        Low power

        Low power

        NSIP pattern

        NSIP pattern

        Lymphoplasmacytic infiltration without lymphoid follicles Lymphoplasmacytic infiltration without lymphoid follicles

        Lymphoplasmacytic infiltration without lymphoid follicles

        Pleural thickening and pleuritis Pleural thickening and pleuritis

        Pleural thickening and pleuritis


        Vascular wall thickening

        Vascular wall thickening

        Emphysematous change

        Emphysematous change

        Low power

        Low power

        UIP pattern

        UIP pattern

        Lymphoid follicles with germinal centers

        Lymphoid follicles with germinal centers

        Honeycomb change

        Honeycomb change


        Honeycomb change

        Honeycomb change

        Lymphoplasmacytic infiltration

        Lymphoplasmacytic infiltration

        Dense fibrosis with lymphocytic infiltration Dense fibrosis with lymphocytic infiltration

        Dense fibrosis with lymphocytic infiltration



        Also see: NSIP


        Images hosted on other servers:
        NSIP pattern NSIP pattern NSIP pattern

        NSIP pattern

        OP pattern OP pattern OP pattern

        OP pattern


        OP pattern

        OP pattern

        LIP pattern LIP pattern LIP pattern LIP pattern

        LIP pattern

        Differential diagnosis
        • Connective tissue disease associated interstitial lung disease (CTD-ILD): clinical and serologic manifestations conclusive to meet rheumatologic criteria
        • Cryptogenic organizing pneumonia (COP): no clinical or serologic autoimmune features
        • Hypersensitivity pneumonitis: history of exposure to causative antigens, airway centered change, granuloma or interstitial giant cells
        • Idiopathic NSIP: no clinical or serologic autoimmune features
        • LIP: no clinical or serologic autoimmune features
        • UIP / IPF: no or only one autoimmune features
        Board review style question #1
        Which 2 cases best meet the criteria for IPAF?

        1. A current smoker with Raynaud’s phenomenon and a histological OP pattern. No serological abnormality is detected.
        2. Patient hospitalized for shortness of breath during the past 3 weeks, now improving over the past week. High titer ANA and radiological NSIP patterns are detected.
        3. Patient with swelling in both wrists for the past year, high titer anti-CCP antibody, high titer CRP, NSIP pattern on CT.
        4. Patient with digital tip ulceration for 6 months, high titer Scl-70 histological UIP pattern with little cellular inflammation.
        5. Patient with 2 year history of pulmonary hypertension who died of acute exacerbation. Histology shows NSIP with OP overlap pattern, diffuse lymphocytic infiltration and pleural thickening.
        Board review style answer #1
        A & D.

          Comments:

        1. IPAF. This case meets clinical and morphologic criteria of IPAF and meets no rheumatologic criteria.
        2. Acute hypersensitivity pneumonitis. This case shows the IPAF findings of serologic and morphologic domain. However, the clinical course is more suggestive for hypersensitivity pneumonitis. NSIP pattern is also common in hypersensitivity pneumonitis.
        3. CTD-ILD. This case meets the classification criteria for rheumatoid arthritis (Ann Rheum Dis 2010;62:2569).
        4. IPAF. This case is suspicious for systemic sclerosis, but does not fulfill the rheumatologic criteria (Ann Rheum Dis 2013;72:1747).
        5. Idiopathic NSIP (with OP). This case shows the findings of morphologic domain only.

        Comment Here

        Reference: Interstitial pneumonia with autoimmune features

        Intrapulmonary thymoma (pending)
        [Pending]

        Intravascular large B cell lymphoma (pending)
        [Pending]

        Intravenous drug abusers
        Clinical features
        • Often incidental findings at autopsy or biopsy
        • Lesions due to injection of insoluble fillers of oral mediation, which lodge in small pulmonary arteries; may include crospovidone, talc, cornstarch or microcrystalline cellulose with characteristic findings
        • All but cornstarch persist indefinitely
        • Patients may have dyspnea or hypoxia; due to related infection, pulmonary edema, diffuse alveolar damage, vasculopathy leading to pulmonary hypertension or interstitial fibrosis
        • Chest xray usually shows a micronodular pattern
        Microscopic (histologic) description
        • Fresh or organizing arterial thrombi with foreign material
        • Perivascular foreign body granulomas contain birefringent foreign material resembling starch, talc or other materials
        • May be associated with acute vasculitis, interstitial inflammation and fibrosis
        Microscopic (histologic) images

        Images hosted on other servers:
        Polarized light

        Polarized light


        Invasive mucinous
        Definition / general
        • Distinct variant of lung adenocarcinoma with a goblet or columnar morphology and abundant intracytoplasmic mucin
        Essential features
        • Rare variant of lung adenocarcinoma with a goblet or columnar morphology and abundant intracytoplasmic mucin
        • No criteria regarding percentage of cells with mucin but typically homogeneous and not admixed with nonmucinous glands
        • Aggressive clinical behavior often associated with intrapulmonary metastasis and recurrence
        Terminology
        ICD coding
        • ICD-10: C34 - Malignant neoplasm of bronchus and lung
        • ICD-O: 8253/3 - Bronchiolo-alveolar carcinoma, mucinous
        Sites
        Pathophysiology
        Etiology
        • Less associated with smoking than other subtypes of lung adenocarcinoma
        Diagrams / tables

        Images hosted on other servers:
        Survival curves

        Survival curves

        Clinical features
        • Frequent intrapulmonary metastasis and recurrence
        • Rare extrapulmonary and distant metastasis (J Thorac Dis 2018;10:3595)
          • 80% are N0 at the time of diagnosis (versus 55% in nonmucinous adenocarcinoma)
          • 80% are M0 at the time of diagnosis (versus 55% in nonmucinous adenocarcinoma)
        • Larger size and higher T category at presentation
        Diagnostic criteria
        • Based on histological and immunohistochemical features
        • Even if pathological results correspond to primary invasive mucinous adenocarcinoma of lung, colorectal adenocarcinoma must be ruled out by endoscopy and radiology
        Radiology description
        • Consolidation with various appearance (Eur J Radiol 2015;84:993, Cancer Imaging 2019;19:47)
          • Solid lesion similar to other lung cancers
          • Mixed solid and gland glass opacity lesion similar to pneumonia
          • Air bronchogram is common
        • Often multifocal with multilobar involvement
        • FDG-PET often demonstrates relatively low accumulation to the lesion
        Radiology images

        Images hosted on other servers:
        Computed tomography Computed tomography Computed tomography Computed tomography Computed tomography Computed tomography

        Computed tomography

        Prognostic factors
        Case reports
        Treatment
        Gross description
        Gross images

        Contributed by Akira Yoshikawa, M.D. and Yale Rosen, M.D.
        Invasive mucinous adenocarcinoma Invasive mucinous adenocarcinoma

        Invasive mucinous adenocarcinoma

        Invasive, diffuse

        Invasive, diffuse

        Lepidic growth pattern

        Lepidic growth pattern

        Microscopic (histologic) description
        • Goblet cells or GI tract type columnar cells
          • No criteria regarding percentage of cells with mucin but typically homogeneous and not admixed with nonmucinous glands
          • Abundant intracytoplasmic mucin
          • Basally oriented nuclei with minimal atypia
          • Adjacent alveolar lumens are often filled with mucin (not specific for invasive mucinous adenocarcinoma)
        • Tend to be well differentiated (J Thorac Dis 2018;10:3595)
          • Often predominantly lepidic growth with slight distortion / destruction of alveolar architecture
          • Admixed with minor papillary, acinar or micropapillary component
        • Spread through air space is common
        Microscopic (histologic) images

        Contributed by Akira Yoshikawa, M.D.

        Intraalveolar mucin

        Nodules with fibrosis

        Tumor periphery

        Diffuse spread

        Acinar growth pattern


        Admixed pneumocytes

        Abundant apical mucin

        Extracellular and apical mucin

        HNF4α

        TTF1 / Napsin A



        Contributed by @zaalruwai83 on Twitter
        Invasive mucinous adenocarcinoma Invasive mucinous adenocarcinoma Invasive mucinous adenocarcinoma Invasive mucinous adenocarcinoma

        Invasive mucinous adenocarcinoma

        Cytology description
        Cytology images

        Contributed by Takashi Hori, C.T. and Akira Yoshikawa, M.D.

        Cluster of tumor cells

        Gold mucin

        Papillary cluster

        Positive stains
        Negative stains
        Molecular / cytogenetics description
        Sample pathology report
        • Lung, right upper lobe, resection:
          • Invasive mucinous adenocarcinoma (see synoptic report)
        Differential diagnosis
        • Metastatic colorectal adenocarcinoma:
          • Endoscopic and radiologic examination critical
          • Usually multifocal
          • Tend to be more solid and dense
          • Proliferation of neoplastic columnar cells with abundant intracytoplasmic mucin
          • CDX2 and CK20 positive
        • Colloid adenocarcinoma (Hum Pathol 2015;46:836):
          • Marked mucin pool replacing normal parenchyma
          • Neoplastic epithelium floating in the mucin
          • Fragments of displaced alveolar walls
          • Minimal nuclear atypia
          • Fibrotic encapsulation in minor cases (formerly called cystadenocarcinoma)
        Board review style question #1

        This patient had multifocal pulmonary lesions. CK20 was positive. Which other marker substantiates the diagnosis and would likely be expressed?

        1. TTF1
        2. PAS
        3. Alcian blue
        4. EpCAM
        5. CDX2
        Board review style answer #1
        E. This is metastatic colorectal adenocarcinoma which usually expresses CDX2 and CK20.

        Comment Here

        Reference: Invasive mucinous adenocarcinoma
        Board review style question #2
        Which of the following findings specifically corresponds to invasive mucinous adenocarcinoma of the lung compared with other sybtypes of lung adenocarcinoma?

        1. Abundant mucin in air spaces
        2. Abundant intracytoplasmic mucin
        3. TTF1 positive
        4. Napsin A positive
        5. p40 positive
        Board review style answer #2
        B. Abundant intracytoplasmic mucin

        Comment Here

        Reference: Invasive mucinous adenocarcinoma

        Langerhans cell histiocytosis
        Definition / general
        • Proliferative disease or disorder of Langerhans cells involving lung, highly associated with smoking
        • Pulmonary Langerhans cell histiocytosis (PLCH) is thought to be distinct from systemic Langerhans cell histiocytosis (LCH)
        • Most common pulmonary histiocytic lesion (Arch Pathol Lab Med 2008;132:1171)
        Essential features
        • Almost exclusively seen in young to middle aged smokers
        • Stellate nodules centered around small airways, with early cellular phase and later fibrotic phase
        • Langerhans cells stain for CD1a, Langerin, and S100
        Terminology
        • Eosinophilic granuloma, Langerhans cell granulomatosis, histiocytosis X ("H-X")
        • Multisystem diseases: Letterer-Siwi disease, Hand-Schüller-Christian disease, Hashimoto-Pritzker syndrome, where lung involvement is histologically indistinguishable from cellular phase of PLCH
        Epidemiology
        • Uncommon, encompassing 3 - 5% of lung biopsies for interstitial lung disease
        • >90% of cases associated with smoking - considered a smoking related disease
        • Usually ages 20 - 50 years
        • More common in Caucasian than African Americans or Asians
        Sites
        • 50% of cases only involve lung
        • 20% of those with multicentric disease (i.e. bone, skin, lymph nodes, spleen, CNS, pituitary, rarely thyroid and thymus) have lung involvement
        Pathophysiology
        • Debatable as reactive versus neoplastic, could arise from smoking related and immunomodulatory processes (Arch Pathol Lab Med 2016;140:230)
        • PLCH may be a reactive process with a subset showing clonality, but extrapulmonary forms of Langerhans cell histiocytosis are thought to be neoplastic
        • Unpredictable course, even with smoking cessation: some spontaneously resolve, others remain stable, some progress to honeycomb fibrosis
        Etiology
        • Disorder of cells with Langerhans cell phenotype
        Clinical features
        • Many patients asymptomatic
        • Chronic cough and exertional dyspnea typical (Eur J Intern Med 2015;26:351)
        • Rarely can present with hemoptysis
        • Recurrent spontaneous pneumothorax in 15 - 25% of cases
        • Pulmonary hypertension and vasculopathy frequent in advanced cases
        • Diabetes insipidus in 15% of patients from pituitary involvement
        • Generally restrictive features in early disease, obstructive features in late disease
        • Secondary malignant and non-malignant neoplasms
        Diagnosis
        • Lung biopsy is necessary for a definitive diagnosis, although may not be required if imaging findings are highly characteristic (Orphanet J Rare Dis 2012;7:16)
        • Transbronchial biopsy may be diagnostic; otherwise, requires surgical lung biopsy
        Laboratory
        • Normal peripheral eosinophil blood count
        Radiology description
        • Upper / middle lung predominance
        • X-ray: small nodules generally up to 1 cm and variably sized cysts
        • High resolution CT: bronchiolocentric stellate nodules possibly with surrounding ground glass opacities, some with faint lucent cavity
          • Advanced disease with cysts, emphysema adjacent to scarring, reticular and nodular opacities, honeycombing, can mimic IPF
        • FDG-PET may show increased uptake, especially in early nodular disease
        Prognostic factors
        • 10 - 20% may progress to respiratory failure
        • Median survival 12 years, 5 year survival 70%, 10 year survival 60%
        • Worse prognosis in cases with pulmonary hypertension
        • Worse prognosis for those with baseline poor respiratory function and those with concurrent neoplasms
        • Worse prognosis with extremes of age, prolonged constitutional symptoms, multiorgan involvement, extensive cysts, honeycomb changes
        Case reports
        Treatment
        • Smoking cessation most effective
        • Corticosteroids +/- chemotherapeutic agents show mixed results
        • Lung transplantation in advanced cases
        Gross description
        • Upper lobe predominance, fine nodular infiltrate, with nodules up to 1.5 cm
        • Advanced disease with cysts, cavitary lesions, and honeycombing
        Gross images

        Images hosted on other servers:
        Tiny nodules

        Tiny nodules

        Microscopic (histologic) description
        • Early cellular phase and older fibrotic lesions may be intermixed, but one pattern typically predominates
        • Scattered, variably sized stellate nodules ("star fish-like" or "Medusa head-like") with interstitial scarring and aggregates of Langerhans cells
        • Layered appearance of nodules, bronchiolocentric distribution with pleural/subpleural sparing
        • Confluence of nodules → serpentine sheath around small airways
        • Langerhans cells are relatively large with abundant, granular, mildly eosinophilic and indistinct cytoplasm, grooved nuclei with indented nuclear membranes / grooves, "crumpled tissue paper" nuclear outlines, pale basophilic nucleus, one or two small nucleoli
        • Also prominent eosinophils, lymphocytes, plasma cells, activated macrophages, multinucleated giant cells
        • Variably sized cystic spaces at periphery of nodules lacking lining cells, secondary to traction on surrounding alveolar walls or airways
        • Frequent hemosiderin, necrosis, alveolar lining cell hyperplasia, pigmented alveolar macrophages ("smoker's macrophages")
        • Variable vasculitis, patchy organization, other smoking related changes such as emphysema, respiratory bronchiolitis, desquamative interstitial pneumonia
        • Older fibrotic lesions have fewer Langerhans cells and eosinophils, more fibrosis
        • May show pulmonary hypertension
        • Sarcomatous variant has significant atypia and mitotic figures
        Microscopic (histologic) images

        Images hosted on other servers:
        Microcystic change

        Microcystic change

        End stage Langerhans

        End stage Langerhans

        Kidney bean shaped nuclei

        Kidney bean shaped nuclei

        CD1a+

        CD1a+

        CD1a, CD56, S100

        CD1a, CD56, S100

        Cytology description
        • Langerhans cells with abundant cytoplasm and grooved nuclei with background eosinophils and lymphocytes
        • Bronchioalveolar lavage may show increased numbers (>5%) of CD1a+ Langerhans cells but test with low sensitivity (Respir Med 2012;106:1286)
        Positive stains
        Electron microscopy description
        • Birbeck's granules (pentilaminar intracytoplasmic structures, tennis racket shaped)
        Electron microscopy images

        Images hosted on other servers:
        Birbeck granules

        Birbeck granules

        Binucleated Langerhans cells, orbit

        Binucleated Langerhans cells, orbit

        Molecular / cytogenetics description
        Differential diagnosis

        Large cell
        Definition / general
        • Resected tumors that lack morphologic or immunohistochemical differentiation toward small cell carcinoma, adenocarcinoma or squamous cell carcinoma (J Thorac Oncol 2015;10:1240)
        • Null or unclear immunophenotype based on IHC analysis with TTF1, p40 and neuroendocrine markers (J Thorac Oncol 2019;14:1213)
        Essential features
        • Malignant, poorly differentiated epithelial neoplasm of lung composed of large, atypical cells
        • No morphologic or immunohistochemical evidence of glandular, squamous or neuroendocrine differentiation
        • Diagnosis of exclusion; diagnosis can only be made on resected tumors
        Terminology
        • Synonymous with large cell undifferentiated carcinoma and large cell anaplastic carcinoma
        • Previously, large cell carcinoma included variants such as basaloid carcinoma, large cell neuroendocrine carcinoma, lymphoepithelioma-like carcinoma, clear cell carcinoma and large cell carcinoma with rhabdoid phenotype
        • As of the 2015 WHO classification, large cell carcinoma is a diagnosis of exclusion, with reclassification of the former large cell carcinoma subtypes into different categories (J Thorac Oncol 2015;10:1243)
        ICD coding
        • ICD-O: 8012/3 - large cell carcinoma, NOS
        • ICD-10: C34.90 - malignant neoplasm of unspecified part of unspecified bronchus or lung
        Epidemiology
        • M > F
        • Average age of 65 years
        • Associated with cigarette smoking
        • Becoming increasingly rare due to immunohistochemical and molecular studies that support the reclassification of entities formerly diagnosed as large cell carcinoma (Sci Transl Med 2013;5:209ra153)
        Sites
        • Peripheral lung
        • 50% with connection to large airways
        Clinical features
        • Cough, chest pain, shortness of breath
        • Typically larger than 5 cm
        Diagnosis
        • Diagnosis of exclusion
        • Small biopsy specimens and cytology specimens should not be used for diagnosis
          • According to the 2021 WHO, only resection specimens with exclusion of a differentiated component, via thorough sampling, can be classified as large cell carcinoma (J Thorac Oncol 2022;17:362)
        • NSCC, NOS (nonsmall cell carcinoma, not otherwise specified) should be used in small biopsies or cytology specimens
        Radiology description
        • Usually large, lobulated or well marginated, peripheral, heterogeneously enhancing mass, often containing central necrosis (Ghosh: Handbook of Imaging in Pulmonary Disease, 1st Edition, 2021)
        • Rapid growth is common with metastasis (chest wall, thoracic lymph nodes, distant) at presentation
        • PET CT preferred over CT for identification of full extent of disease and metastases
        Prognostic factors
        Case reports
        Treatment
        • Dependent on stage
        • Surgical excision, chemotherapy or radiation therapy
        Gross description
        Gross images

        Contributed by Debra L. Zynger, M.D.
        Right lung mass

        Right lung mass

        Microscopic (histologic) description
        Microscopic (histologic) images

        Contributed by Emily O. Symes, M.D.

        Sheets of tumor cells

        Necrosis and lymphovascular invasion

        Extensive necrosis

        Lymphovascular invasion

        Spread through air spaces (STAS)


        High power tumor cells

        TTF1

        CK5/6

        p40

        Napsin A


        PDL1

        Mucin stain

        Virtual slides

        Images hosted on other servers:
        Various images Various images

        Undifferentiated large cell carcinoma

        Various images Various images

        Undifferentiated large cell carcinoma

        Cytology description
        • Large pleomorphic cells with a moderate to abundant amount of cytoplasm, vesicular nuclei, prominent nucleoli
        Positive stains
        Molecular / cytogenetics description
        • Molecular studies show that some null phenotype large cell carcinomas contain mutations seen in conventional adenocarcinoma (e.g., KRAS, NRAS, MET, STK11) and squamous cell carcinoma (e.g., PIK3CA amplification and mutation); this indicates that tumors classified as large cell carcinomas could represent poorly / undifferentiated nonsmall cell lung cancer (NSCLC) (J Thorac Oncol 2019;14:1213, Sci Transl Med 2013;5:209ra153)
        Sample pathology report
        • Lung, left, resection:
          • Large cell carcinoma (1.1 cm) (see comment)
          • Comment: Immunohistochemical studies are performed with appropriate controls. The tumor is negative for TTF1, Napsin A, CK5/6 and p40. There is no morphologic evidence of differentiation to adenocarcinoma or squamous cell carcinoma. The histologic features are not those of neuroendocrine carcinoma. Given these findings, the tumor is best classified as large cell carcinoma. PDL1 immunostain shows weak expression in 20% of tumor cells.
        Differential diagnosis
        Board review style question #1
        Large cell carcinoma of the lung will generally stain for which of the following immunohistochemical markers?

        1. CAM5.2
        2. Chromogranin
        3. p40
        4. SOX10
        5. TTF1
        Board review style answer #1
        A. CAM5.2

        Comment Here

        Reference: Large cell carcinoma
        Board review style question #2

        Which one of the following special stains is shown in the image above?

        1. CK5/6
        2. Chromogranin
        3. TTF1
        4. Mucin
        5. p40
        Board review style answer #2
        D. Mucin. Tumor cells are negative for mucin immunostaining but show degenerative vacuoles, which can mimic mucin on H&E.

        Comment Here

        Reference: Large cell carcinoma

        Large cell neuroendocrine carcinoma
        Definition / general
        • Aggressive, poorly differentiated carcinoma composed of large cells with neuroendocrine architecture, prominent nucleoli, high mitotic activity and necrosis
        • Grouped with other neuroendocrine tumors in the 2021 WHO classification of thoracic tumors (J Thorac Oncol 2022;17:362)
        • More likely to recur and has shorter patient survival than other types of non-small cell lung carcinoma, even in stage I disease (Cancer Control 2006;13:270)
        • May coexist with other lung cancers such as adenocarcinoma and squamous cell carcinoma
        Essential features
        • High grade non-small cell carcinoma with neuroendocrine architecture and immunohistochemical markers, characterized by > 10 mitoses/2 mm2, prominent nucleoli and extensive necrosis (J Thorac Oncol 2022;17:362)
        • Poor overall prognosis
        • Should be distinguished from atypical carcinoid, basaloid squamous cell carcinoma and poorly differentiated adenocarcinoma, although diagnosis can be difficult on small biopsies or cytology specimens
        Terminology
        • Large cell neuroendocrine carcinoma (LCNEC): proposed by Travis et al. in 1991 as distinct from small cell carcinoma (Am J Surg Pathol 1991;15:529)
        • Combined LCNEC: also has components of adenocarcinoma (most common), squamous cell carcinoma, giant cell carcinoma or spindle cell carcinoma (Thorac Surg Clin 2014;24:257)
        ICD coding
        • ICD-O: 8013/3 - large cell neuroendocrine carcinoma
        • ICD-10: C34.90 - malignant neoplasm of unspecified part of unspecified bronchus or lung
        • ICD-11: 2C25.Y - other specified malignant neoplasms of bronchus or lung
        Epidemiology
        Pathophysiology
        • Proposed model of histogenesis of lung neuroendocrine carcinomas, in view of similarities in the expression of primitive neural / neuroendocrine cell specific transcription factors (Pathol Int 2015;65:277)
        Etiology
        Clinical features
        Diagnosis
        • Difficult to make specific diagnosis on small biopsy and cytology specimens
        • Frequently recognized in cytology as non-small cell lung cancer (NSCLC), not otherwise specified or as adenocarcinoma
        • Neuroendocrine features by light microscopy and confirmation by immunohistochemical staining for neuroendocrine markers
        Laboratory
        Radiology description
        • Nonspecific findings, indistinguishable from other NSCLC
        • CT findings: peripherally located tumors, expansively growing with irregular margins, with or without calcification and infrequent cavitation (Clin Imaging 2007;31:379)
        Prognostic factors
        Case reports
        Treatment
        Gross description
        Gross images

        Contributed by Roseann Wu, M.D., M.P.H.
        Peripheral LCNEC

        Peripheral LCNEC

        Frozen section description
        • Neuroendocrine organoid architecture, prominent nucleoli, abundant necrosis
        Microscopic (histologic) description
        Microscopic (histologic) images

        Contributed by Ioanna Abba Nteka, M.D., Aggeliki Cheva, M.D., Ph.D., Antonia Loukousia, M.D., Roseann Wu, M.D., M.P.H. and Kyriakos Chatzopoulos, M.D., Ph.D.
        Nesting, peripheral palisading, necrosis

        Nesting, peripheral palisading, necrosis

        Nuclear pleomorphism, prominent nucleoli, mitosis

        Nuclear pleomorphism, prominent nucleoli, mitosis

        Solid and nested architecture

        Solid and nested architecture

        Vascular invasion

        Vascular invasion


        Cytokeratin

        Cytokeratin

        CD56

        CD56

        Synaptophysin

        Synaptophysin

        Chromogranin

        Chromogranin

        Cytology description
        • Hypercellular, with numerous, single medium to large cells
        • 3 dimensional and variably sized groups
        • Large, pleomorphic cells with irregular vesicular chromatin, prominent nucleoli, abundant cytoplasm, sharp cellular borders
        • Naked nuclei are abundant but with a variable subset of cells showing evident cytoplasm
        • Molding, mitoses, necrotic background
        • Peripheral nuclear palisading; rosette-like structures (Cancer 2008;114:180, Diagn Cytopathol 2001;24:58)
        Cytology images

        Images hosted on other servers:

        Nuclear pleomorphism with prominent nucleoli (Pap)

        Peripheral palisading
        (Diff-Quik)

        Rosette-like structure
        (Diff-Quik)

        Positive stains
        Negative stains
        Electron microscopy description
        • Neurosecretory granules, occasional evidence of granular differentiation and intercellular junctions suggestive of squamous differentiation (Am J Surg Pathol 1991;15:529)
        Molecular / cytogenetics description
        • 3 molecular subsets identified by next generation sequencing; no statistically significant differences in clinicopathologic parameters or survival among subsets:
          • Small cell carcinoma-like: TP53 and RB1 comutation and MYCL amplification
          • Non-small cell carcinoma-like: STK11, KRAS, KEAP1 mutations, no TP53 / RB1 comutation, molecular profile resembling adenocarcinoma, with additional NOTCH family mutations
          • Carcinoid-like: MEN1 mutations, low mutation burden (Clin Cancer Res 2016;22:3618)
        • Other less commonly mutated genes: PIK3CA, PTEN, AKT2, FGFR1, KIT, ERBB2, HRAS, EGFR, NTRK2 and NTRK3 (Clin Cancer Res 2017;23:757, Hum Mutat 2008;29:609)
        • Case reports of tumors harboring an EML4::ALK fusion (Cold Spring Harb Mol Case Stud 2022;8:a00623, Front Oncol 2022;12:823813)
        • BRAF alterations including mutations, amplifications reported in small case series (JCO Precis Oncol 2018;2:1)
        Sample pathology report
        • Lung, left upper lobe, CT guided needle core biopsy:
          • Large cell neuroendocrine carcinoma (see comment)
          • Comment: Multiple sections from the biopsy material reveal an infiltrative malignant tumor consisting of large cells with abundant amphophilic cytoplasm and large, ovoid or round nuclei with prominent nucleoli and increased mitotic activity. Tumor cells form solid nests with peripheral palisading and central necrosis. Immunostains performed on paraffin sections show diffuse and strong expression of keratins AE1 / AE3, synaptophysin, chromogranin, CD56 and nuclear expression of TTF1 and ISNM1. No expression of p40 or napsin A is seen. These findings support the diagnosis.
        Differential diagnosis
        Board review style question #1

        A 76 year old man with a history of heavy smoking is diagnosed with a large pulmonary mass and undergoes lobectomy. The histologic features of the mass can be seen in the photograph above. Which initial panel of immunostains is more likely to be helpful in the differential diagnosis of this pulmonary malignancy from its most frequent counterparts?

        1. BRG1, INI1 and NUT
        2. Chromogranin, synaptophysin and cytokeratin 8/18
        3. ISNM1, p40 and napsin A
        4. p63, TTF1 and CD56
        Board review style answer #1
        C. ISNM1, p40 and napsin A. The differential diagnosis of this malignancy includes large cell neuroendocrine carcinoma, poorly differentiated adenocarcinoma and squamous cell carcinoma. In any case, additional immunostains will be needed to confirm the diagnosis. Answer D is incorrect because although CD56 can be expressed in LCNEC, it should be used in caution when alone, as it is not as specific as other neuroendocrine markers. TTF1 can be expressed in half of LCNECs. Although p63 is a marker of squamous differentiation, it is not as specific as its p40 isoform. Answer B is incorrect because although LCNEC is likely to express all these 3 markers, the panel will not help rule out poorly differentiated adenocarcinoma. Answer A is incorrect because BRG1 expression is lost in undifferentiated SMARCA4 deficient thoracic tumors. INI1 can be partially lost in poorly differentiated non-small cell carcinomas with rhabdoid features or totally lost in SMARCB1 deficient carcinomas. NUT immunopositivity would raise the possibility of NUT carcinoma. None of these markers will be helpful in differentiating LCNEC from its more frequent mimics. Answer C is correct because ISNM1 is a highly sensitive and specific marker of neuroendocrine differentiation . Similarly, p40 isoform of p63 is highly specific for squamous differentiation. Napsin A is most frequently expressed in lung adenocarcinomas. This panel of immunostains will be the most helpful, since ISNM1 positivity will point towards the diagnosis of LCNEC, while p40 and napsin A negativity will make the possibilities of squamous cell carcinoma and adenocarcinoma remote.

        Comment Here

        Reference: Large cell neuroendocrine carcinoma
        Board review style question #2
        Which of the following histologic features is the most helpful in the differential diagnosis of large cell neuroendocrine carcinoma from small cell carcinoma?

        1. Extensive necrosis
        2. Mitotic count > 10/2 mm2
        3. Organoid nesting
        4. Prominent nucleoli
        Board review style answer #2
        D. Prominent nucleoli are a characteristic feature of LCNEC. In contrast, small cell carcinoma cells usually have hyperchromatic nuclei with salt and pepper chromatin distribution and indiscrete nucleoli. Answer B is incorrect because mitotic count is elevated both in LCNEC and small cell carcinoma. Answer A is incorrect because extensive geographic necrosis is a feature of both LCNEC and small cell carcinoma. Answer C is incorrect because although organoid nesting is very characteristic of LCNEC, in can be frequently seen in areas of small cell carcinoma as well.

        Comment Here

        Reference: Large cell neuroendocrine carcinoma

        Legionella
        Clinical features
        • Initial public attention after epidemic in Philadelphia, Pennsylvania (US) convention of American Legion
        • Caused by Legionella species, a short, Gram negative bacillus (Wikipedia)
        • Retrospective review disclosed sporadic cases since early 1900s
        • Pontiac fever is caused by same bacteria, but without pneumonia, and often subclinical
        • Hilar lymph nodes infected in 50% of cases at autopsy; 25% have spread to other organs
        • Causes: contaminated water in water cooling tanks, drinking water conduits, air conditioning systems and other contained indoor water storage systems
        • Immunosuppressed patients, especially organ transplant recipients, are especially vulnerable to severe disease
        • Interaction with macrophages and dendritic cell subtypes appears to be important for resolution (Front Microbiol 2011;2:126)
        Treatment
        • Erythromycin or other antibiotics, but high mortality in immunocompromised
        Microscopic (histologic) description
        • Extensive bronchopneumonia to lobar pneumonia with intra-alveolar neutrophils, macrophages, fibrin; often with leukocytoclastic neutrophilic infiltrate, small vessel vasculitis and necrosis
        Microscopic (histologic) images

        Images hosted on other servers:
        Gram stain

        Gram stain

        Immunofluorescence staining

        Immunofluorescence staining

        Cytology images

        Images hosted on other servers:
        Impression smear from lung

        Impression smear from lung

        Positive stains
        • Dieterle silver stain, monoclonal fluorescent reagent
        Electron microscopy images

        Images hosted on other servers:
        Legionella pneumophila

        Legionella pneumophila


        Lepidic adenocarcinoma
        Definition / general
        • Lung adenocarcinoma with primarily lepidic growth pattern with at least one of the following (Arch Pathol Lab Med 2013;137:685, J Thorac Oncol 2011;6:244):
          • > 0.5 cm of stromal invasion
          • Measuring > 3 cm in greatest dimension with ≤ 0.5 cm of stromal invasion
          • Lymphatic, vascular or pleural invasion
          • Tumor necrosis
          • Spread through airspaces
        Essential features
        • Tumor with noninvasive (lepidic) and invasive components
        • Invasive component should total > 0.5 cm
        • Invasive component can be defined as stromal invasion or the presence of any subtype other than lepidic
        • Presence of lymphovascular invasion, tumor necrosis or pleural invasion also meets the criteria of an invasive component
        • Tumors measuring > 3 cm with < 0.5 cm of invasive or nonlepidic components are also classified as lepidic adenocarcinoma
        • In rare cases, a lepidic carcinoma with < 0.5 cm of invasion that has high risk features (lymphatic, vascular or pleural invasion, tumor necrosis or spread through airspaces) should also be classified as lepidic adenocarcinoma and not minimally invasive adenocarcinoma
        Terminology
        • Lepidic adenocarcinoma
        • Lepidic predominant adenocarcinoma (for mixed patterns where lepidic growth pattern predominates)
        • Older terminology was bronchoalveolar carcinoma; however, this is obsolete and should no longer be utilized
        ICD coding
        • ICD-0: 8250/3 - lepidic adenocarcinoma
        • ICD-10: C34.9 - malignant neoplasm of unspecified part of bronchus or lung
        • ICD-11: XH24W2 - lepidic adenocarcinoma
        Epidemiology
        • More common in Asian populations and not as strongly associated with smoking as other subtypes of adenocarcinoma (Am J Surg Pathol 2014;38:448)
        Sites
        Pathophysiology
        • Hypothesized to form after a multistep progression from atypical adenomatous hyperplasia to adenocarcinoma in situ to lepidic adenocarcinoma (J Thorac Oncol 2008;3:340)
        Clinical features
        Diagnosis
        Radiology description
        Radiology images

        Images hosted on other servers:

        Ground glass opacity

        Prognostic factors
        Case reports
        • 26 year old woman with a lepidic adenocarcinoma demonstrating aerogenous spread of mucin without evidence of invasion (Exp Mol Pathol 2014;96:400)
        • 36 year old woman with a 33 week pregnancy in acute respiratory failure from a lepidic pulmonary adenocarcinoma (Case Rep Oncol 2018;11:822)
        Treatment
        • Associated with a lower risk of recurrence than other subtypes of adenocarcinoma; surgical excision with close follow up may be sufficient for early stage disease (Am J Surg Pathol 2014;38:448)
        Gross description
        • Peripheral ill defined firm white tumor with solid areas; ill defined areas correspond to lepidic growth pattern and the solid areas correspond to invasive component (Hum Pathol 2016;51:41)
        • In rare cases, necrosis and frank pleural invasion may be present
        Microscopic (histologic) description
        • Primary lung adenocarcinoma with lepidic growth pattern measuring ≤ 3 cm in greatest dimension with > 0.5 cm area of stromal invasion or presence of necrosis, lymphovascular invasion, pleural invasion or spread through air spaces
        • Lepidic growth pattern: neoplastic cells resembling bland type II pneumocytes growing along the surface of the alveolar air spaces
        • Foci of stromal invasion characterized by angulated glands, desmoplastic stroma and increased cytologic atypia (Cancer 1995;75:2844)
        • Size of invasion should be measured as the largest focus of invasion (Am J Surg Pathol 2014;38:448, Arch Pathol Lab Med 2013;137:685)
        • If there are multiple foci of invasion, the invasive size can be measured as a sum of the percentage of the invasive components in each section multiplied against the greatest tumor dimension (Am J Surg Pathol 2014;38:448, Surg Today 2019;49:828)
        • Invasion may also be defined as presence of nonlepidic growth pattern (acinar, micropapillary, papillary, solid, colloid or fetal patterns) or the presence of necrosis, lymphovascular invasion, pleural invasion or spread through air spaces
        • For the uncommon occurrence of a lepidic predominant tumor > 3.0 cm with either no invasion or ≤ 0.5 cm of invasion, it is recommended that such tumors be classified as lepidic predominant adenocarcinoma and staged as pT1a, since there is insufficient data to conclude they have the same prognostic features as ≤ 3.0 cm tumors meeting criteria for adenocarcinoma in situ or minimally invasive adenocarcinoma (J Thorac Oncol 2016;11:1204)
        • Mucinous carcinoma can often display a prominent lepidic growth pattern; however, the majority of cases will have areas of invasion and should be classified as an invasive mucinous adenocarcinoma
        Microscopic (histologic) images

        Contributed by Jonathan Keow, M.D., Ph.D. and Matthew J. Cecchini, M.D., Ph.D.
        Low power of lepidic adenocarcinoma

        Lepidic growth pattern

        Medium power of lepidic adenocarcinoma

        With uninvolved lung parenchyma

        High power of lepidic adenocarcinoma with focal invasion

        Lepidic adenocarcinoma with focal invasion

        Noninvasive lepidic adenocarcinoma

        Noninvasive lepidic adenocarcinoma

        High power of lepidic adenocarcinoma with focal invasion

        Lepidic adenocarcinoma

        High power of lepidic adenocarcinoma with focal invasion

        Lepidic adenocarcinoma in a background of emphysema

        Positive stains
        Negative stains
        Sample pathology report
        • Lung, right lower lobe, lobectomy:
          • Invasive adenocarcinoma, lepidic predominant, pT1a NX MX (see synoptic report)
        Differential diagnosis
        Board review style question #1

        In the image above, the abnormal area in the top center is best characterized by which of the following?

        1. Atypical adenomatous hyperplasia
        2. Invasive acinar adenocarcinoma
        3. Noninvasive lepidic adenocarcinoma
        4. Normal lung
        Board review style answer #1
        C. Noninvasive lepidic adenocarcinoma. In this image, the neoplastic cells extend along intact alveolar spaces. There are no architectural changes or features of invasive growth. There is a sharp cut off between these cells and a small amount of normal lung in the top left of the image. While challenging to see at this power, the cells are hyperchromatic with variably spaced overlapping cells that supports the classification as a lepidic adenocarcinoma.

        Comment Here

        Reference: Lepidic adenocarcinoma
        Board review style question #2
        What is the best diagnosis for a 4.0 cm tumor composed primarily of noninvasive lepidic adenocarcinoma and a 0.4 cm invasive acinar component?

        1. Acinar predominant adenocarcinoma
        2. Adenocarcinoma in situ
        3. Lepidic predominant adenocarcinoma
        4. Minimally invasive adenocarcinoma
        Board review style answer #2
        C. Lepidic predominant adenocarcinoma. This tumor is best classified as a lepidic predominant adenocarcinoma because the size is 4 cm which is above the 3 cm cut off for a lepidic adenocarcinoma. Note, the invasive component of 0.4 cm is insufficient to meet criteria for a lepidic adenocarcinoma. The T stage should be assigned based on the size of the invasive component.

        Comment Here

        Reference: Lepidic adenocarcinoma

        Lipoid pneumonia
        Definition / general
        • Usually incidental post mortem finding associated with debilitating disease
        • Lipid either exogenous (from nasal sprays or inhalation of other lipid containing substances) or endogenous (bronchial obstruction)
        Case reports
        • 1 year old man with recent history of meningitis with recurrent episodes of pneumonia (Case of the Week #287)
        Gross description
        • Well circumscribed, firm with prominent lymphatics on lung surface in exogenous type
        Gross images

        Images hosted on other servers:
        Yellow cut surface

        Yellow cut surface

        Microscopic (histologic) description
        • Lipoid material (or empty spaces), inflammatory cells and young fibroblasts
        • Reactive endarteritis, marked alveolar lining cell hyperplasia and lipid laden foamy macrophages
        Microscopic (histologic) images

        Case #287




        Images hosted on other servers:
        Aspiration pneumonia

        Aspiration pneumonia

        Foamy macrophages in alveolar spaces

        Foamy macrophages in alveolar spaces


        Lipoma
        Definition / general
        • Very rare; usually endobronchial, usually men ages 50+ (Chest 2003;123:293)
        • Benign neoplasm composed of mature adipose tissue
        • Treatment of choice is bronchoscopic removal
        Essential features
        • Endobronchial lipomas may show focal cytologic atypia and fibrosis but molecular evidence suggests similarity to lipomas of other sites
        • Endobronchial lipomas clinically mimic other tumors and may cause obstructive symptoms
        • Pulmonary lipomas lack the cartilage, myxoid matrix, entrapped bronchial epithelium, or other mesenchymal components seen in pulmonary hamartomas
        Epidemiology
        • Very rare, 0.1 to 0.5% of all lung tumors
        • Older men, mean age 65, majority with smoking history
        • May be associated with obesity
        Sites
        • Usually endobronchial (large bronchi), very rarely parenchymal
        • More common in right lung
        Pathophysiology
        • Typically sporadic
        Etiology
        • Endobronchial lesions may arise from peribronchial or submucosal adipocytes
        • Parenchymal lesions may arise from adipocytes of subsegmental bronchi or subpleural fatty tissue
        Clinical features
        • Slow growing, indolent
        • Discovered incidentally by imaging or during bronchoscopy for other reasons
        • Larger endobronchial lesions may present with cough, pneumonia, bronchiectasis, empyema, hemoptysis
        • May mimic endobronchial carcinoid, malignancy or asthma / COPD
        Diagnosis
        • CT or MRI can suggest diagnosis
        • Bronchoscopic biopsy or excision for endobronchial lesions
        Radiology description
        • High signal intensity of fat in all MRI sequences
        • CT shows smooth, non-enhancing, homogenous mass with fat density
        • May see peripheral air trapping on CT with obstructive endobronchial lesions
        Prognostic factors
        • Benign, but large lesions may cause obstructive complications
        Case reports
        Treatment
        • Resection is curative
        Clinical images

        Images hosted on other servers:
        Endoscopy Endoscopy

        Endoscopy

        Gross description
        • Endobronchial tumors are generally < 3 cm
        • Well circumscribed, polypoid or sessile, yellowish mass on bronchoscopy
        • Yellow-grey mass with firm capsule
        • Intraparenchymal lesions are well circumscribed, soft, lobulated, yellow
        Gross images

        Images hosted on other servers:
        Complete encapsulation

        Complete encapsulation

        Microscopic (histologic) description
        • Mature adipose tissue underlying respiratory epithelium, with scattered lymphocytes and histiocytes
        • May show fibrosis and hyperchromatic stromal cells
        • Rarely has osteocartilaginous metaplasia
        Microscopic (histologic) images

        Images hosted on other servers:
        Atypical lipoma Atypical lipoma

        Atypical lipoma

        Mature adipose tissue

        Mature adipose tissue

        Molecular / cytogenetics description
        Differential diagnosis

        Loeffler syndrome
        Table of Contents
        Definition / general
        Definition / general
        • Acute eosinophilic pneumonia with transient diffuse pulmonary infiltrates composed primarily of eosinophils and serum eosinophilia
        • Characteristic imaging findings, so biopsy is rarely performed
        • Eosinophils may be present in sputum
        • Often associated with ascaris infection
        • Self limited - lasts only up to 1 month

        Lymphangioleiomyomatosis
        Definition / general
        • Lung mesenchymal neoplasm that arises from abnormal smooth muscle cells around lymphatic vessels and causes parenchymal damage
        Essential features
        • Rare cystic lung disease that causes progressive lung damage; mainly affects young females with tuberous sclerosis complex (TSC)
        Terminology
        • LAM
        ICD coding
        • ICD-O: 9174/3 - lymphangioleiomyomatosis
        • ICD-11: CB07.Z - lymphangioleiomyomatosis, unspecified
        Epidemiology
        Sites
        Pathophysiology
        Etiology
        Clinical features
        Diagnosis
        • Usually a clinical diagnosis
        • Multiple bilateral round lung cysts on high resolution computed tomography (CT) scan
        • Screening CT scans are recommended for females with tuberous sclerosis complex (Am J Respir Crit Care Med 2006;173:105)
        Laboratory
        Radiology description
        Radiology images

        Images hosted on other servers:
        Computed tomography Computed tomography

        Computed tomography

        Prognostic factors
        Case reports
        Treatment
        • Relies on degree of lung function and extent of lung damage
        • Supportive therapy with bronchodilators or oxygen therapy (Thorax 2019;74:999)
        • Observation for patients with mild function impairment
        • mTOR inhibitors (sirolimus, everolimus): for symptomatic patients with severe lung damage and disease progression (N Engl J Med 2011;364:1595)
        • Lung transplantation for cases refractory to medical treatment
        Gross description
        Gross images

        Images hosted on other servers:
        Multiple cysts

        Multiple cysts

        Microscopic (histologic) description
        • Multiple thin walled cysts
        • Cysts are lined or surrounded by nodules of rounded or spindled smooth muscle-like cells with mixed eosinophilic or clear cytoplasm, without cytologic atypia or mitoses
        • Hemosiderin laden macrophages may be present as a clue to organized hemorrhage resulting from cysts infiltrating nearby vessels
        • Multinodular pneumocyte hyperplasia around cysts (found in TSC LAM more than sporadic lymphangioleiomyomatosis)
        • References: Semin Respir Crit Care Med 2020;41:256, Endocrinology 2016;157:3374
        Microscopic (histologic) images

        Contributed by Roseann I. Wu, M.D., M.P.H. and Yale Rosen, M.D.

        Cystic lung with nodule

        Multiple cysts

        Multiple cysts

        Cytology description
        • Well organized, globular cluster consisting of lymphangioleiomyomatosis cells enveloped by lymphatic endothelial cells (Acta Cytol 2009;53:402)
        Cytology images

        Images hosted on other servers:
        Outer flattened cells

        Outer flattened cells

        SMA, HMB45, D20

        SMA, HMB45, D2-40

        Immunofluorescence description
        Immunofluorescence images

        Images hosted on other servers:
        Lymphangioleiomyomatosis cells

        LAM cells

        Positive stains
        Negative stains
        Electron microscopy description
        • Lymphangioleiomyomatosis nodules are lined by type II pneumocytes with short microvilli and electron dense cytoplasmic lamellar bodies (Arch Pathol Lab Med 2000;124:1642)
        Electron microscopy images

        Images hosted on other servers:
        Multiple cysts Multiple cysts

        Type II pneumocytes in LAM nodule

        Molecular / cytogenetics description
        Videos

        Virtual slide review of a case of lymphangioleiomyomatosis

        Overview of radiographic features of lymphangioleiomyomatosis

        Sample pathology report
        • Lung, right upper lobe, wedge biopsy:
          • Lymphangioleiomyomatosis (see comment)
          • Comment: Sections show cystic spaces replacing normal parenchyma. Some of the cystic spaces have mural nodules consisting of epithelioid and spindled cell proliferation. Lesional cells are immunoreactive with ER, SMA and MelanA. Control stains are satisfactory. The clinical presentation, morphology and immunohistochemical stains support the diagnosis above.
        Differential diagnosis
        Board review style question #1

        A 44 year old woman who is a nonsmoker presented to the emergency department with acute chest pain after several months of progressive dyspnea. Physical examination was notable for diminished breath sounds on the right side. Computed tomography of the chest revealed a large pneumothorax and diffuse, intraparenchymal pulmonary cysts. The patient underwent mechanical pleurodesis and a wedge biopsy was performed. The biopsy specimens contained numerous thin walled cysts lined with spindle cells; on immunohistochemistry, the spindle cells coexpressed smooth muscle and melanocytic markers. What is the most likely diagnosis?

        1. Emphysema
        2. Lymphangioleiomyomatosis
        3. Metastasizing pulmonary leiomyoma
        4. Sarcoidosis
        Board review style answer #1
        B. Lymphangioleiomyomatosis. Answer A is incorrect because emphysema can present with pneumothorax and lung cysts on imaging but the process is usually more localized and most patients have a history of smoking. Additionally, cysts seen in emphysematous lung specimens represent lung parenchyma with cystic changes rather than lesional / neoplastic cells with the above mentioned immunophenotype. Answer D is incorrect because while sarcoidosis can present with shortness of breath and cystic changes in the lung, these changes are usually localized. Sarcoidosis patients usually have hilar adenopathy and fibrosis (not a finding in this patient). Answer C is incorrect because metastasizing leiomyoma is a diffuse process that can also be symptomatic and appear cystic; however, on immunohistochemistry, leiomyoma cells are not immunoreactive to melanocytic markers.

        Comment Here

        Reference: Lymphangioleiomyomatosis
        Board review style question #2
        A 44 year old woman who is a nonsmoker presented to the emergency department with acute chest pain after several months of progressive dyspnea. Physical examination was notable for diminished breath sounds on the right side. Computed tomography of the chest revealed a large pneumothorax and diffuse, intraparenchymal pulmonary cysts. The patient underwent mechanical pleurodesis and a wedge biopsy was performed. The biopsy specimens contained numerous thin walled cysts lined with spindle cells; on immunohistochemistry, the spindle cells coexpressed smooth muscle and melanocytic markers. What other findings may be seen on this patient's physical examination?

        1. Characteristic skin changes
        2. Fever
        3. Lower extremity swelling
        4. Tobacco stains
        Board review style answer #2
        A. Characteristic skin changes. Most likely this patient also has TSC gene mutations as these contribute to the development of lymphangioleiomyomatosis (LAM). Skin changes / lesions such as angiofibromas or hypomelanotic macules are hallmarks / diagnostic features of tuberous sclerosis complex. Answer B is incorrect because fever has not been described for this disorder. Answer C is incorrect because lower extremity swelling and smoking history may raise suspicion for pulmonary embolism. Answer D is incorrect because tobacco stains would raise suspicion for complicated emphysema.

        Comment Here

        Reference: Lymphangioleiomyomatosis

        Lymphoepithelial carcinoma
        Definition / general
        • According to the 2021 WHO classification update, lymphoepithelial carcinoma is the currently designated name and is defined as a type of poorly differentiated squamous cell carcinoma associated with variable amount of lymphoplasmacytic infiltrate and frequent association with Epstein-Barr virus (EBV)
          • In the 2015 WHO classification, the preferred name was lymphoepithelioma-like carcinoma and it was in the category of other and unclassified carcinomas
        Essential features
        • Characterized by distinct syncytial growth pattern, vesicular nuclei, prominent eosinophilic nucleoli and variable lymphoplasmacytic infiltrate
        • Immunohistochemical (IHC) staining is the same as that for conventional squamous cell carcinoma (diffusely positive for CK5/6, p40 and p63)
        • With complete surgical resection it has a favorable prognosis when compared with conventional squamous cell carcinoma
        Terminology
        • Formerly known as lymphoepithelioma-like carcinoma
        ICD coding
        • Use the ICD code specific for location of tumor
        • ICD-9: 162 - malignant neoplasm of trachea bronchus and lung
        • ICD-10: C34.90 - malignant neoplasm of unspecified part of unspecified bronchus or lung
        • ICD-11: 2C25 - malignant neoplasms of bronchus or lung
        Epidemiology
        Sites
        Pathophysiology
        • EBV induced carcinogenesis (Nat Commun 2019;10:3108)
          • Driven by dysregulated NFkB pathway, loss of type I IFN genes, APOBEC family gene signature
        Etiology
        Clinical features
        Diagnosis
        • CT images of the thorax and fiberoptic bronchoscopy with tissue biopsy
        • FNA cytology with IHC studies may assist
          • Can be limited by sampling errors
        • Endoscopic examination with or without radiographic imaging of nasopharynx performed to rule out metastatic lymphoepithelial carcinoma
        • Reference: Respirology 2006;11:539
        Radiology description
        • Well defined, solitary, lobulated mass > 1 cm (Clin Radiol 2022;77:e201)
        • Homogeneous density
        • Vascular enhancement
        • High 18F-FDG uptake
        Radiology images

        Images hosted on other servers:

        Xray and CT scans

        Unenhanced thoracic CT

        Prognostic factors
        Case reports
        Treatment
        • Stage I: surgery is the primary form of treatment (Med Oncol 2020;37:20)
          • Complete resection is curative at stage I or II
        • Stage II or higher: surgery + postoperative radiology or chemotherapy
        • Potential options: PDL1 inhibitors
        Gross description
        • Tends to be peripherally located versus central (Front Surg 2021;8:757085)
        • Well circumscribed with irregular borders
        • Not associated with bronchi
        Frozen section description
        • Similar pitfalls to those found with lymphoepithelioma-like carcinomas in other sites (Int J Surg Pathol 2020;28:872)
          • Can resemble lymphoid tissue on low power
          • Epithelial component can look histiocytic
        Microscopic (histologic) description
        • Syncytial growth pattern
        • Large polygonal cells with vesicular nuclei, prominent eosinophilic nucleoli and variably abundant eosinophilic cytoplasm
        • Variable mitosis
        • Variable lymphoplasmacytic infiltrate
        • Reported to also have granulomatous inflammation, focal keratinization and lepidic spreading pattern (Am J Surg Pathol 2019;43:211)
        Microscopic (histologic) images

        Contributed by Heather I-Hsuan Chen-Yost, M.D.

        Lymphoid-like lesion

        Lymphoid infiltrate and syncytial cells

        EBER ISH stain

        p63 immunostain

        CK5/6 immunostain

        Virtual slides

        Images hosted on other servers:

        Wedge excision of 74 year old woman

        Left upper lobe lung resection

        Cytology description
        • Spindle cells are arranged in large cohesive clusters with admixed small lymphocytes (Cytopathology 2019;30:653)
        • Nuclei: pleomorphic, oval, prominent nucleoli
        • Can mimic melanoma, synovial sarcoma
        Positive stains
        Molecular / cytogenetics description
        • TRAF3 makes up ~80% of deletion mutations in lymphoepithelial carcinoma and 5% of simple somatic mutations (Nat Commun 2019;10:3108)
        • Does not show C:G to A:T transversions from tobacco smoking
        • High PDL1 expression (Oncotarget 2015;6:33019)
        • Rarely shows the typical driver mutation for conventional non-small cell lung cancer, such as TP53, KRAS, EGFR mutations or ALK and ROS1 translocations
        Sample pathology report
        • Lung, right lower lobe, lobectomy:
          • Lymphoepithelial carcinoma, 2.3 cm (see synoptic report)
        Differential diagnosis
        Additional references
        Board review style question #1


        Which of the following is most commonly associated with the lung lesion pictured above?

        1. EBV infection
        2. History of autoimmune disease
        3. Radiation exposure
        4. Smoking history
        5. Young age
        Board review style answer #1
        A. EBV infection. Lymphoepithelial carcinoma is a rare subtype of squamous cell carcinoma of the lung that is strongly associated with EBV infection. It tends to be associated with Asian patients in their 50s. Unlike other lung primary carcinomas, it is not known to be associated with a smoking history. It has also not been shown to be associated with radiation exposure or history of autoimmune disease. Radiation exposure can be associated with a differential diagnosis for Hodgkin lymphoma; however, the epithelial cells do not resemble Reed-Sternberg cells and are cohesive. History of autoimmune disease can be associated with lymphoid interstitial pneumonia, which also presents with lymphoid follicles but there will be a background of alveolar disruption and loose epithelioid granulomas. Furthermore, other inflammatory cells such as giant cells and macrophages will also be seen.

        Comment Here

        Reference: Lymphoepithelial carcinoma

        Lymphoepithelioma-like
        Definition / general
        Essential features
        • Pulmonary lymphoepithelioma-like carcinoma (LELC) is an epithelial malignancy characterized by large, syncytial cells with vesicular nuclei and prominent nucleoli, with a prominent lymphocytic infiltrate
        • LELC has a strong association with Epstein-Barr virus (EBV) and is more common in Asian populations
        • LELC shows similar morphologic features to nasopharyngeal carcinoma and may be mistaken for lymphoma due to prominent inflammatory infiltrate
        Terminology
        • Lymphoepithelioma-like carcinoma (LELC) primary to lung first described in 1987 by Bégin et al. (J Surg Oncol 1987;36:280)
        • Previously considered subtype of large cell undifferentiated carcinoma
        • By 2015 WHO, now classified under epithelial tumor, "other and unclassified carcinomas" (J Thorac Oncol 2015;10:1243)
        ICD coding
        • Use code specific for location of tumor
        • C34.90 Malignant neoplasm of unspecified part of unspecified bronchus or lung
        Epidemiology
        Sites
        • Intrapulmonary, either peripheral or central
        Pathophysiology
        Etiology
        Clinical features
        • Nonspecific: cough, dyspnea, chest pain, weight loss
        Diagnosis
        • Exclude metastatic nasopharyngeal carcinoma and other non small cell carcinomas
        • Presence of prominent inflammatory infiltrate helpful
        • Ancillary studies for EBV infection
        Laboratory
        • EBV serology often reveals prior infection, with higher titers associated with larger tumor size and higher stage (Am J Surg Pathol 2002;26:715)
        Radiology description
        • Similar to imaging characteristics of other primary lung cancers
        • Solitary, peripheral pulmonary nodule with direct contact with pleura (J Thorac Imaging 2014;29:246)
        • FDG PET / CT with strong FDG uptake
        Radiology images

        Images hosted on other servers:
        Various images Various images

        Various images

        Prognostic factors
        • Good prognosis: early tumor stage, normal serum lactate dehydrogenase level, normal serum albumin level, no lymph node metastasis
        • Those who underwent complete resection had better overall survival (Cancer 2012;118:4748)
        • Tumor recurrence and necrosis (5% or more of tumor) associated with poor prognosis (Am J Clin Pathol 2001;115:841)
        Case reports
        Treatment
        • Complete surgical resection for early stage disease, with adjuvant chemotherapy and radiotherapy in advanced cases (J Thorac Dis 2017;9:123)
        Gross description
        • Variable size, can be > 10 cm
        • Well circumscribed, lobulated, solid nodule
        • Necrosis common
        Gross images

        Images hosted on other servers:
        Excised mass

        Excised mass

        Microscopic (histologic) description
        • Anastamosing islands, nests, cords or diffuse sheets of tumor cells
        • Syncytial growth of monomorphous, polygonal epithelial cells with large vesicular nuclei, prominent eosinophilic nucleoli, accompanied by marked CD8+ lymphocytic infiltration
        • Admixed lymphocytes, plasma cells, histiocytes, occasional neutrophils or eosinophils
        • Predominantly pushing borders, permeative interface with adjacent lung
        • Intratumoral amyloid deposition in a few cases (Am J Surg Pathol 2002;26:715)
        Microscopic (histologic) images

        Contributed by Roseann Wu, M.D., M.P.H.
        Syncytial growth of monomorphous, polygonal epithelial cells Syncytial growth of monomorphous, polygonal epithelial cells

        Syncytial growth of monomorphous, polygonal epithelial cells

        EBER+

        EBER+



        Images hosted on other servers:
        Various images Various images Various images Various images

        Various images


        Various images Various images Various images

        Various images

        Cytology description
        • Large clusters of neoplastic cells with scant cytoplasm, large hyperchromatic nuclei, irregular nuclear contours, prominent nucleoli, brisk mitotic figures and prominent intratumoral lymphoid infiltration (Diagn Cytopathol 2012;40:820)
        • Stripped, naked tumor nuclei
        • Interspersed lymphocytes, some with crush artifact
        Positive stains
        Negative stains
        Molecular / cytogenetics description
        Differential diagnosis
        Board review style question #1
        Lymphoepithelioma-like carcinoma of the lung has been linked to which pathogen, particularly in Asian populations?

        1. Epstein-Barr virus (EBV)
        2. Human immunodeficiency virus (HIV)
        3. Human papillomavirus (HPV)
        4. Human T lymphotropic virus (HTLV)
        5. Respiratory syncytial virus (RSV)
        Board review style answer #1
        A. Epstein-Barr virus (EBV)

        Comment Here

        Reference: Lymphoepithelioma-like carcinoma

        Lymphoid interstitial pneumonia
        Definition / general
        Essential features
        • Rare type of interstitial lung disease due to different diseases including Sjögren syndrome, rheumatoid arthritis and human immunodeficiency virus (HIV) infection
        • On histology, diffuse infiltration of polyclonal lymphocytes with scant interstitial fibrosis is characteristic
        Terminology
        • Also called lymphocytic interstitial pneumonia
        ICD coding
        • J84.2: lymphoid interstitial pneumonia
        Epidemiology
        • Rare
        • Typical onset at ages 40 - 70 years old but can occur at any age (Chest 2002;122:2150)
        • More common in women
        • No association with smoking history
        Sites
        • Bilateral lower lobes of the lung
        Pathophysiology
        • Pathogenic mechanisms of LIP are still unclear
        • Has aspects of lymphoproliferative disease and lymphoid hyperplasia of polyclonal T or B cells (Chest 2002;122:2150)
        • Although it may transform to lymphoma, especially MALT, the risk is lower than initially reported (Eur Respir J 2006;28:364)
        Etiology
        • Associated with several systemic diseases and conditions (Chest 2002;122:2150, Respirology 2016;21:600, Eur Respir J 2006;28:364)
          • Autoimmune (most common)
            • Sjögren syndrome (SjS); 25% of LIP cases have SjS and 1% of SjS cases present with LIP
            • Rheumatoid arthritis
            • Systemic lupus erythematosus
            • Polymyositis / dermatomyositis
            • Hashimoto disease
            • Hypothyroidism
            • Castleman disease
            • Myasthenia gravis
            • Autoimmune hemolytic anemia
            • Pernicious anemia
            • Primary biliary cirrhosis
          • Infection
            • Human immunodeficiency virus (HIV)
            • Epstein-Barr virus
            • Human T cell lymphotropic virus type 1
            • Legionella pneumonia
            • Mycoplasma
            • Chlamydia
            • Tuberculosis
          • Immunodeficiency
            • Acquired immunodeficiency syndrome (AIDS); especially in children
            • Monoclonal or polyclonal gammopathy
            • Common variable immunodeficiency
          • Idiopathic LIP accounts for 20% of cases (Eur Respir J 2006;28:364)
        Clinical features
        • Very slowly progressive respiratory symptoms
          • Dyspnea on exertion
          • Dry cough
          • Systemic symptoms such as malaise, fever and weight loss
          • Duration of the symptoms prior to diagnosis can exceed a year
        • Bibasilar inspiratory crackles on chest auscultation
        Diagnosis
        • Based on clinical, radiological and pathological findings (multidisciplinary diagnosis)
        • No firm diagnostic criteria currently exist
        Laboratory
        • Dysproteinemia is often present
          • Hypergammaglobulinemia is more common than hypogammaglobulinemia
        • Restrictive pattern on pulmonary function tests
          • Reduced forced vital capacity (FVC)
          • Reduced diffusing capacity of the lung for carbon monoxide (DLCO)
        Radiology description
        • Chest radiography
          • Bibasilar opacities with lower lobe predominance
        • High resolution computed tomography (Eur J Radiol 2015;84:542, Respirology 2016;21:600)
          • Ground glass opacity with / without consolidation with lower lobe predominance
          • Cyst formation and thickening of bronchovascular bundle and interlobular septa are often present
          • Cysts often remain even after resolution of symptoms
        Radiology images

        Images hosted on other servers:
        Ground glass opacity with cyst formation and nodules

        Ground glass opacity with cyst formation and nodules

        Prognostic factors
        Case reports
        Treatment
        • No treatment data from a controlled study is available so far (Respirology 2016;21:600)
          • Corticosteroid therapy is commonly used as a first line treatment and improves the symptoms in most cases
          • Immunosuppression (eg, cyclophosphamide, azathioprine, cyclosporine A) may be used as a second line
        • Treatment for underlying disease is also essential for secondary LIP
        Gross description
        • Ill defined lesion
        • Mild increase in lung weight
        Microscopic (histologic) description
        • Diffuse interstitial infiltration of polymorphous lymphocytes and plasma cells
          • Lymphoid follicles with germinal centers, histiocytes and macrophages are often present
          • Bronchovascular bundles and interlobular septa are usually involved
          • Alveolar structure is often inflated and disrupted
          • Typically CD8+ or CD4+ T cells or B cells predominate, with an admixture of other lymphocytes
          • Immunohistochemistry or molecular testing are necessary to confirm polyclonality
        • Additional findings
          • Loose ill defined epithelioid granulomas
          • Interstitial or intra-alveolar giant cells
          • Intra-alveolar macrophages
          • Type II pneumocyte hyperplasia
          • Cyst formation without marked fibrosis
        • Pertinent negative findings
          • Loose and dense fibrosis (more common in fibrotic / cellular nonspecific interstitial pneumonia (NSIP))
          • Fibroblastic focus (more common in usual interstitial pneumonia or fibrotic NSIP)
          • Honeycomb change (more common in usual interstitial pneumonia or fibrotic NSIP)
          • Organizing pneumonia (more common in hypersensitivity pneumonitis)
        • See: Respirology 2016;21:600, Chest 2002;122:2150, Am J Respir Crit Care Med 2002;165:277
        Microscopic (histologic) images

        Contributed by Akira Yoshikawa M.D. and Yale Rosen, M.D.
        Low power

        Low power

        Bronchovascular bundle

        Bronchovascular bundle

        Architectural destruction

        Architectural destruction

        Septal thickening

        Septal thickening

        Massive lymphocytic infiltration

        Massive lymphocytic infiltration

        Giant cells and lymphocytes

        Giant cells and lymphocytes


        Lymphocytes and histiocytes

        Lymphocytes and histiocytes

        Granuloma and lymphocytes

        Granuloma and lymphocytes

        Lymphoid aggregate and histiocytes

        Lymphoid aggregate and histiocytes

        Diffuse cellular pattern

        Diffuse cellular pattern

        Lymphoplasmacytic infiltration

        Lymphoplasmacytic infiltration


        Lymphoplasmacytic infiltration Lymphoplasmacytic infiltration

        Lymphoplasmacytic infiltration



        Images hosted on other servers:
        LIP

        LIP

        Numerous lymphoid follicles Numerous lymphoid follicles

        Numerous lymphoid follicles

        EBV related LIP

        EBV related LIP

        Cytology description
        • Increase in the number of lymphocytes (especially CD8 positive) of bronchoalveolar lavage, without clonality (Eur Respir J 2006;28:364)
        Positive staining - disease
        Molecular / cytogenetics description
        • EBNA ISH and EBER ISH are helpful to detect EBV infection
        Differential diagnosis
        • Follicular bronchitis / bronchiolitis
          • Lymphocytic infiltration into bronchial / bronchiolar walls with multiple lymphoid follicles
          • No or slight lymphocytic aggregation in intralobular septa
        • Hypersensitivity pneumonitis (HP)
          • History of an antigen exposure such as animals, birds and chemicals
          • Less diffuse but bronchocentric distribution
          • Loose ill defined granuloma and giant cells can be seen in both LIP and HP
        • Lymphoma, especially mucosa (bronchus) associated lymphoid tissue lymphoma
          • Monomorphous infiltration of lymphocytes, distortion of alveolar architecture, Dutcher bodies, pleural infiltration are more common in lymphoma
          • Immunohistochemistry is often helpful
        • Nonspecific interstitial pneumonia
          • Lymphoplasmacytic infiltration is also seen in NSIP but less severe
          • Alveolar structure is usually preserved compared with LIP
        Board review style question #1


        Which of following is not usually seen in this entity?

        1. Cyst formation
        2. Giant cells
        3. Honeycomb change
        4. Lymphocytic interstitial infiltration
        5. Nonnecrotizing granuloma
        Board review style answer #1
        C. Honeycomb change. Lymphoid interstitial pneumonia is predominantly a cellular interstitial pneumonia. Fibrotic processes such as dense fibrosis or honeycomb change usually exclude a diagnosis of LIP.

        Comment Here

        Reference: Lymphoid interstitial pneumonia

        Lymphomatoid granulomatosis
        Definition / general
        • Also called angioimmunoproliferative lesion
        • Lymphoproliferative disorder that is malignant per se, or likely to become malignant, resembles post-transplant lymphoproliferative disorders
        Clinical features
        • Middle aged men and women with well defined, bilateral rounded mass densities on chest X-ray
        • Associated with transplant-related immunosuppression, Sjogren's syndrome, HIV
        • 80% of cases have extrapulmonary involvement (skin, CNS, kidneys, liver, spleen, adrenal glands, heart, GI tract); rarely, there is only extrapulmonary involvement
        • Usually no hilar or mediastinal lymph node involvement
        • May be an EBV related, B-cell proliferation; T-cells present are usually reactive; may have reversal of T-helper / suppressor ratio
        • Large number of atypical lymphoid cells is a poor prognostic factor
        • Median survival 14 months; death due to sepsis, destruction of lung tissue
        Treatment
        • Chemotherapy
        Microscopic (histologic) description
        • Nodular inflammatory infiltrate of large atypical lymphoid cells (prominent nuclei, mitotic activity), plasma cells, immunoblasts, involving walls of pulmonary vessels (angioinvasive, angiocentric, angiodestructive)
        • No multinucleated giant cells
        • Similar infiltrates found in other organs
        Positive stains
        • EBV (50 - 70%)
        Differential diagnosis

        Malakoplakia
        Definition / general
        • Usually in urinary tract; very rare in lung, usually in immunocompromised patients
        • Caused by macrophage dysfunction involving intracellular lysosomal mediated killing of bacteria
        • In AIDS patients, usually due to Rhodococcus equi, an animal pathogen causing opportunistic infections (Chest 1999;115:889)
        Case reports
        Treatment
        • Antibiotics
        Gross description
        • Nodular masses or infiltrates with cavitation
        Gross images

        Images hosted on other servers:
        Grayish white, ill<br>defined lesion with<br>central cavitation

        Grayish white, ill
        defined lesion with
        central cavitation

        Microscopic (histologic) description
        • Intraalveolar histiocytes with pink or foamy cytoplasm (von Hansemann’s histiocytes) that fill and destroy alveoli, not interstitium
        • Histiocytes contain PAS+ bacteria, also Michaelis-Gutmann bodies (round / oval structures, usually intracellular, 5 - 20 microns, with laminated or targetoid appearance that stain deeply with H&E, iron and calcium stains)
        • Also lymphocytes, plasma cells and neutrophils
        Microscopic (histologic) images

        Images hosted on other servers:
        Various images Various images

        Various images

        Differential diagnosis

        Measles
        Definition / general
        • Also called rubeola
        • Infectious acute febrile illness characterized by upper respiratory infection symptoms and maculopapular rash
        • Spread by aerosols and droplets from respiratory secretions or rarely fomites
        • Significant public health problem in United States prior to the development of an effective vaccine; still common in some nonindustrialized countries
        • Recently, recrudescence has occurred in developed world due to some parents refusing to vaccinate their children and importation (Cleve Clin J Med 2010;77:207)
        • Highly infectious, affecting up to 90% of individuals in nonimmune populations
        • Also occurs in adults (MMWR Morb Mortal Wkly Rep 2011;60:666)
        • Pneumonia is serious complication of childhood exanthema
        • May cause bronchiolitis or diffuse alveolar damage
        • Secondary bacterial pneumonia may occur
        Microscopic (histologic) description
        • Epithelial giant cells with viral inclusions
        • Nuclei may contain a single large Cowdry type A inclusion
        • Reticuloendothelial (Warthin-Finkeldey) giant cells may be found in lymphoid tissue
        Microscopic (histologic) images

        Images hosted on other servers:
        Giant cells

        Giant cells

        Videos


        Histopathology Lung - Measles pneumonia
        Differential diagnosis

        Meningioma (pending)
        [Pending]

        Metastases
        Definition / general
        • Lung is a common site of metastases; usually multiple, bilateral, sharply outlined, rapidly growing, more pleomorphic and necrotic than lung primaries
        • May appear as multiple discrete nodules in periphery of lung or as lymphangitic carcinomatosis (peribronchial and perivascular patterns via lymphatics)
        • Rarely appear as intralymphatic microscopic foci that cause pulmonary hypertension
        • Metastases can also be from other lung primaries (Am J Surg Pathol 2009;33:1752)
        Essential features
        • Lung is a common site of metastatic disease and may be the first or only site of metastatic involvement
        • Lung metastases are generally multiple, well circumscribed and tend to grow rapidly
        • A history of malignancy is helpful in determining the primary site but a panel of immunohistochemical stains can help support the diagnosis
        ICD coding
        • ICD-10: C78.00 - Secondary malignant neoplasm of unspecified lung
        • ICD-10: C78.01 - Secondary malignant neoplasm of right lung
        • ICD-10: C78.02 - Secondary malignant neoplasm of left lung
        Epidemiology
        • In autopsy studies, 20 - 50% of patients with malignancy had lung metastasis (Cancer 1981;47:2595)
        Sites
        • Generally peripheral lung but can also be endobronchial
        Pathophysiology
        Etiology
        • Mostly hematogenous or lymphogenous spread from primary site
        Clinical features
        • Usually asymptomatic or may present with nonspecific cough, chest pain, hemoptysis, spontaneous pneumothorax (J Surg Oncol 2014;109:42)
        Diagnosis
        • Typically detected by imaging, specifically CT scans
        • Confirmation by cytologic or histologic sampling
        Radiology description
        • CT scans are preferred modality
        • Single or multiple, peripheral, well circumscribed nodule(s)
        • Limited detection of subcentimeter disease
        Radiology images

        Contributed by Fulvio Lonardo, M.D.
        Pancreas: adenocarcinoma

        Pancreas: adenocarcinoma



        Images hosted on other servers:
        Malignant melanoma<br>shows pulmonary<br>nodules

        Malignant melanoma
        shows pulmonary
        nodules

        Multiple round nodules and masses in both lungs

        Multiple round nodules and masses in both lungs

        Cannonball lung metastases in patient with colon cancer

        Cannonball lung metastases in patient with colon cancer

        Multiple lung metastases

        Multiple lung metastases

        Prognostic factors
        • Clinical variables associated with prolonged survival after lung metastasectomy in colorectal cancer patients
          • Prolonged disease free interval between primary tumor and metastasis, normal prethoracotomy carcinoembryonic antigen, absence of thoracic node involvement and single pulmonary lesion (Ann Surg Oncol 2013;20:572)
        • Complete resection and longer disease free interval > 6 months associated with better survival (Surg Oncol 2012;21:237)
        Case reports
        Treatment
        • Chemotherapy, immunotherapy, other systemic therapy
        • Criteria for pulmonary metastasectomy: primary site of disease controlled, complete resection of lung metastasis feasible, patient able to tolerate procedure, no better alternative treatments (J Surg Oncol 2014;109:42)
        • Stereotactic body radiation therapy (SBRT) and radiofrequency ablation (RFA) may be considered for small, solitary, peripheral tumors
        Gross description
          Patterns associated with specific primaries
        • Central cavitation: colonic adenocarcinoma, leiomyosarcoma, squamous cell carcinoma of upper aerodigestive tract
        • Intrabronchial masses: breast, colon, kidney
        • Lymphangitic carcinomatosis: breast, choriocarcinoma, pancreas, prostate, stomach
        • Nodular metastases: breast, GI, kidney, melanoma, sarcoma
        • Tumor emboli: breast, choriocarcinoma, liver, stomach
        Gross images

        Contributed by Roseann Wu, M.D., M.P.H.
        Lung wiht metastatic tumor

        Lung with metastatic tumor



        Images hosted on other servers:
        Multiple tumor nodules Multiple tumor nodules

        Multiple tumor nodules

        Microscopic (histologic) description
        • Dependent on site of origin (colorectal cancer, bone and soft tissue sarcoma, renal cell carcinoma, melanoma, head and neck tumors, germ cell tumors and many others)
        • Evidence for breast metastasis over lung primary: comedonecrosis, solid nests, trabecular architecture, cribriform growth pattern (Am J Clin Pathol 2009;131:122)
        Microscopic (histologic) images

        Contributed by Roseann Wu, M.D., M.P.H., Debra Zynger, M.D., Case #318 and Fulvio Lonardo, M.D.
        Metastatic colon CA to lung 25x

        Metastatic colon CA to lung 25x

        Clear cell renal<br>cell carcinoma<br>metastatic to lung

        Clear cell renal
        cell carcinoma
        metastatic to lung

        Prostatic<br>adenocarcinoma<br>metastatic to lung

        Prostatic
        adenocarcinoma
        metastatic to lung

        Testicular germ<br>cell tumor (teratoma)<br>metastatic to lung

        Testicular germ
        cell tumor (teratoma)
        metastatic to lung

        Jaw: ameloblastoma<br>metastatic to lung

        Jaw: ameloblastoma
        metastatic to lung


        Pancreas: adenocarcinoma Pancreas: adenocarcinoma

        Pancreas: adenocarcinoma

        Cytology images

        Contributed by Roseann Wu, M.D., M.P.H.
        Metastatic renal cell carcinoma to lung

        Metastatic renal cell carcinoma to lung



        Images hosted on other servers:
        Bladder, urothelial carcinoma

        Bladder, urothelial carcinoma

        Positive stains
        Negative stains
        • TTF1 and Napsin A may be helpful to exclude primary lung adenocarcinoma
        Differential diagnosis
        • Primary lung carcinoma: acini, lepidic growth, nuclear pseudoinclusions, central scar (Am J Clin Pathol 2009;131:122)
        • Inflammatory nodule or benign neoplasm
        Board review style question #1
          For which malignancy is pulmonary metastasectomy most commonly performed and reported?

        1. Colorectal cancer
        2. Germ cell tumors
        3. Melanoma
        4. Osteosarcoma
        5. Renal cell carcinoma
        Board review style answer #1

        Micronodular pneumocyte hyperplasia
        Definition / general
        Essential features
        • Rare pulmonary manifestation of tuberous sclerosis complex that may occur in conjunction with LAM
        • Composed of multiple nodules of enlarged but benign type II pneumocytes that may mimic atypical adenomatous hyperplasia
        • Positive for cytokeratins and TTF1 by immunohistochemical stains, while negative for SMA
        Terminology
        • Also called multifocal micronodular pneumocyte hyperplasia (MMPH)
        • Older terminology: acinar atypical adenomatoid proliferation of epithelium, multiple adenomatoid lesions, micronodular hyperplasia of type II pneumocytes
        Epidemiology
        • Young and middle aged adults, predominantly women
        • Often associated with tuberous sclerosis complex but may be sporadic
        Pathophysiology
        • Functional loss of TSC 1 or TSC2 and hyperphosphorylation of mTOR related protein may cause this benign neoplastic proliferation of pneumocytes (Mod Pathol 2010;23:1251)
        Clinical features
        • Non-specific; may present with pneumothorax, shortness of breath, cough
        Diagnosis
        • Histologic examination
        Radiology description
        Prognostic factors
        • Typically indolent with no clinical significance
        • Rare case studies report respiratory failure
        Case reports
        Gross description
        • Small (1 - 10 mm), well-demarcated, randomly distributed parenchymal lung nodules
        Gross images

        Images hosted on other servers:
        Figure A: Macroscopic examination<br>showing white tinged lesions

        Figure A: Macroscopic examination
        showing white tinged lesions

        Microscopic (histologic) description
        • Multiple, sharply demarcated or circumscribed nodules
        • Tubulopapillary proliferation of type II pneumocytes lining fibrotic and thickened alveolar septa, moderate lymphocytic infiltration, no nuclear atypia (Int J Surg Pathol 2010;18:522)
        • Enlarged type II pneumocytes with abundant eosinophilic cytoplasm, vesicular nuclei, distinct nucleoli, occasional eosinophilic inclusions
        • Increased alveolar macrophages
        Microscopic (histologic) images

        Contributed by Roseann Wu, M.D., M.P.H.
        Micronodular pneumocyte hyperplasia

        Micronodular pneumocyte hyperplasia

        TTF1 stain

        TTF1 stain



        Images hosted on other servers:
        Morphology of pulmonary lesions

        Morphology of pulmonary lesions

        Multifocal micronodular pneumocyte hyperplasia

        Multifocal micronodular pneumocyte hyperplasia

        Positive stains
        • Pan-cytokeratin, TTF1, EMA, surfactant apoproteins A and B
        Negative stains
        Electron microscopy description
        • Osmiophilic lamellar inclusions and surface microvilli of type II pneumocytes
        Molecular / cytogenetics description
        Differential diagnosis
        Additional references

        Minimally invasive
        Definition / general
        Essential features
        • Tumor with noninvasive (lepidic) and invasive components
        • Total tumor size must be less than 3 cm
        • Invasive or nonlepidic components must be less than 0.5 cm
        • Invasive component can be of any subtype other than lepidic
        • No lymphovascular invasion, tumor necrosis or pleural invasion is present
        • Usually nonmucinous but can rarely occur in mucinous tumors
        Terminology
        • Minimally invasive adenocarcinoma (MIA)
        • Microinvasive adenocarcinoma
        • Older terminology used: bronchoalveolar carcinoma; however, this is obsolete and should no longer be utilized
        ICD coding
        • ICD-O: 8250/2 - minimally invasive adenocarcinoma, nonmucinous (international code)
        • ICD-O: 8257/3 - minimally invasive adenocarcinoma, mucinous (international code)
        • ICD-10: C34.9 - malignant neoplasm of unspecified part of unspecified bronchus or lung
        • ICD-11: XH3QM0 - minimally invasive adenocarcinoma, nonmucinous
        • ICD-11: XH2098 - minimally invasive adenocarcinoma, mucinous
        Epidemiology
        Sites
        Pathophysiology
        Etiology
        Clinical features
        Diagnosis
        Radiology description
        Radiology images

        Images hosted on other servers:
        Ground glass lesion on CT scan

        Ground glass lesion on CT scan

        Prognostic factors
        Case reports
        Treatment
        Gross description
        • Peripheral, ill defined, firm, white tumor with solid areas; ill defined areas correspond to lepidic growth pattern and solid areas correspond to invasive component (Hum Pathol 2016;51:41)
        • Necrosis and frank pleural invasion should be absent
        Gross images

        Contributed by Sarah Aziz, M.D.
        Lobectomy specimen

        Lobectomy specimen

        Microscopic (histologic) description
        • Primary lung adenocarcinoma measuring ≤ 3 cm in greatest dimension, with ≤ 0.5 cm area of either stromal invasion or nonlepidic growth pattern (acinar, micropapillary, papillary, solid, colloid, fetal or invasive mucinous patterns)
        • Foci of stromal invasion characterized by angulated glands, desmoplastic stroma and increased cytologic atypia (Cancer 1995;75:2844)
        • Size of invasion should be measured as the largest focus of invasion (Am J Surg Pathol 2014;38:448, Arch Pathol Lab Med 2013;137:685)
        • If there are multiple foci of invasion, the invasive size can be measured as a sum of the percentage of the invasive components in each section multiplied against the greatest tumor dimension (the aggregate invasive size should be ≤ 0.5 cm to render the diagnosis of a minimally invasive adenocarcinoma) (Am J Surg Pathol 2014;38:448, Surg Today 2019;49:828)
        • Necrosis, lymphovascular invasion, pleural invasion and spread through air spaces must be absent
        Microscopic (histologic) images

        Contributed by Jonathan Keow, M.D., Ph.D.
        Noninvasive and invasive components

        Noninvasive and invasive components

        Noninvasive lepidic adenocarcinoma

        Noninvasive lepidic adenocarcinoma

        Focus of invasive adenocarcinoma

        Focus of invasive adenocarcinoma

        Negative stains
        Sample pathology report
        • Lung, right lower lobe, lobectomy:
          • Minimally invasive adenocarcinoma, pT1mi NX MX (see synoptic report)
        Differential diagnosis
        Additional references
        Board review style question #1
        Which of the following tumors would be classified as a minimally invasive adenocarcinoma?

        1. 2 cm lepidic predominant tumor with a 0.8 cm invasive component
        2. 2 cm lepidic predominant tumor with a 0.4 cm invasive component
        3. 2 cm lepidic predominant tumor with a 0.4 cm invasive component with pleural and lymphovascular invasion
        4. 5 cm lepidic predominant tumor with a 0.4 cm invasive component
        Board review style answer #1
        B. 2 cm lepidic predominant tumor with a 0.4 cm invasive component

        Comment Here

        Reference: Adenocarcinoma - minimally invasive

        Minute pulmonary meningothelial-like nodules
        Definition / general
        • Uncommon incidental pulmonary nodules composed of interstitial nodular proliferation of small oval or spindle shaped cells arranged in a zellballen nesting pattern (J Thorac Imaging 2002;17:227)
        Essential features
        • Generally detected incidentally in resected lung specimens (Hum Pathol 2009;40:678)
        • Occasionally detected on thin section computed tomography (Hum Pathol 2009;40:678)
        • More common in patients with malignant pulmonary tumors, especially lung adenocarcinoma, than benign disease (Hum Pathol 2009;40:678)
        • No significant difference in clinicopathologic factors between patients with single and multiple nodules, except for the size of each nodule (Hum Pathol 2009;40:678)
        Terminology
        • Pulmonary chemodectomas were first described by Korn et al. in 1960 (Am J Pathol 1960;37:641)
        • In 1988, Gaffey et al. proposed to change the name of pulmonary chemodectomas to minute meningothelial-like nodules (Am J Surg Pathol 1988;12:167)
        • Terminology includes
          • Pulmonary meningothelial-like nodules (PMLNs)
          • Minute pulmonary meningothelial-like nodules (MPMNs)
          • Pulmonary chemodectoma
          • MPMN-omatosis syndrome
          • Diffuse pulmonary meningotheliomatosis (DPM)
        ICD coding
        • ICD-10: R91.1 - solitary pulmonary nodule
        Epidemiology
        • F > M (10.7% versus 4.5%) (Hum Pathol 2009;40:678)
        • Most frequent in the sixth decade
        • Absent in fetuses, infants and children, indicating that they do not represent congenital rests
        • Reported incidence is 0.3 to 9.5% at autopsy or in surgical specimens (Korean J Pathol 2012;46:87)
        Sites
        Pathophysiology
        Etiology
        Clinical features
        Diagnosis
        • Often identified as incidental findings in histopathological lung sections or in high resolution CT scans conducted for unrelated causes (Korean J Pathol 2012;46:87)
        Radiology description
        • Thin section chest CT reveals randomly distributed, well defined ground glass appearance micronodules, which may simulate metastatic disease (J Comput Assist Tomogr 2001;25:311)
        Radiology images

        Images hosted on other servers:

        CT scan with
        1 cm nodule

        Positron emission tomography

        Chest film shows bilateral diffuse nodules

        CT scan shows nodules

        Prognostic factors
        Case reports
        Treatment
        Gross description
        Microscopic (histologic) description
        • Characteristic nests of bland spindle cells with pale eosinophilic cytoplasm expand alveolar septa
        • Larger lesions connected by intervening collagen, often imparting a stellate configuration
        • Smaller lesions have closely apposed nests with mildly thickened alveolar septa (Am J Surg Pathol 1988;12:167)
        Microscopic (histologic) images

        Contributed by Jian-Hua Qiao, M.D.
        Interstitial nests of spindle cells Interstitial nests of spindle cells

        Interstitial nests of spindle cells

        CD56

        CD56

        EMA

        EMA


        PR

        PR

        Vimentin

        Vimentin

        AE1 / AE3

        AE1 / AE3

        CK5 / 6

        CK5 / 6

        Virtual slides

        Images hosted on other servers:

        Pulmonary meningothelial-like nodule

        Cytology description
        • Single as well as clusters of bland spindle cells are present in aspirate smears
        • However, marked cellular atypia is often seen with cytology diagnosis of atypical / suspicious for malignancy / positive for malignancy
        • Immunohistochemistry of cell block will rule out the possibility of carcinoma or squamous cell carcinoma since the spindle cells are negative for pankeratin (AE1 / AE3), p63 and p40
        Cytology images

        Contributed by Jian-Hua Qiao, M.D.
        Papanicolaou (Pap) stain Papanicolaou (Pap) stain Papanicolaou (Pap) stain

        Papanicolaou (Pap) stain

        p63

        p63

        p40

        p40

        Electron microscopy description
        • Ultrastructural features include interdigitating cellular junctions, desmosomes and scattered intracytoplasmic filaments (Korean J Pathol 2012;46:87)
        Electron microscopy images

        Images hosted on other servers:

        Interdigitating cells and desmosomes

        Molecular / cytogenetics description
        • Mutational analyses were performed on microdissected tissue using 20 polymorphic microsatellite markers targeting 11 genomic regions in an effort to identify genetic similarities of MPMN and meningioma (Am J Surg Pathol 2004;28:207)
        • Loss of heterozygosity was identified in 25% of single MPMN affecting 3 genomic loci
          • No solitary MPMN had > 1 loss of heterozygosity event
          • Multiple loss of heterozygosity were seen only in diffuse pulmonary meningotheliomatosis (MPMN-omatosis syndrome), where 33.3% of MPMNs showed loss of heterozygosity affecting 7 genomic loci
          • Meningioma showed the highest frequency of loss of heterozygosity with major events seen at 22q (60%), 14q (42.8%) and 1p (44.4%) that were not shared by MPMN
          • MPMNs are different from meningioma based on the major molecular genetic events seen in their formation and progression
        • NF2 deletion similar to meningioma found in a limited series (Oncotarget 2018;9:36012)
        Sample pathology report
        • Left lung, mass, CT guided needle core biopsy:
          • Minute pulmonary meningothelial-like nodule
          • Negative for malignancy
          • Adjacent lung parenchyma with mild interstitial fibrosis
        Differential diagnosis
        Additional references
        Board review style question #1
        An 80 year old Caucasian woman had a thyroid mass and a 1 cm pulmonary lesion. CT guided transthoracic needle biopsy was performed to rule out metastatic tumor. Microscopic examination of core needle biopsy tissue reveals the following lesion. What is your diagnosis?



        1. Metastatic thyroid papillary carcinoma
        2. Minute pulmonary meningothelial-like nodule
        3. Pulmonary carcinoid tumor
        4. Pulmonary meningioma
        5. Pulmonary paraganglioma
        Board review style answer #1
        B. Minute pulmonary meningothelial-like nodule. Microscopic lesion is classic of pulmonary meningothelial-like nodule, with irregular and uncircumscribed interstitial nests of spindle cells, morphologically similar to meningothelial cells.

        Comment Here

        Reference: Minute pulmonary meningothelial-like nodules
        Board review style question #2
        Sometimes, immunohistochemistry is necessary to work up this spindle cell lesion. Pulmonary meningothelial-like nodule is most likely positive for which of the following immunostains?

        1. CD56
        2. CK5 / 6
        3. ER
        4. Pankeratin (AE1 / AE3)
        5. Synaptophysin
        Board review style answer #2
        A. CD56. Pulmonary meningothelial-like nodule is positive for CD56, EMA, progesterone receptor / PR and vimentin.

        Comment Here

        Reference: Minute pulmonary meningothelial-like nodules

        Mucoepidermoid carcinoma
        Definition / general
        • Low grade or high grade salivary gland type tumor with mucous secreting cells, squamous cells and intermediate cells
        • Distinguished from other lung cancers by central or peribronchial location, mucous cells, lack of keratinization, expression of p63 and MAML2 rearrangement (Mod Pathol 2014;27:1479)
        Essential features
        • Most common salivary gland type tumor in the lung, tends to arise centrally
        • Similar morphology to mucoepidermoid carcinoma arising in the head and neck with a mixture of mucous cells, squamous cells and intermediate cells; must exclude metastasis
        • Strong association with t(11;19)(q21;p13) and MAML2 rearrangement by fluorescence in situ hybridization (FISH)
        • Squamous component stains with p63 and p40
        Terminology
        • Same diagnostic terminology as used in the salivary gland
        ICD coding
        • C33 Malignant neoplasm of trachea
        • C34.00 Malignant neoplasm of unspecified main bronchus
        • C34.01 Malignant neoplasm of right main bronchus
        • C34.02 Malignant neoplasm of left main bronchus
        • Code more peripheral lesions depending on specific lobe, laterality and extent
        Epidemiology
        Sites
        • Any lung lobe, usually in relation to large bronchi
        Pathophysiology
        Etiology
        • May arise from submucosal bronchial glands
        Clinical features
        • Large central tumors cause obstructive symptoms such as dyspnea, cough, hemoptysis or pneumonia
        • Peripheral lesions may be asymptomatic
        Diagnosis
        • Bronchoscopy may be used to directly visualize and sample an endobronchial tumor
        • Diagnosis may be difficult in small biopsies; may mimic primary non small cell lung carcinoma
        Radiology description
        • Can be difficult to detect on imaging due to small size of tumor, endobronchial location, association with pneumonia or atelectasis (Arch Pathol Lab Med 2007;131:1400)
        • On CT, markedly enhanced homogeneous central bronchial nodule / mass suggests low grade mucoepidermoid carcinoma (MEC); high grade MEC tends to be peripheral with poorly defined margins, lobular, heterogeneous with less enhancement (AJR Am J Roentgenol 2015;205:1160)
        • Uncommon findings included cavitation, diffuse thickening or spiculation (Clin Imaging 2012;36:8)
        Radiology images

        Images hosted on other servers:
        Heterogeneously enhancing lesion

        Heterogeneously enhancing lesion

        Mass shadow measuring 30 mm

        Mass shadow measuring 30 mm

        Mass in right upper lobe and left hilum

        Mass in right upper lobe and left hilum

        Prognostic factors
        Case reports
        Treatment
        Clinical images

        Images hosted on other servers:
        Well circumscribed endobronchial tumor

        Well circumscribed endobronchial tumor

        Gross description
        • Polypoid, exophytic growth or sessile tumor in major bronchi, may be > 5 cm
        • Generally well circumscribed and smooth with tan to yellow cut surface
        • May be solid or cystic and potentially show glistening mucoid material
        Gross images

        Images hosted on other servers:
        Mucoepidermoid carcinoma of airways

        Mucoepidermoid carcinoma of airways

        Tumor in middle lobe bronchus

        Tumor in middle lobe bronchus

        Microscopic (histologic) description
        • Low grade has more cysts and mucous cells; high grade has more solid or nested growth, atypia, mitotic activity, necrosis, hemorrhage
        • Mucus secreting cells, squamoid cells without significant keratinization, intermediate type cells
        • Mucus secreting cells usually large with light blue-gray mucinous cytoplasm; variants include columnar, goblet, cuboidal, clear or oncocytic cells (Arch Pathol Lab Med 2007;131:1400)
        • Squamous cells show intercellular bridges but no keratin whorls or pearls; intermediate cells usually polygonal with bland nucleus (Arch Pathol Lab Med 2007;131:1400)
        • May show areas of papillary growth or spindled cells
        • May have dense lymphoplasmacytic infiltrate (Am J Surg Pathol 2005;29:407)
        • Lack in situ carcinoma of overlying epithelium
        Microscopic (histologic) images

        Contributed by Roseann Wu, M.D., M.P.H. and Yale Rosen, M.D.
        Low grade mucoepidermoid carcinoma

        Low grade mucoepidermoid carcinoma

        Low grade MEC mucicarmine stain

        Low grade MEC mucicarmine stain

        High grade mucoepidermoid carcinoma High grade mucoepidermoid carcinoma

        High grade mucoepidermoid carcinoma

        Mucoepidermoid carcinoma, low grade Mucoepidermoid carcinoma, low grade

        Mucoepidermoid carcinoma, low grade


        Stroma infiltrated by inflammatory cells

        Stroma infiltrated by inflammatory cells



        Images hosted on other servers:
        MEC of lung

        MEC of lung

        EGFR seen in squamous and intermediate cells

        EGFR seen in squamous and intermediate cells

        Pulmonary MEC Pulmonary MEC Pulmonary MEC

        Pulmonary MEC

        MEC-like pulmonary carcinoma

        MEC-like pulmonary carcinoma

        Virtual slides

        Images hosted on other servers:
        Mucoepidermoid carcinoma of the lung, from Juan Rosai’s Collection of Surgical Pathology Seminars Mucoepidermoid carcinoma of the lung, from Juan Rosai’s Collection of Surgical Pathology Seminars Mucoepidermoid carcinoma of the lung, from Juan Rosai’s Collection of Surgical Pathology Seminars Mucoepidermoid carcinoma of the lung, from Juan Rosai’s Collection of Surgical Pathology Seminars

        Mucoepidermoid carcinoma of the lung, from Juan Rosai’s Collection of Surgical Pathology Seminars

        Cytology description
        • Mixture of squamoid cells, mucous cells and intermediate cells
        • Diff-Quik with numerous tight clusters of relatively small and bland cells with well defined but scant cytoplasm, central round and uniform nuclei with small nucleoli; second population of glandular appearing cells with cytoplasmic vacuoles with mucin, focal extracellular metachromatic mucinous material (Diagn Cytopathol 2013;41:1096)
        Positive stains
        Negative stains
        Molecular / cytogenetics description
        • MAML2 rearrangement most common molecular genetic event, found more in low grade tumors (PLoS One 2015;10:e0143169)
        • May stain with epidermal growth factor receptor (EGFR) but does not show the mutation or alterations in copy number (Mod Pathol 2008;21:1168)
        • Reciprocal translocations, including t(1;11)(p22;q13) with overexpression of cyclin D1, t(11;19)(q14-21;p12) and t(11;19)(q21;p13) which encodes MECT1-MAML2 (MECT1 also known as CRTC1) (Arch Pathol Lab Med 2007;131:1400)
        Molecular / cytogenetics images

        Images hosted on other servers:
        <i>MAML2</i> rearrangement by FISH

        MAML2 rearrangement by FISH

        Differential diagnosis
        Additional references
        Board review style question #1
          Rearrangement of which gene would support the diagnosis of mucoepidermoid carcinoma of the lung?

        1. APC
        2. BRAF
        3. MAML2
        4. PIK3CA
        5. PLAG1
        Board review style answer #1

        Multiple pulmonary leiomyomatous hamartomas
        Definition / general
        • Proliferation of multiple smooth muscle nodules in the lung parenchyma
        Essential features
        • Multiple, small, unencapsulated nodules of smooth muscle in fascicles
        • Extremely rare, with controversial relationship to benign metastasizing leiomyoma or leiomyosarcoma
        Terminology
        • Controversial whether are true hamartomas or represent metastasizing leiomyoma or leiomyosarcoma
        Epidemiology
        • Extremely rare, almost always asymptomatic middle aged women
        Sites
        • Throughout lung parenchyma, occasionally bronchovascular distribution
        Etiology
        • Possible origin from contractile system of the lung acini (contractile interstitial cell)
        Clinical features
        • Usually incidental
        Diagnosis
        • Biopsy of nodules
        Radiology description
        • Xray: abnormal nodular shadows
        • CT: multiple small pulmonary nodules
        Prognostic factors
        Case reports
        Treatment
        • Surveillance, enucleation or surgical removal
        Gross description
        • 2 - 15 mm white, firm, solid, well demarcated nodules
        Microscopic (histologic) description
        • Excessive smooth muscle bundles in fascicles, not associated with tumor
        • Spindle cells with eosinophilic, fibrillary cytoplasm and blunt, elongated nuclei
        • Scattered glands and duct-like spaces may be seen
        • No mitoses, no nuclear atypia
        Positive stains
        Negative stains
        Differential diagnosis

        Mycobacteria non-TB

        Mycoplasma pneumoniae
        Definition / general
        • Strict aerobe that lacks a true cell wall, 0.2 - 0.8 μm and among the smallest free living bacterias
        • Formerly called atypical pneumonia (atypical due to moderate sputum production and consolidation and only minimal leukocytosis)
        Clinical features
        • Common in children and young adults, but may occur in older age groups
        • Causes community acquired pneumonia (eMedicine), either interstitial (usually) or bronchopneumonia (rare)
        • Often asymptomatic
        • Superimposed bacterial infection may occur
        • Diagnosis: rarely biopsied, diagnose with complement fixation antigen assays; cold agglutinins present in 50% of cases
        Treatment
        Gross description
        • Reddish blue, congested, patchy lungs; usually no pleuritis
        Microscopic (histologic) description
        • Bronchiolitis, interstitial and minimal intra-alveolar involvement with widened alveolar septa due to lymphoplasmacytic inflammatory cells
        • Intra-alveolar proteinaceous material
        • Neutrophilic infiltrate in bronchioles acutely, bronchiolar metaplasia, lymphoplasmacytic infiltrate in bronchial wall and pneumocyte hyperplasia
        Videos


        Gliding phase
        Differential diagnosis

        Myoepithelioma (pending)
        [Pending]

        Nocardia
        Clinical features
        • Ubiquitous saphrophyte; disease caused by inhalation or direct implantation into skin
        • More than 30 species with clinical significance (Clin Microbiol Rev 2006;19:259)
        • Opportunistic lung infection associated with transplantation, chemotherapy, immunosuppression, steroids
        • Disease is relatively uncommon in HIV likely due to sulfonamide prophylaxis directed against P. jeroveci; HAART treatment may be effective (Can J Infect Dis Med Microbiol 2009;20:e103)
        • Complication of alveolar proteinosis
        Gross images

        Images hosted on other servers:
        Abscess

        Abscess

        Microscopic (histologic) description
        • Focal bronchopneumonia with microabscesses, variable histiocytic response and ill defined granulomas
        • If identified by anatomic pathologist, clinical microbiology laboratory should be notified as routine culture may not isolate the organism
        Microscopic (histologic) images

        Contributed by Yale Rosen, M.D.
        Small abscess

        Small abscess

        Gram stain Gram stain

        Gram stain



        Images hosted on other servers:
        Modified acid fast stain shows weakly acid fast branching, filamentous bacilli

        Modified acid fast stain
        shows weakly acid fast
        branching, filamentous bacilli

        Positive stains
        • Modified acid fast, Fite, Gram stain and GMS
        • Gram stain: slender, slightly beaded, branching filamentous bacilli

        Nodular amyloid (amyloidoma)
        Definition / general
        • Rare, presents as asymptomatic nodule
        • Usually in asymptomatic elderly with nodule on chest Xray, but no evidence of systemic disease
        • Good prognosis; typically doesn't progress to lymphoproliferative disorders
        • Nodular involvement may be associated with systemic involvement
        Case reports
        • 85 year old woman with asymptomatic pulmonary nodules (Case #197)
        Gross images

        Contributed by Dr. Sajna V.M. Kutty, MVR Cancer Centre and Research Institute, Kozhikode, Kerala, India

        Rib tumor

        Microscopic (histologic) description
        • Well circumscribed amyloid, often with lymphocytes (T cells) and plasma cells
        • Granulomatous reaction to amyloid common, often calcification and ossification
        Microscopic (histologic) images

        Contributed by Dr. Sajna V.M. Kutty, MVR Cancer Centre and Research Institute, Kozhikode, Kerala, India

        Amyloid deposits

        Apple green birefringence



        Case #197

        Lung nodules - H&E

        Congo red - polarizing microscopy

        Cytology images

        Case #197

        Fine needle aspiration of lung nodule

        Positive stains
        • Congo red staining: glassy, salmon pink amorphous material with apple green birefringence under polarized microscopy
        • Clonal restriction of plasma cells (kappa or lambda staining, not both)
        Electron microscopy description
        • Amyloid fibrils
        Differential diagnosis
        • Primary pulmonary lymphomas with amyloid production: < 1% of pulmonary lymphomas have amyloid deposits, usually age 70+ years with marginal zone or SLL / CLL subtypes, often with lymphatic tracking and reactive lymphoid follicles; the presence of evenly distributed lymphocytes in the nodule and invasion of the pleura is specific for lymphoma
        • Hyalinizing granuloma: history of exposure to Histoplasma or TB, collagen is Congo red negative

        Nonspecific interstitial pneumonia
        Definition / general
        Essential features
        • An interstitial lung disease with a histologic NSIP pattern, due to a variety of etiologies
        • The NSIP pattern is the second most common pattern, and always part of the differential diagnosis of the usual interstitial pneumonia (UIP) pattern, but NSIP has a better prognosis than UIP
        • Multidisciplinary discussion is necessary to make the diagnosis and decide treatment direction
        Terminology
        • NSIP is the name of the morphologic (histologic) pattern
        • When no specific cause is identified, the clinical, radiological and pathological diagnosis of "idiopathic NSIP" is rendered
        ICD coding
        • Idiopathic nonspecific interstitial pneumonia: J84.113
        Epidemiology
        • Idiopathic NSIP is the second most common idiopathic interstitial pneumonia, following idiopathic pulmonary fibrosis
        • The mean onset is 50 - 60 years old
        • Female predominant
        • No clear association with tobacco use
        Sites
        • Bottom of lower lobes of bilateral (or "bibasilar") lung
        Pathophysiology / etiology
        Diagrams / tables

        Images hosted on other servers:
        Schema of NSIP pattern

        Schema of NSIP pattern

        Clinical features
        • Chronic and slowly progressive respiratory failure
          • Shortness of breath
          • Dyspnea on exertion
          • Cough
          • Fatigue
          • Weight loss
        • Abnormal chest auscultation
          • End inspiratory fine crackles in bibasilar lung
        • Restrictive pattern in pulmonary function tests:
          • Decreased forced vital capacity (FVC)
          • Decreased diffusing capacity of the lung for carbon monoxide (DLCO)
        Diagnosis
        • It is often challenging to make the diagnosis for cases with an NSIP pattern, since it includes a variety of diseases
        • Therefore, two levels of diagnosis are recommended: histological diagnosis of NSIP pattern and multidisciplinary (clinical-radiological-pathological) diagnosis for its etiology (i.e. idiopathic, connective tissue, etc.), based on:
          • Clinical features, including laboratory tests
          • High resolution computed tomography (HRCT)
          • Surgical lung biopsy
        • The guidelines suggest multidisciplinary discussion by experienced physicians, radiologists and pathologists, especially when they disagree (Am J Respir Crit Care Med 2011;183:788, Am J Respir Crit Care Med 2004;170:904, Thorax 2003;58:143)
        • Some NSIP cases with inconclusive autoimmune features may fit the criteria for idiopathic NSIP under current guidelines, or be classified as interstitial pneumonia with autoimmune features (IPAF) (Eur Respir J 2015;46:976)
        • Some cases with NSIP pattern may be included in the newly described category of "Unclassifiable interstitial pneumonia (UCIP)" (Respirology 2016;21:51)
        Laboratory
        • Serological autoantibody tests: can be positive but not conclusive for connective tissue disease
          • Antinuclear antibody
          • Rheumatoid factor
          • Anti-CCP antibody
          • Anti-SS-A, B antibody
          • Anti-Sm antibody
          • Anti-Scl-70 antibody
          • Anti-ARS antibody
        • Antibodies tests of hypersensitivity pneumonitis: pertinent negative for idiopathic NSIP (Am J Respir Crit Care Med 2012;186:314)
          • Avian antigens: pigeon, parakeet, budgerigar, chicken
          • Fungus: trichosporon, aspergillus
          • Bacteria: actinomycete
          • Mycobacteria: Mycobacterium avium-intracellulare
          • Chemicals
        • Increased serum KL-6 (normal limit is < 500 IU)
        Radiology description
        • Chest xray and HRCT can detect lesions: however, it is usually not sensitive enough to render the diagnosis (PLoS One 2016;11:e0166168, Thorax 2003;58:143)
        • HRCT findings suggestive for NSIP pattern:
          • Diffuse ground glass opacity
          • Reticular opacity
          • Traction bronchiectasis
        • Not suggestive findings:
          • Irregular linear opacity
          • Honeycombing
          • Nodular opacity
        Radiology images

        Images hosted on other servers:
        NSIP pattern on HRCT

        NSIP pattern on HRCT

        Prognostic factors
        • Better prognosis than UIP / IPF, with a 5 year survival of ~ 70% for all NSIP (Am J Respir Crit Care Med 1998;157:199)
        • Survival rate depends on the background etiology (Eur Respir J 2015;45:746)
          • 5 year survival of 61% for idiopathic NSIP
          • 5 year survival of 42% for NSIP with chronic hypersensitivity pneumonitis
          • 5 year survival of 77% for autoimmune NSIP
        Case reports
        Treatment
        • There is no standard treatment for idiopathic NSIP, although 80% of cases could be improved or stabilized by glucocorticoids with / without other immunosuppressive drugs such as azathioprine and cyclophosphamide (Am J Respir Crit Care Med 1998;158:1286, Respir Med 1999;93:113)
        • Treat background diseases, including autoimmune diseases
        • Remove suspicious drugs; reduce inhalation exposure to possible disease promoters
        Gross description
        • Diffuse involvement with mild to moderate increase in lung weight
        • Fibrotic changes in lower lobes
          • Homogeneous and diffuse compared to UIP
          • Shrunken lung
          • Traction bronchiectasis can be seen
          • Honeycomb change can be seen but limited
        Microscopic (histologic) description
        • Since there are few specific findings for NSIP pattern, it is essential to exclude other lung diseases on histology (Am J Respir Crit Care Med 2008;177:1338)
        • Characteristic findings of NSIP pattern
          • Diffuse and uniform inflammation ("temporal homogeneity") on low power of alveolar wall, bronchovascular bundles and pleura
            • There are usually no normal alveolar walls in the affected lobules
          • Cellular or fibrotic change
            • Lymphocytic or plasmacytic infiltration
            • Loose fibrosis
            • Lung architecture is frequently preserved
            • "Cellular NSIP" or "fibrotic NSIP" can be stated specifically in pathologist report
        • Features of interstitial pneumonia with autoimmune features (IPAF) (Chest 2010;138:251):
          • Lymphoid aggregates with germinal center
          • Extensive pleuritis
          • Prominent plasmacytic infiltration
          • Dense perivascular collagen
        • It is quite rare to see pure NSIP on histology - typically there are focal findings of other interstitial lung disease (Histopathology 2014;65:549)
        • Pertinent negative findings (Am J Respir Crit Care Med 2008;177:1338)
          • UIP-like change
            • Marked architectural distortion
            • Dense fibrosis with smooth muscle hyperplasia or elastosis
            • Fibroblastic foci
            • Honeycomb change
          • Hypersensitivity pneumonitis-like change
            • Airway centered change
            • Peribronchiolar metaplasia
            • Granulomas or interstitial giant cells with cholesterol cleft
          • Extensive organizing pneumonia (areas more than 20% of disease)
          • Desquamative interstitial pneumonia-like change
            • Aggregation of respiratory bronchiolitis macrophages in alveolar sacs and respiratory bronchioles
          • Diffuse alveolar damage-like change
            • Hyaline membrane
            • Alveolar hemorrhage
          • Lymphoid interstitial pneumonia-like change
            • Marked and extensive infiltration of lymphocytes in alveolar wall
            • Light chain restriction
          • Smoking related interstitial fibrosis (J Clin Pathol 2013;66:882)
            • Alveolar hyalinized septal thickening with little cellular infiltration
            • Centriacinar emphysema
        Microscopic (histologic) images
        Scroll to see all images.

        Contributed by Akira Yoshikawa, M.D.
        Low power

        Low power

        Uniform involvement

        Uniform involvement

        Less inflamed

        Less inflamed

        Lymphocytic infiltration and fibrosis

        Lymphocytic infiltration and fibrosis

        Lymphoid follicles with germinal center

        Lymphoid follicles with germinal center

        Lymphoplasmacytic infiltration

        Lymphoplasmacytic infiltration


        Low power

        Low power

        Uniform involvement

        Uniform involvement

        Sparse architectural distortion

        Sparse architectural distortion

        Peribronchiolar metaplasia

        Peribronchiolar metaplasia

        A few honeycomb changes

        A few honeycomb changes

        Dense fibrosis

        Dense fibrosis


        Low power

        Low power

        Uniform involvement

        Uniform involvement

        Lymphocytic infiltration and fibrosis

        Lymphocytic infiltration and fibrosis

        Pleuritis

        Pleuritis

        Organizing pneumonia

        Organizing pneumonia

        Lymphocytic aggregation

        Lymphocytic aggregation


        Airway centered change

        Airway centered change

        Low power

        Low power

        Uniform involvement with lymphoid follicles

        Uniform involvement with lymphoid follicles

        Lymphoid follicles with germinal center

        Lymphoid follicles with germinal center

        Pleuritis with lymphoid follicles

        Pleuritis with lymphoid follicles

        Dense fibrosis are sparsely seen

        Dense fibrosis are sparsely seen



        Images hosted on other servers:
        Diffuse and uniform inflammation Diffuse and uniform inflammation Diffuse and uniform inflammation Diffuse and uniform inflammation

        Diffuse and uniform inflammation

        Cellular NSIP

        Cellular NSIP


        Fibrotic NSIP Fibrotic NSIP

        Fibrotic NSIP

        Pleuritis

        Pleuritis

        Organizing pneumonia in NSIP

        Organizing pneumonia in NSIP

        Series of slides of fibrotic NSIP

        Series of slides of fibrotic NSIP

        Positive stains
        Differential diagnosis
        • Desquamative interstitial pneumonia: smoking history, aggregates of alveolar macrophages
        • Diffuse alveolar damage: hyaline membrane, alveolar hemorrhage
        • Hypersensitivity pneumonitis: exposure to antigens, airway centered change
        • Infectious disease (TB, fungal)
        • Interstitial pneumonia with autoimmune features (IPAF): lymphoid aggregates with germinal center, extensive pleuritis, prominent plasmacytic infiltration, dense perivascular collagen
        • Lymphoid interstitial pneumonia: viral infection such as HIV, CMV and HTLV1
        • Organizing pneumonia: marked organizing pneumonia on histology
        • Smoking related interstitial fibrosis: smoking history, acellular fibrosis
        • UIP / IPF: patchy and peripheral involvement, fibroblastic focus, honeycomb change

        NUT carcinoma
        Definition / general
        • NUT carcinoma is an aggressive malignancy characterized by NUT gene rearrangements
        • Monotonous, poorly differentiated morphology with foci of abrupt keratinization
        Essential features
        • NUT carcinoma is an aggressive malignancy more frequently arising in the thorax or aerodigestive tract, affecting patients of any age with equal gender distribution
        • Morphologically it shows diffuse architecture and consists of monotonous cells with frequent cytoplasmic clearing, large nuclei with prominent nucleoli and foci of abrupt keratinization
        • Due to its largely nonspecific morphology, the differential diagnosis is broad
        • Speckled nuclear immunohistochemical expression of NUT protein in ≥ 50% of cells is a diagnostic hallmark
        • NUT gene rearrangements are present and can be detected by FISH
        Terminology
        • NUT midline carcinoma (no longer recommended)
        ICD coding
        • ICD-O: 8023/3 - NUT carcinoma
        Epidemiology
        Sites
        Pathophysiology
        Etiology
        Clinical features
        Diagnosis
        • Diagnosis of NUT carcinoma can be challenging because of nonspecific morphologic features
        • Features may not be uniformly present in small biopsy specimens
        • A combination of subtle morphologic changes with NUT immunohistochemistry can aid in the diagnosis (Virchows Arch 2021;478:21)
        Laboratory
        Prognostic factors
        Case reports
        Treatment
        Gross description
        • Because of the advanced stage at diagnosis, examination of complete resection specimens is rare
        • Tumor can be deceivingly well demarcated from adjacent structures (Int J Surg Pathol 2016;24:85)
        Microscopic (histologic) description
        Microscopic (histologic) images

        Contributed by Kyriakos Chatzopoulos, M.D., Ph.D. and Kassiani Boulogeorgou, M.D.

        Cytomorphology of NUT carcinoma

        NUT abrupt keratinization

        NUT immunostain

        Cytology description
        Positive stains
        Negative stains
        Electron microscopy description
        Molecular / cytogenetics description
        Sample pathology report
        • Mediastinum, anterior, excisional biopsy:
          • NUT carcinoma (see comment)
          • Comment: Infiltrative, poorly differentiated carcinoma, with extensive necrosis and focal keratinization. Immunohistochemically, tumor cells express p63, p40, keratins 5/6 and NUT. Break apart FISH is positive for NUT rearrangement. These findings support the diagnosis.
        Differential diagnosis
        Board review style question #1

        A 36 year old man presents with cough and shortness of breath and is diagnosed with a large mediastinal mass and undergoes open surgical biopsy. A low power hematoxylin and eosin microscopic picture is shown above. If the mass undergoes genetic testing, which of the following is the most likely finding?

        1. BRD4::NUT fusion transcript
        2. Isochromosome 12p
        3. MYC amplification
        4. SMARCA4 point mutation
        Board review style answer #1
        A. BRD4::NUT fusion transcript

        BRD4::NUT fusion transcript is detected in 70% of NUT carcinomas. Answer B is incorrect because isochromosome 12p is a common finding in mediastinal germ cell tumors. Answer C is incorrect: although MYC is overexpressed in NUT carcinoma, this is because it is downstream in the BRD4::NUT fusion transcript cascade and not because of gene amplification. Answer D is incorrect because SMARCA4 point mutations are frequent in SMARCA4 deficient thoracic tumors (Cancer Genet Cytogenet 2010;203:16, Histopathology 2021;78:593, Oncogene 2014;33:1736, Virchows Arch 2021;478:21).

        Comment Here

        Reference: NUT carcinoma
        Board review style question #2
        A 42 year old woman presents with hemoptysis and bone pain. Imaging reveals a right hilar pulmonary mass and multiple lytic bone lesions. The patient mentions a history of facial nodule removed a year ago. Biopsy of a bone lesion reveals metastatic carcinoma with monomorphous morphology and foci of abrupt keratinization. Immunohistochemistry shows nuclear positivity for NUT protein. Which of the following genetic findings would make the diagnosis of NUT carcinoma more likely?

        1. Detection of YAP1::NUT fusion transcript
        2. Positive break apart NUT FISH
        3. Presence of a complex karyotype
        4. Wild type TP53 gene
        Board review style answer #2
        B. Positive break apart NUT FISH

        Although rare NUT fusions are nondetectable by FISH, the combination of this finding with a positive NUT immunostain has high sensitivity and specificity for diagnosing NUT carcinoma. Answer A is incorrect because a YAP1::NUT fusion, along with the patients reported history of facial nodule would be consistent with the diagnosis of porocarcinoma rather than NUT carcinoma. Answer C is incorrect because most NUT carcinomas have noncomplex karyotypes, with the only finding being a t(15;19) reciprocal translocation. Answer D is incorrect because TP53 is frequently mutated in NUT carcinoma (Lung Cancer 2015;90:484, J Cutan Pathol 2021;48:403, Pathol Int 2018;68:583, Cancer Med 2021;10:5757).

        Comment Here

        Reference: NUT carcinoma

        Organizing pneumonia
        Definition / general
        • Organizing pneumonia (OP) can be defined as either clinicopathological diagnosis, histological pattern or microscopic findings
        • Histologic features include polypoid fibroblastic aggregations, which plug alveolar sacs, ducts and bronchioles
        Essential features
        • Organizing pneumonia is one of the most commonly seen lung lesions and is associated with a variety of diseases, such as infections and systemic diseases
        • Cryptogenic organizing pneumonia is a relatively rare disease but often needs to be considered, since its clinical and radiological manifestations are often varied and nonspecific
          • Cryptogenic organizing pneumonia is a diagnosis that is not made by a pathologist; rather, it is a diagnosis of exclusion made by the multidisciplinary care team
        • Histologically, both cryptogenic organizing pneumonia and secondary organizing pneumonia are characterized by polypoid fibroblastic aggregations, which plug alveolar sacs, ducts and bronchioles
        Terminology
        • Organizing pneumonia (OP):
          • Can be either clinicopathological diagnosis (cryptogenic organizing pneumonia and secondary organizing pneumonia), histological pattern (organizing pneumonia pattern), or microscopic findings (e.g., Masson body)
          • Since 1901, organizing pneumonia has been described with the name bronchiolitis obliterans as an interstitial lung disease with granulation tissue plugs within alveolar ducts and small airways secondary to a variety of causes, including infection, fume exposure, drugs, collagen vascular disease, allergic reactions and obstruction (Chest 1983;83:161)
          • Davison et al. (1983) and Epler et al. (1985) reported a series of cases with organizing pneumonia and no evidence of infection or other aetiological agents; after that, the term bronchiolitis obliterans organizing pneumonia (BOOP) was commonly used for a while (Q J Med 1983;52:382, N Engl J Med 1985;312:152)
        • Cryptogenic organizing pneumonia (COP):
          • Idiopathic form of organizing pneumonia
          • In 2002, the American Thoracic Society / European Respiratory Society suggested the term cryptogenic organizing pneumonia (COP) to avoid confusion with airway disease (such as constrictive bronchiolitis obliterans) and categorized cryptogenic organizing pneumonia into acute / subacute interstitial pneumonia (Am J Respir Crit Care Med 2002;165:277, Am J Respir Crit Care Med 2013;188:733)
        • Cicatricial organizing pneumonia (ciOP) (Histopathology 2022;80:279):
          • Also known as collagenized organizing pneumonia, cicatricial variant of organizing pneumonia, fibrosing organizing pneumonia, scarred organizing pneumonia
          • Chronic fibrotic changes of conventional organizing pneumonia with formation of dense collagen fibers within the alveolar space, with no architectural destruction
        ICD coding
        • ICD-10: J84.116 - cryptogenic organizing pneumonia
        • ICD-11: CB03.2 - cryptogenic organizing pneumonitis
        Epidemiology
        Sites
        • Bilateral or unilateral lobes of the lung
        Pathophysiology
        • Organizing pneumonia is a repair process (wound healing) of the lung in response to preceding alveolar injury (Thorax 2000;55:318, Eur Respir J 2006;28:422)
          • Injury to capillary endothelial cells and alveolar epithelial cells results in the leakage of plasma protein, especially coagulation factors
          • Intra-alveolar coagulation of proteins and coagulation factors generate fibrin clotting on alveolar surfaces
          • Fibroblasts / myofibroblasts migrate into the damaged area, proliferate and generate loose fibrosis in the form of a small polyp
        • IL6, IL8 and TGFβ1 may play an important role in pathogenesis (Adv Exp Med Biol 2016;911:77)
        • Galectin 9 and regulatory T cells are increased in the lung with cryptogenic organizing pneumonia (Lung 2015;193:683)
        Etiology
        Diagrams / tables

        Images hosted on other servers:
        Typical organizing pneumonia

        Classification of interstitial lung diseases

        Typical organizing pneumonia

        Imaging patterns of cryptogenic organizing pneumonia

        Clinical features
        Diagnosis
        • Based on clinical features, radiology and histopathology (Am J Respir Crit Care Med 2002;165:277):
          • Clinical investigation for a possible cause of the disease is necessary
          • Surgical lung biopsy or transbronchial lung biopsy is required to establish a firm diagnosis since the clinical and radiological findings are often not specific
          • However, a biopsy may not be necessary if the clinical and radiological features are suggestive enough
        Laboratory
        • Increased serum surfactant proteins A and D
        • Negative serum antibodies of connective tissue diseases and hypersensitivity pneumonitis
        • Reference: Respiration 2019;98:534
        Radiology description
        • Simple chest radiography
          • Bilateral or unilateral ground glass opacity and consolidation
        • High resolution computed tomography (Respirology 2016;21:810, Chest 2017;151:1356)
          • Typical pattern:
            • Patchy ground glass opacity and consolidation with / without air bronchogram
            • Often bilateral and asymmetrical
            • Often peripheral and migratory
            • Size varies from a few centimeters to a whole lobe
            • Typical organizing pneumonia sometimes looks similar to eosinophilic pneumonia, pulmonary lymphoma and lepidic adenocarcinoma
          • Less common patterns:
            • Focal organizing pneumonia: nodular or mass-like consolidation mimicking lung cancer (Int J Clin Exp Pathol 2015;8:511)
            • Infiltrative organizing pneumonia: diffuse infiltrative opacity
            • Reversed halo sign: central ground glass opacity surrounded by round consolidation
            • Crazy paving pattern: areas of ground glass opacities superimposed to focal thickening of pulmonary parenchyma
            • Progressive fibrosis pattern: subpleural basal reticulations and architectural distortion, mimicking nonspecific interstitial pneumonia and usual interstitial pneumonia
            • Perilobular pattern: curved or arcade-like bands of parenchymal consolidation with blurred borders and thickening of the interlobular septa, resembling a Roman arch
            • Linear and band-like opacities: thick radial bands of consolidation containing an air bronchogram or subpleural curvilinear bands, parallel to the pleura
        Radiology images

        Images hosted on other servers:
        Typical organizing pneumonia Typical organizing pneumonia

        Typical organizing pneumonia

        Typical organizing pneumonia Typical organizing pneumonia Typical organizing pneumonia

        Typical organizing pneumonia


        Chest radiograph Chest radiograph

        Chest radiograph

        Focal organizing pneumonia Focal organizing pneumonia

        Focal organizing pneumonia

        Infiltrative organizing pneumonia

        Infiltrative organizing pneumonia

        Prognostic factors
        Case reports
        Treatment
        Gross description
        • Multiple patchy fibrotic lesions
        • Ill defined, soft to firm gray areas
        • Volume of the lung is usually normal
        • Other changes overlap with secondary organizing pneumonia
        Gross images

        Contributed by Yale Rosen, M.D.
        Pale area of nodular consolidation

        Pale area of nodular consolidation

        Mass-like lesions

        Mass-like lesions



        Images hosted on other servers:
        Typical organizing pneumonia

        Cryptogenic organizing pneumonia

        Microscopic (histologic) description
        • Organizing pneumonia (Semin Respir Crit Care Med 2012;33:462):
          • Fibroblastic plugs in alveolar sacs and ducts (organizing pneumonia) and bronchiolar lumen
          • Formed by spindled fibroblasts in pale staining matrix of immature loose collagen with polypoid shape (Masson body) or serpiginous or elongated form
          • Organizing pneumonia sometimes extends from one alveolus to the next through interalveolar fenestrae (butterfly pattern)
        • Mild to moderate cellular infiltrate in background (Eur Respir J 2006;28:422, Clin Med Insights Circ Respir Pulm Med 2016;9:123):
          • Thickened alveolar septa with lymphocytes, plasma cells and histiocytes
          • Alveolar architecture is usually preserved in cryptogenic organizing pneumonia
            • Because interstitial dense fibrosis, architectural destruction and honeycomb change are not components of cryptogenic organizing pneumonia, the organizing pneumonia lesion is likely to be secondary to other lung disease if these findings are mixed or overlap (J Clin Pathol 2009;62:387)
        • Airspace changes (Thorax 2000;55:318):
          • Foamy macrophage accumulation in surrounding airspace may be present
          • Occasional fibrin deposition
            • If prominent, the lesion is more likely to be an infection, eosinophilic pneumonia (especially after corticosteroids), vasculitis (e.g., granulomatosis with polyangiitis) or acute fibrinous organizing pneumonia (AFOP)
        • Cicatricial form of cryptogenic organizing pneumonia (Hum Pathol 2017;64:76):
          • Organizing granulation tissue is collagenized or hyalinized and harbors eosinophilic, lamellar and dense fibers
          • Airspaces can be filled with collagenized organizing pneumonia but alveolar architecture is mostly preserved
        Microscopic (histologic) images

        Contributed by Akira Yoshikawa, M.D. and Yale Rosen, M.D.
        Patchy involvement

        Patchy involvement

        Fibroblast plug Fibroblast plug

        Fibroblast plug

        Fibroblast plug

        Fibroblast plug

        Fibroblast plug Fibroblast plug

        Fibroblast plug


        Fibroblast plug Fibroblast plug

        Fibroblast plug

        Fibroblast plug Fibroblast plug

        Fibroblast plug

        Mononuclear cells infiltrate

        Mononuclear cell infiltrate

        Foamy macrophages

        Foamy macrophages


        Cicatricial variant Cicatricial variant

        Cicatricial variant

        Cicatricial variant

        Cicatricial variant

        Virtual slides

        Images hosted on other servers:
        Cryptogenic organizing pneumonia Cryptogenic organizing pneumonia

        Cryptogenic organizing pneumonia

        Cryptogenic organizing pneumonia

        Cicatricial organizing pneumonia

        Cryptogenic organizing pneumonia

        Organizing pneumonia in transplant biopsy

        Positive stains
        • Elastica van Gieson staining is helpful to evaluate if architecture of alveoli is preserved or not
        Negative stains
        • Organizing polyps are negative for elastica van Gieson staining except in cicatricial variant
        Electron microscopy description
        Videos

        Cryptogenic organizing pneumonia

        Focal organizing pneumonia

        Fibrosing organizing pneumonia

        Sample pathology report
        • Lung, right middle lobe, transbronchial biopsy:
          • Organizing pneumonia (see comment)
          • Negative for granuloma, vasculitis and malignancy
          • Comment: Based on the provided patient information, the diagnosis of cryptogenic organizing pneumonia is favored. However, secondary organizing pneumonia from various causes (infection, drug toxicity, autoimmune disease, malignancy, etc.) needs to be excluded clinically.
        Differential diagnosis
        • Acute interstitial pneumonia (AIP) / acute respiratory distress syndrome (ARDS):
          • Diffuse alveolar damage (DAD) of AIP or ARDS may have a prominent organizing pneumonia (organizing DAD) radiologic pattern
          • If organizing pneumonia is so diffusely distributed that it almost completely occupies the surgical sample, organizing DAD should be favored
          • Hyaline membranes, architectural destruction and myofibroblastic aggregation with less extracellular matrix are other clues of DAD pattern
        • Eosinophilic pneumonia:
          • Intra-alveolar eosinophils
          • Pink macrophages (not foamy), more fibrin, frequent type II pneumocyte atypia
        • Granulomatosis with polyangiitis:
          • Geographic necrosis, hemorrhage, capillaritis
          • More eosinophils
          • More fibrin deposition
        • Inflammatory myofibroblastic tumor:
          • Nodule / mass of spindle cells
          • More plasma cells
          • ALK+, SMA+, desmin+
        • Inflammatory pseudotumor (IgG4 related / NOS):
          • Nodule / mass of spindle cells
          • SMA+, ALK+
        • Nonspecific interstitial pneumonia (NSIP):
          • Organizing pneumonia is often seen in NSIP pattern but it does not exceed more than 20% of area
          • Interstitial change (cellular or fibrotic) is more predominant
        • Spindle cell carcinoma (sarcomatoid carcinoma):
          • Nodule / mass of atypical spindle cells
        • Usual interstitial pneumonia (UIP):
          • Fibroblastic focus in UIP pattern is another type of myofibroblastic aggregation but usually adjacent to dense fibrotic lesion, unlike in organizing pneumonia
          • Dense interstitial fibrosis, architectural destruction and honeycomb change are other clues of UIP
        Board review style question #1
        Which of the following findings is against the histologic diagnosis of organizing pneumonia?

        1. Interstitial mononuclear infiltrate
        2. Fibrin deposition
        3. Fibroblastic focus
        4. Foamy macrophage accumulation
        5. Masson body
        Board review style answer #1
        C. Fibroblastic focus. This finding is rather suggestive for usual interstitial pneumonia (UIP) pattern.

        Comment Here

        Reference: Organizing pneumonia
        Board review style question #2

        A 70 year old woman presented with cough, chest tightness and dyspnea. She was a former smoker with 20 pack years. High resolution CT scan of the chest showed bilateral patchy ground glass opacities in the lungs. A transbronchial biopsy was performed. What is the pathologic diagnosis based on the image provided?

        1. Bronchial asthma
        2. Desquamative interstitial pneumonia
        3. Diffuse alveolar damage
        4. Organizing pneumonia
        5. Respiratory bronchiolitis
        Board review style answer #2
        D. Organizing pneumonia. Fibroblast plug is the diagnostic feature for organizing pneumonia.

        Comment Here

        Reference: Organizing pneumonia

        Other fungi
        Table of Contents
        Blastomyces | Candida | Cryptococcus | Mucor
        Blastomyces

        Clinical features
        • See also Skin - nontumor chapter
        • Dimorphic fungus found in soil in Central and Southeastern United States and Canada (bordering Great Lakes, St. Lawrence River, Mississippi River and Ohio River basins), Mexico, Africa, India, Middle East
        • More common in males
        • Due to inhalation of soil containing microfoci of mycelia (Clin Microbiol Rev 2010;23:367)
        • Difficult to isolate in clinical microbiology laboratory
        • May present as consolidative pneumonia, ARDS or nodules resembling carcinoma; prefers upper lobes
        • May also infect skin and bone
        • Grossly often resembles tuberculosis

        Microscopic (histologic) description
        • Mixed acute and granulomatous inflammation caused by large budding yeasts (15 - 10 μm) with broad based buds and refractile walls
        • Easily seen with H&E

        Microscopic (histologic) images

        Images hosted on other servers:
        Various images (skin or sputum)

        Typical appearance of B. dermatitidis

        Various images (skin or sputum)

        Microabscess with a nonbudding yeast cell

        Various images (skin or sputum) Various images (skin or sputum)

        Budding yeast of B. dermatitidis

        Candida

        Definition / general
        • See also Skin - nontumor chapter
        • Normal flora of mouth, GI tract, vagina
        • May be commensal or pathogen
        • Commonly present in upper airways
        • Often present in aspiration pneumonia or pulmonary abscess due to colonization but invasive disease due to candida is rare in lung except in setting of candidal sepsis
        • Candida pneumonia associated with malignancy or immunosuppression (West J Med 1979;131:196)

        Microscopic (histologic) description
        • Pseudohyphae, occasionally true hyphae and budding yeasts
        • In some cases, only yeast may be present

        Microscopic (histologic) images

        Contributed by Yale Rosen, M.D.
        Lung: GMS staining Lung: GMS staining

        Lung: GMS staining



        Images hosted on other servers:
        Candida

        Candida



        Positive stains
        Cryptococcus

        Clinical features
        • Yeast, mostly encapsulated, found in pigeon droppings, that may cause mild infection after inhalation; usually confluent bronchopneumonia with "yeast lakes" of microorganisms and possibly coin lesions but no evident host response (eMedicine: Cryptococcosis [Accessed 22 March 2021])
        • May also cause meningitis
        • Latent infections can reactivate in immunosuppressed
        • CNS disease is a major concern in immunocompromised
        • Most commonly an opportunistic infection but disease may occur in immunocompetent patients
        • Major virulence factor is capsular polysaccharide glucuronoxylomannin, which hinders phagocytosis by alveolar histiocytes and inflammatory cell recruitment and migration
        • Other virulence factors include melanin production (Fontana-Masson stain may be positive; melanin may have antioxidant properties) and enzymes that increase invasiveness

        Case reports

        Microscopic (histologic) description
        • Somewhat pleomorphic, round / oval yeast, 4 - 10 microns
        • Thick, mucinous capsule stains bright red with mucicarmine; some are unencapsulated
        • Narrow necked budding takes place
        • Smaller, unencapsulated forms resemble Histoplasma capsulatum

        Microscopic (histologic) images

        Case #298
        Frozen section Frozen section Frozen section Frozen section

        Frozen section

        Oil immersion images Oil immersion images

        Oil immersion images


        Oil immersion images Oil immersion images

        Oil immersion images

        H&E H&E H&E H&E

        H&E


        PAS stain

        PAS stain



        Images hosted on other servers:
        Various images

        Large mucoid capsule

        Various images

        Cryptococcosis in patient with AIDS

        Mucor

        Definition / general
        • Also known as zygomycosis or mucormycosis
        • Ubiquitous fungi of class Zygomycetes, includes Mucor, Rhizopus, Absidia and Cunninghamella
        • Opportunistic infection especially associated with diabetes; other predisposing factors are neutropenia, corticosteroid therapy, iron overload and mucocutaneous trauma
        • Pulmonary and sinusoidal infection caused by inhaled spores or secondary to rhinocerebral mucormycosis

        Case reports

        Microscopic (histologic) description
        • Large, nonsepta hyphae with 90 degree angle branching and nonparallel walls, angioinvasive causing tissue necrosis and hemorrhage

        Microscopic (histologic) images

        Images hosted on other servers:
        Culture shows young sporangia of Mucor species

        Culture shows young sporangia of Mucor species


        Other parasites
        Cryptosporidium

        Definition / general
        • Waterborne protozoal infection that mostly infects intestinal epithelium causing diarrhea or malabsorption
        • Often causes self limited illness in immunocompetent patients
        • In HIV / AIDS, may spread to hepatobiliary tree or airways

        Treatment

        Microscopic (histologic) description
        • Characteristic spores on luminal epithelial cells
        • Mucosa with mild lymphocytic infiltrate

        Cytology images

        Images hosted on other servers:
        Cryptosporidium in sputum

        Cryptosporidium in sputum

        Dirofilaria

        Clinical features
        • Dog heartworm; may infect humans as secondary end stage host, particularly in southern coastal states in US
        • Adult worms die in right ventricle, embolize in pulmonary arterial circulation
        • Embolization of dead worms may be clinically silent or cause chest pain, fever, chills, hemoptysis or blood eosinophilia
        • Evokes necrotizing granulomatous response with vasculitis in lung tissue
        • Rarely see dead worms
        • Usually self limited in humans but may cause lung infarct
        • Usually diagnosed after resection of a nodule to exclude malignancy
        • Chest Xray: solitary peripheral pulmonary nodule or coin lesion (Int J Immunopathol Pharmacol 2010;23:345)

        Case reports

        Microscopic (histologic) description
        • Rounded infarct with coagulative necrosis, well demarcated from surrounding normal lung by epithelioid histiocytes and fibrous connective tissue
        • Focal calcifications and lymphoid aggregates
        • Necrotic nematode has homogenous cuticle without external ridges
        • Longitudinal muscle layer just internal to cuticle and internal cuticular ridges

        Microscopic (histologic) images

        Contributed by Dr. Vladimir Zaitsev
        Pulmonary embolization Pulmonary embolization Pulmonary embolization Pulmonary embolization

        Pulmonary embolization


        Pulmonary embolization Pulmonary embolization Pulmonary embolization

        Pulmonary embolization

        Echinococcus

        Clinical features
        • Also called hydatid cyst
        • Humans become infected by eating food contaminated with tapeworm eggs, becoming intermediate hosts
        • Eggs from dog tapeworm Echinococcus granulosus or E. multilocularis where foxes are the most common definitive hosts; other species rarely cause hydatid diseae in humans
        • For E. granulosus, sheep are most important intermediate hosts; for E. multilocularis, rodents are most important intermediate hosts
        • E. granulosus is more common in humans and is most common cestode infection of the lung
        • Eggs hatch in duodenum and spread to liver, lung, bone or elsewhere
        • E. granulosus cysts are most common in liver, 5 - 15% occur in lung; pulmonary disease is often secondary to hepatic disease (World J Surg 2001;25:46)
        • Larvae lodge in capillaries and incite a mononuclear and eosinophilic inflammatory cell response
        • Many larvae die, some encyst
        • Pulmonary cysts may be asymptomatic or cause respiratory compromise by compressing airways or lung parenchyma; rarely complicated by Aspergilloma (Br J Radiol 2008;81:e279)
        • Cyst rupture may cause fatal anaphylactic shock or pneumonia with consequent development of numerous new cysts throughout lung

        Case reports

        Microscopic (histologic) description
        • Cysts gradually enlarge and years later may be several centimeters in diameter
        • Cyst is bilayered and surrounded by fibroblasts, mononuclear cells, eosinophils, multinucleated giant cells
        • Daughter cysts usually develop in large mother cyst
        • Daughter cysts develop as projections from a germinative layer and form brood capsules
        • Degenerating scolices of developing worms produce sediment so called "hydatid sand"

        Microscopic (histologic) images

        Contributed by Hanni Gulwani, M.B.B.S.

        Various images



        Cytology images

        Images hosted on other servers:
        2 cm lung nodule 2 cm lung nodule 2 cm lung nodule 2 cm lung nodule

        2 cm lung nodule

        BAL fluid - scolices

        BAL fluid - scolices

        Filaria

        Definition / general
        • Due to microfilaria of Wuchereria bancrofti or Brugia malayi, which circulate in pulmonary capillaries and cause an immediate type of eosinophilic hypersensitivity reaction
        • Patients have high levels of IgE and high titers of filarial antibodies
        • Adult worms not seen in lungs
        • Restricted to tropical regions
        • Tropical eosinophilia may occur from filaria in lymph nodes or spleen (Meyers-Kouwenaar syndrome)
        • Symptoms and signs: cough, typically nocturnal, dypsnea, chest tightness, wheezing, fevers, weight loss and eosinophilia

        Microscopic (histologic) description
        • Rare microfilaria within pulmonary capillaries with marked eosinophilic infiltrate
        • Eosinophilic abscesses develop into granulomata
        Paragonimus

        Clinical features
        • Freshwater trematode (lung fluke) found in North America that infects crayfish and crabs as intermediate hosts (MMWR Morb Mortal Wkly Rep 2010;59:1573)
        • Humans acquire infection by ingesting raw or undercooked infected shellfish; rarely contaminated seaweed or watercress
        • Larvae mature in gut and migrate through diaphragm to infect lungs
        • Most cases in Asia due to P. westermani; schistosomiasis may also involve the lung
        • May mimic lung tumor (Korean J Parasitol 2011;49:69)
        • May require serology to confirm diagnosis

        Case reports

        Microscopic (histologic) description
        • Necrotizing or non-necrotizing granulomas with giant cells containing eggs with operculum; also organizing pneumonia (Am J Surg Pathol 2011;35:707)
        • Variable vasculitis and pleuritis; also lymphocytes, plasma cells, eosinophils
        • Egg is birefringent

        Cytology images

        Images hosted on other servers:

        Paragonimus egg

        Various images

        Paragonimus
        ova found in
        bronchoalveolar
        lavage fluid

        Toxoplasma

        Definition / general
        • Associated with AIDS, immunosuppression including therapy and hematologic malignancy
        • May be transmitted in utero
        • Ubiquitous intracellular coccidian parasite that causes focal parenchymal necrosis, diffuse interstitial pneumonia
        • Cats are definitive hosts, humans are intermediate
        • Very high prevalence in France; in US, from 3% to 20% depending on geography, ethnic group
        • Humans acquire transmission from ingestion of material contaminated with cat feces or cysts contained in undercooked meat (pork, beef, mutton); infection may be acquired in utero
        • Most disease in CNS, eye and heart; pulmonary involvement is in 70% of disseminated cases
        • Pulmonary involvement is indicative of disseminated disease and risk of death from CNS disease or bronchopneumonia
        • May cause severe morbidity or death in affected neonates

        Gross description
        • Lungs heavily congested, combined weight approximately 2000 grams

        Microscopic (histologic) description
        • Organisms present in histiocytes, alveolar lining cells, endothelial cells and pseudocytes abundant in necrotic areas

        Positive stains

        Other pneumoconiosis
        Aluminum
        • Chronic exposure to high concentrations of fumes during aluminum arc welding causes severe pneumoconiosis with diffuse pulmonary accumulation of aluminum metal and reduction in lung function (distinct from siderosis or welder's pneumoconiosis)
        • Disease also described in manufacturers of aluminum abrasives (Shaver's disease), aluminum smelters and polishers
        Anthracosis
        • Presence of carbon particles in lung
        • Not a pathologic condition necessarily, often due to urban living
        • Carbon particles are relatively inert and usually don't elicit reactive fibrosis
        • When extensive, may cause coal workers' pneumoconiosis (below)
        Berylliosis
        • Due to heavy exposure to dusts or fumes of beryllium, more common in nuclear and aerospace industries
        • Acute disease has disappeared due to exposure standards
        • Low dose exposure may cause granulomatous lesions that mimic sarcoidosis
        • Chronic berylliosis due to cell mediated immunity
        • 2% of those exposed develop disease; delayed hypersensitivity leads to noncaseating granulomas in lungs and hilar nodes, which become progressively fibrotic; no symptoms until late
        • Heavy beryllium exposure is linked to lung cancer
        Siderosis
        • Associated with iron workers and welders
        • Welders exposed to fumes with metal oxide particles 0.1 to 1.0 μm in diameter (iron, titanium, manganese and aluminum)
        • Lungs usually heavily pigmented without significant fibrosis
        • Aluminum (separate topic) has higher risk for significant pulmonary disease
        • Concurrent cigarette smoking, asbestos exposure may complicate clinical and pathologic findings
        • Micro: dust within interstitium, usually iron predominates
        Case reports
        Gross description
        • Aluminum: slate gray metallic appearance in scarred areas and peribronchial lymph nodes
        Microscopic (histologic) description
        • Aluminum: patchy pleural thickening, emphysematous blebs, pneumothorax and considerable fibrous replacement of lung tissue with aluminum containing macrophages; granulomatous disease has been described

        Papanicolaou system (pending)
        [Pending]

        Paraganglioma
        Definition / general
        • Very rare neuroendocrine tumor in lung
        • Usually benign
        • Metastases to lung are more common than a lung primary
        Essential features
        • Classic "zellballen" / organoid pattern with S100+ sustentacular cells, similar to pheochromocytoma
        • Primary pulmonary paraganglioma is extremely rare and requires ancillary studies to differentiate from carcinoid tumor
        • Considered malignant if metastasis is demonstrated
        Terminology
        Epidemiology
        • Extremely rare (< 25 case reports)
        • Thoracic paragangliomas comprise 1 - 2% of all paragangliomas (including pheochromocytomas)
        • Middle aged males predominate in sporadic cases, but middle aged females predominate in MEN 2 syndromes and present earlier
        Sites
        • Various organs
        • Thoracic paragangliomas mostly in mediastinal compartment but can also be primary intrapulmonary mass
        • Rarely may be endobronchial
        Etiology
        • Arise from paraganglion cells; neural crest origin
        • Intrapulmonary paragangliomas may arise from paraganglia-like structures in interstitium
        Clinical features
        • Mostly asymptomatic or non-specific symptoms
        • Obstructive symptoms if endobronchial
        • May show symptoms depending on specific neuropeptide production, if functional (i.e. hypertension from elevated norepinephrine)
        • May invade bronchus and metastasize to lymph nodes
        Diagnosis
        • Usually solitary nodules discovered incidentally
        • Diagnosis requires histologic examination with ancillary studies
        Laboratory
        • May show elevated serum levels of specific neuropeptide
        Radiology description
        • Xray: abnormal lung shadow
        • CT: well circumscribed to slightly irregular, solid mass
        • PET scan: may show FDG avid mass
        Radiology images

        Images hosted on other servers:
        Pulmonary nodule

        Pulmonary nodule

        Prognostic factors
        • Mostly benign behavior
        • If metastases, call "malignant paraganglioma", which may show reduced sustentacular cells and decreased neuropeptide markers
        • Features most predictive of malignancy: extra-adrenal location, coarse nodularity of tumor, confluent tumor necrosis, absence of hyaline globules (Hum Pathol 1990;21:1168)
        Case reports
        Treatment
        • Surgical resection (local excision or lobectomy)
        Gross description
        • Solitary or multiple, usually peripheral mass, often microscopic but may be several centimeters
        • Round to lobulated, well circumscribed but may show infiltrating border
        • Grey, pink, tan to orange color, may show areas of hemorrhage
        Gross images

        Images hosted on other servers:
        Bulging mass

        Bulging mass

        Microscopic (histologic) description
        • Organoid, trabecular and alveolar patterns, variable "zellballen"
        • Zellballen pattern (nested islands of chief cells) with inconspicuous sustentacular cells and capillary network at periphery (Am J Surg Pathol 2004;28:825)
        • Variable nuclear size, some cells with pleomorphism or hyperchromasia
        • Round to oval nuclei, prominent nucleoli, clear to eosinophilic and finely granular cytoplasm
        • Nuclear pseudoinclusions, intercellular hyaline globules, spindle cell or oncocytic changes may be seen
        • Presence of mitoses and vascular invasion not helpful for distinguishing benign and malignant
        • Necrotic foci may be seen
        • May show partly infiltrating border of lymphocytes
        Microscopic (histologic) images

        Images hosted on other servers:
        Histology of nodule

        Histology of nodule

        Lung tumor cells

        Lung tumor cells

        Cytology description
        • Papillary-like clusters of epithelioid cells with round to oval nuclei, evenly dispersed chromatin, micronucleoli, occasional anisonucleosis (Acta Cytol 2001;45:459)
        • Abundant cells with round or oval nuclei and marked anisokaryosis with tendency to form acini or follicular structures
        • Binucleation and intranuclear cytoplasmic inclusions (Acta Cytol 1988;32:386)
        Positive stains
        Negative stains
        Electron microscopy description
        • Dense core granules with internal, submembranous, eccentric halo emanating in a bubble-like fashion ("norepinephrine-type" granules)
        Molecular / cytogenetics description
        • Mutations in RET gene on chromosome 10
        Differential diagnosis

        Patterns of injury
        Definition / general
        • Major categories of interstitial lung disease
        • Fibrosing: UIP, NSIP, COP, lung disease with connective tissue disease, pneumoconiosis, drug reaction, radiation pneumonitis
        • Granulomatous: sarcoidosis, hypersensitivity pneumonitis
        • Eosinophilic
        • Smoking related: DIP, respiratory bronchiolitis-associated interstitial lung disease
        • Other: pulmonary alveolar proteinosis
        • Source: Kumar et al.: Robbins and Cotran Pathologic Basis of Disease, 8th Edition, 2009

        • Intraalveolar reaction: DIP, pulmonary alveolar proteinosis, infection, extrinsic allergic alveolitis, chronic eosinophilic pneumonia
        • Small airway disease: bronchiolitis obliterans, respiratory bronchiolitis, mycoplasma infection, viral infection, extrinsic allergic alveolitis, eosinophilic pneumonia
        • Large airway disease: allergic bronchopulmonary aspergillosis, bronchocentric granulomatosis, TB, fungi, Granulomatosis with polyangiitis (Wegener's)
        • Granulomatous vasculitis: Granulomatosis with polyangiitis (Wegener's), sarcoidosis, Churg-Strauss, bronchocentric granulomatosis, fungi, TB
        • Small vessel disease: primary pulmonary hypertension, thromboembolism, polyarteritis nodosa, veno-occlusive disease, Churg-Strauss syndrome
        • Hemorrhage: Goodpasture's, SLE, immune complex glomerulonephritis, idiopathic pulmonary hemosiderosis, Granulomatosis with polyangiitis (Wegener's)
        • Lymphoid infiltrates: lymphocytic interstitial pneumonia, lymphoma, lymphoid aggregates, extrinsic allergic alveolitis
        • Eosinophils: chronic eosinophilic pneumonia, Churg-Strauss syndrome, bronchocentric granulomatosis, Langerhans cell histiocytosis

        PEComa (clear cell tumor)
        Definition / general
        • Benign/borderline peripheral lung neoplasm derived from perivascular epithelioid cells
        • Member of perivascular epithelioid cell tumor (PEComa) or "myomelanocytoma" family, which includes angiomyolipoma, lymphangioleiomyomatosis (LAM)
        • Grouped with LAM under PEComatous tumors in 2015 WHO Classification of Lung Tumors (J Thorac Oncol 2015;10:1243)
        • Sheets of large cells with clear to eosinophilic granular cytoplasm and PAS+ glycogen granules
        Essential features
        • Extremely rare, benign pulmonary tumor derived from perivascular epithelioid cells (PEComa family of tumors)
        • Clear to eosinophilic, finely granular cytoplasm containing abundant PAS+ glycogen
        • Stains with melanocytic and smooth muscle markers by immunohistochemistry
        Terminology
        • First described as clear cell tumor of lung but also known as sugar tumor due to intracytoplasmic glycogen (Yale J Biol Med 1971;43:213)
        • Considered a benign PEComa in the lung, as opposed to malignant PEComa
        ICD coding
        • D14.31 Benign neoplasm of right bronchus and lung
        • D14.32 Benign neoplasm of left bronchus and lung
        • D14.30 Benign neoplasm of unspecified bronchus and lung
        Epidemiology
        • Extremely rare, case reports and small case series
        • Generally older adults >40 years old, may occur in children
        • No clear gender predilection
        Sites
        • Generally arises in peripheral lung, rarely in trachea or bronchi
        Pathophysiology
        • May rarely arise in patients with tuberous sclerosis along with lymphangioleiomyomatosis (LAM) and micronodular pneumocyte hyperplasia (MMPH) (Am J Surg Pathol 1997;21:1242)
        Etiology
        • Mesenchymal perivascular epithelioid cells (PEComa family)
        Clinical features
        • Generally asymptomatic or nonspecific pulmonary symptoms
        Diagnosis
        • Generally incidental finding on imaging
        • Requires tissue diagnosis with immunohistochemistry
        Radiology description
        Radiology images

        Images hosted on other servers:
        CT of clear cell tumor of lung

        CT of clear cell tumor of lung


        Prognosis and treatment
        • Rare case reports of malignant features and visceral metastasis
        • Aggressive features may include nuclear pleomorphism, >5 cm size, increased mitoses, necrosis (consider malignant PEComa)
        Case reports
        Treatment
        • Simple surgical excision
        Gross description
        • Small (< 5 cm), sharply outlined, glistening, red-tan to brown mass, usually in peripheral lung
        • Uniform cut surface, though may have foci of necrosis
        Gross images

        Images hosted on other servers:
        Well circumscribed tumors Well circumscribed tumors

        Well circumscribed tumors

        4 cm, gray-white, circumscribed tumor<br>confined to the lower lobe under the pleura

        4 cm, gray-white, circumscribed tumor
        confined to the lower lobe under the pleura

        Microscopic (histologic) description
        • Organoid (thick cords and rounded nests with sinusoid-type vascular network) or sheets of epithelioid cells
        • Clear to eosinophilic granular cytoplasm, well defined cell borders, numerous PAS+ glycogen granules
        • Small, uniform, rounded nuclei with small nucleoli, variably prominent sclerotic vasculature, may have extracellular amorphous eosinophilic material with variable calcification
        • Occasionally “spider cells” with nuclear condensation of eosinophilic cytoplasm with extensions to the cell membrane
        • May show entrapped small airways, necrosis, sparse lymphocytes
        • No fat, no / rare mitoses
        • Rarely malignant features (J Clin Oncol 2010;28:e626, Histopathology 2011;58:498)
        Microscopic (histologic) images

        Images hosted on other servers:
        (A) Core needle biopsy shows neoplastic cells in nested and insular pattern surrounding numerous thinned wall sinusoid-like vessels<br>(B) intranuclear cytoplasmic invaginations<br>(C) HMB45+ staining

        (A) Core needle biopsy shows neoplastic cells in nested and insular pattern surrounding numerous thinned wall sinusoid-like vessels
        (B) intranuclear cytoplasmic invaginations
        (C) HMB45+ staining


        Tumor with malignant histology Tumor with malignant histology Tumor with malignant histology Tumor with malignant histology Tumor with malignant histology

        Tumor with malignant histology

        Various images

        Various images

        Cytology description
        • Small and large cohesive clusters of bland epithelioid to spindle cells with oval or elongated nuclei, indistinct nucleoli, finely vacuolated cytoplasm, delicate transgressing vessels, background naked nuclei, rare intranuclear pseudoinclusion (Diagn Cytopathol 2008;36:89)
        Cytology images

        Images hosted on other servers:
        Benign sclerosing perivascular<br>epithelioid cell tumor (PEComa)

        Benign sclerosing perivascular
        epithelioid cell tumor (PEComa)

        (A) Large, irregular, cohesive clusters of columnar and spindle-shaped cells in background of single naked nuclei (Pap)
        (B) several large clusters with open cribriform-like spaces (Pap)
        (C) spindle-shaped and polygonal cells with round to oval nuclei and vacuolated ill-defined cytoplasm (Pap)
        (D) cell block shows neoplastic cells arranged in a nested and insular pattern surrounding sinusoid-like vessels; inset shows sinusoid-like vessel amidst tumor

        Negative stains
        Electron microscopy description
        Molecular / cytogenetics description
        • TSC2 loss of heterozygosity when arises in patient with tuberous sclerosis
        Differential diagnosis

        Pleomorphic adenoma
        Definition / general
        • Pulmonary adenoma includes bronchial adenoma, alveolar adenoma, papillary adenoma
        • Pleomorphic adenoma could be considered a type of bronchial adenoma
        • Benign tumor with biphasic growth, resembling counterpart in salivary glands; need to exclude metastasis from salivary glands

        • Related entities:
          • Alveolar adenoma:
            • Benign tumor in asymptomatic patients
            • Presents as coin lesion, usually in peripheral lung, consisting of small cystic spaces lined by type II pneumocytes and containing fluid
            • Interstitial cellular component consists of epithelioid and spindle shaped cells
          • Bronchial gland adenoma:
            • Includes oncocytoma and mucus gland adenoma, a rare, solitary, benign, well circumscribed, multicystic, predominately exophytic bronchial tumor
          • Carcinoma ex pleomorphic adenoma:
          • Intracapsular carcinoma ex pleomorphic adenoma:
            • Malignant component does not breach the capsule of the parent tumor, no metastatic potential
          • Papillary adenoma:
            • Arises in peripheral lung, composed of type II pneumocytes (club cells), TTF1+
            • Rarely has infiltrative features (Virchows Arch 2000;436:289)
        Essential features
        • Primary pulmonary pleomorphic adenoma is a very rare benign tumor that tends to arise in the large airways
        • Tumors tend to show small, branching, double layered ductules rather than ducts and less cartilaginous stroma as compared to pleomorphic adenomas arising in the salivary gland
        • Keratin and vimentin staining of the epithelial component supports diagnosis of primary pulmonary tumor as opposed to only keratin staining in salivary gland tumors (Arch Pathol Lab Med 2003;127:621)
        • Prognosis is generally good, but cases of carcinoma ex pleomorphic adenom have been reported
        Terminology
        • Pleomorphic adenoma is also known as mixed tumor
        • Overlaps with “myoepithelioma”, which could be used for lesions with less cartilaginous and ductal differentiation
        Epidemiology
        • Very rare (< 40 cases reported), predominantly in young adults, age range 8 - 74 years
        • Possibly shows slight F > M
        Sites
        • Usually in trachea and major bronchi, rarely in distal bronchi
        Etiology
        • Controversial, possibly from tracheal and bronchial submucosal glands or primitive stem cells
        Clinical features
        • Frequently asymptomatic but can present with dyspnea, hemoptysis, fever, weight loss or effusion
        • Depending on location, may cause obstruction of airway
        Diagnosis
        • Broader differential diagnosis on biopsy specimens but generally straightforward on resection specimen
        Radiology images

        Images hosted on other servers:
        Enhanced and nonenhanced CT scan

        Enhanced and nonenhanced CT scan

        Prognostic factors
        • Generally indolent; no features to predict rare cases of metastasis
        Case reports
        Treatment
        • Complete conservative surgical excision, but may recur many years later
        • Endobronchial resection may be an option in some cases (Intern Med 2008;47:1117)
        Gross description
        • Bronchial lesions are typically polypoid
        • Peripheral lesions are usually attached to bronchi, well circumscribed, no capsule, 1 - 16 cm, gray - white, soft, rubbery with chondroid cut surface, may have tumor tongues outside circumscribed margin
        Gross images

        Images hosted on other servers:
        Cut surface shows a well defined<br>mass, whitish to yellowish,<br>partly myxoid, solid and cystic

        Cut surface shows a well defined
        mass, whitish to yellowish,
        partly myxoid, solid and cystic

        Microscopic (histologic) description
        • Generally less cartilaginous stroma than salivary gland counterpart, with small branching ductules rather than ducts
        • Nests, tubules, trabeculae
        • Mixture of round or oval epithelial cells and myoepithelial cells in chondromyxoid or fibromyxoid stroma with focal hyalinization
        • Generally nuclear atypia / necrosis / hemorrhage / mitoses, occasional multinucleated giant cells
        • May show myoepitheliomatous, plasmacytoid, squamous features
        • Lumina may contain PAS+ eosinophilic secretions
        Microscopic (histologic) images

        Contributed by Roseann Wu, M.D., M.P.H.
        Various images Various images Various images Various images Various images

        Various images



        Images hosted on other servers:
        Metastasizing tumor with benign features

        Metastasizing tumor with benign features

        Cytology description
        • Fibrillary stroma mixed with bland basaloid epithelial cells
        Positive stains
        Negative stains
        Differential diagnosis
        Additional references

        Pleomorphic carcinoma
        Definition / general
        • Pleomorphic carcinoma:
          • Subtype of sarcomatoid carcinoma; usually aggressive, malignant epithelial neoplasm composed of cells with significant cytologic atypia and nuclear pleomorphism
          • Contains at least 10% spindle cells and / or giant cells
          • Included under recent WHO classification of "carcinomas with pleomorphic, sarcomatoid or sarcomatous elements"
        • Giant cell carcinoma:
          • Subtype of sarcomatoid carcinoma consisting of purely giant, pleomorphic tumor cells
          • Should not show differentiated non small cell components
          • Tumor stains for cytokeratins
        • Spindle cell carcinoma:
          • Subtype of sarcomatoid carcinoma (WHO)
          • < 1% of all primary lung carcinomas
        Essential features
        • Giant cell carcinoma:
          • Classified as a subtype of sarcomatoid carcinoma that is composed almost entirely of tumor giant cells with no differentiated carcinomatous elements
          • Tumor giant cells may or may not stain for human chorionic gonadotrophin
          • Metastatic sarcoma or germ cell tumor should be excluded
        Terminology
        ICD coding
        • Giant cell carcinoma:
          • Use code specific for location of tumor
          • ICD-10: C34.90 - malignant neoplasm of unspecified part of unspecified bronchus or lung
        Epidemiology
        • Pure giant cell carcinoma is rare, 0.3 - 2% of lung cancers
        • Mean age: 65 years, range: 42 - 81 years
        • > 90% men, 92% smokers
        Sites
        • Giant cell carcinoma:
          • May show predilection for upper lobes
          • Giant cell tumors frequently metastasize to small intestine
        Pathophysiology
        • Giant cell carcinoma: may have increased incidence of gastrointestinal tract involvement (Cancer 1992;70:606)
        Etiology
        • Giant cell carcinoma: appears to be a morphologic phenotype expressed by a heterogeneous group of tumors (Histopathology 1998;32:225)
        Clinical features
        • Pleomorphic carcinoma:
        • Giant cell carcinoma:
          • Cough, chest pain, dyspnea, malaise (varies by location)
        Diagnosis
        • Giant cell carcinoma: cannot be made on small biopsies or cytology; definite diagnosis only on resected tumor
        Radiology images

        Images hosted on other servers:
        CT of lung nodule

        CT of lung nodule

        Prognostic factors
        • Giant cell carcinoma:
        • Pleomorphic carcinoma:
          • Stage 1 tumors have same prognosis as other stage 1 non small cell carcinomas; at higher stages, may have worse prognosis than other non small cell carcinomas of similar stage (Am J Surg Pathol 2003;27:311)
        Case reports
        Treatment
        • Giant cell carcinoma: not usually surgical since metastatic at diagnosis but resection and radiation may prolong survival
        Clinical images

        Images hosted on other servers:
        Endobronchial tumor

        Endobronchial tumor

        Gross description
        • Pleomorphic carcinoma:
          • 2 - 17 cm, necrosis and hemorrhage common
        • Giant cell carcinoma:
          • Solid, well demarcated, tan, peripheral mass with areas of necrosis and hemorrhage
        Gross images

        Images hosted on other servers:
        8 x 7 cm tumor

        8 x 7 cm tumor

        Microscopic (histologic) description
        • Pleomorphic carcinoma:
          • Non small cell lung carcinoma with at least 10% neoplastic spindle or giant cells, usually with epithelial cells
          • Epithelial component 10 - 85%, usually adenocarcinoma or large cell carcinoma, also squamous cell carcinoma (Am J Surg Pathol 2008;32:1727)
          • Usually poorly differentiated
          • Spindle cells resemble MPNST, MFH or fibrosarcoma
          • Giant cells usually bizarre with multilobulated nuclei, abundant eosinophilic cytoplasm accompanied by heavy neutrophilic infiltrate with occasional ingested white blood cells
          • Stroma often myxoid, frequent inflammatory infiltrate, collagen fibers
          • Numerous mitotic figures
          • Massive necrosis common
          • Vascular invasion in 58%
        • Giant cell carcinoma:
          • Dyscohesive, polygonal, mono and multinucleated giant cells with abundant cytoplasm and prominent nucleoli
          • Neoplastic, highly pleomorphic giant cells, often in inflammatory stroma with emperipolesis (Arch Pathol Lab Med 2010;134:1645)
          • Giant cells are multinucleated, may resemble syncytiotrophoblasts and produce human chorionic gonadotropin but are usually fewer than in primary choriocarcinoma of lung (Histopathology 2000;36:17)
          • 2 types of giant cells: βHCG positive syncytiotrophoblast-like giant cells with smudged nuclei and coarse chromatin with nuclear molding and βHCG negative giant cells with admixed neutrophils and cell in cell features (Histopathology 2016;68:680)
        • Spindle cell carcinoma:
          • Carcinoma composed exclusively of spindle shaped tumor cells
          • Tumor cells often obliterate vessels
        Microscopic (histologic) images

        Contributed by Roseann Wu, M.D., M.P.H.
        Giant cell CA lung HE

        Giant cell CA lung HE

        Giant cell CA LUNG HE

        Giant cell CA lung HE



        Images hosted on other servers:
        8 x 7 cm tumor

        8 x 7 cm tumor

        Diffuse proliferation of atypical, giant and bizarre cells

        Diffuse proliferation of atypical, giant and bizarre cells

        Virtual slides

        Images hosted on other servers:
        Giant cell carcinoma - Rosai collection Giant cell carcinoma - Rosai collection Giant cell carcinoma - Rosai collection Giant cell carcinoma - Rosai collection

        Giant cell carcinoma


        Giant cell carcinoma - Rosai collection Giant cell carcinoma - Rosai collection Giant cell carcinoma - Rosai collection

        Giant cell carcinoma

        Cytology description
        • Giant cell carcinoma:
          • Pleomorphic cells in flat loose clusters, with abundant, thick, well demarcated cytoplasm, multinucleation (Acta Cytol 2011;55:173)
          • Sharp clear nuclear membranes, acidophilic cytoplasm with finely granular or foamy appearance, irregular coarse chromatin, bizarre mitoses, neutrophil engulfment (Diagn Cytopathol 2007;35:555)
        Cytology images

        Contributed by Roseann Wu, M.D., M.P.H.
        Giant cell CA ThinPrep

        Giant cell CA ThinPrep

        Positive stains
        • Pleomorphic carcinoma:
          • Sarcomatoid component: CK7 (63%), TTF1 (43%), surfactant protein A (6%)
          • Epithelial component: CK7 (76%), TTF1 (59%), surfactant protein A (39%)
        • Giant cell carcinoma:
        Negative stains
        Differential diagnosis
        Board review style question #1
        Giant cell carcinoma of the lung is most commonly associated with which epithelial component resembling non small cell carcinoma?

        1. Adenocarcinoma
        2. Adenosquamous carcinoma
        3. Large cell carcinoma
        4. Squamous cell carcinoma
        5. None of the above
        Board review style answer #1
        E. None of the above. Giant cell carcinoma is a subtype of sarcomatoid carcinoma composed almost entirely of tumor giant cells with no differentiated carcinomatous elements.

        Comment Here

        Reference: Pleomorphic carcinoma

        Pleuropulmonary blastoma
        Definition / general
        • Pleuropulmonary blastoma (PPB) is a rare, primitive primary neoplasm of the thorax in young children
        • The tumor, which is often but not always associated with cystic lung lesions, may arise in pulmonary parenchyma, the mediastinum and pleura
        • It was initially proposed to be a distinct entity in 1988 (Cancer 1988;62:1516)
        • An international registry has been established (ppbregistry.org)
        Essential features
        • Three subtypes of PPB exist (Types I, II and III), and are a continuum from the least to most malignant lesions (Cancer 2015;121:276, Pediatr Dev Pathol 2015;18:504)
          • Type I is multicystic; it is now considered the same clinical and pathologic entity as CPAM (congenital pulmonary airway malformation) type IV (Neonatology 2017;111:76)
          • Type II shows thickening areas (nodules) within this cystic lesion
          • Type III shows solid masses
        Terminology
        • PPB
        • Pneumoblastoma
        • Mesenchymal cystic hamartoma
        • Cystic mesenchymal hamartoma
        • Pulmonary rhabdomyosarcoma
        • Rhabdomyosarcoma in lung cyst
        • Pediatric pulmonary blastoma
        ICD coding
        • M8973/3
        Epidemiology
        • PPB is encountered in childhood, mostly in the first years of life (90% are between 0 - 2 years old)
        • Type I may be found in very young children (birth to 2 years of age)
        • Types II and III tend to be found after 2 years of age (Pediatr Dev Pathol 2015;18:504)
        Sites
        • PPB may arise in the lung, mediastinum and pleura
        Pathophysiology
        Etiology
        • PPB is associated with PPB family tumor and dysplasia syndrome in 33% of cases
        • Many of these patients have a mutation of the DICER1 gene (Cancer 2015;121:276)
        Clinical features
        • Children usually present with difficulty breathing or other respiratory problems including persistent pneumonitis, coughing or atelectasis
        • 10% may also present with multilocular cystic nephroma and very rarely, Wilms tumor
        Diagnosis
        • MRI or CT can help diagnose, but biopsy is suggested
        Radiology description
        Prognostic factors
        • The prognosis of PPB, especially Types II and III is poor because of frequent relapses and distant metastases which are often seen in the brain and bone (Pediatr Pulmonol 2015;50:698)
        Case reports
        Treatment
        • At present, the treatment of PPB is multimodal and includes surgery, chemotherapy or radiation therapy
        • The combination depends on the type and aggressiveness of the disease (Pediatr Pulmonol 2015;50:698)
        Gross description
        • Type I PPB is a peripherally located, multicystic and thin walled structure
        • Type II PPB is a mixed solid and cystic tumor characterized by variable thickened or nodule-like areas
        • Type III PPB is characterized by a well circumscribed, mucoid, white-tan solid mass attached to the pleura and involves a lobe or entire lung; necrosis and hemorrhage may be present in the friable areas
        Gross images

        Contributed by Erdener Özer, M.D., Ph.D. and Case #166
        Type III PPB characterized by well circumscribed whitish tan solid tumor Type III PPB

        Type III PPB



        Images hosted on other servers:
        Type I PPB has<br>multiloculated cyst<br>filled with scanty<br>clear serous fluid

        Type I PPB has
        multiloculated cyst
        filled with scanty
        clear serous fluid

        Microscopic (histologic) description
        • Type I PPB is a peripherally located, multicystic and thin walled structure
        • Type II PPB is a mixed solid and cystic tumor with variable thickened or nodule-like areas
        • Type III PPB is an apparently heterogeneous tumor composed of one or more of the following elements:
          • Primitive blastema-like small cells with hyperchromatic nuclei, high nuclear to cytoplasmic ratio and abundant mitoses
          • Spindled and ovoid cells embedded in a myxoid stroma
          • Nodules of immature or malignant chondroid elements
          • Isolated or clusters of large anaplastic cells with pleomorphic nuclei, atypical mitotic figures or eosinophilic hyaline bodies
        Microscopic (histologic) images

        Contributed by Erdener Özer, M.D., Ph.D., Dr. Roopa Arora and Case #166
        Types I and III PPB Types I and III PPB Types I and III PPB

        Types I and III PPB

        Pleuropulmonary blastoma Pleuropulmonary blastoma Pleuropulmonary blastoma

        Pleuropulmonary blastoma


        Pleuropulmonary blastoma

        Pleuropulmonary blastoma

        SMA

        SMA

        Type III PPB Type III PPB Type III PPB

        Type III PPB


        Type III PPB Type III PPB Type III PPB Type III PPB Type III PPB

        Type III PPB

        Virtual slides

        Images hosted on other servers:
        Cystic pleuropulmonary blastoma Cystic pleuropulmonary blastoma

        Cystic pleuropulmonary blastoma

        Cytology description
        • Cellular smear with both dispersed single cells and cohesive aggregates (Arch Pathol Lab Med 2000;124:416)
        • Uniform cells (small round blue cells) may lack cytoplasm, typically display nuclear molding with round or oval nuclei containing darkly stained fine chromatin
        • Scattered malignant giant cells have more abundant cytoplasm with indistinct edges and more dispersed chromatin
        • Some malignant cells may be spindled with high N/C ratios, eosinophilic cytoplasm and irregular nuclear membranes
        • There is no evidence of rosette or glandular formation (Journal of Clinical and Diagnostic Research 2011;5:1656)
        Positive stains
        Differential diagnosis

        Plexiform arteriopathy
        Definition / general
        • Historical term based on lesions induced by congenital heart disease
        • Heath and Edwards Classification from 1958 (modified below) still in widespread use
        • Open lung biopsy in patients with congenital heart disease may be performed to determine if corrective vascular surgery will be beneficial
        Microscopic (histologic) description
        Grade I (early):
        • Muscularization and media hypertrophy (> 7% of lumen) of pulmonary arteries

        Grade II:
        • Intimal hyperplasia causing attenuation of vascular lumen

        Grade III:
        • Subintimal fibrosis with onion ring appearance
        • Marked reduplication of internal elastic membrane
        • Arteries and arterioles resemble pipes

        Grade IV to V:
        • Dilation and plexiform lesions, aneurysmal dilation of small pulmonary arteries, plexiform and glomeruloid nodules
        • Fibrin thrombi within plexiform lesion, old / new hemorrhage present

        Grade VI:
        • Uncommon, acute necrotizing arteritis with fibrinoid necrosis and acute inflammation of vessel wall, similar to polyarteritis nodosa
        • Associated with extreme pulmonary hypertension
        Microscopic (histologic) images

        Images hosted on other servers:
        Plexiform lesion

        Plexiform lesion

        Differential diagnosis
        • Secondary pulmonary hypertension: similar histology, different history

        Pneumocystis
        Definition / general
        • Pneumocystis pneumonia (PCP) is a serious infection caused by the fungus Pneumocystis jirovecii in immunocompromised patients
        Essential features
        • One of the most common AIDS defining diseases in HIV infected patients
        • Substantial mortality and morbidity
        • Direct microscopic examination of bronchoalveolar lavage, sputum and lung biopsy are the gold standard methods of diagnosis
        • Cup shaped organisms with a central dot-like density
        Terminology
        • Pneumocystis jirovecii pneumonia (PJP)
        • Formerly known as Pneumocystis carinii
        ICD coding
        • ICD-10: B59 - pneumocystosis
        Epidemiology
        • Incidence of Pneumocystis pneumonia has declined substantially with widespread use of Pneumocystis pneumonia prophylaxis and antiretroviral therapy
        • Recent incidence among patients with AIDS in Western Europe and the United States is < 1 case per 100 person years
        • Commonly seen in HIV and immunocompromised patients (hematologic malignancies followed by autoimmune diseases) (Int J Infect Dis 2022;121:172)
        Sites
        • Lung and rarely extrapulmonary sites, which include lymph nodes, spleen, liver, bone marrow, adrenal glands, gastrointestinal tract, kidneys, thyroid gland, heart, pancreas, eyes, ears, lumina of blood vessels, perivascular involvement and lymphatic invasion (Arch Intern Med 1991;151:1205)
        Pathophysiology
        • Lung injury during infection is mostly due to host inflammatory response mediated by CD4+ T cells, alveolar macrophages, neutrophils and the soluble mediators
          • Inflammatory response is primarily responsible for the damage of the alveolar capillary interface activity (N Engl J Med 2004;350:2487)
        Etiology
        • Immunocompromised conditions (e.g., cancer, HIV, transplant recipients on immunosuppressant therapies) (N Engl J Med 2004;350:2487)
        Diagrams / tables

        Images hosted on other servers:
        Life cycle

        Life cycle

        Clinical features
        • Symptoms include fever, nonproductive cough, difficulty breathing, chest pain, fatigue and chills (Respiration 2018;96:52)
        • These symptoms develop over several days to weeks
        Diagnosis
        • Sputum, bronchoalveolar lavage or lung biopsy
        • Direct microscopic examination, polymerase chain reaction (PCR, highly sensitive but less specific), blood test to detect beta D glucan (Med Mycol 2020;58:1015)
        • In direct microscopy, Grocott-Gomori methenamine silver (GMS) stain should be performed on tissue biopsy and lavage specimens; immunofluorescence may be performed
        • Papanicolaou (Pap) and Romanowsky (Giemsa, Diff-Quik) barely stain the cyst wall of the organisms and infection can easily be missed
        Laboratory
        • Hypoxemia is the most characteristic laboratory abnormality
        • Elevation of lactate dehydrogenase levels to > 500 mg/dL is common but also nonspecific
        Radiology description
        • Chest radiograph typically demonstrates diffuse, bilateral, symmetrical ground glass interstitial infiltrates emanating from the hila in a butterfly pattern (AIDS 1998;12:885)
        • In early disease, a chest radiograph may be normal
        • Atypical radiographic presentations also occur, e.g., nodules, blebs / cysts, asymmetric disease, upper lobe localization, intrathoracic adenopathy and pneumothorax
        • Spontaneous pneumothorax in a patient with HIV infection should raise the suspicion of PCP
        • Cavitation and pleural effusion are uncommon in the absence of other pulmonary pathogens or malignancy and their presence may indicate an alternative diagnosis or an additional pathology such as tuberculosis (TB), Kaposi sarcoma or bacterial pneumonia
        Radiology images

        Images hosted on other servers:
        Chest imaging

        Chest imaging

        Bilateral well defined cystic spaces in lung

        Bilateral well defined cystic spaces in lung

        Prognostic factors
        • Treatment changes course of disease
        Case reports
        • 45 year old woman presented with dyspnea on exertion and significant weight loss within the preceding 4 months (BMC Infect Dis 2023;23:185)
        • 50 year old man presented with hypoxia, 1 week history of pleuritic chest pain and fever (Turk Patoloji Derg 2020;1:246)
        • 59 year old man presented with Pneumocystis jirovecii associated pneumonia and hypercalcemia due to ectopic production of 1,25 dihydroxyvitamin D, 6 years after renal transplantation (Acta Clin Belg 2021;76:75)
        Treatment
        • Trimethoprim and sulfamethoxazole
        Gross description
        • Lungs are heavy, firm and gray-brown
          • There is fine porosity due to trapped pockets of air
          • There may be small yellow indurated areas in the lung parenchyma and occasional emphysematous subpleural blebs (Can Med Assoc J 1969;100:846)
        • Lung nodules with or without cavitation have been demonstrated in response to granulomatous inflammation (Thorax 2002;57:435)
        Gross images

        Images hosted on other servers:
        Confluent consolidation and fine porosity

        Confluent consolidation and fine porosity

        Frozen section description
        • Touch preparations with Giemsa stain of fresh frozen lung infected with Pneumocystis is preferred over histologic sections as morphologic features are more apparent in the former (Can Med Assoc J 1969;100:846)
        Frozen section images

        Images hosted on other servers:
        Touch preparation of fresh frozen lung

        Touch preparation of fresh frozen lung

        Microscopic (histologic) description
        • Alveolar spaces filled with pink, foamy amorphous material composed of proliferating fungi and cell debris
        • Pneumocystis organisms can be identified in the GMS (Grocott-Gomori methenamine silver) stained tissue sections within airspaces
        • They are thin walled, oval, round and crescentic forms about the size of red blood cells (7 microns) (Can Med Assoc J 1969;100:846)
        • Granulomas, although rare, are mostly necrotizing and multiple
        • More than 1 infection, such as coexisting cytomegalovirus infection, may be present in these patients so it is important to look for other infectious organisms
        Microscopic (histologic) images

        Contributed by Aliya N. Husain, M.D.
        Intra-alveolar Pneumocystis jirovecii organisms

        Intra-alveolar Pneumocystis jirovecii organisms

        Granulomatous inflammation

        Granulomatous inflammation

        Foamy amorphous material

        Foamy amorphous material

        Intra-alveolar pneumocystis organisms

        Intra-alveolar Pneumocystis organisms

        Cytology description
        • Characteristic morphological finding is round to oval and cup shaped dark black (GMS stain) cysts with central dot-like density
        Cytology images

        Images hosted on other servers:
        Bronchoalveolar lavage washings

        Bronchoalveolar lavage washings

        Immunofluorescence description
        • Direct immunofluorescence testing on sputum and bronchoalveolar lavage specimens
        Immunofluorescence images

        Images hosted on other servers:
        Direct immunofluorescence testing

        Direct
        immunofluorescence
        testing

        Negative stains
        Electron microscopy description
        Molecular / cytogenetics description
        • Polymerase chain reaction is sensitive and rapid in detecting the presence of Pneumocystis jirovecii in bronchoalveolar lavage specimens (J Mycol Med 2022;32:101241)
        Sample pathology report
        • Lung, transbronchial biopsy:
          • Organisms consistent with Pneumocystis jirovecii infection (see comment)
          • Comment: Grocott-Gomori methenamine silver (fungal) stain is positive for organisms morphologically consistent with Pneumocystis jirovecii (control is appropriate).
        Differential diagnosis
        • Pneumocystis jirovecii lack budding and have a central dark dot-like density that is not seen in the following organisms: Candida, Histoplasma and Cryptococcus
        • Alveolar proteinosis:
          • Diffuse pulmonary opacification and intra-alveolar PAS positive material and lipid
          • Exudative pulmonary edema, sputum contains gelatinous material
          • No inflammation
          • Alveolar contents contain type II pneumocytes and necrotic alveolar macrophages
          • GMS stain is negative
        • Anti-GBM disease:
          • Necrotizing, hemorrhagic, interstitial pneumonitis; associated with rapidly proliferative glomerulonephritis, linear immunoglobulin deposits on basement membranes of alveolar septal walls
          • Also, intra-alveolar hemorrhage, septal thickening and hypertrophy, organization of blood in alveolar spaces
        • Organizing pneumonia:
          • Bronchiolar and alveolar plugs of loose fibrous tissue (Masson bodies)
        Board review style question #1

        A 50 year old man with hypoxia and bilateral interstitial infiltrates on chest imaging showed intra-alveolar foamy material. Grocott-Gomori methenamine (GMS) stain is shown in the figure above. What is the diagnosis?

        1. Alveolar proteinosis
        2. Candida
        3. Cryptococcus
        4. Pneumocystis pneumonia
        Board review style answer #1
        D. Pneumocystis pneumonia. Pneumocystis jirovecii are round to oval and cup shaped GMS positive cysts, approximately the size of red blood cells, with a central dot. Answer C is incorrect because Cryptococcus is larger, usually 10 - 20 microns, budding and does not reveal a central dot with GMS stain. Answer A is incorrect because GMS is negative in alveolar proteinosis. Answer B is incorrect because Candida is larger, forms pseudohyphae and budding yeasts and does not reveal a central dot with GMS stain.

        Comment Here

        Reference: Pneumocystis
        Board review style question #2

        A 60 year old woman with dry cough, fever and shortness of breath revealed bilateral pulmonary nodules on imaging. Biopsy revealed features as shown in the figure above. What is the diagnosis?

        1. Blastomyces and respiratory syncytial virus (RSV)
        2. Cryptococcus neoformans and herpes virus
        3. Histoplasma capsulatum and adenovirus
        4. Pneumocystis pneumonia and cytomegalovirus (CMV)
        Board review style answer #2
        D. Pneumocystis pneumonia and cytomegalovirus (CMV). The presence of intra-alveolar amorphous foamy material is characteristic of pneumonia caused by Pneumocystis jirovecii. Enlarged cells with single intranuclear inclusion and multiple small intracytoplasmic inclusions are diagnostic for cytomegalovirus. Answers A, B and C are incorrect because intra-alveolar amorphous foamy material is rarely seen in Cryptococcus, Histoplasma and Blastomyces pulmonary infection. Adenovirus has smudged intranuclear inclusions. Herpes virus has glassy intranuclear inclusions and respiratory syncytial virus (RSV) causes multinucleation with small intracytoplasmic eosinophilic inclusions.

        Comment Here

        Reference: Pneumocystis

        Preinvasive-general
        Definition / general
          Pulmonary preneoplastic changes include (Respir Res 2002;3:20):
        • Bronchial squamous dysplasia and in situ carcinoma preceding invasive squamous cell carcinoma and basaloid carcinoma
        • Atypical adenomatous hyperplasia preceding bronchioloalveolar carcinoma
        • Diffuse idiopathic pulmonary neuroendocrine cell hyperplasia, a proposed precursor for carcinoid tumor
        Essential features
        • Squamous dysplasia / CIS is a multifocal and clonal condition strongly associated with cigarette smoking i.e. “field cancerization”
        • High grade squamous dysplasia / CIS is associated with an increased risk of invasive squamous cell carcinoma
        • Squamous dysplasia / CIS tends to arise in large central airways
        Terminology
        • “Dysplasia” usually used for squamous bronchial lesions
        • Angiogenic squamous dysplasia = capillary blood vessels closely juxtaposed to and projecting into metaplastic or dysplastic squamous bronchial epithelium (Clin Cancer Res 2000;6:1616)
        ICD coding
        • D02.20: Carcinoma in situ of unspecified bronchus and lung
        • D02.21: Carcinoma in situ of right bronchus and lung
        • D02.22: Carcinoma in situ of left bronchus and lung
        Epidemiology
        Sites
        • Squamous dysplasia / CIS with propensity for large central airways, usually around bifurcations, less commonly in trachea
        Pathophysiology
        • May precede mass by many years
        • Pre invasive lesions and subsequent cancers are clonally related (J Pathol 2011;224:153)
        • Evidence for stepwise progression relatively weak, but concept of field carcinogenesis is strongly supported (Cancer Metastasis Rev 2010;29:5)
        Etiology
        • Associated with smoking
        • Possible progression from basal cells or metaplastic goblet cells to squamous metaplasia, dysplasia, CIS
        Clinical features
        • Usually not symptomatic on its own
        Diagnosis
        • Image enhanced endoscopy i.e. autofluorescence bronchoscopy (AFB), high magnification bronchovideoscopy (HMS), narrow band imaging (NBI), endobronchial ultrasonography (EBUS), optical coherence tomography (OCT) (Clin Chest Med 2013;34:373)
        • Frequently encountered in resection specimens, but not often on endoscopic biopsy specimens
        Prognostic factors
        • Some preneoplastic lesions regress, while others progress
        • No difference in progression rate and time to progression based on initial histologic grading; cannot differentiate the potentially more malignant lesions (Clin Cancer Res 2005;11:537)
        • Persistence of dysplasia associated with development of invasive carcinoma (Cancer Prev Res 2016;9:96)
        • Squamous dysplasia with high telomerase activity, increased Ki67 and p53 positivity tend to persist and might progress to carcinoma (Lung Cancer 2004;46:187)
        • CIS is strong predictor of progression to invasive squamous cell carcinoma (Cancer Metastasis Rev 2010;29:5)
        Case reports
        • Three men ages 63, 65 and 71 years old with dysplastic lesions in bronchi which progressed to squamous cell carcinoma (Br J Cancer 1997;75:678)
        Treatment
        • Bronchoscopic followup of severe dysplasia and CIS, endobronchial or surgical techniques (Chest 2007;132:221S)
        Gross description
        • Either unremarkable mucosa or papillary and granular with loss of folds
        Microscopic (histologic) description
        • Histological patterns of bronchial epithelial dysplasia: basal cell dysplasia, columnar cell dysplasia, bronchial epithelial dysplasia with transitional differentiation and squamous dysplasia (Mod Pathol 2006;19:429)
        • Squamous dysplasia: focal to full thickness replacement of epithelium by squamous cells with increased nuclear to cytoplasmic ratio, nuclear pleomorphism, mitotic activity but intact basement membrane
          • No invasive growth although may extend into ducts of submucosal glands
          • Graded with a 4 tier (mild / moderate / severe / CIS) or 2 tier system (low grade / high grade) (J Clin Pathol 2001;54:257)
          • Mild dysplasia: minimal abnormalities with basal expansion, increased cellularity, vertically oriented nuclei, limited to bottom third of epithelium, mitoses absent or rare, maturation present
          • Moderate dysplasia: more abnormalities, partial maturation, extending to lower two thirds of epithelium, mitoses limited to lower two thirds
          • Severe dysplasia: cellular pleomorphism, coarse chromatin, frequent nucleoli, basal zone to upper third, mitoses confined to lower two thirds, superficial cell flattening
          • CIS: lack of maturation, significant cytologic abnormalities, coarse chromatin, inconsistent nuclear orientation, mitoses present in full thickness
        • Diffuse idiopathic pulmonary neuroendocrine cell hyperplasia (DIPNECH): preinvasive proliferation of pulmonary neuroendocrine cells
          • Proposed criteria for diagnosis on lung resection is multifocal neuroendocrine cell hyperplasia as defined by 5 or more pulmonary neuroendocrine cells in at least 3 separate small airways combined with 3 or more carcinoid tumorlets (Semin Diagn Pathol 2015;32:438, Lung 2015;193:659)
        Microscopic (histologic) images

        Contributed by Roseann Wu, M.D., M.P.H.
        Moderate to severe dysplasia

        Moderate to severe dysplasia



        Images hosted on other servers:
        Missing Image

        Various images

        Missing Image

        Normal bronchial mucosa

        Positive stains
        Molecular / cytogenetics description
        Differential diagnosis
        • Squamous metaplasia and reactive / reparative atypia
        • Basal cell hyperplasia
        • Squamous papilloma
        Additional references

        Primary pulmonary myxoid sarcoma with EWSR1::CREB1 fusion
        Definition / general
        • Low grade tumor
        • Typically in (or close to) large airways
        • Lobulated tumor with spindle and stellate cells arranged in lace-like strands and cords within prominent myxoid stroma
        Essential features
        • Endobronchial nodular mass with reticular / lace-like growth pattern and abundant myxoid stroma
        • Expresses EMA and vimentin, negative for most other markers
        • FISH positive for EWSR1 rearrangement (85%)
        Terminology
        • Primary pulmonary myxoid sarcoma
        ICD coding
        • ICD-O: 8842/3 - primary pulmonary myxoid sarcoma
        • ICD-10: C34.9 - malignant primary bronchiogenic / bronchogenic neoplasm
        Epidemiology
        • Mean age = 46.3 years; F:M = 1.4:1 (24 cases)
        • 40% with cough, hemoptysis, chest pain; 20% present as incidental lung mass (Am J Surg Pathol 2011;35:1722)
        Sites
        Pathophysiology
        Etiology
        Diagnosis
        • Relies on histology, immunohistochemistry and FISH in biopsy or surgical resection
        Radiology description
        • Mass related to bronchus, predominantly endobronchial
        Radiology images

        Contributed by Hongxing (Simon) Gui, M.D., Ph.D.
        Endobronchial mass

        Endobronchial mass

        Prognostic factors
        • Patients with EWSR1 rearrangement have favorable prognosis; wild type EWSR1 portends poor clinical outcome (Pol J Pathol 2017;68:261)
        Case reports
        Treatment
        Gross description
        Microscopic (histologic) description
        • Lobulated architecture, reticular or lace-like pattern with anastomosing cords and strands, abundant myxoid stroma, often lightly basophilic (Am J Surg Pathol 2011;35:1722)
        • Oval, spindle to polygonal cells with minimal to moderate atypia, infrequent mitotic figures with the majority < 5/10 high power fields
        • Admixed lymphoplasmacytic infiltrate
        • Some cells are chondrocyte-like or mimicking physaliferous cells in chordoma (Diagn Pathol 2020;15:15)
        Microscopic (histologic) images

        Contributed by Hongxing (Simon) Gui, M.D., Ph.D.
        Nodular mass

        Nodular mass

        Reticular pattern

        Reticular pattern

        Multinodular growth

        Multinodular growth

        Spindle cells

        Spindle cells


        Hybrid patterns

        Hybrid patterns

        Angiomatoid fibrous histiocytoma area

        Angiomatoid fibrous histiocytoma area

        Spindle cells in lacy pattern

        Spindle cells in lacy pattern

        Positive stains
        Negative stains
        Electron microscopy description
        • Focal dense plaques present in the plasma membrane, external lamina and intermediate junctions are rarely observed (Virchows Arch 2014;465:453)
        Molecular / cytogenetics description
        Sample pathology report
        • Lung, right lower lobe, lobectomy:
          • Primary pulmonary myxoid sarcoma (see comment)
          • Comment: H&E sections demonstrate a well circumscribed endobronchial tumor composed of lobules of spindled and stellate cells arranged in a reticular pattern within a myxoid background. A chronic inflammatory infiltrate is seen throughout the tumor. Immunohistochemically, the tumor cells are positive for EMA and negative for AE1 / AE3, CAM 5.2, pancytokeratin, p40, desmin, SMA, CD34, TTF1, KIT, vimentin, CD1a and S100. A FISH test for EWSR1 was positive for rearrangement in the tumor cells. The morphology in combination with the FISH test result confirm the above diagnosis.
        Differential diagnosis
        Board review style question #1

        Which of the following is a good prognostic factor for this primary pulmonary lung lesion?

        1. Brisk lymphoplasmacytic infiltrate
        2. FISH positive for EWSR1 translocation
        3. Lack of tumor necrosis
        4. Middle age onset
        5. Size > 5 cm
        Board review style answer #1
        B. FISH positive for EWSR1 translocation. Primary pulmonary myxoid sarcoma tumors (shown in photo) with EWSR1 rearrangement have a relatively good prognosis.

        Comment Here

        Reference: Primary pulmonary myxoid sarcoma with EWSR1::CREB1 fusion
        Board review style question #2
        Which of the following entities is the closest mimicker of primary pulmonary myxoid sarcoma?

        1. Epithelioid hemangioendothelioma
        2. Extraskeletal myxoid chondrosarcoma
        3. Inflammatory myofibroblastic tumor
        4. Myoepithelioma
        5. Myxoid angiomatoid fibrous histiocytoma
        Board review style answer #2
        E. Myxoid angiomatoid fibrous histiocytoma. Myxoid angiomatoid fibrous histiocytoma and primary pulmonary myxoid sarcoma have many overlapping features.

        Comment Here

        Reference: Primary pulmonary myxoid sarcoma with EWSR1::CREB1 fusion

        Pseudomonas
        Clinical features
        • Aerobic, mobile, flagellated Gram negative bacillus that survives in water, soil and vegetation
        • Worldwide distribution (eMedicine)
        • Resistant to many disinfectants, common in hospital environment, common cause of hospital acquired infections
        • Commonly colonizes humans, but rarely causes disease in healthy subjects
        • Affects primarily infants, cystic fibrosis, immunocompromise, critically ill, neutropenia and burns / on ventilators
        • Immunocompromised usually have paucicellular pattern, rapidly progressive course with bronchopneumonia and bacteremia
        • Immunocompetent have a cellular pneumonia, more protracted course
        • Perivascular bacterial infiltration is somewhat specific for pseudomonas

        Infants:
        • Rare: 0.3% of neonatal ICU admissions
        • Usually low birth weight (1.2% of low birth weight admissions)
        • Mortality 32% - 87% with death within 1 - 2 days
        • Diagnosis made by culture
        Case reports
        • 49 year old woman with fatal pneumonia likely associated with contaminated hot tub (Infection 2011;39:265)
        Gross images

        Contributed by Yale Rosen, M.D.
        Pseudomonas pneumonia Pseudomonas pneumonia

        Pseudomonas pneumonia

        Microscopic (histologic) description
        • Necrotizing pneumonia with 2 patterns: paucicellular coagulative confluent bronchopneumonia with perivascular bacillary infiltration (bacteremic) or cellular pneumonia without evidence of perivascular organisms (nonbactermic)
        • Nonbacteremic: often occurs in debilitated patients with chronic lung or heart disease through aspiration
        Additional references

        Pulmonary artery intimal sarcoma
        Definition / general
        • Malignant mesenchymal tumor arising from large pulmonary blood vessels
        Essential features
        • Predominantly intraluminal growth in pulmonary arterial circulation
        • Arises from intimal layer of elastic arteries, usually sparing media
        • Causes obstruction of the vessel lumen leading to pulmonary embolism (PE) or pulmonary hypertension
        • Causes embolic seeding to peripheral organs
        • Microscopy: spindle cells with variable smooth muscle expression and variable atypia; however, may be completely undifferentiated with or without heterologous elements (osteosarcoma, chondrosarcoma)
        • > 70% cases are positive for MDM2 (mouse double minute 2) amplification
        • References: J Thorac Dis 2019;11:S9, Pathol Res Pract 2021;224:153548
        ICD coding
        • ICD-O: 9137/3 - intimal sarcoma
        • ICD-11: 2C25.Y & XH36H7 - other specified malignant neoplasms of bronchus or lung & intimal sarcoma
        Epidemiology
        • Rare tumor
        • Estimated prevalence: 0.001 - 0.003% (maybe be underestimated due to similar presentation as pulmonary thromboembolism)
        • Found in 1 - 4% of pulmonary endarterectomy specimens in patients with clinical features of chronic thromboembolic pulmonary hypertension
        • Females affected more than males (M:F = 0.7:1)
        • Median age at diagnosis: 48 years
        • References: CJC Open 2020;2:711, Aorta (Stamford) 2016;4:142, Korean J Radiol 2018;19:792, Cancer 1993;71:1761
        Sites
        • Pulmonary arteries
          • Anywhere from pulmonary valve to lobar branches
          • Bilateral or unilateral
          • Can also grow backwards into heart
        • Rare reports of intimal sarcoma arising from other vessels and the heart
          • Larger vessels: aorta (known as aortic intimal sarcoma, which is even rarer)
          • Peripheral arteries: iliac and femoral arteries; or even more rarely, may arise from the renal arteries, carotid arteries or inferior vena cava
          • Heart: undifferentiated pleomorphic sarcomas (UPS) of the heart is suggested to represent the cardiac analog of intimal sarcoma
        • References: Case Rep Oncol 2016;9:267, Radiographics 2021;41:361, Cancer 1993;71:1761, J Thorac Oncol 2016;11:441 Mod Pathol 2021;34:2122
        Pathophysiology
        • Originates from subendothelial pluripotent cells within intimal lining
        • Frequent amplifications and gains as well as other alterations, in 12q12-15, 4q12, 7p12 and others, where several genes, including MDM2, CDK4, PDGFRA, EGFR and other genes, are located
        • References: Pathol Res Pract 2021;224:153548, Radiographics 2021;41:361
        Etiology
        • Not known
        Diagrams / tables

        Images hosted on other servers:

        Most common histologic patterns

        Clinical features
        • Mimics pulmonary embolism and pulmonary hypertension
        • Most common presentation: breathlessness
        • Other symptoms: chest / back pain, cough, hemoptysis, fatigue, syncope, fever
        • Clues to diagnosis:
          • Absence of risk factors for pulmonary embolism (but half of patients with pulmonary embolism also have no risk factors)
          • Patient fails anticoagulation therapy
        • References: Radiographics 2021;41:361, Case Rep Oncol 2021;14:318, Intern Med 2019;58:3599
        Diagnosis
        • Diagnostic imaging modalities: CT angiography, fluorine 18-fluorodeoxyglucose PET (18F-FDG PET), MRI and echocardiography
        • Biopsies are typically not performed
        • Diagnosis is usually made on resection and histopathologic exam
        • Reference: Radiographics 2021;41:361
        Laboratory
        Radiology description
        • Heterogenous lobulated mass in an elastic artery (most often a pulmonary artery)
        • Forms acute angle with vessel wall (versus pulmonary embolism)
        • Possible imaging features:
          • Tumor typically enlarges and molds to fill the central vascular lumen and its branches
          • Can also grow backwards across the pulmonary valve, into the right ventricle, even into the right atrium and vena cava = continuous or multifocal appearance
          • May occlude vascular lumen
          • May invade through the vascular wall into surrounding tissue
          • Intralesional blood flow suggests neoplasia
        • CT: scan with contrast demonstrates a large filling defect
        • PET: most intimal sarcomas are F-FDG avid
        • MRI, T1 weighted: isointense to mildly hyperintense
        • MRI, T2 weighted: isointense to highly intense
        • Imaging differential diagnosis = more common cardiovascular lesions: thrombus, tumor embolism, valvular vegetation
        • References: Case Rep Oncol 2016;9:267, Radiographics 2021;41:361
        Radiology images

        Contributed by Andre L. Moreira, M.D., Ph.D.
        Contrast CT: filling defect

        Contrast CT: filling defect



        Images hosted on other servers:

        Transesophageal echo (TEE): large mobile masses

        Echo: masses below pulmonary valve

        Contrast CT: filling defect

        CT showing filling defect


        CT: filling defect

        Intense avidity on PET

        CT: multiple lung nodules

        Prognostic factors
        Case reports
        Treatment
        • Surgical resection:
          • Pulmonary endarterectomy (PEA)
          • Full thickness resection
          • Pneumonectomy
        • Full thickness resection = complete excision of affected pulmonary artery depending on extent of tumor
        • Neoadjuvant or adjuvant chemotherapy or radiation therapy may be considered
          • Possible chemotherapy regimens: anthracyclines, ifosfamide, gemcitabine, taxanes, platinum based antineoplastic drugs
          • Immunotherapy (not yet standardized)
        • Reference: Radiographics 2021;41:361
        Clinical images

        Images hosted on other servers:

        Glossy white tumor, pulmonary trunk

        Intraoperative images of tumor

        Gross description
        Gross images

        Contributed by Andre L. Moreira, M.D., Ph.D.
        Pulmonary artery with tumor

        Pulmonary artery with tumor



        Images hosted on other servers:

        Mass mimicking saddle pulmonary embolism

        Yellow solid tumor mass

        Occlusion, main pulmonary artery

        Filling arterial branches (arrows)

        Distending arterial branches (arrows)

        Microscopic (histologic) description
        Microscopic (histologic) images

        Contributed by Andre L. Moreira, M.D., Ph.D. and Borislav A. Alexiev, M.D. (Case #535)
        Atypical spindle cell neoplasm

        Atypical spindle cell neoplasm

        Atypical spindle cells arranged in fascicles

        Atypical spindle cells arranged in fascicles

        Atypical spindle cells

        Atypical spindle cells

        Atypical spindle cells, fascicles

        Atypical spindle cells, fascicles

        High grade nuclei, mitoses <p>High grade nuclei, mitoses</div>

        High grade nuclei, mitoses


        Polypoid tumor with fibrin

        Polypoid tumor with fibrin

        Tumor in fibrin

        Tumor in fibrin

        Tumor in artery wall

        Tumor in artery wall

        Atypical mitoses

        Atypical mitoses

        Myxoid and hypocellular areas

        Myxoid and hypocellular areas

        Nuclear pleomorphism

        Nuclear pleomorphism


        Tumor necrosis

        Tumor necrosis

        Vascularity within tumor Vascularity within tumor

        Vascularity within tumor

        Smooth muscle actin IHC

        Smooth muscle actin IHC

        MDM2 IHC

        MDM2 IHC

        Mimicker of blood clot

        Mimicker of blood clot


        Mimicker of organized clot Mimicker of organized clot

        Mimicker of organized clot

        Mimicker of IMT

        Mimicker of IMT


        Cytology description
        • Pleomorphic malignant spindle cells
        • Loosely cohesive clusters of tumor cells
        • Reference: Cytojournal 2015;12:3
        Cytology images

        Images hosted on other servers:

        Clusters of spindled cells in PAIS

        Pleomorphic cells with vacuolated cytoplasm

        Pleomorphic spindle cells in PAIS

        Positive stains
        Molecular / cytogenetics description
        • MDM2 gene amplification: most important in diagnosis
        • Alterations have been reported in 12q12–15, 4q12, 7p12, 9p21, leading to changes or amplification of MDM2, CDK4, SAS, PDGFRA, EGFR, TERT, KIT, KDR, NOTCH2, ERBB3, GLI, VEGFR2, FBXW7, FAT3 or FANCM
        • In one study, by FISH, 81% of intimal sarcomas had PDGFRA amplification, 65% had MDM2 amplification and 76% had EGFR amplification
        • Cardiac intimal sarcomas from the left atrium tends to show MDM4 and CDK6 amplifications; they lack MDM2 amplification
        • References: Mod Pathol 2021;34:2122, Pathol Res Pract 2021;224:153548, Zhonghua Bing Li Xue Za Zhi 2020;49:816, J Thorac Dis 2019;11:S9
        Molecular / cytogenetics images

        Contributed by Borislav A. Alexiev, M.D. (Case #535)
        <i>MDM2</i> gene in red

        MDM2 gene in red



        Images hosted on other servers:
        FISH study with <i>MDM2</i>

        FISH study with MDM2

        Cardiac: <i>MDM4</i> and <i>CDK6</i>

        Cardiac: MDM4 and CDK6

        <i>MDM2</i> FISH

        MDM2 FISH

        Sample pathology report
        • Mass, pulmonary artery, excision:
          • Intimal sarcoma (see comment)
          • Comment: The sections show a spindle cell proliferation arranged in fascicles, with hyperchromatic pleomorphic nuclei. The lesion invades the media of the artery (supported by elastic stain). The sections also reveal areas of necrosis, consisting of > 50% of the submitted specimen. The mitotic activity is 12 mitoses/10 high power fields. The tumor cells are positive for SMA, vimentin and CD31; negative for cytokeratin, desmin, myogenin, CD34, beta catenin and STAT6. MDM2 immunohistochemistry shows positivity in the tumor nuclei. FISH study shows MDM2 and CDK4 amplification. The histologic, immunohistochemical and molecular findings are consistent with a diagnosis of intimal sarcoma.
        Differential diagnosis
        • Lung thromboemboli:
          • Fibrin with or without organization, platelets (CD61+)
          • Lines of Zahn
          • History of risk factors for thromboembolism
          • No atypical spindle cells, mitosis or necrosis
        • Lung infarct:
          • Gross: wedge shaped area of hemorrhagic or tan lung parenchyma
          • Histology: no atypical spindle cells or mitosis
        • Sarcomatoid carcinoma:
          • Atypical spindle cells with mitoses or necrosis
          • Epithelioid cells may be present
          • Cytokeratin positive (may be focal)
        • Metastatic sarcoma from another organ:
          • Atypical spindle cells with mitoses or necrosis
          • History, immunohistochemical stains, molecular studies and previous pathology are helpful to differentiate
        Board review style question #1

        A 50 year old woman, who completed a long distance flight the previous week, came to the ER with chest pain and breathlessness. D dimer level was elevated. CT scan with contrast showed a large filling defect in the left pulmonary artery. She was started on anticoagulation therapy but did not improve. PET / CT showed that the left pulmonary artery mass was FDG avid. The lesion was resected and the pathology revealed the findings shown above. Immunostains showed that the cells were positive for SMA and CD31 and negative for AE1 / AE3. What is the diagnosis?

        1. High grade sarcoma, consistent with intimal sarcoma
        2. Metastatic carcinoma
        3. Sarcomatoid carcinoma
        4. Thromboembolus
        Board review style answer #1
        A. High grade sarcoma, consistent with intimal sarcoma

        Comment Here

        Reference: Pulmonary artery intimal sarcoma
        Board review style question #2
        A malignant spindle cell tumor is seen arising from the intimal layer of the pulmonary artery. Immunostains showed that the tumor cells were positive for SMA and CD31 and negative for AE1 / AE3. The provisional diagnosis was pulmonary artery intimal sarcoma. What is the most common molecular alteration seen in this entity?

        1. ETV6::NTRK3
        2. EWSR1::FLI1
        3. FUS::CREB3L2
        4. MDM2 amplification
        Board review style answer #2
        D. MDM2 amplification

        Comment Here

        Reference: Pulmonary artery intimal sarcoma

        Pulmonary blastoma
        Definition / general
        • Rare lung malignancy, 0.25 - 1% of lung cancers
        • Biphasic components with admixed epithelium and primitive stroma
        • Highly aggressive type of sarcomatoid carcinomas (BMC Res Notes 2014;7:294)
        Essential features
        • Peripheral solitary large mass, well circumscribed, unencapsulated
        • Well differentiated glandular component and blastematous stroma
        Terminology
        • Separated from fetal adenocarcinoma (epithelium only) and pleuropulmonary blastoma (mesenchymal only)
        ICD coding
        • ICD-O: 8972/3 - pulmonary blastoma
        Epidemiology
        Sites
        Pathophysiology
        Etiology
        Clinical features
        Diagnosis
        Radiology description
        Radiology images

        Images hosted on other servers:
        Missing Image

        Mass near hilum

        Missing Image

        Compression of vena cava

        Missing Image

        Uptake within mass

        Missing Image

        Right mass

        Prognostic factors
        Case reports
        Treatment
        Gross description
        • Mostly peripheral, large, well circumscribed, unencapsulated mass, average 9.1 cm (Lung Cancer 2011;73:127)
        • Lobulated with cystic, necrotic and hemorrhagic degeneration
        Gross images

        Images hosted on other servers:
        Missing Image

        Solid mass

        Missing Image

        Hemorrhagic mass

        Microscopic (histologic) description
        Microscopic (histologic) images

        Contributed by Hongxing Gui, M.D., Ph.D. and University of Michigan Virtual Slide Box
        Epithelial and mesenchymal components Epithelial and mesenchymal components

        Epithelial and mesenchymal components

        Glandular epithelium and blastematous stroma Glandular epithelium and blastematous stroma

        Glandular epithelium and blastematous stroma


        TTF1

        TTF1

        Vimentin

        Vimentin

        Beta catenin

        Beta catenin

        Positive stains
        Negative stains
        Electron microscopy description
        Molecular / cytogenetics description
        Sample pathology report
        • Lung, right lower lobe, lobectomy:
          • Pulmonary blastoma, 9 cm, margins negative for tumor (see comment)
          • Comment: H&E sections demonstrate an admixture of malignant epithelial and mesenchymal components. Immunostains performed with adequate controls reveal that the glandular cells are strongly positive for pancytokeratin and TTF1. The stromal cells are vimentin positive. Beta catenin reveals nuclear staining in both epithelial and stromal components. The findings are consistent with the diagnosis of pulmonary blastoma, a subgroup of sarcomatoid carcinoma.
        Differential diagnosis
        • Fetal type adenocarcinoma:
          • Epithelial component only, no blastematous stroma
        • Pleuropulmonary blastoma:
          • Pediatric tumor, most diagnosed before 4 years of age
          • No epithelial component; admixture of blastematous and sarcomatoid components
          • Epithelium (if seen) is benign entrapped respiratory type
          • 40% with DICER1 mutation
        • Biphasic synovial sarcoma:
        • Carcinosarcoma:
          • Non-small cell carcinoma (squamous cell carcinoma or adenocarcinoma) and heterologous sarcomatous component
          • Older age
          • Absence of morules and no nuclear staining of beta catenin
        Gene mutations commonly seen in different tumors
        CTNNB1 mutation P53 mutation DICER1 mutation
         Fetal adenocarcinoma   +   -   - 
         Pulmonary blastoma   +   +   Rare 
         Carcinosarcoma   -   +   - 
         Pleuropulmonary blastoma   -   +   + 
        Note: +, present; -, absent
        Board review style question #1
          Which of the following is true for pulmonary blastoma?

        1. Associated with smoking
        2. Low to moderate grade tumor
        3. Lung counterpart of Wilms tumor
        4. Most common in young patients < 15 years old
        5. Mostly in a central location
        Board review style answer #1
        A. Smoking is a risk factor of pulmonary blastoma

        Comment Here

        Reference: Pulmonary blastoma
        Board review style question #2
        Missing Image Missing Image


          What distinguishes pulmonary blastoma, shown here, from primary lung carcinosarcoma?

        1. Fewer mitotic figures
        2. Less common for metastasis
        3. Less cytologic atypia
        4. Less necrosis
        5. Nuclear immunostaining of beta catenin
        Board review style answer #2
        E. Pulmonary blastoma is positive for beta catenin, which is negative in lung carcinosarcoma

        Comment Here

        Reference: Pulmonary blastoma

        Pulmonary capillary hemangiomatosis
        Definition / general
        • Proliferation of benign appearing capillaries in alveolar septa that appear to compress pulmonary veins
        • May represent a secondary angioproliferative process of veno-occlusive disease caused by postcapillary obstruction rather than a distinct entity (Am J Surg Pathol 2006;30:850)
        • Poor prognosis, median survival after symptom onset is 3 years
        Essential features
        • Disordered proliferation of capillaries within interstitial tissue, including involvement of larger pulmonary vessel walls and airways
        • Closely related to pulmonary veno-occlusive disease (PVOD) and could be secondary process, manifests as pulmonary hypertension
        • CD31 and CD34 immunohistochemistry can help distinguish capillary proliferation from congestion
        Terminology
        • Controversial whether a distinct entity from pulmonary veno-occlusive disease (PVOD)
        • Consider diagnosis of "secondary PCH" or PCH-like changes for proliferations seen in association with other disorders, reserving "primary PCH" for extremely rare cases without recognizable causative background disease (Am J Surg Pathol 2006;30:850)
        Epidemiology
        • Very rare, mostly in adults
        • Mean age 30 years, range 2-71 years
        Sites
        Pathophysiology
        • Chronic passive congestion may lead to capillary proliferation
        Etiology
        • Not known; thought to be neoplastic but not entirely clear
        Clinical features
        • Clinically mimics PVOD or atypical interstitial lung disease due to presence of pulmonary hypertension
        • Nonspecific symptoms: progressive dyspnea, cough, chest pain, fatigue, small amount of hemoptysis
        • Pulmonary function tests show normal FVC and FEV with reduced diffusion capacity
        Diagnosis
        • Clinically and radiographically indistinguishable from PVOD; diagnosis requires microscopic diagnosis by lung biopsy
        • Diagnosis frequently made on explant specimen or after death
        Radiology description
        • Findings seen in pulmonary hypertension; i.e. cardiomegaly, enlarged pulmonary arteries
        • Chest Xray: interstitial infiltrates, thick interlobular septa, bibasilar reticulonodular or micronodular areas of opacity, lymphoadenopathy
        • HRCT: diffuse centrilobular ground glass opacity (Cardiovasc Pathol 2013;22:287), peripheral sparing
        Prognostic factors
        • Prostacyclin therapy (used in pulmonary hypertension) may cause sudden respiratory distress and death in these patients
        Case reports
        Treatment
        • Lung transplantation is only definitive treatment but recurrence has been reported
        • Drugs for other causes of pulmonary hypertension are relatively ineffective
        • Case reports suggest Interferon α-2a or doxycycline may be effective
        Clinical images

        Images hosted on other servers:
        Category 1 pulmonary hypertension<br>diagnosed postmortem as secondary to<br>pulmonary capillary hemangiomatosis (PCH)

        Category 1 pulmonary hypertension
        diagnosed postmortem as secondary to
        pulmonary capillary hemangiomatosis (PCH)

        Category 1 pulmonary hypertension<br>due to a second case of pulmonary<br>capillary hemangiomatosis

        Category 1 pulmonary hypertension
        due to a second case of pulmonary
        capillary hemangiomatosis

        Gross description
        • Multiple, red-brown, ill defined, nodular lesions
        • Congested, edematous, without significant fibrosis
        Gross images

        Images hosted on other servers:
        Left lung with pulmonary parenchyma

        Left lung with pulmonary parenchyma

        Microscopic (histologic) description
        • Preserved architecture with areas of involvement admixed with areas of normal lung
        • Proliferation of benign appearing capillaries expanding alveolar septa that appear to compress pulmonary veins
        • Proliferation around bronchovascular bundles creating nodular appearance
        • At least 2 layers of aberrant capillaries within alveolar wall (Arch Pathol Lab Med 2015;139:274)
        • Bland endothelial cells, no mitoses
        • Intra-alveolar hemosiderin-laden macrophages, small areas of acute or old hemorrhage, hemosiderosis
        • Small pulmonary arteries with intimal thickening/medial hypertrophy
        Microscopic (histologic) images

        Contributed by Yale Rosen, M.D.
        Pulmonary veno-occlusive disease - Capillary dilatation

        Pulmonary veno-
        occlusive disease -
        Capillary dilatation

        Pulmonary capillary hemangiomatosis Pulmonary capillary hemangiomatosis

        Pulmonary capillary hemangiomatosis



        Images hosted on other servers:
        (A)small veins demonstrate marked myointimal thickening; adjacent alveolar septa are<br>thickened by endothelial cell proliferation of pulmonary capillary hemangiomatosis;<br>(B) CD31 highlights endothelial cell proliferation

        (A)small veins demonstrate marked myointimal thickening; adjacent alveolar septa are
        thickened by endothelial cell proliferation of pulmonary capillary hemangiomatosis;
        (B) CD31 highlights endothelial cell proliferation


        Pulmonary capillary hemangiomatosis Pulmonary capillary hemangiomatosis Pulmonary capillary hemangiomatosis

        Pulmonary capillary hemangiomatosis

        Cytology description
        • Alveolar lavage often with increased hemosiderin-laden macrophages
        Positive stains
        Molecular / cytogenetics description
        Differential diagnosis
        • Capillary congestion
        • Veno-occlusive disease: associated with PCH; eccentric intimal thickening of venules in lobular septa with some septal veins completely occluded, increased elastic fibers in venous media (highlighted with elastin stain); intimal fibrosis narrows and occludes the pulmonary veins, causing dilatation of capillaries

        Pulmonary edema
        Table of Contents
        Clinical features
        Clinical features
        • Due to hemodynamic disturbances (cardiogenic) or local microvascular injury

        Hemodynamic disturbances:
        • Due to increased hydrostatic pressure from congestive heart failure
        • Lungs are wet and heavy, fluid initially at base of lower lobes because hydrostatic pressure is greater here
        • Congestion, fluid and hemosiderin laden macrophages (heart failure cells) are present
        • Later fibrosis and thickening of alveolar walls (brown induration of lung)

        Local microvascular injury:
        • Injury causes leakage of fluids and proteins into interstitial space, eventually into alveoli
        • When diffuse, contributes to acute respiratory distress syndrome

        Pulmonary hamartoma
        Definition / general
        • Benign neoplasm composed of varying amounts of mesenchymal elements (cartilage, fat, connective tissue, smooth muscle) with entrapped respiratory epithelium
        • At least 2 mesenchymal elements should be present for diagnosis
        Essential features
        • Most common benign pulmonary neoplasm, often incidentally found
        • Benign neoplasm composed of varying amounts of at least 2 mesenchymal elements combined with entrapped respiratory epithelium
        • Radiographic features often very characteristic
        Terminology
        • Other names previously used in the literature (chondroid hamartoma, mesenchymoma, chondromatous hamartoma, hamartochondroma) are no longer recommended
        ICD coding
        • ICD-O: 8992/0 - pulmonary hamartoma
        • ICD-11: 2F00.Y & XH3UD9 - other specified benign neoplasm of middle ear or respiratory system & pulmonary hamartoma
        Epidemiology
        Sites
        Pathophysiology
        Etiology
        • Thought to arise from mesenchymal tissue
        • Chromosomal aberrations may be a primary event in pulmonary hamartoma but unclear what other genetic events may be necessary to induce tumor development (Cancer Genet Cytogenet 2002;138:160)
        Clinical features
        Diagnosis
        Radiology description
        • Small coin lesion, often solitary, characterized by smooth edge, focal fat or fat alternating with calcific foci (Ann Thorac Med 2015;10:231)
        • Calcifications often described as popcorn calcification
        • Intralesional fat and popcorn-like calcifications allow a confident diagnosis on CT (J Thorac Imaging 2016;31:11)
        Radiology images

        Contributed by Jeffrey P. Kanne, M.D.
        Hamartoma with calcifications

        With calcifications

        Hamartoma CT scan

        CT scan

        Prognostic factors
        Case reports
        Treatment
        Clinical images

        Images hosted on other servers:

        Bronchoscopy

        Large endobronchial hamartoma

        Large endobronchial mass

        Gross description
        • Pale, firm, round to multilobulated, well circumscribed nodule (most are < 4 cm in diameter)
        • Endobronchial hamartoma manifests as yellow to gray sessile polyps of large airways
        • References: Mayo Clin Proc 1996;71:14, Thorax 1987;42:790
        Gross images

        Contributed by @Andrew_Fltv on Twitter
        Pulmonary hamartoma Pulmonary hamartoma Pulmonary hamartoma

        Pulmonary hamartoma



        Images hosted on other servers:

        Pneumonectomy specimen

        Various images

        Firm and discrete lesions

        Various images

        Resected endobronchial tumor

        Microscopic (histologic) description
        • Composed predominantly of varying degrees of mesenchymal tissue: hyaline cartilage, fat, smooth muscle and bone
        • Other connective tissue elements may be present, represented by bland spindle cells, fibrous tissue or myxoid change
        • Variably conspicuous, entrapped benign epithelial cells
        • Reference: Thorax 1987;42:790
        Microscopic (histologic) images

        Contributed by Hui-Hua Li, M.D., Ph.D. and Jefree J. Schulte, M.D.
        Hamartoma with cartilage

        With cartilage

        Fat rich hamartoma

        Fat rich

        Hamartoma, typical features

        Typical features

        Hamartoma, fat / myxoid stroma

        Fat / myxoid stroma


        Hamartoma, bland spindle cells

        Bland spindle cells

        Pulmonary hamartoma core biopsy

        Core biopsy

        Pulmonary hamartoma biopsy, cartilage

        Biopsy, cartilage

        Pulmonary hamartoma biopsy, epithelium

        Biopsy, epithelium



        Contributed by @Andrew_Fltv on Twitter
        Pulmonary hamartoma Pulmonary hamartoma

        Pulmonary hamartoma

        Cytology description
        • FNA is highly reliable means of diagnosis (Cytopathology 2008;19:185)
        • Misclassification as malignancy can occur, especially at the time of immediate adequacy assessment, prior to review of all cytologic preparations (Cytopathology 2008;19:185, BMC Pulm Med 2003;3:2)
        • FNA often shows fibromyxoid tissue, fat or cartilage associated with benign reactive epithelial cells
        • Lack of cytologic atypia and bland chromatin should point towards a benign diagnosis (Acta Cytol 2009;53:201)
        Cytology images

        Images hosted on other servers:

        FNA smears

        Positive stains
        • Immunohistochemical markers are not typically needed to confirm diagnosis
        • Immunohistochemical profile should be typical for the mesenchymal tissue being stained
        • Interestingly, sex steroid receptors (ER, PR, AR) are often expressed (Am J Surg Pathol 2006;30:819)
        Molecular / cytogenetics description
        • Often not needed for diagnostic purposes
        • High frequency of translocation t(3;12)(q27-28;q14-15) resulting in gene fusion of the HMGA2 and LPP genes (see Pathophysiology and Etiology above)
        Sample pathology report
        • Lung, left, needle core biopsy:
          • Pulmonary hamartoma (see comment)
          • Comment: The histologic sections reveal the presence of a benign pulmonary lesion composed of hyaline cartilage, fibroadipose tissue, bone and bland spindle cells in myxoid stroma. The mesenchymal elements appear to entrap benign respiratory epithelium. These findings support the above diagnosis.
        • Lung, right, wedge resection:
          • Pulmonary hamartoma (3.2 cm) (see comment)
          • Comment: The histologic sections reveal the presence of a well circumscribed benign pulmonary lesion composed of hyaline cartilage, fibroadipose tissue, smooth muscle and bland spindle cells set in fibromyxoid stroma. The mesenchymal elements appear to entrap benign respiratory epithelium. These findings support the above diagnosis.
        Differential diagnosis
        • Pulmonary chondroma:
          • Lacks entrapped epithelium
          • Does not contain secondary mesenchymal elements
          • Often seen in patients with Carney triad
        • Other soft tissue tumors (including leiomyoma and lipoma):
          • Only 1 mesenchymal component may be sampled at the biopsy
          • Radiographic (popcorn calcifications and fat) findings may help suggest the diagnosis of hamartoma
        • Primary / metastatic sarcoma:
          • Cytologic atypia and features of necrosis should be present for diagnosis of sarcoma
        Board review style question #1

        A 67 year old man died in a work related accident. At autopsy, a 1 cm, circumscribed nodule is identified in the peripheral lung. The cut surface reveals a tan-white rubbery nodule. The histologic findings are seen in the image above. The incidentally discovered tumor is best classified as

        1. Non small cell carcinoma
        2. Pulmonary chondroma
        3. Pulmonary hamartoma
        4. Pulmonary sequestration
        Board review style answer #1
        C. Pulmonary hamartoma

        Comment Here

        Reference: Pulmonary hamartoma
        Board review style answer #2
        A 60 year old man presents to the emergency department following a motor vehicle accident. A chest CT scan reveals a circumscribed solitary pulmonary nodule containing fibroadipose tissue and calcifications. The patient meets with a thoracic surgeon, who does not immediately recommend excision. The patient is followed by serial CT scans and the nodule shows minimal growth over the next year. The characteristics of this pulmonary nodule are those of

        1. High grade neuroendocrine carcinoma of the lung
        2. Metastatic carcinoma to the lung
        3. Non small cell lung carcinoma
        4. Pulmonary hamartoma
        Board review style answer #2
        D. Pulmonary hamartoma

        Comment Here

        Reference: Pulmonary hamartoma

        Pulmonary hypertension
        Clinical features
        • Defined clinically as mean resting pulmonary artery pressure > 25 mm Hg or systolic > 30 mm Hg, usually not symptomatic until 60 mm Hg
        • Also defined as 25% or more of systemic pressure (normal is 10%)
        • Classified based on consensus conference (J Am Coll Cardiol 2009;54:43)

        Pulmonary arterial hypertension (PAH):
        • Idiopathic
        • Familial
        • Associated with:
          • Collagen vascular disease
          • Congenital systemic to pulmonary shunts
          • Portal hypertension
          • HIV infection
          • Drugs and toxins (historically diet drug fenfluramine and phentermine fen-phen)
          • Other
        • Associated with significant venous or capillary involvement:
          • a) Pulmonary venous occlusive disease
          • b) Pulmonary capillary hemangiomatosis

        Pulmonary venous hypertension:
        • Left sided atrial or ventricular heart disease
        • Left sided valvular heart disease

        Pulmonary hypertension associated with hypoxemia:
        • COPD
        • Interstitial lung disease
        • Sleep-disordered breathing
        • Alveolar-hypoventilation disorders
        • Chronic exposure to high altitudes

        Pulmonary hypertension due to chronic thrombotic or embolic disease:
        • Thromboembolic obstruction of proximal pulmonary arteries
        • Thromboembolic obstruction of distal pulmonary arteries
        • Nonthrombotic pulmonary embolism (tumor, parasites and foreign material)
        • Miscellaneous (sarcoidosis, Langerhan cell histiocytosis and compression of pulmonary vessels)

        • In secondary forms, endothelial cell dysfunction initiates the disorder; causes decreased production of nitric oxide and prostacyclin and increased levels of endothelin, leading to endothelial cell activation, constriction and thrombogenesis; usually women ages 20 - 40 years; some patients have vasospastic element

        Idiopathic:
        • Sporadic, requires exclusion of other known causes
        • Usually women 20 - 40 years old
        • Sometimes children are initially asymptomatic, then shortness of breath, fatigue, angina, progressing to right ventricular hypertrophy, cor pulmonale, pulmonary emboli or pneumonia

        Familial:
        • Rare, autosomal dominant with incomplete penetrance
        • Mutations in bone morphogenic protein receptor 2 signalling pathway (BMPR2)
        Treatment
        • Vasodilators, calcium channel blockers, nitric oxide, antithrombotic medications, prostacyclin analogues, endothelial cell receptor antgonists, phosphodiesterase-5 inhibitors and lung transplant in selective patients
        • Disease reversible if arterial lesions restricted to medial hypertrophy, intimal thickening of longitudinal smooth muscle or cellular intimal proliferation
        Gross images

        Images hosted on other servers:
        Atherosclerosis

        Atherosclerosis

        Microscopic (histologic) description
        • Organizing thrombi (suggests recurrent pulmonary emboli)
        • Interstitial fibrosis if hypoxia
        • Changes in major vessels / branches are similar to systemic atherosclerosis
        • Small vessels have medial hypertrophy and intimal fibrosis, which may narrow lumina to pinpoint
        • Plexogenic arteriopathy: tuft of capillaries spanning lumina of arteries, changes can be quantified using Reid index

        Pulmonary placental transmogrification
        Definition / general
        • Pulmonary placental transmogrification is a rare lesion, characterized by cystic lesions of the lung
        • First described in 1979 by McChesney (Lab Invest 1979;40:245)
        Essential features
        • Placental transmogrification or placentoid bullous lesion of the lung is an unusual condition in which the alveoli develop a peculiar villous configuration that resembles placental villi at low microscopic magnification
        • Villous appearance probably results from the development of edema and fibrosis in the residual strands of alveolar tissue present in the enlarged airspaces of severe emphysema
        • Placental transmogrification of the lung has been described in patients with severe emphysema induced by cigarette smoking, congenital giant bullous emphysema and fibrochondromatous hamartomas of the lung
        Terminology
        • Pulmonary placental transmogrification, bullous emphysema, placentoid bullous lesion
        ICD coding
        • ICD-10: J98.4 - Other disorders of lung
        Epidemiology
        • Usually occurs in men aged 20 - 50
        • 48% have a history of smoking
        Sites
        • Lung parenchyma, usually subpleural and unilateral
        Etiology
        • Etiology and pathogenesis remains unclear
        • Hypotheses
          • Variant of giant bullous emphysema
          • Initial clear cell proliferation followed by emphysema-like cystic change; uncertain if proliferation is clonal (Hum Pathol 2004;35:517)
          • Congenital hamartomatous malformation
        • Can have fatty infiltration (Mod Pathol 1997;10:846)
          • May be the result of metaplastic mesenchymal differentiation
          • Occasionally, pulmonary lipomatosis is reported as a variant of placental transmogrification
        Clinical features
        • Patients might be asymptomatic or present with dyspnea, chronic obstructive lung disease, pneumothorax or a combination of these
        • Chest tightness, cough and chest pain
        Radiology description
        Prognostic factors
        • A few patients with incomplete resection of pulmonary bullae have recurrent disease
        Case reports
        Treatment
        • Surgical resection, including video assisted thoracoscopic surgery (VATS), pneumonectomy or lobectomy
        • Surgical resection is usually curative and leads to successful improvement of symptoms and quality of life
        • Lymph node dissection is unnecessary
        Gross description
        • Resembles cystic lesions with variable amounts of intracystic papillary proliferation
        • Affected areas are replaced or filled by gelatinous tissues described as bubbly, vesicular, grape-like or sponge-like
        Gross images

        Images hosted on other servers:

        Massively dilated bullae

        Fatty infiltration

        Fatty infiltration

        Microscopic (histologic) description
        • Bullous emphysema / cystically dilated airspaces, usually subpleural
        • Intracystic proliferation of variable sized papillary structures, which are morphologically similar to mature chorionic villi
        • Cores of papillary / villous structures contain congested capillary sized vessels
        • Papillary / villous lesions are surrounded by hyperplastic alveolar pneumocytes
        • Infiltration of mature adipocytes is sometimes identified in papillary / villous lesions
        • Pleural reactive changes are present and usually secondary to pneumothorax, including granulation proliferation, acute hemorrhage and infiltration of eosinophils
        Microscopic (histologic) images

        Contributed by Jian-Hua Qiao, M.D.
        Foci of subpleural bullous emphysema Foci of subpleural bullous emphysema

        Foci of subpleural bullous emphysema

        Clusters of mature placental chorionic villi-like structures Clusters of mature placental chorionic villi-like structures Clusters of mature placental chorionic villi-like structures Clusters of mature placental chorionic villi-like structures

        Clusters of mature placental chorionic villi-like structures



        Images hosted on other servers:

        Fatty infiltration

        Positive stains
        Negative stains
        Molecular / cytogenetics description
        • Interstitial clear cells were dissected by laser capture microdissection in 2 cases, DNA of interstitial clear cells were extracted and used for studies of 19 microsatellites by PCR (polymerase chain reaction) and automatic sequencer (Hum Pathol 2004;35:517)
          • Microsatellite alterations were observed in 13 markers in case 1 and in 8 markers in case 2
          • Loss of heterozygosity (LOH) was found in 1 chromosomal region in case 1 and none of the tested regions in case 2
        Differential diagnosis
        Board review style question #1
        Which of the following statements applies to current practice in pulmonary placental transmogrification?

        1. Complete mediastinal or hilar lymph node dissection is necessary
        2. It usually occurs in women aged 20 - 50
        3. Lesion is negative for TTF1
        4. No patients had a history of smoking
        5. Surgical resection is the treatment of choice
        Board review style answer #1
        E. Surgical resection is usually curative and leads to successful improvement of symptoms and quality of life.

        Comment Here

        Reference: Pulmonary placental transmogrification
        Board review style question #2
        A young man with pneumothorax and pleural blebs had a VATS lung wedge biopsy performed. Microscopic examination revealed a subpleural emphysematous lesion with the following papillary lesions in dilated airspaces. What is your diagnosis?



        1. Bronchogenic cyst
        2. Congenital cystic adenomatoid malformation
        3. Cystic non small cell lung cancer
        4. Intralobar pulmonary sequestration
        5. Pulmonary placental transmogrification
        Board review style answer #2
        E. Pulmonary placental transmogrification. Papillary lesions are morphologically identical to mature chorionic villi of placental tissue. When these papillary / villous lesions present in dilated airspaces of resected lung, it is diagnostic for pulmonary placental transmogrification

        Comment Here

        Reference: Pulmonary placental transmogrification

        Pulmonary zoonoses
        Clinical features
        • Zoonoses are diseases and infections naturally transmitted between people and vertebrate animals (WHO: Zoonotic Disease: Emerging Public Health Threats in the Region [Accessed 18 August 2021])
        • 3 classes:
          1. Endemic zoonoses which are present in many places and affect many people and animals
          2. Epidemic zoonoses which are sporadic
          3. Emerging and re-emerging zoonoses which are newly appearing in a population or have existed previously but are rapidly increasing in incidence
        Hantavirus
        Rhodococcus
        • Zoonotic infection originally identified in lungs of foals; also cattle, swine, sheep, deer, goats, dogs, cats, other species
        • Worldwide distribution in water and soil, especially near feces of herbivores
        • Infection by inhalation or ingestion
        • Resides primarily within lung macrophages of infected patients, also some alveolar cells (Microbes Infect 2011;13:438)
        • R. equi is most common human pathogen of the Rhodococci
        • Generally occurs in AIDS patients (may cause malakoplakia) (Clin Infect Dis 1999;28:1334)
          • Also other immunodeficient patients; very rarely in immunocompetent
        • Incidence decreasing, likely due to HAART therapy and azithromycin prophylaxis
        • Rhodo since salmon pink pigment in culture
        • Treated with antibiotics but mortality is 50% in AIDS patients, 20% in other immunocompromised and 10% in immunocompetent
          Case reports
          Microscopic (histologic) description
          • Hantavirus: interstitial pneumonitis with mononuclear infiltrate, edema, focal hyaline membranes, pleura effusions; atypical lymphocytes in pulmonary vasculature may mimic lymphoma
          • Rhodococcus: epithelioid macrophages containing gram positive, partially acid fast coccobacilli that resemble mycobacteria; may produce malakoplakia
          Microscopic (histologic) images

          Images hosted on other servers:
          Various images Various images

          Rhodococcus

          Electron microscopy images

          Images hosted on other servers:
          Transmission electron<br>micrograph of<br>Sin Nombre hantavirus

          Transmission electron
          micrograph of
          Sin Nombre hantavirus


          Radiation induced lung disease
          Table of Contents
          Clinical features | Treatment
          Clinical features
          • Incidence of severe disease decreasing due to better therapeutic modalities
          • Symptoms: fever, shortness of breath, radiologic infiltrates, diffuse alveolar damage and type II pneumocyte atypia
          • Acute radiation pneumonitis: 1 - 6 months after therapy, occurs in 10% - 20% of patients
          • Chronic radiation pneumonitis: interstitial fibrosis, epithelial atypia and foam cells in vessel walls
          Treatment
          • Steroids

          Radiology-pathology correlation (pending)
          [Pending]

          Respiratory bronchiolitis
          Definition / general
          • Common histologic marker of current or former smoker status
          • Usually an incidental pathologic finding but can occasionally be associated with clinically significant interstitial lung disease (respiratory bronchiolitis interstitial lung disease [RB-ILD])
          Essential features
          • Respiratory bronchiolitis present in virtually all smokers
          • Characterized by the accumulation of lightly pigmented (smoker's) macrophages in distal airways and peribronchiolar airspaces with or without mild inflammation and fibrosis in adjacent interstitium
          • Diagnosis of respiratory bronchiolitis interstitial lung disease relies on pathologic, clinical and radiographic correlation and is a diagnosis of exclusion
          Terminology
          • Smoker's bronchiolitis
          ICD coding
          • ICD-10: J84.115 - respiratory bronchiolitis interstitial lung disease
          Epidemiology
          Sites
          • Distal airways of lungs
          Pathophysiology
          • Unknown at this time
          Etiology
          • Smoking
          Clinical features
          • Nearly all patients with respiratory bronchiolitis asymptomatic
          • Respiratory bronchiolitis interstitial lung disease patients often symptomatic (Chest 2005;127:178, Chest 2007;131:664)
            • 90% dyspnea; 50% cough
            • Pulmonary function tests: obstructive or restrictive > normal > mixed patterns
          Diagnosis
          • Diagnosis of respiratory bronchiolitis interstitial lung disease requires correlating clinical, radiologic and pathologic findings:
            • Clinical: patient symptoms, pulmonary function testing
            • Radiologic: chest Xray, high resolution computed tomography (HRCT)
            • Pathologic: histologic examination of lung wedge biopsy
          • Diagnosis of respiratory bronchiolitis interstitial lung disease only applicable after other forms of diffuse lung disease have been ruled out
          Radiology description
          Radiology images

          Images hosted on other servers:

          RB-ILD chest radiograph and CT scan

          Prognostic factors
          Case reports
          • 11 year old boy with respiratory bronchiolitis interstitial lung disease due to heavy second hand smoke exposure (Pediatrics 2015;136:e1026)
          • 33 year old man with respiratory bronchiolitis interstitial lung disease thought to be related to electronic nicotine delivery system (Respirol Case Rep 2017;5:e00230)
          • 2 cases of respiratory bronchiolitis interstitial lung disease in men under 50 initially diagnosed with idiopathic pulmonary fibrosis (Chest 2000;117:282)
          Treatment
          • Smoking cessation
          Gross description
          • No gross abnormality
          Microscopic (histologic) description
          • Lightly pigmented (smoker's) macrophages limited to distal airspaces and peribronchiolar airspaces
          • Pigment within macrophage cytoplasm is yellow to light brown and finely granular
          • Minimal to absent peribronchiolar interstitial thickening by fibrosis
          • No diffuse interstitial fibrosis or inflammation
          Microscopic (histologic) images

          Contributed by Kristine Konopka, M.D.

          Macrophage accumulation

          Smoker's macrophages

          Iron stain

          Positive stains
          • Iron (e.g. Prussian blue)
          Negative stains
          • None relevant to this diagnosis
          Sample pathology report
          • Lung, right upper, middle and lower lobes, wedge biopsies:
            • Respiratory bronchiolitis (see comment)
            • Comment: Respiratory bronchiolitis (RB) is a common incidental finding in current or former smokers and is usually of little clinical consequence. In a subset of patients, however, this is the only pathologic finding and seems to account for clinical interstitial lung disease. In this scenario, a diagnosis of respiratory bronchiolitis interstitial lung disease may be applicable.
          Differential diagnosis
          • Desquamative interstitial pneumonia (DIP):
            • Substantial histologic overlap with respiratory bronchiolitis interstitial lung disease
            • Diffuse accumulation of pigmented macrophages beyond the peribronchiolar airspaces into the distal airspaces
            • Degree of interstitial fibrosis and inflammation usually greater than in respiratory bronchiolitis interstitial lung disease
          • Hemosiderosis:
            • Potential etiologies: diffuse alveolar hemorrhage or chronic passive vascular congestion
            • Hemosiderin pigment darker and coarser than smoker's pigment
            • Iron stain can help in highlighting iron encrustation of vascular elastica (endogenous pneumoconiosis), a finding seen in chronic hemorrhage or chronic passive congestion
          Board review style question #1

          The findings in this lung wedge biopsy are associated with

          1. Aspiration
          2. Cigarette smoking
          3. Exposure to inhaled bird antigens
          4. Median survival of 2.5 years from diagnosis
          5. Type I hypersensitivity reaction
          Board review style answer #1
          B. Cigarette smoking. The image shows respiratory bronchiolitis, characterized by the accumulation of pigmented (smoker's) macrophages within distal airspaces. Aspiration usually results in amorphous or birefringent particulates within distal airspaces. Answer C is associated with hypersensitivity pneumonia, while type I hypersensitivity reactions are associated with atopic asthma. Respiratory bronchiolitis is a marker of current or former smoker status and is usually of no clinical consequence with no deaths attributable to it.

          Comment Here

          Reference: Respiratory bronchiolitis
          Board review style question #2
          A 64 year old man with extensive smoking history undergoes lung wedge biopsy for evaluation of progressive exertional dyspnea and a restrictive defect on pulmonary function tests. Sections of the wedge biopsy show accumulation of lightly pigmented macrophages within distal airspaces. This is accompanied by patchy interstitial fibrosis that is associated with areas of architectural distortion and lung remodeling in the forms of scarring and honeycomb change. Based on these findings, which of the following is the best diagnosis?

          1. Desquamative interstitial pneumonia (DIP)
          2. Diffuse alveolar hemorrhage
          3. Respiratory bronchiolitis interstitial lung disease (RB-ILD)
          4. Smoking related interstitial fibrosis (SRIF)
          5. Usual interstitial pneumonia (UIP)
          Board review style answer #2
          E. Usual interstitial pneumonia (UIP). Although the features of respiratory bronchiolitis are present in the wedge biopsy sections and the patient has clinically significant interstitial lung disease, the diagnosis of respiratory bronchiolitis interstitial lung disease requires that other forms of diffuse lung disease have been ruled out. In this case, the findings of patchy interstitial fibrosis and honeycombing are consistent with usual interstitial pneumonia, which is the best explanation for this patient's symptoms and pulmonary function testing results.

          Comment Here

          Reference: Respiratory bronchiolitis

          Rheumatoid lung disease
          Definition / general
          • Variable presentation of chronic pleuritis, effusion, diffuse interstitial fibrosis, intrapulmonary rheumatoid nodules, rheumatoid nodules with pneumoconiosis (Caplan syndrome), pulmonary hypertension, bronchiolitis obliterans, amyloidosis and vasculitis
          • Note: 30% with rheumatoid arthritis have abnormal pulmonary function
          • Rheumatoid nodules are associated with active joint disease and elevated titers of rheumatoid factor
          Treatment
          • Steroids (thus, must first rule out an infectious process)
          Gross description
          • Pleural fibrosis, rheumatoid nodules and pneumonia
          Microscopic (histologic) description
          • Rheumatoid granuloma usually in lung periphery near pleura
          • Round with zone of pallisading histiocytes perpendicular to necrobiotic center
          • Adjacent vasculitis
          • Nodules have carbon and silica within the necrotic center in Caplan syndrome

          RSV
          Clinical features
          • Leading cause of lower respiratory tract infection in infants and young children (eMedicine)
          • Often in children under 2 years old; may cause death in infants 1 - 6 months
          • Usually self limited infection in older children
          • Infects respiratory epithelium, leading to acute inflammatory response with epithelial sloughing and diffuse alveolar damage in severe cases
          Microscopic (histologic) description
          • Prominent airway obstruction (Mod Pathol 2007;20:108)
          • Giant cells with inconspicuous, round, pink intracytoplasmic inclusions and acute inflammation
          • Affects small bronchiole epithelium, type 1 and 2 pneumocytes, not basal cells
          • Inflammation centered on bronchial and pulmonary arterioles, due to monocytes, T cells, neutrophils
          Microscopic (histologic) images

          Images hosted on other servers:
          Giant cells

          Giant cells


          Sarcoidosis
          Definition / general
          • Systemic disease characterized by granulomatous inflammation, typically nonnecrotizing, with 90% of overall cases showing involvement of the lungs and hilar lymph nodes
          • Subset of cases may involve the lung with sparing of lymph nodes
          Essential features
          • Granulomatous involvement of one or more tissues that is not attributable to other etiologies (including infectious)
          • Clinical correlation is necessary even in the presence of classic histology
          Terminology
          ICD coding
          • ICD-10:
            • D86.2 - sarcoidosis of lung with sarcoidosis of lymph nodes
            • D86.0 - sarcoidosis of lung
            • D86.9 - sarcoidosis, unspecified
          Epidemiology
          • Age of onset / diagnosis has shifted in recent years, with most cases now diagnosed at an older median age; most cases first occur in the fifth and sixth decades of life
          • Women are affected at twice the rate of men; Black patients have the highest incidence of disease, followed by White patients
          • Reference: Ann Am Thorac Soc 2016;13:1244
          Sites
          • Lungs, lymph node, cardiovascular, liver, skin, eye (see below under Clinical features)
          Pathophysiology
          • Believed to result from stimulation of CD4+ T cells by an unknown antigen
            • Downstream effects of cytokines and chemokines stimulate macrophages and lead to granuloma formation
          • Increased IFNγ and TNFα present in affected tissues
          • Increased CD4+ T cells are seen in bronchoalveolar lavages (BAL) performed on sarcoidosis patients compared to controls
          • Reference: Chest 2018;153:1432
          Etiology
          • Incompletely understood at this time
          • Sarcoidosis occurs more frequently in those with chronic inhalational exposures of silica and metal particulates
          • Reference: JAMA 2022;327:856
          Clinical features
          • Presentation: cough, dyspnea and other pulmonary symptoms are common; nonpulmonary presentations often include skin or eye lesions
          • Acute presentations may display fever, arthralgia and painful skin lesions
          • May be discovered incidentally following chest radiographs which often show hilar adenopathy and reticular opacities
          • Lungs
            • Reticular opacities on chest radiograph may be evident
            • High resolution CT may show multiple subcentimeter nodules with a perilymphatic distribution
            • Most cases will be associated with hilar lymphadenopathy
            • Pulmonary involvement in the absence of lymphadenopathy portends a worse prognosis
          • Lymph nodes
            • The vast majority of patients show hilar or mediastinal lymphadenopathy
            • Involved lymph nodes may be calcified or sclerotic
          • Cardiovascular
            • Symptomatic cardiac involvement is rare but autopsy studies have shown that > 50% of cases may show subclinical cardiac involvement
          • Liver
            • A subset of patients display liver involvement, often accompanied by hepatomegaly
            • Cirrhosis is a rare complication
          • Skin
            • Skin involvement in ~25%
            • Findings can include erythema nodosum or presence of numerous skin colored papules, especially on the face / nose
          • Eye
            • Common manifestations include uveitis and keratoconjunctivitis
          • References: JAMA 2022;327:856, Intern Emerg Med 2018;13:325, Ann Am Thorac Soc 2016;13:1244, EClinicalMedicine 2021;37:100966
          Diagnosis
          • Sarcoidosis is a clinicopathologic diagnosis; may be made in the appropriate clinical setting after exclusion of relevant differential diagnoses
          Laboratory
          • Multiple inflammatory markers may be altered in patients with sarcoidosis though most abnormalities are nonspecific
          • Angiotensin converting enzyme (ACE) is commonly elevated
            • Tracking ACE as well as certain acute phase reactants including C reactive protein and serum amyloid A may be useful in tracking disease activity in established disease
          • Elevated CD4 / CD8 ratio may be present on bronchoalveolar lavage (BAL)
          • Many patients display hypergammaglobulinemia
          • ~10% of patients have hypercalcemia
          • Hepatic involvement can cause abnormal liver function tests
          • References: JAMA 2022;327:856, Eur Respir J 1997;10:2716
          Radiology description
          • High resolution CT findings include perihilar opacities, micronodules with perilymphatic and bronchocentric distribution and variable fibrosis
          • Chest radiographs demonstrate bilateral hilar adenopathy in the majority of cases
          • Reference: Semin Respir Crit Care Med 2020;41:758
          Radiology images

          Images hosted on other servers:

          Some HRCT patterns

          Prognostic factors
          • 5 year mortality is ~7%
            • Most deaths (> 60%) are attributable to chronic pulmonary disease
          • Isolated pulmonary involvement is associated with worse outcomes
          • Clinical and radiographic findings are used to stage the disease (stages 0 - IV) (Radiopaedia: Thoracic sarcoidosis (staging) [Accessed 2 June 2023])
            • Lower stages are associated with increased rates of resolution
            • Higher stages are associated with increased mortality
          • Reference: JAMA 2022;327:856
          Case reports
          Treatment
          • Oral corticosteroids are primary therapy
          • Refractory cases may be treated with immunosuppressive agents
          • Reference: JAMA 2022;327:856
          Gross description
          • In most cases, the cut surface will demonstrate the presence of multiple small white to tan nodules along the bronchovascular bundle and in the pleura
          • Some cases may show larger, confluent nodularity
          Gross images

          Contributed by @AKhannaMBBS on Twitter
          Sarcoidosis Sarcoidosis

          Sarcoidosis



          Images hosted on other servers:

          Hilar lymphadenopathy in sarcoidosis

          Frozen section description
          • Well circumscribed granulomas, often containing multinucleated giant cells
          Microscopic (histologic) description
          • Well circumscribed granulomas that may coalesce to form large nodules
            • Typically nonnecrotizing
            • Multinucleated giant cells are common and may contain cytoplasmic inclusions (e.g., asteroid bodies or Schaumann bodies)
            • Distribution is submucosal and perilymphatic (along bronchovascular structures, intralobular septa and occasionally, the pleura)
            • Involvement of lung parenchymal or larger vessels may be seen
          • Lymphoplasmacytic infiltrate is variably present surrounding the sarcoid granulomas
          • Variable degrees of associated fibrosis are often present and can be extensive, especially in longstanding disease
          • Asteroid bodies and Schauman bodies are characteristic but not pathognomonic
            • Asteroid body: stellate cytoplasmic inclusion with peripheral clearing
            • Schaumann (conchoidal) body: cytoplasmic inclusion composed of concentric calcifications
          • References: JAMA 2022;327:856, Am J Respir Crit Care Med 2020;201:e26
          Microscopic (histologic) images

          Contributed by Aliya N. Husain, M.D. and @AKhannaMBBS on Twitter
          Multinulceated giant cells and fibrosis Multinulceated giant cells and fibrosis

          Multinulceated giant cells and fibrosis

          Granuloma with asteroid body

          Granuloma with asteroid body

          Mediastinal node with granulomas and fibrosis

          Mediastinal node with granulomas and fibrosis


          Granulomas of bronchovascular bundle and pleura

          Granulomas of bronchovascular bundle and pleura

          Granulomas with fibrosis

          Granulomas with fibrosis

          Sarcoidosis Sarcoidosis

          Sarcoidosis

          Cytology description
          • Epithelioid histiocytes with curved, often boomerang shaped nuclei; characteristically present in a syncytium
          • Fibrotic granulomas may be hypocellular
          Negative stains
          • Special stains for organisms (e.g., GMS, AFB) should be negative
          Sample pathology report
          • Lung, left upper lobe, wedge biopsy:
            • Numerous well formed granulomas, consistent with sarcoidosis (see comment)
            • Comment: The specimen shows numerous well formed granulomas, which demonstrate a predominantly bronchovascular and submucosal distribution and show associated fibrosis. Several associated multinucleated giant cells are present. The patient's clinical history and radiographic findings (hilar adenopathy and perilymphatic nodularity) are noted.
          Differential diagnosis
          • Berylliosis:
            • Diagnosis requires clinical history of exposure
          • Infectious causes (fungi, atypical mycobacteria):
            • GMS and AFB should be routinely performed on all pulmonary biopsies with granulomas
              • Tissue cultures increase sensitivity for infectious agents
              • If clinical suspicion for infectious etiology is high and tissue cultures are negative, RNA sequencing should be considered
          • Foreign body granulomas:
            • Polarization can aid in identifying foreign bodies; however, polarizing material may occasionally be present in giant cells of sarcoidosis
          Board review style question #1
          A 55 year old woman with clinically suspected sarcoidosis undergoes biopsy of mediastinal lymph nodes and wedge biopsy of involved lungs. Which of the following findings are most consistent with sarcoidosis?

          1. Lungs with poorly formed granulomas along the bronchovascular structures and lymph nodes with granulomatous lesions containing refractile material
          2. Lungs with poorly formed interstitial granulomas and lymph nodes with granulomatous lesions containing refractile material
          3. Lungs with well formed granulomas along bronchovascular structures and pleura and lymph nodes with well formed granulomas
          4. Lungs with well formed interstitial granulomas with refractile material and lymph nodes with reactive changes
          Board review style answer #1
          C. Lungs with well formed granulomas along bronchovascular structures and pleura and lymph nodes with well formed granulomas. Answers A, B and D are incorrect because sarcoid associated granulomas tend to be well formed and typically do not contain foreign material including refractile particles.

          Comment Here

          Reference: Sarcoidosis
          Board review style question #2

          The image above shows a mediastinal lymph node biopsy in a patient whose CT findings include mediastinal and hilar opacities as well as micronodularity of perilymphatic and bronchocentric distribution. What is the most likely diagnosis and which stains should be performed on the biopsy prior to signout?

          1. Diagnosis: infectious process; stains: Gram stain, AFB
          2. Diagnosis: infectious process; stains: trichrome, Gram stain
          3. Diagnosis: sarcoidosis; stains: AFB, GMS
          4. Diagnosis: silicosis; stains: AFB, GMS
          Board review style answer #2
          C. Diagnosis: sarcoidosis; stains: AFB, GMS. This section demonstrates a lymph node with multiple well formed, nonnecrotizing granulomas as well as fibrosis. These findings are characteristic of sarcoidosis. Answer D is incorrect because no fine particulate is seen, which makes silicosis less likely. Answers A and B are incorrect because the differential for granulomatous inflammation also includes infectious processes including fungal and atypical mycobacterial infections and GMS and AFB should be performed on all pulmonary / mediastinal lymph node specimens with granulomas. Negative special stains do not rule out this possibility; if clinical suspicion is high and special stains and cultures are negative, RNA sequencing may be considered to more definitively rule out infectious etiology. Remember that sarcoidosis is a diagnosis of exclusion.

          Comment Here

          Reference: Sarcoidosis

          SARS
          Clinical features
          • Severe Acute Respiratory Syndrome
          • Caused by SARS associated coronavirus, a new member of Coronaviridae (Am J Clin Pathol 2004;121:574)
          • Transmission by respiratory secretions
          • Antibodies may not appear until 28 days after onset; molecular tests have greatest yield during second week of illness (Arch Pathol Lab Med 2004;128:1346)
          • Outbreaks worldwide in 2002 - 2003 with more than 8,000 cases and over 750 deaths; the last reported case was laboratory associated and occurred in 2004
          • Virus localized to pneumocytes (Am J Clin Pathol 2004;121:574)
          Microscopic (histologic) description
          • Diffuse alveolar damage (DAD) varying based on duration of illness
          • 10 or fewer days: acute phase DAD, airspace edema, bronchiolar fibrin and small airway injury
          • 11+ days: organizing phase DAD, type II pneumocyte hyperplasia and marked reactive atypia, squamous metaplasia, multinucleated giant cells and acute bronchopneumonia
          • Acute phase DAD: hyaline membranes lining alveolar walls, interstitial and airspace edema, interstitial infiltrates of inflammatory cells and vascular congestion
          • Organizing phase DAD: fibroblast proliferation in interstitium and air spaces
          • Small airway injury: loss of cilia, bronchiole epithelial denudation, deposition of fibrin within the lumen and on exposed basement membranes
          • May have coinfections with CMV, Aspergillosis
          Positive stains
          Additional references

          Sclerosing pneumocytoma
          Definition / general
          • Pulmonary pneumocytoma are indolent tumors with a characteristic dual population of cuboidal surface cells resembling type II pneumocytes and stromal round cells
          • Displays varying amounts of hemangiomatous, papillary, sclerotic and solid histologic patterns
          Essential features
          • First described by Liebow and Hubbell in 1956 as sclerosing hemangioma (Cancer 1956;9:53)
          • Renamed pulmonary sclerosing pneumocytoma and categorized into adenoma in WHO 4th edition
          • Characteristic bland cytologic features with multiple growth patterns and a dual cell population of surface and round cells that can be highlighted by immunohistochemistry
          • Difficult to establish diagnosis on small biopsies, cytology or frozen sections and permanent sections are preferred
          Terminology
          • Obsolete term (not recommended): pulmonary sclerosing hemangioma
          ICD coding
          • ICD-O: 8832/0 - sclerosing pneumocytoma
          • ICD-11: 2F0Z & XH7436 - benign neoplasms of respiratory and intrathoracic organs, unspecified & sclerosing pneumocytoma
          Epidemiology
          Sites
          • Typically solitary and peripheral masses, in lower lobes of lung
          • Rarely multifocal endobronchial masses or localized in hilum, visceral pleura or mediastinum
          Pathophysiology
          Etiology
          Clinical features
          • Typically asymptomatic and incidental findings by routine radiological CT / Xray scans
          • Patients can present with symptoms based on localization and tumor size
          • Nonspecific respiratory symptoms: cough, sputum, hemoptysis and chest pain
          • Cases with lymph node metastasis have been reported, however, this does not adversely affect prognosis (Arch Pathol Lab Med 2003;127:321)
          Diagnosis
          • Histological diagnosis supported by clinical and radiological CT / Xray findings and corresponding ancillary immunohistochemical testing
          • Preoperative diagnosis: solitary pulmonary nodule, challenging histological diagnosis on small biopsies, cytology, cell block
          • Intraoperative diagnosis: malignancy cannot be ruled out, challenging histological diagnosis on frozen sections
          • Reference: World J Surg Oncol 2013;11:85
          Radiology description
          • Significant size variability, ranging from 1 to 8 cm in diameter, majority are < 3.5 cm
          • Xray: solitary, well defined nodular lesion (oval to rounded shadow), may show calcification and crescent radiolucency at the periphery (air crescent sign)
          • CT: well defined intraparenchymal nodular mass (often juxtapleural), may show calcification, air crescent sign and inhomogeneous enhancement
          • Can occasionally present as lobulated, pure or mixed ground glass nodules or with pleural traction, mimicking malignant tumors
          • Reference: Medicine (Baltimore) 2015;94:e498
          Radiology images

          Images hosted on other servers:

          Well defined nodular lesion in peripheral lung

          Prognostic factors
          Case reports
          Treatment
          • Mainstay treatment is surgical resection
          • Generally, sublobar wedge resection and lymph node dissection (with lymph node metastasis) is curative for peripheral small sized tumors (Korean J Thorac Cardiovasc Surg 2011;44:39)
          • Limited resection is favored with preservation of lung function if sufficient resection margin can be achieved (World J Surg Oncol 2022;20:140)
          Gross description
          • Well circumscribed mass, with varied gray-tan to yellow tumors
          • Can also present dark red with hard texture (hemorrhagic)
          • May find cystic degeneration and calcification
          • Significant size variability, ranging from 1 to 8 cm in diameter, majority are < 3.5 cm
          Gross images

          Images hosted on other servers:

          Well demarcated hemorrhagic peripheral lung lesion

          Frozen section description
          • Key diagnostic clues intraoperatively
            • Gross appearance is consistent with sclerosing pneumocytoma
            • Recognition of the dual cell population
            • Identification of different architectural patterns: papillary, sclerotic, solid and hemorrhagic
              • Clinical history of a nonsmoker younger patient with a well circumscribed peripheral lesion
          • Major diagnostic pitfalls: solid predominant pattern, hypercellularity, glandular spaces, desmoplasia-like sclerosis, cellular atypia, coagulative necrosis (Histopathology 2018;72:500)
          Frozen section images

          Contributed by Matthew J. Cecchini, M.D., Ph.D.
          Variable architecture

          Variable architecture

          Cell populations

          Cell populations

          Microscopic (histologic) description
          • Key feature is the presence of 2 cell types: cuboidal surface cells and round stromal cells
            • Cuboidal surface cells resemble type II pneumocytes
            • Round stromal cells have well defined border, with bland monotonous cells and fine chromatin
          • 4 histologic growth patterns have been described
            • Papillary: surface cells overlying hypercellular fibrovascular cores of round cells
            • Sclerotic: composed of hyalinized collagen, hemosiderin deposition in macrophages, cholesterol clefts and dystrophic calcification
            • Solid: sheets of round cells with tubular / incomplete adenoid structures surrounded by surface cells
            • Hemorrhagic: blood filled spaces lined by cuboidal epithelial cells
          • Most tumors contain at least 3 of the 4 growth patterns in various proportions (Histopathology 2018;72:500)
          Microscopic (histologic) images

          Contributed by Matthew J. Cecchini, M.D., Ph.D. and Pooja Navale, M.D.
          Core needle biopsy with various growth patterns

          Core needle biopsy with various growth patterns

          Core needle biopsy with 2 cell population

          Core needle biopsy with 2 cell population

          Well demarcated lesion

          Well demarcated lesion

          Solid growth architecture

          Solid growth architecture

          Papillary architecture

          Papillary architecture


          Hemorrhagic areas

          Hemorrhagic areas

          Prominent foamy macrophages

          Prominent foamy macrophages

          Whole mount view

          Whole mount view

          Solid tumor

          Solid tumor

          Surface cells and round cells

          Surface cells and round cells


          TTF1 highlights both cell populations

          TTF1 highlights both cell populations

          Pancytokeratin highlights the surface cells

          Pancytokeratin highlights the surface cells

          TTF1

          TTF1

          EMA

          EMA

          CAM 5.2

          CAM 5.2

          Cytology description
          • Cytomorphological findings of sclerosing pneumocytoma overlap with lung adenocarcinoma (Cancer Cytopathol 2020;128:414)
          • Most common cytological features (Diagn Cytopathol 2014;42:242)
            • Moderately to highly cellular
            • Papillary structures and large sheets
            • Small to medium sized cells with moderate / abundant cytoplasm
            • Bland round to oval nuclei with inconspicuous nucleoli
            • Hemorrhagic background (fresh blood, siderophages, free hemosiderin pigment)
          Positive stains
          Negative stains
          Molecular / cytogenetics description
          Videos

          Sclerosing pneumocytoma in small biopsy

          Sclerosing pneumocytoma in resection

          Sample pathology report
          • Right lung, lower lobe, wedge resection:
            • Sclerosing pneumocytoma (see microscopic description)
              • 1.5 cm in greatest dimension
              • Tumor present 1.0 cm from nearest (parenchymal) margin
            • 2 lymph nodes, negative for malignancy
          Differential diagnosis
          Board review style question #1
          What is the immunophenotype of the round cells in a sclerosing pneumocytoma?

          1. TTF1 negative, pancytokeratin negative
          2. TTF1 negative, pancytokeratin positive
          3. TTF1 positive, pancytokeratin negative
          4. TTF1 positive, pancytokeratin positive
          Board review style answer #1
          C. TTF1 positive, pancytokeratin negative. The round cells have a unique immunophenotype of TTF1 positive and cytokeratin negative, which is proposed to correspond to a primitive respiratory epithelium-like immunophenotype. Answer A is incorrect because the round cells have TTF1 expression. Answer B is incorrect because the round cells are keratin negative and TTF1 positive. Answer D is incorrect because the round cells are keratin negative.

          Comment Here

          Reference: Sclerosing pneumocytoma
          Board review style question #2

          The pigment seen adjacent to a sclerosing pneumocytoma likely relates to which of the following?

          1. Alveolar hemorrhage (hemosiderin deposition)
          2. Cannabis use
          3. Cigarette smoking
          4. Melanin pigment
          Board review style answer #2
          A. Alveolar hemorrhage (hemosiderin deposition). These tumors are often associated with hemorrhage and the pigment typically relates to hemosiderin. This can be a clue to the diagnosis. Answer B is incorrect because cannabis pigment has a more coarse brown appearance than hemosiderin. Answer C is incorrect because the smoking related pigment has a fine yellow-brown appearance with black specks. Answer D is incorrect because melanin has a fine dusty appearance compared to hemosiderin. Melanin is not typically associated with sclerosing pneumocytoma.

          Comment Here

          Reference: Sclerosing pneumocytoma

          Sequestrations
          Definition / general
          • Congenital lung malformation that is defined by an anomalous systemic vascular supply and sequestration from the tracheobronchial tree
          Essential features
          • Systemic feeding vessel is usually identified radiographically, documented in the surgical note and identifiable in intact gross specimens
          • Extralobar bronchopulmonary sequestrations are outside the normal pleural investment and can be associated with other congenital anomalies
          • Intralobar bronchopulmonary sequestrations are found within the normal pleura and are associated with an increased risk of infection
          • Sequestrations may rarely have an aberrant airway connection to the GI tract but usually the main airway ends blindly in the parenchyma at the site of the feeding vessel (Pediatr Dev Pathol 2007;10:75)
          Terminology
          • Also called pulmonary sequestration, intralobar sequestration (ILS), extralobar sequestration (ELS)
          • Hybrid lesion is a bronchopulmonary sequestration with cystic parenchymal changes similar to a type 2 congenital pulmonary airway malformation (CPAM)
          ICD coding
          • ICD-10: Q33.9 - congenital malformation of lung, unspecified
          Epidemiology
          Sites
          • Intralobar bronchopulmonary sequestrations are invested within the normal pleura and are almost always found in the lower lobes of the lung (Thorax 1979;34:96, J Pediatr Surg 2019;54:1286)
            • Systemic blood supply is most often from the thoracic aorta
            • Venous return is typically via the pulmonary veins
          • Extralobar bronchopulmonary sequestrations are distinct from the lung and outside the normal pleural investment
            • Most commonly found within the thorax but can also be seen within the abdomen, diaphragm, retroperitoneum and neck (J Pediatr Surg 2019;54:1286)
            • Systemic blood supply is most often from the thoracic or abdominal aorta
            • May rarely communicate with the gastrointestinal tract
          Etiology
          • Numerous hypotheses have been proposed
          Diagrams / tables

          Images hosted on other servers:

          ILS as seen at surgery

          Clinical features
          • Among historic symptomatic cases, ~40 - 60% of patients with extralobar bronchopulmonary sequestrations have other congenital anomalies, including congenital diaphragmatic hernia and congenital heart disease (Thorax 1979;34:96)
          • Intralobar bronchopulmonary sequestrations are most often isolated anomalies but can be associated with other findings in ~15% of symptomatic patients (Thorax 1979;34:96)
            • Intralobar sequestrations that are not diagnosed prenatally may present with recurrent infection
          Diagnosis
          • Often diagnosed on prenatal ultrasound but may not be detected until later in life
          • Can be an incidental finding
          Radiology description
          • Xray: homogenous opacity in the lung base; may have air fluid levels (AJR Am J Roentgenol 2000;175:1005)
          • Ultrasound: homogenous, echogenic solid mass with an aberrant systemic artery (J Ultrasound Med 2018;37:371)
          • CT: solid mass with emphysematous changes at the margins; may or may not have associated cystic changes and air fluid levels
          Radiology images

          Images hosted on other servers:

          Xray

          Ultrasound

          CT scan

          Prognostic factors
          Case reports
          Treatment
          • Surgical resection is curative and routinely performed for symptomatic lesions
          • Asymptomatic lesions are often prophylactically resected to prevent subsequent infections and respiratory symptoms (Semin Pediatr Surg 2015;24:187)
          Gross description
          • Intralobar bronchopulmonary sequestration (Semin Pediatr Surg 2015;24:176)
            • Aberrant systemic feeding vessel is often apparent as a tied / stapled vascular margin that is distinct from the hilar structures
            • Variable parenchymal findings, ranging from grossly unremarkable lung to cysts measuring up to 2.5 cm in greatest dimension
            • May have a mucocele
          • Extralobar bronchopulmonary sequestration (Semin Pediatr Surg 2015;24:176)
            • Irregularly shaped portion of lung entirely invested by pleura with a vascular pedicle
            • Variable parenchymal findings, ranging from grossly unremarkable lung to cysts measuring up to 2.5 cm in greatest dimension
            • May have a mucocele
            • May have prominent fine lymphatics underlying the pleural surface
          Gross images

          Contributed by Jennifer Pogoriler, M.D., Ph.D.
          ILS with systemic vessel

          ILS with systemic vessel

          Cut surface ILS with systemic vessel

          Cut surface ILS with systemic vessel

          ELS and feeding vessel

          ELS and feeding vessel

          Cut surface of ELS

          Cut surface of ELS

          ELS with prominent lymphatics

          ELS with prominent lymphatics

          ELS with cystic changes

          ELS with cystic changes

          Microscopic (histologic) description
          • Systemic feeding vessel is often accompanied by hilar type structures, including an airway, nerves, and lymph nodes (Pediatr Dev Pathol 2007;10:75)
          • Systemic artery branches within the lesion are often somewhat thicker than normal pulmonary arteries and may show pulmonary hypertension-like changes in older patients (Histopathology 2010;57:121)
          • Cases universally have at least mild parenchymal maldevelopment with enlarged and simplified alveoli
          • ~50% of cases also have cystic parenchymal maldevelopment
            • Cysts are typically round and lined by ciliated columnar epithelium; epithelial complexity is rare
            • Histologically similar to a type 2 CPAM
            • Sequestrations with cystic parenchymal maldevelopment are often referred to as a hybrid lesion (Pediatr Dev Pathol 1999;2:454, J Pediatr Surg 1997;32:986)
          • Often have prominent evidence of mucostasis, including pools of mucin and foamy intra-alveolar macrophages (Am J Surg Pathol 2019;43:47)
            • May have foci of skeletal muscle within septa between larger cysts
          • Prominent lymphangiectasia is seen in a subset of extralobar bronchopulmonary sequestrations
          Microscopic (histologic) images

          Contributed by Jennifer Pogoriler, M.D., Ph.D.
          ILS aberrant hilum

          ILS aberrant hilum

          Normal lung adjacent to ILS

          Normal lung adjacent to ILS

          ILS with parenchymal maldevelopment

          ILS with parenchymal maldevelopment

          Cystic changes in ELS

          Cystic changes in ELS


          ELS with prominent lymphangiectasia

          ELS with prominent lymphangiectasia

          Skeletal muscle in septae

          Skeletal muscle in septa

          High power skeletal muscle

          High power skeletal muscle

          Sample pathology report
          • Lung, left lower lobe, lobectomy:
            • Intralobar bronchopulmonary sequestration with associated cystic parenchymal maldevelopment (see comment)
            • Comment: These features have been referred to as hybrid lesion.
          Differential diagnosis
          • Type 1 CPAM:
            • No systemic feeding vessel or aberrant hilum
            • Gross: readily identifiable cysts
            • Variably sized cystic spaces with epithelial complexity
            • Mucinous cell clusters seen in ~75%
            • Mucostasis is rare
          • Type 2 CPAM:
            • No systemic feeding vessel or aberrant hilum
              • Morcellated specimens may require correlation with operative note
            • Gross: may have cysts measuring up to 2.5 cm
            • Spectrum of histologic changes from mild parenchymal maldevelopment to simple round cystic spaces, similar to bronchopulmonary sequestrations
            • Prominent mucostasis
          • Intrapulmonary bronchogenic cyst:
            • No systemic feeding vessel or aberrant hilum
            • Gross: single, thick walled cyst
            • May also have cystic changes in the adjacent parenchyma
          Board review style question #1

          Diagnosis of a bronchopulmonary sequestration requires which of the following?

          1. Cystic parenchymal changes
          2. Epithelial complexity
          3. Prominent lymphangiectasia
          4. Systemic feeding vessel
          Board review style answer #1
          D. Systemic feeding vessel. A sequestration should be sequestered from both the normal pulmonary arterial circulation as well as from the tracheobronchial tree. Although the parenchyma is maldeveloped, frank cystic change is seen only in a subset of cases. Epithelial complexity is rarely seen. This is a feature more typical of type 1 congenital pulmonary airway malformation. Lymphangiectasia may be present in a subset of extralobar sequestrations but is not a required feature.

          Comment Here

          Reference: Sequestrations
          Board review style question #2

          Gross examination of the lobectomy specimen shown above reveals a sutured vessel along the periphery of the lobe that is distinct from the hilum. What is the best diagnosis?

          1. Bronchogenic cyst
          2. Extralobar bronchopulmonary sequestration
          3. Intralobar bronchopulmonary sequestration
          4. Type 2 congenital pulmonary airway malformation
          5. Type 3 congenital pulmonary airway malformation
          Board review style answer #2
          C. Intralobar bronchopulmonary sequestration. This image shows a systemic feeding vessel associated with a lower lobe (note diaphragmatic surface). This is an intralobar sequestration. An extralobar sequestration does not share pleura with the normal lung. Congenital pulmonary airway malformations and intrapulmonary bronchogenic cysts do not have a systemic feeding vessel.

          Comment Here

          Reference: Sequestrations

          Silicosis
          Definition / general
          • Silica: crystalline silicon dioxide
          • Most prevalent chronic occupational disease in the world, due to foundry work, sandblasting, stone cutting and coal mining
          • Decades of exposure usually required for symptoms
          • Causes a progressive, nodular fibrosing pneumoconiosis
          • Acute silicosis: less common, due to heavy exposure; similar to alveolar proteinosis with generalized accumulation of lipoproteinaceous material within alveoli
          • Crystalline forms of silica are more fibrogenic than amorphous forms; quartz is particularly fibrogenic, although quartz plus other minerals are less fibrogenic
          • Quartz causes directly injury to membranes via SiOH groups and by free radicals generated by crushing silica
          • Silica also causes macrophages to release mediators which stimulate fibroblasts, including tumor necrosis factor
          • Talc, vermiculite and mica are noncrystalline silicates that less commonly cause pneumoconiosis
          • Detect on routine chest xray as a fine nodularity in upper lobes, but normal pulmonary function
          • No symptoms until progressive massive fibrosis, then disease progresses with impaired pulmonary function
          • Disease may progress even after exposure to silica ceases
          • Not associated with lung cancer
          Gross description
          • Early, tiny, discrete pale to black (if coal dust present) nodules in upper zones of lungs, progressing to hard collagenous scars
          • Nodules have stellate shape at the edges, may cavitate due to tuberculosis or ischemia
          • Fibrosis present in hilar nodes and pleura
          • May see eggshell calcification in nodes on xray
          Microscopic (histologic) description
          Early lesions:
          • Small nodules of fibroblasts and histiocytes with abundant silica, that become less cellular and more hyalinized with time
          • With progressive massive fibrosis, see hyalinized and condensed collagen, needle-like spicules with pointed ends, 5 microns or less, birefringent with polarization and intra- or extracellular
          Microscopic (histologic) images

          Images hosted on other servers:
          Silicotic nodule

          Silicotic nodule

          Silica crystals (polarized light)

          Silica crystals (polarized light)


          Small cell carcinoma
          Definition / general
          • High grade neuroendocrine tumor that arises in the hilum of smokers, with a poor prognosis and no current targeted therapy
          Essential features
          • High grade, usually advanced at diagnosis
          • Almost exclusively associated with smoking / tobacco exposure
          • Positive for at least one neuroendocrine marker
          ICD coding
          • ICD-10: C34.90 - malignant neoplasm of unspecified part of unspecified bronchus or lung
          Epidemiology
          Sites
          • Lung: central / bronchial / hilar; rarely a peripheral nodule
          • Submucosal growth
          • Metastasis to liver, adrenals, bone, bone marrow, brain; often widespread
          Pathophysiology
          • Arises from neuroendocrine cells of basal bronchial epithelium
          Etiology
          • Smoking (rare in nonsmokers)
          Clinical features
          Diagnosis
          • Biopsy: based on hematoxylin and eosin morphology
          Laboratory
          Radiology description
          Radiology images

          Images hosted on other servers:

          Right hilar mass

          Left hilar mass

          Prognostic factors
          • Adverse (Cancer Treat Res 2016;170:301)
            • Continued smoking → chemotherapy resistance
            • Labs: elevated lactate dehydrogenase (LDH), alkaline phosphatase, albumin
            • Metastasis to the liver, bone marrow and brain
            • Presence of paraneoplastic syndromes
          • Favorable (Lung Cancer 2019;130:216)
            • Women
          Case reports
          Treatment
          Gross description
          • Central or hilar mass
          • White-tan, soft, friable, necrotic (Am J Surg Pathol 2002;26:1184)
          • Peripheral nodules: circumscribed, with fleshy cut surface
          Gross images

          Contributed by Carolyn Glass, M.D.

          Central tumor with bronchial spread



          Images hosted on other servers:

          Hilar mass

          Central tumor

          Spreading along bronchi

          Frozen section description
          • Rarely seen at frozen section due to being diagnosed on cytology and usually treated nonoperatively
          Microscopic (histologic) description
          • Round / oval blue cells with minimal cytoplasm; usually small to medium sized
          • Nuclear features: finely dispersed chromatin, no distinct nucleoli, molding, smudging, high mitotic rate
          • Stroma: thin, delicate, scant, fibrovascular
          • Necrosis and apoptosis of individual cells common
          • Patterns: sheets, clusters, ribbons, rosettes, peripheral palisading
          • Other / rarer features (Surg Oncol Clin N Am 2016;25:447)
            • Azzopardi phenomena: basophilic nuclear material lining blood vessel walls
            • Metastatic cells usually have more cytoplasm
            • Scattered giant cells
          Microscopic (histologic) images

          Contributed by Carolyn Glass, M.D.

          Sheets of tumor cells

          Crushed blue cells

          Cellular morphology

          CK7

          TTF1

          CD56

          Virtual slides

          Images hosted on other servers:

          Small cell lung carcinoma

          Cytology description
          • Nuclei: oval / elongated, hyperchromatic, absent nucleoli, granular cytoplasm, smooth membrane
          • Scant cytoplasm
          • Pattern: molding, individual cells or loose clusters, crush artifact
          • Necrosis and apoptosis of individual cells and tumor background
          • Hypercellular (Int J Clin Exp Pathol 2010;3:367)
          Positive stains
          Negative stains
          Electron microscopy description
          Electron microscopy description

          Images hosted on other servers:

          Dense core granules

          Molecular / cytogenetics description
          Sample pathology report
          • Lung, left upper lobe, endobronchial biopsy:
            • Small cell carcinoma (see comment)
            • Comment: Positive immunoreactivity for keratin, CD56 and TTF1 with negative staining for CD45 supports the diagnosis of small cell carcinoma.
          Differential diagnosis
          Board review style question #1
          A 65 year old man presents to the clinic with generalized fatigue for 6 months. He says he has been feeling very down lately and that his mood has had a negative impact on his relationship with his wife of 30 years. He also reports recent acne flares, which he says he has not struggled with since college. He is a 25 pack per year smoker and has hypertension, previously well controlled on his current regimen. Vitals taken today show a BP of 155/95. Physical exam reveals an adipose deposit on the dorsal upper thorax and abdominal striae. A chest Xray reveals a hilar lung mass. Which of the following tumors most commonly produces the paraneoplastic syndrome seen in this patient?

          1. Adenocarcinoma of the lung
          2. Large cell carcinoma of the lung
          3. Small cell lung cancer
          4. Squamous cell lung cancer
          Board review style answer #1
          C. Small cell lung cancer is associated with paraneoplastic syndromes and one of the most common is Cushing syndrome, as in this patient. This is caused by increased production of adrenocorticotropic hormone by the small cell tumor. Symptoms include weight gain, fatigue, glucose intolerance, moon face, buffalo hump, muscular weakness, skin manifestations (thinning, acne, abdominal striae), psychological symptoms (depression, anxiety, decreased libido), hypertension, increased risk of bone fracture and reproductive symptoms (irregular or absent menstruation in females and erectile dysfunction in males). Other paraneoplastic syndromes associated with small cell lung carcinoma include syndrome of inappropriate antidiuretic hormone secretion and Lambert-Eaton.

          Comment Here

          Reference: Small cell carcinoma
          Board review style question #2
          A 59 year old female smoker with no significant past medical history comes to the clinic complaining of dyspnea. She states that a month ago, she began experiencing shortness of breath during her morning 2 mile walk, which she had previously enjoyed without difficulty. Over the last week, she has started becoming short of breath throughout the day and complains of headaches that are not relieved with ibuprofen. When asked about her diet, she says she has decreased her salt intake, as she thinks her face has become swollen. On physical exam, you note edema of the right arm and distention of right sided neck veins. Xray of the chest shows a right sided lung mass at the hilum. Fine needle aspiration of this mass would most likely reveal which of the following?

          1. Granulomas with central necrosis and giant cells
          2. Islands of large eosinophilic cells containing keratin
          3. Nests of cells with large irregular shaped nuclei that are forming glands
          4. Sheets of small round blue cells with finely dispersed chromatin
          Board review style answer #2
          D. Sheets of small round blue cells with finely dispersed chromatin. Small cell lung cancer is histologically described as small round / oval cells with high nuclear to cytoplasmic ratio. The nuclei have finely dispersed or salt and pepper chromatin and absent nucleoli. Small cell lung cancer is most often located at the hilum and grows along the bronchi. Growth of the tumor can cause compression of the superior vena cava leading to superior vena cava syndrome, characterized by swelling of the face and upper limbs, cough and distention of the neck veins, as venous blood flow is obstructed.

          Comment Here

          Reference: Small cell carcinoma
          Board review style question #3

          A patient undergoes a biopsy of a lung mass with the histology shown in the image. Which of the following immunohistochemical results would be most typical of the lesion?

          1. INSM1+, chromogranin+, CK7+ diffuse, p40+, Ki67 low
          2. INSM1+, chromogranin-, CK7+ diffuse, p40+, Ki67 high
          3. INSM1-, chromogranin+, CK7- punctate, p40+, Ki67 high
          4. INSM1+, chromogranin+, CK7- punctate, p40-, Ki67 high
          5. INSM1-, chromogranin-, CK7- punctate, p40-, Ki67 low
          Board review style answer #3
          D. INSM1+, chromogranin+, CK7- punctate, p40-, Ki67 high. The tumor shown is small cell carcinoma of the lung. Of the choices listed, INSM1+, chromogranin+, CK7- punctate, p40-, Ki67 high (answer D) is the most likely immunoprofile.

          Comment Here

          Reference: Small cell carcinoma

          Squamous / glandular / mixed papilloma
          Definition / general
          Epidemiology
          • Solitary endobronchial papillomas in adults is rare, < 0.5% of lung tumors
          • Squamous papillomas usually occur in middle aged male smokers
          Terminology
          • Also called squamous cell papilloma, solitary tracheobronchial papilloma
          • Termed papillomatosis if multiple lesions present
          Clinical features
          • Associated with HPV 6 and HPV 11; high risk HPV may be seen in cases associated with carcinoma (Hum Pathol 1994;25:1191)
          • Noninvasive but may have dysplasia, recur or develop into squamous cell carcinoma
          • Nonspecific lower respiratory tract complaints include hemoptysis, recurrent pneumonia, asthma-like symptoms, dry cough
          • Rarely arises in lower bronchial tree, but usually manifests as endobronchial exophytic growth
          Diagnosis
          • May be difficult on frozen section and small biopsy specimens to distinguish from squamous carcinoma
          Radiology description
          • May be incidental finding on imaging or show symptoms related to obstruction
          • Xray: either normal, infiltrative shadow, hilar mass or lobar collapse
          • CT: mass shadowing, can be PET avid
          Radiology images

          Images hosted on other servers:
          Pedunculated mass

          Pedunculated mass

          Case reports
          Treatment
          • Endoscopic removal, cryotherapy, fulguration
          • Surgical resection
          Gross description
          • Tan-white, friable, pedunculated / polypoid, smooth to verrucoid, glistening
          • Wart-like, cauliflower-like
          • Generally less than a few centimeters in size
          Microscopic (histologic) description
          • Usually exophytic, papillary lesion with arborizing fibrovascular cores lined by keratinizing or nonkeratinizing mature squamous epithelium
          • Rarely inverted pattern
          • Lesion may grow into adjacent alveolar spaces
          • May contain areas lined by ciliated or non-ciliated columnar cells with cuboidal cells or mucin-filled cells (mixed squamous and glandular papilloma)
          • May exhibit viral cytopathic effect: enlarged hyperchromatic nuclei, nuclear wrinkling, polychromasia, binucleate forms, perinuclear halos
          • Mild to moderate stromal inflammation which may be related to airway obstruction
          • No mitoses, no necrosis
          Microscopic (histologic) images

          Contributed by Roseann Wu, M.D., M.P.H.



          Images hosted on other servers:
          Various images

          Various images

          Solid mass

          Solid mass

          Schneiderian papilloma

          Schneiderian papilloma

          Bronchiak papilloma

          Bronchial papilloma

          Cytology description
          • Moderate cellularity, single and loosely clustered squamous cells
          • Cytoplasmic keratinization and dark pyknotic nuclei with variable atypia, may show cells resembling koilocytes
          • Background acute inflammation
          Cytology images

          Images hosted on other servers:
          Cytomorphology

          Cytomorphology

          Positive stains
          Negative stains
          Molecular / cytogenetics description
          • Subset positive for HPV by ISH or PCR
          Differential diagnosis
          Additional references

          Squamous cell carcinoma
          Definition / general
          • Malignant tumor arising from epithelial cells with squamous differentiation demonstrated either immunohistochemically or morphologically, with the presence of keratinization or intercellular bridges
          • Can be keratinizing or nonkeratinizing and subtypes include solid, cystic, papillary, pseudoglandular, alveolar filling and sarcomatoid / spindle cell change
          Essential features
          • Morphologically identified with keratinization, keratin pearls and intercellular bridges
          • Poorly differentiated lesions may lose morphologic features of squamous differentiation and require IHC to separate from other non-small lesions, p40 positivity will help
          • Primary and metastatic tumors have an overlapping morphology and immunohistochemical profiles; in most cases it is not possible to determine the primary site of squamous cell carcinoma
          Terminology
          • Epidermoid carcinoma: historical and no longer recommended
          ICD coding
          • ICD-10: C34.90 - malignant neoplasm of unspecified part of unspecified bronchus or lung
          • ICD-11: 2C25.2 - squamous cell carcinoma of bronchus or lung
          Epidemiology
          • Strong association with history of tobacco smoking (80% of men and 90% of women) (Thorax 1973;28:354)
          • Primarily in patients over 50 years of age
          • 85% of primary lung cancers are non-small cell carcinomas and squamous cell makes up 30% of these cases (Clin Cancer Res 2012;18:2443)
          Sites
          • Can present in any site of the lungs or bronchus but is more commonly central
          • Metastasis is frequently observed (Ann Oncol 2018;29:iv192)
            • Mediastinal lymph nodes are the primary site
            • Hematogenous spread to distant organs
              • Bone is the most common
              • Liver is the second most common
          Pathophysiology
          • Squamous cells lining the respiratory tract transformed from carcinogen exposure, most frequently smoking
          • Occupational heavy metal exposure has also been reported (Int Arch Occup Environ Health 2009;82:867)
          • Extrapulmonary squamous cell carcinoma is also possible
          Etiology
          Clinical features
          • Wide range of nonspecific pulmonary symptoms reported including cough, chest pain, shortness of breath, hemoptysis, wheezing, weight loss, recurrent infections, loss of appetite and fatigue (StatPearls: Squamous Cell Lung Cancer [Accessed 31 May 2023])
          • If the mass compresses the recurrent laryngeal nerve it may also present with hoarseness
          Diagnosis
          • Can be made on biopsy or cytology specimen, though a lack of keratinization or in poorly differentiated cases immunohistochemical testing may be required (Thorax 2016;71:177)
          Laboratory
          • Humoral hypercalcemia of malignancy secondary to paraneoplastic syndrome is seen in up to 10% of cases and is associated with a poor prognosis (World J Clin Oncol 2014;5:197)
          Radiology description
          • CT is the imaging of choice for lung cancer evaluation and demonstrates a nonspecific mass with irregular borders, often with central cavitation
          • Centrally located tumors are more common and often cause obstruction with associated pneumonitis or lung collapse
          • Squamous cell carcinoma cannot be distinguished definitively from radiological findings as other lesions may have the same radiologic appearance
          • Reference: Radiopaedia: Squamous Cell Carcinoma of the Lung [Accessed 31 May 2023]
          Radiology images

          Images hosted on other servers:
          Central mass with cavitation

          Central mass with cavitation

          Peripheral mass with collapse

          Peripheral mass with collapse

          Prognostic factors
          • Stage at time of diagnosis is the best predictor of prognosis
          Case reports
          Treatment
          Gross description
          • Often cavitary mass with white, smooth cut surfaces that frequently contain hemorrhage and necrosis
          • Mass is more often central than peripheral and may be within the bronchus
          Gross images

          Contributed by @yro854 and @Andrew_Fltv on Twitter
          Squamous cell carcinoma Squamous cell carcinoma

          Squamous cell carcinoma



          Images hosted on other servers:
          Central cavitation

          Central cavitation

          Tumor obstructing bronchus

          Tumor obstructing bronchus

          Tumor extending to pleura

          Tumor extending to pleura

          Microscopic (histologic) description
          • Presence of keratinization including keratin pearls, intercellular bridging, abundant inflammation and necrosis are all frequent but not required for diagnosis
          • There is currently no established grading system for squamous cell carcinoma of the lung and cases and intuitively graded based on the degree of perceived differentiation
          • Poorly differentiated squamous cell carcinoma exhibits severe cellular and nuclear atypia, abundant mitosis and often requires IHC studies to differentiate from other poorly differentiated lesions
          • Reference: Acad Pathol 2017;4:2374289517705950
          Microscopic (histologic) images

          Contributed by Alex McGeough, M.D. and Andrey Bychkov, M.D., Ph.D.
          Tumor budding

          Tumor budding

          Well differentiated squamous cell carcinoma

          Well differentiated

          Intercellular bridging

          Intercellular bridging

          Intercellular bridging

          Intratumoral necrosis

          Poorly differentiated squamous cell carcinoma

          Poorly differentiated

          Moderately differentiated squamous cell carcinoma

          Moderately differentiated


          Prominent cytologic atypia

          Prominent cytologic atypia

          Keratin pearls

          Keratin pearls

          Basaloid type

          Basaloid type

          Palisading nuclei in basaloid type

          Palisading nuclei in basaloid type

          p40 positive in basaloid type

          p40 positive in basaloid type


          Bronchial biopsy

          Bronchial biopsy

          Nonkeratinizing, moderately differentiated squamous cell carcinoma

          Nonkeratinizing

          p40 staining

          p40 staining

          CK 5/6

          CK 5/6

          SCC cocktail

          SCC cocktail

          Cytology description
          • Atypical cells with keratinization are seen in well differentiated cases
          • Higher grade lesions may have similar cytologic appearance to adenocarcinoma and IHC studies can be performed on the cell block to differentiate
          • Often demonstrates stretching of the cytoplasm referred to as tadpole cells
          • Reference: Int J Clin Exp Pathol 2010;3:367
          Cytology images

          Images hosted on other servers:
          Keratinization and tadpole cell

          Keratinization and tadpole cell

          Positive stains
          Negative stains
          • Negative for neuroendocrine markers, TTF1 and CK20
          Electron microscopy description
          • Abundant tonofilaments, complex desmosomes, basal lamina
          Molecular / cytogenetics description
          • Abundant molecular alterations are present and testing does not have the prognostic value seen in adenocarcinoma
          • Molecular testing for targetable driver mutations is recommended in cases of adenosquamous carcinoma
          • EGFR and ALK targetable variants can occur in squamous cell carcinoma; as such, molecular testing may be considered in young or nonsmoking patients
          Sample pathology report
          • Lung, right upper lobe, wedge resection:
            • Invasive, moderately differentiated, keratinizing squamous cell carcinoma (1 cm greatest dimension) (see synoptic report)
            • Pathologic tumor stage: pT1a, pN0, pMX
            • Margins of resection are free of tumor (4 cm bronchial margin of resection)
          Differential diagnosis
          Board review style question #1

          What histologic feature seen in the above tumor best supports a diagnosis of squamous cell carcinoma?

          1. Generous eosinophilic cytoplasm
          2. Intercellular bridges
          3. Lack of glandular structures
          4. Presence of nucleoli
          Board review style answer #1
          B. Intercellular bridges. Intercellular bridging can be easily appreciated between adjacent tumor cells, which is a defining histologic feature of squamous cell carcinoma. Answers A and C - D are incorrect because these are nonspecific findings seen in other lung lesions.

          Comment Here

          Reference: Lung - Squamous cell carcinoma
          Board review style question #2
          What molecular testing is routinely recommended in squamous cell carcinoma of the lung?

          1. EGFR
          2. KRAS
          3. PDL1
          4. PTEN
          Board review style answer #2
          C. PDL1. PDL1 is the correct answer as PDL1 expression assessed by the tumor proportion score (TPS) may predict response to immunotherapy. Answers A and B are incorrect because EGFR and KRAS are only routinely recommended in lung adenocarcinoma. Answer D is incorrect because PTEN is associated with colorectal cancer and thyroid cancer.

          Comment Here

          Reference: Lung - Squamous cell carcinoma
          Board review style question #3

          The above tumor is found in the lung. Which stain will allow differentiation between basaloid squamous cell carcinoma and other basaloid neoplasms?

          1. CK5/6
          2. CK7
          3. p40
          4. TTF1
          Board review style answer #3
          C. p40 is the stain of choice for the identification of squamous cell carcinoma and will help demonstrate squamous differentiation. Answers A and B are incorrect because while CK7 and CK5/6 would also both be positive in this lesion, they would not prove squamous differentiation, as they are positive in the majority of lung carcinomas. Answer D is incorrect because TTF1 would support neuroendocrine or adenocarcinoma differentiation instead of squamous.

          Comment Here

          Reference: Lung - Squamous cell carcinoma

          Squamous cell carcinoma in situ
          Definition / general
          • Squamous cell carcinoma in situ (SCIS) is a preinvasive lesion originating from the bronchial epithelium and represents a precursor of squamous cell carcinoma (SCC)
          • Recognition of SCIS can often be complex because it forms a histologic continuity with severe dysplasia; in SCIS there is a complete loss of maturation, severe cytologic atypia and mitoses in all epithelial levels (basal, middle and upper) with an intact basement membrane
          Essential features
          • SCIS is a preinvasive squamous lesion that is a precursor SCC of the lung and bronchus
          • Histologically there is severe full epithelial thickness dysplasia, mitoses on all levels with complete loss of maturation from the base to the luminal surface
          • The most studied and successful treatment choice is endobronchial photodynamic therapy
          Terminology
          • Recommended: squamous cell carcinoma in situ (SCIS)
          • Acceptable: high grade intraepithelial neoplasia
          • Not recommended: bronchial premalignancy, early noninvasive cancer
          ICD coding
          • ICD-O: 8070/2 - in situ neoplasm of the lung (such as squamous carcinoma in situ)
          • ICD-10: D02.2 - carcinoma in situ of bronchus and lung
          • ICD-11: 2E62.2 & XH7WM7 - carcinoma in situ of bronchus or lung & squamous cell carcinoma in situ, NOS
          Epidemiology
          • No clear data regarding incidence but epidemiology is similar to lung SCC (Eur Respir J 2009;33:656)
          • Data for preinvasive bronchial lesions are scant and tend to involve a population that is at risk (i.e., smokers or are in a screening program for lung cancer) (J Clin Pathol 2001;54:257)
          • Risk factors (similar to invasive SCC)
            • Smoking combustible tobacco products - the most detrimental
            • Radon exposure both indoors and in mines
            • Outdoor air pollution
            • Asbestos exposure (Chest 2013;143:e1S)
          Sites
          • Anywhere from the main right or left bronchus to distal bronchioles
          • Can be unifocal or multifocal throughout the tracheobronchial tree (Cancer Prev Res (Phila) 2014;7:1)
          • Mirrors the localization of SCC; as such, at least two - thirds are centrally located with the remaining one - third in the periphery (Ann Am Thorac Soc 2017;14:118)
          Pathophysiology
          • Analogous to that of lung SCC
          • SCIS is clearly linked to exposure to carcinogens found in cigarettes - tobacco mutation signature
          • Causes a series of genetic and epigenetic alterations
          • TP53, CDKN2A, SOX2 and AKT2 genes frequently exhibit somatic and copy number alterations
          • With cell cycle and DNA repair pathway alterations and chromosomal instability
          • Reference: Nat Med 2019;25:517
          Etiology
          • Like that of lung SCC
          • Main etiological factor is smoking combustible tobacco products
          • Indoor radon exposure and other ionizing radiation
          • Asbestos exposure or other professional exposure
          • Underlying chronic lung disease (e.g., pulmonary fibrosis, COPD, etc.)
          • Indoor and outdoor air pollution (Clin Cancer Res 2005;11:537)
          Clinical features
          • SCIS is typically asymptomatic and is incidentally diagnosed during bronchoscopy
          • Also seen in resection specimens for lung cancer
          Diagnosis
          • 40% of SCIS can be detected by white light reflectance bronchoscopy (WLB)
          • Combining white light reflectance bronchoscopy with autofluorescence bronchoscopy (AFB) can detect lesions as small as 0.5 mm
          • Autofluorescence bronchoscopy has a greater sensitivity than white light reflectance bronchoscopy for diagnosing SCIS, with combination obtaining a sensitivity of 85%
          • Using infrared light illumination with optical coherence tomography can provide
            • Cross sectional images with a spatial resolution ranging from 3 to 15 μm
            • Penetration depth of 2 mm
            • Nearly histological in quality
          • Diagnosis and exclusion of invasion are through biopsy
          • References: Thorac Surg Clin 2013;23:153, J Thorac Oncol 2011;6:1336, Clin Cancer Res 2008;14:2006
          Radiology description
          • SCIS cannot be visualized on CT or PET
          Prognostic factors
          • SCIS is very difficult to detect
          • Has the potential to be multifocal, with ~30% of patients developing synchronous or metachronous lesions
          • Excellent 5 year survival of > 90%
          • Reference: Chest 2013;143:e263S
          Case reports
          Treatment
          • Endobronchial photodynamic therapy has been the most evaluated technique
            • Complete response rates between 32% and 100%
          • Other endobronchial therapies include
            • Endobronchial brachytherapy
            • Bronchoscopic electrocautery
            • Cryotherapy
            • Nd:YAG laser therapy
          • Reference: Chest 2013;143:e263S
          Clinical images

          Images hosted on other servers:
          Right main bronchus under a) white light and b) autofluorescence

          Right main bronchus
          under a) white
          light and b) auto-
          fluorescence

          Gross description
          • Polypoid / nodular lesions of 1 - 2 mm in diameter (25%) or flat lesions (75%) that are > 10 mm and appear as a focal thickening with marked irregular mucosa
          • Localization
            • In the segmental bronchi, close to points of branching
            • Spreading to the nearby lobar and subsegmental branches
            • Less common in the trachea
          • Reference: J Thorac Cardiovasc Surg 1993;106:1098
          Microscopic (histologic) description
          • Diagnostic criteria are based on epithelial thickness, cell size, maturation / orientation and nuclei
            • Thickness
              • May or may not be increased
            • Cell size
              • Markedly increased
              • May have marked anisocytosis and pleomorphism
            • Maturation / orientation
              • No progression from the base to the luminal surface
              • Expanded basal area
              • Cellular crowding throughout
              • No intermediate zone
              • Flattening of the surface is limited to the most superficial cells
            • Nuclei
              • High and variable N:C ratio
              • Coarse and uneven chromatin
              • Nuclear angulations and folding
              • Prominent or absent nuclei
              • No consistent orientation to the epithelial surface
              • Mitotic figures throughout the full thickness
          • Reference: Histopathology 2001;38:202
          Microscopic (histologic) images

          Contributed by Gheorghe-Emilian Olteanu, M.D., Ph.D., Ana Mataić, M.D. and Luka Brčić, M.D., Ph.D.
          Normal respiratory epithelium

          Normal respiratory epithelium

          Normal respiratory epithelium and SCIS

          Normal respiratory epithelium and SCIS

          Upper layer spot necrosis

          Upper layer spot necrosis

          Flattening of the upper layer

          Flattening of the upper layer

          Polypoid growth pattern

          Polypoid growth pattern

          Pleomorphic cells

          Pleomorphic cells

          Positive stains
          Molecular / cytogenetics description
          • 5q loss of heterozygosity
          • Somatic and copy number alterations in the TP53, CDKN2A, SOX2 and AKT2 genes
          Videos

          SCIS histopathology - lung, bronchus

          Sample pathology report
          • Bronchus, endobronchial biopsy:
            • Squamous cell carcinoma in situ (see comment)
            • Comment: Severe, full epithelial thickness dysplasia and mitoses on all levels with complete loss of maturation. No invasion noted.
          Differential diagnosis
          • Severe dysplasia:
            • Cellular pleomorphism, coarse chromatin, frequent nucleoli, basal zone to upper third, mitoses confined to lower two - thirds, superficial cell flattening
          Board review style question #1
          Which of the following is true about squamous cell carcinoma in situ (SCIS)?

          1. SCIS can be visualized on CT or PET
          2. SCIS does not represent a precursor of squamous cell carcinoma
          3. SCIS is a preinvasive lesion originating from the bronchial epithelium
          4. Smoking combustible tobacco products is not a risk factor for squamous cell carcinoma in situ
          Board review style answer #1
          C. Squamous cell carcinoma in situ (SCIS) is a preinvasive lesion originating from the bronchial epithelium. When the bronchial epithelium is exposed to carcinogens and irritants, it may show squamous metaplasia, which is not considered a preneoplastic lesion. Further accumulation of somatic genetic alterations leads to squamous dysplasia and squamous cell carcinoma in situ, which are considered preinvasive squamous lesions.

          Answer A is incorrect because squamous cell carcinoma in situ cannot be visualized on CT or PE but it can be detected in up to 40% of cases by white light reflectance bronchoscopy (WLB); the sensitivity goes up when white light reflectance bronchoscopy is combined with autofluorescence bronchoscopy (AFB). Answer B is incorrect because SCIS does represent a precursor of squamous cell carcinoma. Answer D is incorrect because the main etiological factor for SCIS is smoking combustible tobacco products.

          Comment Here

          Reference: Lung - Squamous cell carcinoma in situ
          Board review style question #2
          Regarding the histologic features of squamous cell carcinoma in situ (SCIS), which of the following is true?

          1. Diagnostic criteria are not based on epithelial maturation / orientation
          2. Epithelial thickness must be increased
          3. Mitotic figures are found only in the lower third
          4. There is no progression in maturation from the base to the luminal surface
          Board review style answer #2
          D. There is no progression in maturation from the base to the luminal surface and no consistent orientation of nuclei to the epithelial surface. Answer A is incorrect because diagnostic criteria for squamous cell carcinoma in situ are based on epithelial thickness, cell size, maturation / orientation and nuclei. Answer B is incorrect because epithelial thickness may or may not be increased. Answer C is incorrect because mitotic figures are present throughout the full thickness.

          Comment Here

          Reference: Lung - Squamous cell carcinoma in situ
          Board review style question #3

          A patient with a nodular mass in the lower left lobe underwent a lobectomy. The tumor was located at 0.5 cm from the resection margin. The histological image above shows part of the bronchial resection margin. Regarding these findings, which of the following statements is true?

          1. In the report, the resection margins are stated as uninvolved by carcinoma in situ
          2. Ki67 is positive only in cells located in the lower part of the epithelium
          3. The lesion is diffusely and strongly positive for p40
          4. There is a progression in maturation from the base to the luminal surface
          Board review style answer #3
          C. The lesion is diffusely and strongly positive for p40. The picture shows the part of the bronchial resection margin that is involved by carcinoma in situ. Answer D is incorrect because the lesion shows no progression in maturation from the base to the luminal surface. Answer B is incorrect because on IHC, the lesion is diffusely and strongly positive for p40 and Ki67 shows increased proliferative activity with diffuse expression. Answer A is incorrect because lobectomy specimens contain bronchial and vascular margins and the presence of carcinoma in situ at a surgical margin is an important finding and must be reported as such.

          Comment Here

          Reference: Lung - Squamous cell carcinoma in situ

          Staging
          Pathologic TNM staging of carcinomas of the lung, AJCC 8th edition
          Definition / general
          • Applicable to carcinomas of the lung, including non small cell and small cell carcinomas and bronchopulmonary carcinoid tumors, does not apply to sarcomas or other rare tumors of the lung
          • WHO has defined new entities of adenocarcinoma in situ and minimally invasive adenocarcinoma, which are assigned to Tis and T1mi categories, respectively
          Essential features
          • T, N and M categories code the anatomic extent of lung cancer and predict overall survival of patients with non small cell and small cell carcinomas
          • The 8th edition AJCC / TNM classification has provided a more comprehensive stratification of tumors based on subdivision of T, N and M categories and also adopted the newly introduced histological entities of lung carcinoma
          ICD coding
            ICD-10:
          • C34.0: Main bronchus
          • C34.1: Upper lobe, lung
          • C34.2: Middle lobe, lung
          • C34.3: Lower lobe, lung
          • C34.8: Overlapping lesion of lung
          • C34.9: Lung, not otherwise specified (NOS)
          Diagrams / tables

          Images hosted on other servers:
          Missing Image

          Staging chart

          Primary tumor (pT)
          • pTX: primary tumor cannot be assessed or tumor proven by the presence of malignant cells in sputum or bronchial washings but not visualized by imaging or bronchoscopy
          • pT0: no evidence of primary tumor
          • pTis: carcinoma in situ, squamous cell carcinoma in situ, adenocarcinoma in situ (pure lepidic pattern and ≤ 3 cm)
          • pT1mi: minimally invasive adenocarcinoma (≤ 3 cm with a predominantly lepidic pattern and ≤ 5 mm of invasion)
          • pT1a: tumor ≤ 1 cm or rarely a superficial, spreading tumor of any size with invasive component limited to the bronchial wall that may extend proximal to the main bronchus
          • pT1b: tumor > 1 cm but ≤ 2 cm
          • pT1c: tumor > 2 cm but ≤ 3 cm
          • pT2: tumor > 3 cm but ≤ 5 cm or involves the main bronchus regardless of distance to the carina without involvement of the carina, invades visceral pleura (PL1 or PL2) or is associated with atelectasis or obstructive pneumonitis that extends to the hilar region involving part or all of the lung
            • pT2a: tumor > 3 cm but ≤ 4 cm or has 1 of the above features and size cannot be determined
            • pT2b: tumor > 4 cm but ≤ 5 cm
          • pT3: tumor > 5 cm but ≤ 7 cm or directly invades parietal pleura (PL3), chest wall (including superior sulcus tumors), phrenic nerve or parietal pericardium or presence of a separate tumor nodule in the same lobe
          • pT4: tumor > 7 cm or tumor of any size invading diaphragm, mediastinum, heart, great vessels, trachea, recurrent laryngeal nerve, esophagus, vertebral body or carina or presence of a separate tumor nodule in an ipsilateral different lobe

          Notes:
          • Tumor size has important prognostic relevance; each centimeter increase in size, from less than 1 cm to up to 5 cm, yields a significantly different prognosis
          • Multiple separate nodules are classified into four disease patterns (J Thorac Oncol 2016;11:639):
            • Multiple primary tumors
            • Nodules of the same histopathological type
            • Multiple tumors with predominant ground glass features on CT and a lepidic pattern on pathological examination
            • Diffuse pneumonic type lung cancer
          • Multiple primary tumors
            • Considered separate primary tumors if different histologic types (e.g., squamous carcinoma and adenocarcinoma) or are squamous carcinomas that have arisen from carcinoma in situ
            • Different biomarker pattern and absence of nodal or systemic metastases favor multiple primary tumors
          • Single primary tumor
            • Considered to have arisen from a single tumor if an exact match by comparative genomic hybridization
            • Similar histologic appearance, matching biomarker patterns and significant nodal or systemic metastases favor a single primary tumor
          Regional lymph node (pN)
          • pNX: regional lymph nodes cannot be assessed
          • pN0: no regional lymph nodes metastasis
          • pN1: metastasis in ipsilateral peribronchial, ipsilateral hilar or intrapulmonary lymph node, including involvement by direct extension
          • pN2: metastasis in ipsilateral mediastinal or subcarinal lymph node
          • pN3: metastasis in contralateral mediastinal, contralateral hilar, ipsilateral / contralateral scalene or supraclavicular lymph node

          Notes:
          • In addition to anatomic extent of nodal disease, quantification based on the number of involved lymph node stations has prognostic impact
          Distant metastasis (pM)
          • pM0: no distant metastasis
          • pM1a: separate tumor nodule in a contralateral lobe, pleural nodules, pericardial nodules, malignant pleural effusion or malignant pericardial effusion
          • pM1b: single extrathoracic metastasis in a single organ (including a single non regional node)
          • pM1c: multiple extrathoracic metastases in a single organ or in multiple organs

          Notes:
          • Most pleural and pericardial effusions are a result of the tumor
          • Rarely, fluids are negative for tumor, non bloody and not an exudate; clinical judgment is required in these cases
            • Number of distant metastases has more prognostic relevance than organ location
          AJCC prognostic stage grouping
          Occult carcinoma: TX N0 M0
          Stage 0: Tis N0 M0
          Stage IA1: T1mi N0 M0
          T1a N0 M0
          Stage IA2: T1b N0 M0
          Stage IA3: T1c N0 M0
          Stage IB: T2a N0 M0
          Stage IIA: T2b N0 M0
          Stage IIB: T1a - c N1 M0
          T2a - b N1 M0
          T3 N0 M0
          Stage IIIA: T1a - c N2 M0
          T2a - b N2 M0
          T3 N1 M0
          T4 N0 - 1 M0
          Stage IIIB: T1a - c N3 M0
          T2a - b N3 M0
          T3 - 4 N2 M0
          Stage IIIC: T3 - 4 N3 M0
          Survival data
          Overall survival
          2 years 5 years
          Clinical stage  Pathological stage  Clinical stage  Pathological stage 
          Stage IA1   97% 97% 92% 90%
          Stage IA2 94% 94% 83% 85%
          Stage IA3 90% 92% 77% 80%
          Stage IB 87% 68% 89% 73%
          Stage IIA 79% 82% 60% 65%
          Stage IIB 72% 76% 53% 56%
          Stage IIIA 55% 65% 36% 41%
          Stage IIIB 44% 47% 28% 24%
          Stage IIIC 24% 30% 13% 12%
          Stage IVA 23% NA 10% NA
          Stage IVB 10% NA 0% NA
          Registry data collection variables
          • For surgically resected non small cell lung cancer
            • Gender
            • Age
            • Weight loss
            • Performance status
            • Resection margins
            • Adequacy of mediastinal dissection
          • For advanced non small cell lung cancer
            • EGFR mutation
            • ALK gene rearrangement
            • Gender
            • Symptoms
            • Weight loss
            • Performance status
            • Chemoradiotherapy
            • Chemotherapy
          • For small cell lung cancer
            • Performance status
            • Age
            • Comorbidity
            • Chemotherapy
            • Thoracic radiotherapy
            • Prophylactic cranial radiotherapy
          • Pathological variables for surgically resected non small cell lung cancer (Gospodarowicz: Prognostic Factors in Cancer, 3rd Edition, 2006):
            • Histologic type
            • Differentiation grade
            • Vascular invasion
            • Lymphatic permeation
            • Perineural invasion
            • Type of visceral pleura invasion: PL1 versus PL2
            • Positive pleural lavage cytology
            • SUVmax of primary tumor
            • Molecular / biologic markers
          Histologic grade
          • GX: grade of differentiation cannot be assessed
          • G1: well differentiated
          • G2: moderately differentiated
          • G3: poorly differentiated
          • G4: undifferentiated
          Videos

          Staging animation

          Updates on TNM staging (2017)

          8th TNM classification (2018)

          Additional references
          Board review style question #1
            A CT detected a ground glass nodule in the upper lung. Surgical resection revealed a 2 cm adenocarcinoma with pure lepidic pattern and no stromal invasion. What is the pT category?

          1. pT0
          2. pT1a
          3. pT1b
          4. pT1mi
          5. pTis
          Board review style answer #1
          E. pTis. The pTis category for adenocarcinoma in situ was added in the AJCC / TNM 8th edition staging scheme. By definition, this is a localized, small (≤ 3 cm) adenocarcinoma with growth restricted to neoplastic cells along pre-existing alveolar structures (lepidic growth) and lacking stromal, vascular, alveolar space or pleural invasion. Radiologic equivalent is a pure ground glass nodule on CT. In contrast, minimally invasive adenocarcinoma (pT1mi) is defined as a lepidic predominant adenocarcinoma measuring up to 3 cm with an invasive component measuring up to 0.5 cm.

          Comment Here

          Reference: Lung - Staging
          Board review style question #2
            What is the correct pM category for a lung carcinoma with an ipsilateral malignant pleural effusion?

          1. pM0
          2. pM1a
          3. pM1b
          4. pM1c
          Board review style answer #2
          B. pM1a. A lung carcinoma with pleural or pericardial nodules or with a proven malignant pleural or pericardial effusion is defined as pM1a.

          Comment Here

          Reference: Lung - Staging

          Staphylococcus aureus
          Clinical features
          • Uncommon life threatening pneumonia that may be community acquired or hospital acquired
          • Traditionally caused by S. aureus, but infections by S. epidermidis are increasing (eMedicine)
          • Variable clinical presentation, but patients are acutely ill, often with septicemia or a viral infection
          • High risk: comatose patients, neurosurgery or HIV+ patients
          • Abscesses and empyema are common complications, have high rate of severe morbidity and mortality
          • Methicillin resistant Staphylococcus aureus (MRSA) pneumonia can be deadly (Clin Microbiol Rev 2010;23:616); both community and hospital acquired cases are developing resistance to clindamycin, but are sensitive to trimethoprim / sulfamethoxazole (Am J Med Sci 2012;343:196)
          • Panton-Valentine leukocidin (PVL) is a rare (< 5% of stains) Staphylococcus aureus cytotoxin which causes WBC destruction and tissue necrosis; emerging as a serious problem worldwide (Clin Infect Dis 1999;29:1128, Wikipedia)
          • No vaccine available (BMC Public Health 2011;11 Suppl 3:S27)
          Diagnosis
          Microscopic (histologic) description
          • Pyogenic response with tissue destruction

          Strongyloides
          Clinical features
          • Endemic in Southeastern United States, South America, Southeast Asia and Sub-Saharan Africa
          • Infection occurs when larvae in soil penetrate the skin and travel to the lung via the venous circulation; the worms then travel up the trachea to the oropharynx and are swallowed setting up infection in the small intestine
          • Female worms in the small intestinal mucosa produce eggs asexually (parthenogenesis) that are passed to the soil where they hatch to continue cycle of infection
          • Larvae hatched in the intestines may penetrate the colonic mucosa to travel to the lung to reinitiate infection (autoinfection)
          • Disease usually affects the gastrointestinal tract
          • In immunocompromised patients, severe disease with large burden of parasites may occur; patients may present with alveolar hemorrhage (Prim Care Respir J 2009;18:337)
          • May predispose to invasive infections caused by enteric organisms (Am J Clin Pathol 2007;128:622)
          Diagrams / tables

          Images hosted on other servers:
          Life cycle

          Life cycle

          Microscopic (histologic) description
          • In the lung, eosinphilic pneumonia with hemorrhage may occur
          • Worms may be found in airways, alveoli and blood vessels
          Microscopic (histologic) images

          Case #133
          Small bowel infection

          Small bowel infection



          Images hosted on other servers:
          Small bowel infection Small bowel infection

          Small bowel infection

          Cytology images

          Contributed by anonymous
          Bronchial wash

          Bronchial wash

          Pap stain

          Pap stain

          Diff-Quik

          Diff-Quik



          Images hosted on other servers:
          Bronchial wash

          Bronchial wash

          BAL fluid

          BAL fluid


          Transplantation / rejection
          Definition / general
          • Lung transplant is the treatment for end stage lung nontumoral diseases
          • Lung allograft rejection is a significant cause of acute graft dysfunction and eventually chronic graft failure (Arch Pathol Lab Med 2017;141:437)
          Essential features
          • Lung allograft rejection is a significant cause of acute graft dysfunction and eventually chronic graft failure
          • Rejection of allograft is through a cellular mediated response or antibody mediated response
          • Acute cellular rejection resolves in 80 - 90% with immunosuppression
          • Chronic lung allograft dysfunction is the main cause of mortality in 50% of lung transplant recipients after 5 years
          Terminology
          • Acute cellular rejection (ACR)
          • Chronic lung allograft dysfunction (CLAD): irreversible graft loss, 20% decrease in forced expiratory volume in 1 second (FEV1)
          • Bronchiolitis obliterans syndrome (BOS): 85% of chronic lung allograft dysfunction cases
          • Obliterative bronchiolitis (OB): clinical bronchiolitis obliterans syndrome
          • Restrictive allograft syndrome (RAS [rCLAD]): 15% of chronic lung allograft dysfunction cases
          ICD coding
          • ICD-10:
            • T86.810 - lung transplant rejection
            • T86.811 - lung transplant failure
            • T86.812 - lung transplant infection
            • T86.818 - other complications of lung transplant
            • T86.819 - unspecified complication of lung transplant
          Epidemiology
          • 17.1 - 64% incidence of acute cellular rejection in the first year post-lung transplant (J Thorac Dis 2019;11:S1732)
          • Acute rejection is responsible for 3.6% of deaths among lung transplant recipients in the first 30 days (J Thorac Dis 2017;9:5440)
          • After 5 years, chronic lung allograft dysfunction is the main cause of mortality in 50% of lung transplant recipients (J Thorac Dis 2017;9:2684)
          Sites
          • Respiratory system
          Pathophysiology
          • Immune response to the allograft is characterized by the following processes
            • Hyperacute rejection
              • Due to preformed donor specific antibodies, which activate the complement system and platelet aggregation leading to vascular thrombosis and graft necrosis (J Heart Lung Transplant 2016;35:397)
            • Acute cellular rejection
              • Recipient T lymphocytes recognize donor human leukocyte antigen (HLAs) or other antigens
              • Pathways: direct (donor dendritic cells presenting major histocompatibility complex directly to recipient T cells) and indirect (recipient dendritic cells presenting alloantigen from donor antigen presenting cells to T cells) (J Thorac Dis 2017;9:5440)
            • Antibody mediated rejection
              • Donor specific antibodies produced by B cells and plasma cells, target foreign HLA on the donor capillary endothelial cells
              • HLA donor specific antibodies cause persistent insult to the graft epithelium, resulting in epithelial ischemia and antibodies to self antigens such as K-α 1 tubulin (Kα1T) and collagen V (Col V0) (Arch Pathol Lab Med 2017;141:437)
          Etiology
          Clinical features
          • Shortness of breath
          • Pulmonary edema
          • Diffuse alveolar hemorrhage (J Thorac Dis 2019;11:S1732)
          • Fever
          • Cough
          • Decline in pulmonary function tests
          Diagnosis
          • Bronchoscopy
            • Bronchoalveolar lavage (BAL): cytological and microbiological analysis
            • Transbronchial biopsy forceps and cryobiopsy (Ann Thorac Surg 2019;108:1052)
          • Clinical
          Laboratory
          Radiology description
          • Chest Xray
            • Ground glass opacities, pulmonary edema and pleural effusions
          • High resolution CT
            • Ground glass opacities with interlobular septal thickening, without consolidation and atelectasis (J Thorac Dis 2017;9:5440)
          Radiology images

          Images hosted on other servers:
          Hyperacute lung transplant rejection

          Hyperacute lung transplant rejection

          Acute rejection (A1)

          Acute rejection (A1)

          Restrictive allograft syndrome

          Restrictive allograft syndrome

          Prognostic factors
          • Hyperacute rejection has high mortality (J Heart Lung Transplant 2016;35:397)
          • Acute rejection resolves in 80 - 90% with immunosuppression
          • Chronic lung allograft dysfunction is a major cause of death after 6 months posttransplant
          • Immunosuppressant therapy causes secondary infections (CMV, EBV) and posttransplant lymphoproliferative disorder (Respiration 2015;90:451)
          • Posttransplant lymphoproliferative disorder
            • Majority occur within the first year posttransplant (> 85%)
            • Risk factors include immunosuppressive burden and the oncogenic nature of the Epstein-Barr virus (EBV) (World J Transplant 2020;10:29)
            • Treatment includes reduction of immunosuppressive therapy, rituximab, antivirals and chemotherapy
          Case reports
          Treatment
          • Reserved for symptomatic grades of acute cellular rejection (> A2) and consists of intravenous methylprednisolone, tapering to oral
          • Consider using tacrolimus if on cyclosporine or adding mammalian target of rapamycin (mTOR) inhibitor, such as everolimus (J Thorac Dis 2017;9:5440)
          • Extracorporeal photophoresis stimulates and expands the number of peripheral regulatory T cells, stabilizing acute cellular rejection and reducing lung function decline in chronic lung allograft dysfunction / bronchiolitis obliterans syndrome (Clin Transplant 2017;31:e13041)
          Gross description
          • Antibody mediated rejection: unfixed lungs are hyperemic and heavy
          • Chronic rejection: hyperinflated lungs, with bronchiectasis, fibrosis of lung parenchyma (J Thorac Dis 2017;9:2684)
          Gross images

          Contributed by M. Ángeles Montero-Fernández, M.D., Ph.D.
          Explant lung with diffuse alveolar damage

          Explant lung with diffuse alveolar damage

          Idiopathic pleuroparenchymal fibroelastosis

          Idiopathic pleuroparenchymal fibroelastosis

          Chronic lung allograft dysfunction

          Chronic lung allograft dysfunction

          Microscopic (histologic) description
          • Acute cellular rejection is characterized by infiltration of lymphocytes / monocytes around blood vessel endothelium (A grade) and airways / bronchioles (B grade) (J Thorac Dis 2017;9:5440)
          • Classified by the International Society for Heart and Lung Transplantation (ISHLT) working party (see table below)
          • Perivascular inflammation
            • Minimal (A1): infrequent circumferential monocyte cuffing displaying a ring of 2 - 3 cell thick layers in the perivascular adventitia (venules and arterioles); no eosinophils or endotheliatis
            • Mild (A2): more frequent circumferential monocyte cuffing with > 3 cell thick layer perivascular infiltrates; eosinophils and endothelialitis can be seen; no extension to the adjacent alveolar interstitium
            • Moderate (A3): extension of perivascular inflammation into neighboring alveolar septa; neutrophils, eosinophils and endothelialitis are common findings
            • Severe acute rejection (A4): diffuse inflammation, hemorrhage, parenchymal necrosis, necrotizing vasculitis and hyaline membrane formation (J Thorac Dis 2017;9:5440, Surgical Pathology 2020:13:119)
          • Airway inflammation (grades B and E)
            • Inflammation of noncartilaginous bronchioles (B) and large cartilaginous bronchi (E)
              • Low grade (B1R E1): submucosal lymphocytic inflammation, without epithelial injury
              • High grade (B2R E2): submucosal lymphocytic inflammation, with epithelial injury with or without ulceration (Surgical Pathology 2020:13:119)
          • Chronic vascular rejection (D0: absent; D1: present): fibrotic remodeling of veins and arteries
          • Chronic rejection is characterized as present or absent
            • Bronchiolitis obliterans (OB): partial or complete occlusion of bronchiole lumen, with patchy submucosal fibrosis, with or without inflammatory infiltrates
            • Restrictive allograft syndrome (RAS): pleuroparenchymal fibroelastosis with or without bronchiolitis obliterans or diffuse alveolar damage

          Revised working formulation for classification and grading of pulmonary allograft rejection
          A: Perivascular inflammation
          AX: Ungradable
          B / E: Airway inflammation C: Chronic airway rejection
          D: Chronic vascular rejection
          Grade 0: none Grade 0: none 0: absent
          Grade 1: minimal Grade 1R*: low grade 1: present
          Grade 2: mild Grade 2R*: high grade
          Grade 3: moderate BX / EX: ungradable CX / DX: ungradable
          Grade 4: severe
          Microscopic (histologic) images

          Contributed by M. Ángeles Montero-Fernández, M.D., Ph.D.
          Alveolar septal capillaritis

          Alveolar septal capillaritis

          Minimal acute cellular rejection (A1)

          Minimal acute cellular rejection (A1)

          Moderate acute cellular rejection (A3)

          Moderate acute cellular rejection (A3)

          Severe acute cellular rejection (A4)

          Severe acute cellular rejection (A4)


          Lymphocytic bronchiolitis (B2R)

          Lymphocytic bronchiolitis (B2R)

          Obliterative bronchiolitis

          Obliterative bronchiolitis

          Acute fibrinous and organizing pneumonia

          Acute fibrinous and organizing pneumonia

          Acute lung injury

          Acute lung injury

          C4d

          C4d

          Cytology description
          • Allograft rejection is associated with increased neutrophils and epithelial cells in bronchoalveolar lavage (Respir Res 2018;19:102)
          Immunofluorescence description
          • In antibody mediated rejection, capillary C4d staining in alveolar tissue may support the presence of an antibody mediated process (Am J Transplant 2018;18:936)
          Immunofluorescence images

          Contributed by M. Ángeles Montero-Fernández, M.D., Ph.D.
          C4d

          C4d

          Positive stains
          • Proposed scoring threshold for positive C4d: diffuse staining (> 50%) of alveolar interstitial capillaries
          Negative stains
          • Proposed scoring threshold for negative C4d: focal staining (< 50%) of alveolar interstitial capillaries
          Molecular / cytogenetics description
          • MicroRNA has shown promising potential in distinguishing patients with acute cellular rejection from those without rejection (Transplant Direct 2015;1:e44)
          • Donor derived cell free DNA increases with acute rejection and also correlates with lung function decline and histological rejection grading (JHLT 2021;40:822)
          Videos

          Indications, patient selection and outcomes

          Sample pathology report
          • Left lung, lower lobe, transbronchial biopsy:
            • Mild acute cellular rejection, mild lymphocytic bronchiolitis and no evidence of antibody mediated rejection (A2 B1R C0, pAMR0) (see comment)
            • Comment: There are 5 adequate pieces of lung, including 1 bronchiole and several vessels. 2 perivascular lymphocytic infiltrates that extend to the septa are noted. Occasional eosinophils and focal endotheliitis are noted. Bronchiolar submucosal lymphocytic infiltrate without epithelial injury is also seen. No bronchiolitis obliterans or capillaritis identified. Immunohistochemistry stain for C4d is negative.
          Differential diagnosis
          • Hyperacute (antibody mediated) rejection:
            • Donor transmitted infection:
              • Specific criteria of infection as granulomas, intra-alveolar neutrophils with cellular debris, viral inclusions, fungal structures
            • Ischemia reperfusion injury (acute lung injury):
              • Usually up to 2 weeks after the transplant
              • Intra-alveolar or membranous fibrin
              • Type 2 pneumocyte hyperplasia, increase of interstitial neutrophils due to ischemia, organizing pneumonia
          • Acute rejection:
          • Chronic rejection:
            • Nonspecific large airway fibrosis
            • Constrictive bronchiolitis:
              • Fibrotic bronchiolar stenosis nonrelated to transplant
            • Organizing pneumonia:
              • Nonspecific alveolar fibroblastic process
          Board review style question #1

          Which of the following is the main cause of mortality in lung allograft patients?

          1. Acute cellular rejection
          2. Antibody mediated rejection
          3. Chronic lung allograft dysfunction
          4. Hyperacute rejection
          Board review style answer #1
          C. Chronic lung allograft dysfunction. The main cause of mortality in lung transplant patients is chronic lung allograft dysfunction (CLAD), affecting 50% of transplant recipients after 5 years (J Thorac Dis 2017;9:2684). Answers A and B are incorrect because these conditions are treatable, although they may evolve to CLAD which may eventually cause graft failure and dead. Answer D is incorrect because hyperacute rejection is currently an infrequent complication.

          Comment Here

          Reference: Lung - Transplantation / rejection

          Tuberculosis
          Definition / general
          • Infectious disease caused by Mycobacterium tuberculosis
          • Disease can be manifested as primary, secondary and miliary tuberculosis
          Essential features
          • Infectious disease caused by M. tuberculosis
          • Hallmark is necrotizing granulomatous inflammation, composed of central necrotic zone surrounded by epithelioid histiocytes and Langhans type giant cells
          • Presence of M. tuberculosis should be confirmed by microbiologic cultures or nucleic acid amplification testing (NAAT)
          Terminology
          • TB
          • Mycobacterium tuberculosis (MTB)
          ICD coding
          • ICD-10: A15.0 - tuberculosis of lung
          • ICD-11: 1B10 - tuberculosis of the respiratory system
          Epidemiology
          Sites
          • Most commonly affects the respiratory system but other systems can be involved in disseminated disease (gastrointestinal [GI] system, lymphoreticular system, skin, central nervous system, musculoskeletal system, reproductive system and liver) (J Family Community Med 2019;26:83)
          Pathophysiology
          • Infection begins when M. tuberculosis enters lungs via inhalation, reaches the alveolar space and encounters resident alveolar macrophages
          • If alveolar macrophages do not eliminate the bacteria, M. tuberculosis invades the lung interstitial tissue, either by direct infection of alveolar epithelium or as a result of the infected alveolar macrophage migrating to the interstitium
          • Dendritic or other inflammatory cells transport M. tuberculosis to pulmonary lymph nodes for T cell priming; this yields recruitment of immune cells to form a granuloma
          • Granuloma may contain infection (latent TB)
          • Bacteria replicate within the growing granuloma; if the granuloma fails to contain the infection, bacteria may disseminate to other organs and can enter the bloodstream, leading to reentry and release into the respiratory tract; the infected host is now infectious and symptomatic (active TB disease)
          • Primary TB is defined as an infection caused by MTB in a previously uninfected host; secondary TB usually occurs due to reactivation of latent TB after initial primary infection; miliary TB is the disseminated form with hematogenous spread of MTB to the lungs and other organs
          • Selected reviews on the pathophysiology of M. tuberculosis infection: Nat Rev Dis Primers 2016;2:16076, FEMS Microbiol Rev 2019;43:341
          Etiology
          Diagrams / tables

          Images hosted on other servers:

          Mycobacterium tuberculosis infection

          Clinical features
          • Chronic and persistent cough (often productive), weight loss, fever, night sweats and hemoptysis (J Biomed Sci 2020 Jun;27:74)
          • History of latent TB or exposure to infected individual may be identified
          Diagnosis
          • Based on a combination of exposure history, clinical symptoms, radiologic and laboratory findings (Nat Rev Dis Primers 2016;2:16076)
          • Diagnosis can be made with lung or lymph node biopsy, surgical resection specimen, cytology specimen and autopsy specimen, although lung biopsy does not have a routine role in diagnosis
          Laboratory
          • Active TB disease
            • Imaging techniques (chest Xrays and PET CT)
            • Sputum smears (AFB or Ziehl-Neelsen staining)
            • Cultures
            • Molecular tests (nuclear amplification and gene based tests)
          • Latent TB infection
            • Tuberculin skin testing (Mantoux test with purified protein derivative [PPD])
            • Interferon release assays (IGRA)
          • Selected references with discussions on diagnostic techniques: Nat Rev Dis Primers 2016;2:16076, J Biomed Sci 2020 Jun;27:74, N Engl J Med 2021;385:2271
          Radiology description
          • Often identified on chest Xray or CT
          • Commonly presents as cavitary lesion in upper lobe in infected immunocompetent hosts
          • Immunocompromised patients can show lower lobe disease with adenopathy and pleural effusion
          • Review of radiographic findings: Insights Imaging 2022;13:3
          Radiology images

          Images hosted on other servers:

          Chest Xray and high resolution CT of active tuberculosis

          Prognostic factors
          Case reports
          Treatment
          Gross description
          • Cavitary disease
          • Tuberculomas: localized conglomerates of necrotizing granulomatous infection
          • Classically described findings:
            • Ghon focus: subpleural, often upper lobe nodule
            • Ghon complex: Ghon focus plus lymphatic or hilar lymph node involvement
            • Caseous necrosis (cheese-like, tan to white grumous material)
          • Can cause empyema (chronic or active infection of the pleural space), resulting in pus accumulation in pleural cavity
          • Bronchiectasis (irreversible dilatation of the bronchi and bronchioles)
          • References: Semin Diagn Pathol 2017;34:518, StatPearls: Ghon Complex [Accessed 23 May 2022]
          Gross images

          Contributed by Sajna V.M. Kutty, M.D.
          65 year old male chronic smoker with 3 cm lung mass

          65 year old male chronic smoker with 3 cm lung mass



          Images hosted on other servers:
          Lung with cavity

          Lung with cavity and caseous necrosis

          Histomorphological patterns

          Histomorphologic
          patterns of
          postprimary
          tuberculosis

          Microscopic (histologic) description
          • Hallmark is necrotizing granulomatous inflammation, composed of central necrotic zone surrounded by epithelioid histiocytes with varied number of multinucleated giant cells and lymphocytes
          • Multinucleated giant cells may contain Langhans type giant cells (nuclei arranged in a horseshoe shaped pattern at the periphery of the cell) but Langhans type giant cells are not specific for TB infection
          • Organisms are usually present within the central zone of necrosis, seen on special stains (in some cases)
          • Nonnecrotizing granulomas can be present as well
          • References: Semin Diagn Pathol 2017;34:518
          Microscopic (histologic) images

          Contributed by Hui-Hua Li, M.D., Ph.D., Jefree J. Schulte, M.D. and Aliya N. Husain, M.D.
          Necrotizing granuloma core biopsy

          Necrotizing granuloma core biopsy

          Giant cells

          Giant cells

          Large necrotizing granuloma

          Large necrotizing granuloma

          Necrotizing granuloma

          Necrotizing granuloma

          Positive AFB stain

          Positive AFB stain



          Contributed by @AnaPath10 on Twitter
          Tuberculosis Tuberculosis Tuberculosis

          Tuberculosis

          Virtual slides

          Images hosted on other servers:

          Necrotizing granulomatous inflammation

          Miliary TB, H&E

          Miliary TB, AFB

          Cytology description
          • Epithelioid granulomas, Langhans type giant cells, necrosis and acute or chronic inflammation
          • Sources of sampling: aspirates from transbronchial lung biopsy, mediastinal lymph nodes and other smears
          Cytology images

          Images hosted on other servers:

          Lymph node tuberculosis

          Lymph node tuberculosis

          Positive stains
          Videos

          Case of pulmonary tuberculosis with granulomas

          Histopathology of pulmonary tuberculosis

          Sample pathology report
          • Left lung, transbronchial biopsy:
            • Necrotizing granulomatous inflammation with AFB positive bacilli (see comment)
            • Comment: AFB histochemical stain highlights occasional AFB positive bacilli within necrotizing granulomas. GMS histochemical stain is negative for fungal elements. Correlation with microbial cultures is needed.
          • Station 7 lymph node, fine needle aspiration:
            • Necrotizing granulomatous inflammation with AFB positive bacilli (see comment)
            • Comment: AFB histochemical stain highlights occasional AFB positive bacilli within necrotizing granulomas. GMS histochemical stain is negative for fungal elements. Correlation with microbial cultures is needed.
          Differential diagnosis
          • Nontuberculous mycobacterial disease / atypical mycobacterial infection:
            • Including M. avium complex (MAC), M. kansasii, M. xenopi and M. abscessus
            • Cannot be distinguished from tuberculosis based on gross or microscopic appearance
            • Positive identification of the organism by culture or polymerase chain reaction (PCR) techniques is necessary for precise speciation
          • Granulomatous fungal infections:
            • Including histoplasmosis, blastomycosis, cryptococcosis and coccidioidomycosis
            • Often presents as asymptomatic solitary pulmonary nodules
            • Combination of necrotizing and nonnecrotizing granulomas with overlapping histologic features making it difficult to predict a specific pathogen based on the histologic findings alone
          • Sarcoidosis:
            • Noncaseating granuloma composed of epithelioid cells, giant cells and lymphocytes
            • Well formed granulomas tend to be distributed along lymphatic pathways and may coalesce to form macroscopic nodules (nodular sarcoidosis)
          • Granulomatosis with polyangiitis (Wegener):
            • Combination of necrotizing granulomatous inflammation and necrotizing vasculitis targeting small to medium size vessels
          Board review style question #1

          A 72 year old woman who is HIV positive and has a significant medical history of diabetes mellitus presents to the ED with persistent cough, hemoptysis and chest pain. Imaging findings are concerning for infectious disease. A nodular lesion in the lung is biopsied (see image above). Special stains reveal AFB positive bacilli. Which of the following statements is true?

          1. Correlation with microbial cultures or PCR is required for definitive diagnosis
          2. Granulomatous changes would be expected to be observed near the lymphatics of the lung (lymphatic or lymphangitic distribution)
          3. These findings are consistent with sarcoidosis
          4. These findings are diagnostic of Mycobacterium tuberculosis
          Board review style answer #1
          A. Correlation with microbial cultures or PCR is required for definitive diagnosis

          Comment Here

          Reference: Tuberculosis
          Board review style question #2
          A 63 year old man is diagnosed with Mycobacterium tuberculosis infection following bronchoscopy, with smears showing acid fast bacilli and culture growing M. tuberculosis. Which of the following statements is true?

          1. If alveolar macrophages fail to eradicate M. tuberculosis, the resultant inflammatory response is typically characterized by eosinophil infiltration
          2. M. tuberculosis cannot grow outside of the lung parenchyma
          3. M. tuberculosis first infects type 1 pneumocytes and replicates within the cytoplasm of the pneumocyte
          4. Resident macrophages within the lungs are the primary cell that is infected, upon initial infection by M. tuberculosis
          Board review style answer #2
          D. Resident macrophages within the lungs are the primary cell that is infected, upon initial infection by M. tuberculosis

          Comment Here

          Reference: Tuberculosis

          Typical carcinoid tumor / neuroendocrine tumor, grade 1
          Definition / general
          • Pulmonary carcinoids are well differentiated neoplasms, with neuroendocrine differentiation further divided into typical and atypical carcinoids (neuroendocrine tumors grade 1 and 2)
          • Classification is based on mitotic count per 2 mm2 and the presence / absence of necrosis (Ki67 proliferative index is currently not recommended to distinguish between typical and atypical carcinoids)
          Essential features
          • Typical carcinoids are classified according to a mitotic count < 2 per 2 mm2, the absence of necrosis and a tumor size > 0.5 cm
          • Histologic features include neuroendocrine growth pattern and monotonous tumor cells with salt and pepper chromatin and inconspicuous nucleoli and moderate to abundant eosinophilic cytoplasm
          • Treatment is based on surgical resection; prognosis is excellent
          Terminology
          • Not recommended: well differentiated lung neuroendocrine tumor
          • G1 - G3 neuroendocrine neoplasm classification is not currently applied to pulmonary carcinoids (Mod Pathol 2018;31:1770)
          • Neuroendocrine tumor (NET) terminology has been added, according to the unifying nomenclature proposed by the WHO classification of tumors group
          ICD coding
          • ICD-O: 8240/3 - typical carcinoid
          • ICD-10: D3A.090 - benign carcinoid tumor of the bronchus and lung
          • ICD-11: 2C25.4 & XH9LV8 - carcinoid or other malignant neuroendocrine neoplasms of bronchus or lung & neuroendocrine tumor, grade 1
          Epidemiology
          Sites
          • Anywhere from the trachea to the distal bronchioles
          • Central airways (85%), mostly main and lobar bronchi > peripheral airways (15%) (Cancer 2008;113:5)
          • Central
            • Most common type; usually slow growing and associated with obstruction, infection, hemorrhage
            • 5% metastasize, usually to regional lymph nodes
            • Rarely distant osteoblastic metastases to bone
            • 10 year survival: 70%
            • Cytology (bronchoalveolar lavage [BAL] and brushings) often negative since tumor is covered by mucosa
          • Peripheral
            • Arise in peripheral lung, often beneath the pleura
            • Usually asymptomatic and incidental
            • Excellent prognosis
            • Rare nodal metastases are usually cured by excision
          Etiology
          • Unclear
          • May arise in the context of diffuse idiopathic pulmonary neuroendocrine cell hyperplasia and tumorlets
          • No relation to high grade pulmonary neuroendocrine carcinoma (Mod Pathol 2018;31:1770)
          Clinical features
          • Mostly due to tumor location (Chest 2017;151:1141)
            • Peripheral carcinoids are commonly asymptomatic
            • Centrally located carcinoids may present with dyspnea, cough, wheezing, hemoptysis, recurrent infection and pneumonia due to airway obstruction
          • Paraneoplastic syndromes are uncommon and usually present in the setting of liver metastases (CMAJ 2017;189:E398)
          • Bronchoscopy shows polypoid endobronchial lesion in central airway
          Diagnosis
          • Diagnosis of carcinoid can be made on biopsy or cytology specimen but the distinction between typical and atypical carcinoid usually requires a surgical specimen, unless necrosis or mitoses are seen (Ann Oncol 2015;26:1604)
          Laboratory
          Radiology description
          • Chest CT scan
          • Positron emission tomography (PET) scan: low to moderate uptake (mean standardized uptake value [SUV]: 3.4)
          Radiology images

          Contributed by Philippe Joubert, M.D., Ph.D. and Case #266

          CT scan: endoluminal tumor

          Central carcinoid tumor

          Prognostic factors
          Case reports
          Treatment
          Clinical images

          Contributed by Simon Martel, M.D.

          Bronchoscopy: endoluminal tumor

          Gross description
          • Well circumscribed, round to oval tan colored tumors
          • Localization
            • Frequently in bronchial lumen, sessile or pedunculated with partial or complete obstruction of the lumen
            • Peripheral tumors may not be evidently located in airways
          • Size range: 0.5 - 9.5 cm
          • Reference: Lung Cancer Int 2015;2015:545601
          Gross images

          Contributed by Sylvain Trahan, M.D. and Yale Rosen, M.D.

          Endoluminal tumor

          Peripheral tumor

          Large polypoid tumor

          Frozen section description
          • Homogenous population of cells with mild atypia
          • Organoid pattern helps to distinguish from inflammatory cells
          • Fine chromatin
          • Mitoses are rarely seen on frozen section
          • Hyalinization of the stroma and the presence of blood vessels may help to distinguish from carcinoma
          • Reference: Hum Pathol 2009;40:1
          Frozen section images

          Contributed by Philippe Joubert, M.D., Ph.D.

          Relatively uniform cells with mild atypia

          Microscopic (histologic) description
          • Diagnostic criteria
            • Neuroendocrine tumor with size ≥ 5 mm with < 2 mitoses/2 mm2 and absence of necrosis
          • Neuroendocrine growth pattern (organoid, trabecular, rosette formation, nested) or pseudoglandular, follicular and papillary growth
          • Tumor cells are uniform with a polygonal shape, round to oval nuclei with salt and pepper chromatin as well as inconspicuous nucleoli and moderate to abundant eosinophilic cytoplasm
            • Spindle cells and clear cell features can be seen
          • Stroma is fine and highly vascularized; hyalinization, cartilage or bone formation are possible
          • Reference: Curr Oncol 2018;25:S86
          Microscopic (histologic) images

          Contributed by Philippe Joubert, M.D., Ph.D., Jijgee Munkhdelger, M.D., Ph.D. and Andrey Bychkov, M.D., Ph.D.

          Endobronchial carcinoid

          Rosettes

          Trabecular and solid

          Solid and zellballen

          Trabecular

          Pseudorosettes


          Pseudogland formation

          Salt and pepper chromatin

          Spindle cell features

          Focal cytologic atypia

          Clear cells

          Stroma ossification


          High vascularization

          Lymph node metastasis

          Lymphovascular invasion

          Submucosal nodule

          Uniform small cells


          Uniform small cells

          AE1 / AE3

          CD56

          TTF1

          Ki67

          Typical carcinoid immunoprofile

          Cytology description
          • Loosely cohesive groups and isolated cells sometimes around branching capillaries; rosette structures may be seen (Cibas: Cytology - Diagnostic Principles and Clinical Correlates, 4th Edition, 2014)
          • Uniform population of small tumor cells with round / elongated or plasmacytoid shape
          • Smooth nuclear outlines, uniform with salt and pepper chromatin and a small nucleolus; no or rare mitoses
          • Scant granular cytoplasm
          • Absence of molding, nuclear crush and necrosis
          Cytology images

          Contributed by Philippe Joubert, M.D., Ph.D.

          Loosely cohesive groups

          Cells branching around a capillary

          Rosette-like structures

          Chromatin detail

          Cell block

          Positive stains
          Negative stains
          Molecular / cytogenetics description
          • Low somatic mutation rate
          • Mutations in the chromatin remodeling genes, including MEN1 (Nat Commun 2014;5:3518)
          Videos

          Typical carcinoid tumor

          Sample pathology report
          • Right lung, superior lobe, transbronchial biopsy:
            • Carcinoid tumor, not otherwise specified
          Differential diagnosis
          Board review style question #1

          A 55 year old woman had a lower left lobectomy showing a well circumscribed flesh colored tumor. Histologic details are shown in the image above. Regarding this entity, which of the following statements is true?

          1. CDX2 is usually negative in the lung and is useful to differentiate from a metastasis of an intestinal origin
          2. Ki67 proliferative index is essential to grade this tumor
          3. Rb expression is typically lost in this tumor
          4. TTF1 is always positive
          Board review style answer #1
          A. CDX2 is usually negative in the lung and is useful to differentiate from a metastasis of an intestinal origin. The image shows a tumor with a trabecular pattern and pseudorosettes. The stroma is highly vascularized. Tumor cells are monotonous with scant to moderate eosinophilic cytoplasm. Nuclei are round to oval with salt and pepper chromatin and inconspicuous nucleoli. No mitoses are seen. These features are consistent with a typical carcinoid lung tumor.

          Ki67 immunostaining, counter to well differentiated neuroendocrine tumors from the gastrointestinal tract, is not essential to grade the tumor. The difference between typical and atypical tumor relies on mitosis count and the presence of necrosis. However, Ki67 is useful to differentiate from high grade neuroendocrine tumors (small cell and large cell neuroendocrine carcinomas) on small biopsies with artifacts. TTF1 is positive in only half of cases with a higher frequency in peripheral tumors. Rb expression is preserved in typical carcinoid lung tumors, unlike in small cell tumors.

          Comment Here

          Reference: Carcinoid tumor / neuroendocrine tumor, grade 1
          Board review style question #2
          Regarding typical carcinoid tumors of the lung, which of the following statements is true?

          1. Diagnostic criteria are based on mitotic rate and Ki67 proliferation index
          2. Most patients are smokers
          3. They are mostly found in peripheral airways
          4. They may arise in the context of diffuse idiopathic pulmonary neuroendocrine cell hyperplasia and tumorlets
          Board review style answer #2
          D. They may arise in the context of diffuse idiopathic pulmonary neuroendocrine cell hyperplasia and tumorlets. Development of carcinoid tumors of the lung is not well understood but some cases are associated with diffuse idiopathic pulmonary neuroendocrine cell hyperplasia and tumorlets. The other statements are false. Even though lung carcinoids are found in association to airways and can be found all over the bronchial tree, most of them are centrally located (C). Typical carcinoids are not related to smoking (B). The diagnostic criteria of carcinoid tumors of the lung differ from the ones of well differentiated neuroendocrine tumors of the gastrointestinal tract (A). In the lung, the classification is based on mitotic count per 2 mm2 and presence / absence of necrosis. Typical carcinoids have a mitotic rate of 2 mitoses/2 mm2 and no necrosis, while atypical carcinoids have a mitotic rate of 2 - 10 mitoses/2 mm2 or a presence of necrosis.

          Comment Here

          Reference: Carcinoid tumor / neuroendocrine tumor, grade 1

          UIP / IPF
          Definition / general
          • Idiopathic pulmonary fibrosis (IPF) is a specific form of chronic, fibrosing interstitial lung disease (ILD) of unknown cause, associated with histologic and radiological features of usual interstitial pneumonia (UIP) (Am J Respir Crit Care Med 2022;205:e18)
          • Progressive pulmonary fibrosis (PPF) is a relatively new clinical term introduced in 2022 and used to describe a patient with an ILD other than IPF with worsening clinical course (at least 2 of 3: worsening symptoms, radiological progression and physiological progression)
          Essential features
          • Chronic and progressive respiratory failure due to lung fibrosis
          • One of the most common interstitial lung diseases and the most lethal entity
          • Characterized by bibasilar patchy fibrosis and honeycomb change
          Terminology
          • Usual interstitial pneumonia (UIP) is the name of morphological pattern in histology and radiology and is also seen in other etiologies, such as connective tissue diseases and hypersensitivity pneumonitis
          • IPF was previously called chronic interstitial pneumonitis or cryptogenic fibrosing alveolitis
          • PPF is not a pathologic diagnosis and its definition is agnostic to the underlying condition
          ICD coding
          • ICD-10: J84.1 - other interstitial pulmonary diseases with fibrosis
            • Diffuse pulmonary fibrosis
            • Fibrosing alveolitis (cryptogenic)
            • Hamman-Rich syndrome
            • Idiopathic pulmonary fibrosis
            • Usual interstitial pneumonia
          • ICD-11: CB03.4 - idiopathic pulmonary fibrosis
          Epidemiology
          Sites
          • May diffusely involve bilateral lungs but typically shows lower lobe, subpleural and basilar predominance
          Pathophysiology
          • Damage of alveolar epithelium (or pneumocytes) is a key event
          • Aberrant Wnt / β catenin signaling pathway may be related to abnormal remodeling (Am J Pathol 2003;162:1495)
          • Increased TGFβ1 activity likely has role in abnormal fibrosis (Annu Rev Pathol 2014;9:157)
          Etiology
          Diagrams / tables

          Images hosted on other servers:
          Schema of UIP pattern

          Schema of UIP pattern

          IPF diagnosis by lung biopsy and HRCT

          IPF diagnosis by lung biopsy and HRCT

          Clinical features
          • Manifestations of chronic respiratory syndrome:
            • Chronic dyspnea
            • Dry cough
            • Fatigue
            • Digital clubbing
          • Abnormal chest auscultation
            • End inspiratory fine crackles in bibasilar lung
          • Restrictive pattern in pulmonary function tests
            • Decreased total lung capacity (TLC)
            • Decreased forced vital capacity (FVC)
            • Decreased diffusing capacity of the lung for carbon monoxide (DLCO)
            • Reduced diffusing capacity is mainly due to ventilation - perfusion mismatch from ventilation of lung tissue with capillary destruction and perfusion of under ventilated alveoli; smaller component due to reduced diffusion across fibrotic alveolar septa
          Diagnosis
          • Diagnosis of UIP / IPF is often challenging and strict criteria are still being discussed
          • Based on clinical features (including laboratory tests), high resolution computed tomography (HRCT) and surgical lung biopsy (SLB)
            • It is necessary to rule out other etiologies such as particle exposure, connective tissue diseases and hypersensitivity pneumonia
            • Transbronchial lung biopsy has limited role in establishing diagnosis of UIP
            • Cryobiopsy has been shown to produce high diagnostic accuracy and good interobserver agreement for UIP pattern (Respiration 2019;98:421)
          • Guidelines suggest multidisciplinary decision by physicians, radiologists and pathologists, especially when there is a discrepancy between them (Am J Respir Crit Care Med 2022;205:e18)
          • Current guidelines do not recommend lung biopsy for cases with typical UIP pattern on HRCT; nevertheless, histopathology remains an important diagnostic modality (Eur J Radiol 2014;83:20, Am J Respir Crit Care Med 2004;170:904, Eur Respir J 2010;35:1322, Thorax 2003;58:143)
          Laboratory
          • Negative serum antibodies of connective tissue diseases
          • Negative serum antibodies of hypersensitivity pneumonitis
          Radiology description
          • Chest Xray can detect lesions; however, it is usually not sensitive enough to render the diagnosis
          • Honeycomb change: clustered cystic airspaces, 3 to 10 mm in diameter (Eur J Radiol 2014;83:27)
          • UIP pattern on HRCT (Am J Respir Crit Care Med 2022;205:e18)
            • Distribution
              • Subpleural and basal predominant
              • Often heterogeneous (areas of normal lung interspersed with fibrosis)
              • Occasionally diffuse
              • May be asymmetric
            • Features
              • Honeycombing with or without traction bronchiectasis / bronchiolectasis
              • Presence of irregular thickening of interlobular septa
              • Usually superimposed with a reticular pattern, mild ground glass opacity (GGO)
              • May have pulmonary ossification
          • Probable UIP pattern on HRCT
            • Distribution
              • Subpleural and basal predominant
              • Often heterogeneous (areas of normal lung interspersed with reticulation and traction bronchiectasis / bronchiolectasis)
            • Features
              • Reticular pattern with traction bronchiectasis / bronchiolectasis
              • May have mild GGO
              • Absence of subpleural sparing
          • Indeterminate for UIP
            • Diffuse distribution without subpleural predominance
            • CT features of lung fibrosis that do not suggest any specific etiology
          Radiology images

          Images hosted on other servers:
          Usual interstitial pneumonia (UIP) pattern Usual interstitial pneumonia (UIP) pattern Usual interstitial pneumonia (UIP) pattern

          Usual interstitial pneumonia (UIP) pattern

          UIP and possible UIP pattern

          UIP and possible UIP pattern

          Prognostic factors
          Case reports
          Treatment
          Gross description
          • Fibrotic changes in lower lobes
            • Shrunken lung with hobnailed pleura due to scarring
            • Elastic hard consistency
            • Diffuse (relatively subpleural dominant) destruction of lung mesenchyme
            • Multiple air cysts due to honeycomb change
            • Traction bronchiectasis can be seen
          Gross images

          Images hosted on other servers:
          Honeycomb lung

          Honeycomb lung

          Cobblestone appearance of the pleura

          Cobblestone appearance of the pleura

          Microscopic (histologic) description
          • Major findings are consistent with UIP pattern (Am J Respir Crit Care Med 2022;205:e18)
            • Patchy dense fibrosis with architectural distortion
              • Lesion representing UIP pattern is mainly composed of hyalinized collagen
              • Often destructs alveolar architecture
              • Smooth muscle and elastofibrosis are often present
              • Only mild infiltration by inflammatory cells should be seen (usually near areas of honeycomb change)
              • Honeycombing: cystic spaces lined by bronchiolar epithelium and fibrotic wall
            • Predilection for subpleural and paraseptal lung parenchyma
              • Spatial heterogeneity (dense fibrosis alternating with areas of the uninvolved lung) is evident on low power
              • Normal lung parenchyma can remain in the center of the lobule
            • Fibroblastic foci
              • Active fibrotic lesions composed of myofibroblasts
              • Aggregation of spindle cells with gray to pale purple matrix adjacent to dense fibrosis
            • Absence of features that suggest an alternate diagnosis
          • Clinical guidelines suggest 4 levels of certainty for pathologic diagnosis (Am J Respir Crit Care Med 2018;198:e44)
            • Definite UIP pattern
              • Dense fibrosis with architectural distortion (i.e., destructive scarring or honeycombing)
              • Predominant subpleural or paraseptal distribution of fibrosis
              • Patchy involvement of lung parenchyma by fibrosis
              • Fibroblast foci
              • Absence of features to suggest an alternate diagnosis
            • Probable UIP pattern
              • Some histologic features of UIP are present but to an extent that precludes a definite diagnosis of UIP / IPF
              • Absence of features to suggest an alternate diagnosis
              • Or honeycombing only
            • Indeterminate for UIP
              • Fibrosis with or without architectural distortion, with features favoring either a pattern other than UIP or features favoring UIP secondary to another cause
              • Some histologic features of UIP but with other features suggesting an alternative diagnosis
              • Features of an alternative diagnosis: a cellular inflammatory infiltrate away from areas of honeycombing; prominent lymphoid hyperplasia, including secondary germinal centers; and a distinctly bronchiolocentric distribution that could include extensive peribronchiolar metaplasia
            • Alternative diagnosis
              • Features of other histologic patterns of idiopathic interstitial pneumonia (e.g., absence of fibroblast foci or loose fibrosis) in all biopsies
              • Histologic findings indicative of other diseases; hypersensitivity pneumonitis, Langerhans cell histiocytosis, sarcoidosis, lymphangioleiomyomatosis (LAM)
            • Some criteria of these findings (e.g., how many granulomas to be considered significant) are still not clear
          • Additional findings
            • Squamous metaplasia: squamoid epithelium without cilia, indicator of acute exacerbation (Eur Respir J 2003;22:821)
            • Interstitial emphysema: a cystic space without epithelial lining, predictor of pneumothorax (Am J Surg Pathol 2014;38:339)
            • Smoking related changes including respiratory bronchiolitis, accumulation of smoker’s macrophages and desquamative interstitial pneumonia-like reaction
            • Pulmonary alveolar proteinosis (PAP)-like change: eosinophilic and fine granular proteinaceous material in airspace (Respir Investig 2016;54:272)
            • Centriacinar emphysema, especially in smokers
            • Peribronchiolar metaplasia, replacement of bronchiolar epithelium in alveolar walls and terminal bronchioles (Am J Surg Pathol 2005;29:948)
            • Fat metaplasia
            • Bone metaplasia
            • Kuhn hyaline (Mallory hyaline): dense, waxy, eosinophilic clumps within cytoplasm of reactive type 2 pneumocytes (Hum Pathol 1980;11:59)
            • Fibrin deposition
          Microscopic (histologic) images

          Contributed by Akira Yoshikawa, M.D. and Yale Rosen, M.D.
          Spatial heterogeneity Spatial heterogeneity

          Spatial heterogeneity

          Spatial heterogeneity

          Spatial heterogeneity

          Subpleural fibrosis

          Subpleural fibrosis

          Subpleural and paraseptal fibrosis

          Subpleural and paraseptal fibrosis

          Paraseptal fibrosis

          Paraseptal fibrosis


          Dense fibrosis

          Dense fibrosis

          Dense fibrosis with architectural destruction

          Dense fibrosis with architectural destruction

          Honeycomb change Honeycomb change

          Honeycomb change

          Honeycomb change Honeycomb change

          Honeycomb change


          Fibroblastic foci

          Honeycomb change

          Fibroblast focus Fibroblast focus

          Fibroblast focus

          Fibroblast focus

          Fibroblast focus

          Fibroblast focus Fibroblast focus

          Fibroblast focus


          Peribronchiolar metaplasia Peribronchiolar metaplasia

          Peribronchiolar metaplasia

          PAP-like change

          PAP-like change

          Respiratory bronchiolitis

          Respiratory bronchiolitis

          Squamous metaplasia

          Squamous metaplasia

          Fat metaplasia

          Fat metaplasia


          Bone metaplasia

          Bone metaplasia

          Elastosis

          Elastosis

          Virtual slides

          Images hosted on other servers:

          Usual interstitial pneumonia

          Usual interstitial pneumonia

          Positive stains
          Electron microscopy description
          • Fibrosis due to migration of activated mesenchymal cells through defects in epithelial lining and its basement membrane from interstitial to intraluminal compartments
          • Replacement of alveolar type I cells by hyperplastic alveolar type II cells
          Videos

          Usual interstitial pneumonia

          Usual interstitial pneumonia

          Usual interstitial pneumonia

          Sample pathology report
          • Lung, right upper, middle and lower lobes, wedge biopsy:
            • Usual interstitial pneumonia (UIP) (see comment)
            • Comment: Sections from the lung wedge biopsies show a fibrosing interstitial pneumonia, characterized by predominantly honeycombing and dense fibrosis. The areas of fibrosis are primarily located in the peripheral portions of the specimen. There appears to be a temporally variegated appearance, with areas of relatively uninvolved lung, along with areas showing patchy interstitial fibrosis. Fibroblast foci are frequently identified. Mild chronic inflammatory infiltrate is noted. There is no evidence of opportunistic microorganisms, granulomatous inflammation and vasculitis.
          Differential diagnosis
          Board review style question #1

          A lung wedge biopsy of a 55 year old man is shown. What is the most likely diagnosis?

          1. Diffuse alveolar damage
          2. Emphysema
          3. Nonspecific interstitial pneumonia
          4. Organizing pneumonia
          5. Usual interstitial pneumonia
          Board review style answer #1
          E. Usual interstitial pneumonia. Sections show patchy fibrosis adjacent to the preserved alveoli. Fibroblast foci are also present.

          Comment Here

          Reference: UIP / IPF
          Board review style question #2
          A 70 year old man presented with progressive dyspnea over the past few years. Which of the following findings, if present, would suggest an alternative diagnosis other than idiopathic pulmonary fibrosis (IPF)?

          1. Fibroblastic foci
          2. Granuloma
          3. Honeycomb change
          4. Subpleural and paraseptal fibrosis
          5. Temporal heterogeneity
          Board review style answer #2
          B. Granuloma. The presence of granuloma is not a feature of UIP / IPF and would be suggestive of an alternative diagnosis.

          Comment Here

          Reference: UIP / IPF

          Vaping associated acute lung injury
          Definition / general
          • Inhalational acute lung injury secondary to the use of e-cigarettes or other vaping devices which aerosolize fluid that is subsequently inhaled by the user
          Essential features
          • Spectrum of acute lung injury, from organizing pneumonia to diffuse alveolar damage
          • Airway centered foamy macrophages and pneumocytes
          • Bronchiolitis often also present
          • Intra-alveolar fibrin in some cases
          Terminology
          • The Centers for Disease Control and Prevention has divided cases of vaping associated lung injury into confirmed and probable based upon the following criteria (MMWR Morb Mortal Wkly Rep 2019;68:787):

            Confirmed Probable
            Use of an e-cigarette (vaping) or dabbing Within 90 days before symptom onset
            Imaging Pulmonary infiltrate, such as opacities on plain film chest radiograph or ground glass opacities on chest computed tomography
            Infectious workup Absence of pulmonary infection on initial workup Infection identified via culture or PCR but clinical team believes this is not the sole cause of the underlying respiratory disease process or minimum criteria to rule out pulmonary infection not met (testing not performed) and clinical team believes this is not the sole cause of the underlying respiratory disease process
            Alternative diagnosis No evidence in medical record of alternative plausible diagnoses (e.g., cardiac, rheumatologic or neoplastic process)
          ICD coding
          Epidemiology
          Sites
          • Lung
          Pathophysiology
          • Currently not well understood; however, it is suspected that one or more compounds in the vaping fluid produce an acute lung injury
          • Initial outbreak of illness was linked to the use of vitamin E (tocopheryl acetate) as a cutting agent for the THC, which peaked in the summer of 2019 (Arch Pathol Lab Med 2020;144:1490)
          • It is not clear if other cutting agents or contaminate can produce similar acute lung injury reactions
          Etiology
          • Vaping is the action of using an e-cigarette, a battery powered vaporizer that uses an atomizer (small heating element and a wicking material), to aerosolize a liquid
            • When the user inhales, a flow sensor activates the heating element that atomizes the liquid
          • E-cigarettes use e-liquid or vape juice that usually contains predominantly propylene glycol and glycerin, with a small amount of nicotine, flavorings, additives and differing amounts of contaminants (MMWR Morb Mortal Wkly Rep 2019;68:787, Arch Pathol Lab Med 2020;144:1490)
            • Some also contain cannabidiol or cannabis oil but the precise composition is rarely known
          • Dabbing is a related process that involves inhaling or smoking concentrated extracts of cannabis
          • Production of toxic ketene gas upon heating of vitamin E acetate has been reported (Proc Natl Acad Sci U S A 2020;117:6349)
            • This offers a potential explanation for the lung injury seen in cases of EVALI related to vitamin E
          Clinical features
          • Patients are typically healthy prior to presentation with a history of vaping use
          • Presentation is typically in an acute / subacute time frame with respiratory complaints that can mimic an infectious or inflammatory process
          • Majority (but not all cases) have reported the use of tetrahydrocannabinol or cannabinoid containing products (MMWR Morb Mortal Wkly Rep 2019;68:860)
          Diagnosis
          • As defined by the Centers for Disease Control and Prevention, confirmed cases will have a history of vaping with exclusion of other infectious and inflammatory conditions (MMWR Morb Mortal Wkly Rep 2019;68:787)
          • Biopsy is not required for diagnosis but can be supportive
          Laboratory
          • Negative infectious testing, including respiratory viral panel, influenza PCR, urine antigen for Streptococcus, sputum culture or bronchioloalveolar lavage culture
          • Oil red O staining has been performed on bronchoalveolar lavage fluids in a number of cases with variable results; however, positive staining or a lipid laden macrophages index has not been shown to be a sensitive or specific marker for vaping associated lung injury (Am J Clin Pathol 2020;153:30)
          Radiology description
          Radiology images

          Contributed by Matthew J. Cecchini, M.D., Ph.D. and Brandon T. Larsen, M.D., Ph.D.

          Ground glass & consolidation

          Ground glass

          Prognostic factors
          • Unknown
          Case reports
          Treatment
          Microscopic (histologic) description
          • A spectrum of acute lung injury can be observed, including (Am J Clin Pathol 2020;153:30, N Engl J Med 2019;381:1780)
            • Organizing pneumonia, with mucopolysaccharide rich intraluminal plugs of proliferating fibroblasts within alveolar spaces and distal bronchioles
            • Diffuse alveolar damage, with diffuse alveolar septal thickening, type II pneumocyte hyperplasia and hyaline membranes
            • Acute fibrinous pneumonitis, with abundant intra-alveolar fibrin accompanied by alveolar septal edema and variable degrees of organization
          • Majority of cases have foamy macrophages and foamy pneumocytes; however, this is a nonspecific finding and can also been seen in other scenarios, such as amiodarone exposure
          • Neutrophils and eosinophils have also been observed in a subset of cases but eosinophils are usually rare
          • No large lipid inclusions or other features of exogenous lipoid pneumonia have been observed in published cases
          Microscopic (histologic) images

          Contributed by Matthew J. Cecchini, M.D., Ph.D. and Brandon T. Larsen, M.D., Ph.D.

          Diffuse alveolar damage

          Acute lung injury with organizing pneumonia

          Acute fibrinous pneumonitis

          Bronchiolitis

          Foamy change

          Cytology description
          Negative stains
          • Negative stains for fungal and acid fast organisms
          Sample pathology report
          • Lung, right, transbronchial biopsy:
            • Diffuse alveolar damage (see comment)
            • Comment: The provided clinical history of vaping is noted. Diffuse alveolar damage has been observed as one of the patterns of acute lung injury observed in patients with vaping associated lung injury. However, diffuse alveolar damage is a nonspecific pattern of acute lung injury that can be secondary to many etiologies including infection, septicemia / shock and adverse drug reaction. These other potential causes of acute lung injury must be excluded clinically.
          Differential diagnosis
          • Diffuse alveolar damage:
            • Infection (Pneumocystis, Legionella, Mycoplasma, viral), other toxic inhalants, drug toxicity, shock, sepsis, aspiration, radiation, systemic connective tissue diseases, idiopathic
          • Organizing pneumonia:
            • Infection (viral, bacterial, postobstructive), other toxic inhalants, drug toxicity, radiation, collagen vascular disease, aspiration, idiopathic
          Board review style question #1
          Which of the following is a specific and sensitive confirmatory test for vaping associated acute lung injury?

          1. Congo red
          2. Martius scarlett blue
          3. Masson trichrome
          4. No confirmatory test exists
          5. Oil red O
          Board review style answer #1
          D. No confirmatory test exists. The histologic findings in vaping associated acute lung injury are nonspecific and there are no confirmatory tests to confirm the etiology of vaping associated lung disease. All tests, including oil red O, are nonspecific. The diagnosis of vaping associated acute lung injury is based on features of acute lung injury in the setting of appropriate exposure history and exclusion of other infectious and other causes of acute lung injury.

          Comment Here

          Reference: Vaping associated acute lung injury
          Board review style question #2

          The acute lung injury present in these biopsies from a patient with vaping associated acute lung injury is best characterized as which of the following patterns?

          1. Diffuse alveolar damage
          2. Ischemia
          3. Organizing pneumonia
          4. Vasculitis
          Board review style answer #2
          C. Organizing pneumonia

          Comment Here

          Reference: Vaping associated acute lung injury

          Varicella zoster
          Clinical features
          • Overwhelmingly a benign disease in childhood (Eur J Clin Microbiol Infect Dis 2011;30:435)
          • Adults with infection have 25× risk of children in developing pneumonia; occurs in 10% - 15% of adult patients (Ann Thorac Med 2007;2:163)
          • Greatest risk of severe disease in immunosuppressed patients, especially bone marrow transplant recipients, patients with chronic lung disease, pregnant women and neonates
          • Incidence has declined with effective treatment, but is increasing due to unvaccinated individuals
          • Diagnosis usually apparent from skin lesions and clinical history, but immunohistochemistry, culture or PCR may be necessary to rule out HSV
          • Pneumonia usually occurs 2 - 7 days after rash appears
          • Pulmonary calcifications may occur after recovery
          • Untreated pneumonia has 10% - 40% mortality rate
          Gross description
          • Heavy, firm and purple lungs
          • May have hemorrhagic lesions on pleura resembling skin lesions
          Microscopic (histologic) description
          • Necrosis of bronchial and alveolar epithelium, acute inflammation and hemorrhage, eosinophilic nuclear inclusions and rare multinucleated cells at edge of necrotic foci

          Veliparib
          Definition / general
          • A poly (ADP-ribose) polymerase (PARP) inhibitor blocks PARP1 and PARP2, thereby preventing repair of DNA damage
          • Developed by Abbvie
          • Synonym: ABT-888, 912444-00-9, CHEBI:62880, 2-[(R)-2-methylpyrrolidin-2-yl]-1H-benzimidazole-4-carboxamide, veliparib
          Trade name
          • Veliparib®
          Indications
          Pathophysiology
          • Base excision repair and homologous recombination mediated double strand breaks system responsible for the repair of damaged DNA (Figure 1, panel A)
          • In cells with BRCA1/2 mutated cells, base excision repair and other DNA repair mechanisms compensate for the nonfunctional homologous recombination (Figure 1, panel B)
          • In cells with nonfunctioning base excision repair because of PARP1 inhibition, at least one copy of BRCA1/2 with functioning homologous recombination can repair the damaged DNA (Figure 1, panel C)
          • In mutated BRCA1/2 cancer cells, PARP inhibitors prevents the base excision repair leading to cell death (Figure 1, panel D)
          • Reference: Mod Pathol 2020;33:944, see diagram below
          Diagrams / tables

          Images hosted on other servers:

          Mechanism of PARP1 inhibitor in cancer cells

          Clinical information
          Uses by pathologists
          • None at this time
          Side effects
          • Gastrointestinal: nausea, vomiting
          • Fatigue
          • Myelosuppression (anemia, neutropenia, thrombocytopenia)
          • Secondary leukemia and myelodysplastic syndrome (late serious effects)
          • Reference: Onco Targets Ther 2015;8:1931
          Drug administration
          • Drug taken by mouth
          Board review style question #1
            Veliparib is a potential therapeutic agent inhibiting which of the following proteins?

          1. BRAF
          2. BRCA1 and BRCA2
          3. PARP1 and PARP2
          4. TP53
          Board review style answer #1
          C. PARP1 and PARP2

          Comment Here

          Reference: Veliparib

          WHO classification
          Definition / general
          WHO (2015)
          WHO classification of tumors of the lung (Travis: WHO Classification of Tumours of the Lung, Pleura, Thymus and Heart, 4th Edition, 2015)
            Epithelial tumorsICD-O codes
          • Adenocarcinoma 8140/3
            • Lepidic adenocarcinoma 8250/3
            • Acinar adenocarcinoma 8551/3
            • Papillary adenocarcinoma 8260/3
            • Micropapillary adenocarcinoma8265/3
            • Solid adenocarcinoma8230/3
            • Invasive mucinous adenocarcinoma8253/3
              • Mixed invasive mucinous and nonmucinous adenocarcinoma 8254/3
            • Colloid adenocarcinoma 8480/3
            • Fetal adenocarcinoma8333/3
            • Enteric adenocarcinoma 8144/3
            • Minimally invasive adenocarcinoma
              • Nonmucinous 8256/3
              • Mucinous8257/3
            • Preinvasive lesions
              • Atypical adenomatous hyperplasia 8250/0
              • Adenocarcinoma in situ8140/2
                • Nonmucinous8250/2
                • Mucinous8253/2
          • Squamous cell carcinoma8070/3
            • Keratinizing squamous cell carcinoma8071/3
            • Nonkeratinizing squamous cell carcinoma8072/3
            • Basaloid squamous cell carcinoma8083/3
            • Preinvasive lesion
              • Squamous cell carcinoma in situ8070/2
          • Neuroendocrine tumors
            • Small cell carcinoma8041/3
              • Combined small cell carcinoma8045/3
            • Large cell neuroendocrine carcinoma8013/3
              • Combined large cell neuroendocrine carcinoma8013/3
            • Carcinoid tumors
              • Typical carcinoid8240/3
              • Atypical carcinoid8249/3
            • Preinvasive lesion
              • Diffuse idiopathic pulmonary neuroendocrine cell hyperplasia8040/0
          • Large cell carcinoma8012/3
          • Adenosquamous carcinoma8560/3
          • Pleomorphic carcinoma8022/3
          • Spindle cell carcinoma 8032/3
          • Giant cell carcinoma8031/3
          • Carcinosarcoma8980/3
          • Pulmonary blastoma8972/3
          • Other and unclassified carcinomas
            • Lymphoepithelioma-like carcinoma8082/3
            • NUT carcinoma8023/3
          • Salivary gland type tumors
            • Mucoepidermoid carcinoma8430/3
            • Adenoid cystic carcinoma8200/3
            • Epithelial myoepithelial carcinoma8562/3
            • Pleomorphic adenoma8940/0
          • Papillomas
            • Squamous cell papilloma8052/0
              • Exophytic8052/0
              • Inverted8053/0
            • Glandular papilloma8260/0
            • Mixed squamous and glandular papilloma8560/0
          • Adenomas
            • Sclerosing pneumocytoma8832/0
            • Alveolar adenoma8251/0
            • Papillary adenoma8260/0
            • Mucinous cystadenoma8470/0
            • Mucous gland adenoma8480/0
          Mesenchymal tumors
          • Pulmonary hamartoma8992/0
          • Chondroma9220/0
          • PEComatous tumors
            • Lymphangioleiomyomatosis9174/1
            • PEComa, benign8714/0
              • Clear cell tumor8005/0
            • PEComa, malignant8714/3
          • Congenital peribronchial myofibroblastic tumor8827/1
          • Diffuse pulmonary lymphangiomatosis
          • Inflammatory myofibroblastic tumor8825/1
          • Epithelioid hemangioendothelioma9133/3
          • Pleuropulmonary blastoma8973/3
          • Synovial sarcoma9040/3
          • Pulmonary artery intimal sarcoma9137/3
          • Pulmonary myxoid sarcoma with EWSR1-CREB1 translocation8842/3
          • Myoepithelial tumors
            • Myoepithelioma8982/0
            • Myoepithelial carcinoma8982/3
          Lymphohistiocytic tumors
          • Extranodal marginal zone lymphoma of mucosa associated lymphoid tissue (MALT lymphoma)9699/3
          • Diffuse large B cell lymphoma9680/3
          • Lymphomatoid granulomatosis9766/1
          • Intravascular large B cell lymphoma9712/3
          • Pulmonary Langerhans cell histiocytosis9751/1
          • Erdheim-Chester disease9750/1
          Tumors of ectopic origin
          • Germ cell tumors
            • Teratoma, mature9080/0
            • Teratoma, immature9080/1
          • Intrapulmonary thymoma8580/3
          • Melanoma8720/3
          • Meningioma, NOS9530/0
          Metastatic tumors

          • ICD-O note: behavior is coded: /0 for benign tumors; /1 for unspecified, borderline, or uncertain behavior; /2 for carcinoma in situ and grade III intraepithelial neoplasia; and /3 for malignant tumors
          Major updates
          • Conceptual changes (J Thorac Oncol 2015;10:1243)
            • Use of immunohistochemistry throughout the classification
            • New emphasis on genetic studies, in particular, integration of molecular testing to help personalize treatment strategies for advanced lung cancer patients
            • New classification for small biopsies and cytology similar to that proposed in the 2011 IASLC / ATS / ERS classification
          • Changes in terminology and diagnostic criteria (J Thorac Oncol 2015;10:1243)
            • Revised classification of lung adenocarcinoma
            • Restricted the diagnosis of large cell carcinoma only to resected tumors that lack any clear morphologic or immunohistochemical differentiation with reclassification of the remaining former large cell carcinoma subtypes into different categories
            • Reclassified squamous cell carcinomas into keratinizing, nonkeratinizing and basaloid subtypes with the nonkeratinizing tumors requiring immunohistochemistry proof of squamous differentiation
            • Grouped neuroendocrine tumors in a single category
            • Added NUT carcinoma
            • Changed the term sclerosing hemangioma to sclerosing pneumocytoma
            • Changed the name hamartoma to pulmonary hamartoma
            • Created a group of PEComatous tumors that include (a) lymphangioleiomyomatosis, (b) PEComa, benign (with clear cell tumor as a variant) and (c) PEComa, malignant
            • Introduced the entity pulmonary myxoid sarcoma with an EWSR1-CREB1 translocation
            • Added the entities myoepithelioma and myoepithelial carcinomas, which can show EWSR1 gene rearrangements
            • Recognition of usefulness of WWTR1-CAMTA1 fusions in diagnosis of epithelioid hemangioendotheliomas
            • Added Erdheim-Chester disease to the lymphoproliferative tumor and group of tumors of ectopic origin to include germ cell tumors, intrapulmonary thymoma, melanoma and meningioma
          Board review style question #1
            Which new entity has been added in the 2015 WHO classification of lung tumors?

          1. Adenosquamous carcinoma
          2. MALT lymphoma
          3. NUT carcinoma
          4. Sclerosing hemangioma
          5. Hyalinizing granuloma
          Board review style answer #1
          C. NUT carcinoma. NUT midline carcinoma is a recently recognized highly aggressive tumor defined by a NUT fusion. Being exceptionally rare in the lung, it was introduced in the 2015 WHO classification of lung tumors. Adenosquamous carcinoma and MALT lymphoma are in the WHO classification but not new entities in the 2015 edition. Sclerosing hemangioma is an outdated term; this tumor has been recently renamed to sclerosing pneumocytoma. Hyalinizing granuloma is an inflammatory lesion, not a tumor.

          Comment Here

          Reference: Lung - WHO classification
          Board review style question #2
            Which is the name of an adenocarcinoma with this growth pattern according to the 2015 WHO classification of lung tumors?

          1. Acinar
          2. Bronchioloalveolar
          3. Lepidic
          4. Micropapillary
          5. Papillary
          Board review style answer #2
          C. Lepidic. Lepidic adenocarcinoma consists of neoplastic cells growing along the surface of alveolar walls (lepidic pattern) with no architectural disruption and no lymphovascular / pleural invasion. It was formerly known as bronchioloalveolar carcinoma (obsolete term).

          Comment Here

          Reference: Lung - WHO classification
          Back to top
          Recent Lung Pathology books

          Cagle: 2015

          Churg: 2019

          Duarte Achcar: 2016

          Fraig: 2015

          Husain: 2021

          IAC-IARC-WHO: 2023

          IARC: 2015

          IARC: 2021

          Jiang: 2023

          Katzenstein: 2016

          Mukhopadhyay: 2016

          Mukhopadhyay: 2023

          Papotti: 2015

          Popper: 2016

          Smith: 2023

          Suster : 2020

          Suster: 2022

          VandenBussche: 2017

          VandenBussche: 2019

          Zhang: 2018



          Find related Pathology books: cardiovascular, cytopathology, GI, lung, mediastinum/serosa, pediatric
          Image 01 Image 02